Combine Chemistry Rotation

Pataasin ang iyong marka sa homework at exams ngayon gamit ang Quizwiz!

Which of the following 2 hour postprandial glucose values demonstrates unequivocal hyperglycemia diagnostic for diabetes mellitus? A. 160 mg/dL (8.8 mmol/L) B. 170 mg/dL (9.4 mmol/L) C. 180 mg/dL (9.9 mmol/L) D. 200 mg/dL (11.0 mmol/L)

Correct Answer: D

metabolic acidosis causes

DKA, severe diarrhea, renal failure, shock

Paget's disease

a bone disease of unknown cause characterized by the excessive breakdown of bone tissue, followed by abnormal bone formation --highest ALP values

Hemoglobin derivatives

hemoglobin breakdown products include porphyrins, bilirubin, and urobilirubin

chylomicron & VLDL

how are triglycerides tranported?

liver disease

increase ALP+ increase 5'NT=

Unconjugated bilirubin

indirect bilirubin; relatively insoluble in water

Hypocalcemia

low levels of calcium in the blood --tetany

anemia of chronic disease (inflammation)

what disease state(s) would yield these results? serum iron: decreased transferrin saturation: decreased TIBC: decreased serum ferritin: increased

20-250µg/dL

what is the normal range for serum ferritin in males?

250-425µg/dL

what is the normal ranger for TIBC?

CK and MB

trauma to the skeletal muscle causes increase in total CK and MB isoenzymes, but % activity MB is < 3%

When quantifying serum total proteins, upon what is the intensity of the color produced in the biuret reaction dependent? A. Molecular weight of the protein B. Acidity of the medium C. Number of peptide bonds D. Nitrogen content of the protein

A commonly used method to quantify serum total proteins is the biuret procedure. The biuret reaction is based on the complexing of cupric ions in an alkaline solution with the peptide linkages of protein molecules. Because the amino acids of all proteins are joined together by peptide bonds, this method provides an accurate quantification of the total protein content of serum. The greater the amount of protein in a specimen, the greater will be the number of available peptide bonds for reaction and the more intense the colored reaction will be. In the biuret reaction, the intensity of the reddish violet color produced is proportional to the number of peptide bonds present. Generally, one cupric ion complexes with four to six peptide linkages. However, a colored product may be formed when the cupric ion links through coordinate bonds with at least two peptide linkages, with the smallest compound able to react being the tripeptide. Therefore, not only will proteins contribute to the formation of the colored product, but so, too, will any tripeptides and polypeptides present in a serum sample.

In potentiometry, which of the following is considered the standard electrode? A. Hydrogen electrode B. Calcium electrode C. Potassium electrode D. Copper electrode

A. A half-cell, also called an electrode, is composed of a single metallic conductor surrounded by a solution of electrolyte. An electrochemical cell consists of two half-cells. If two different kinds of half-cells are connected in such a way as to make a complete circuit, a current will flow because of the potential difference between the two electrodes. The connection must be between the two metallic conductors and also between the two electrolyte solutions, usually by means of a salt bridge. In the analytical technique of potentiometry, a comparison is made between the voltage of one half-cell connected to another half-cell. It is customary that all half-cell potentials be compared to the potential generated by a standard electrode. The universally accepted standard half-cell with which all other halfcells are compared is the standard hydrogen electrode, arbitrarily assigned a potential E° of 0.000 volt.

Which of the following is not a typical finding in magnesium deficiency tetany? A. High serum phosphate level B. Normal serum calcium level C. Normal blood pH value D. Low serum potassium level

A. A low ionized serum magnesium level is characteristic of a magnesium deficiency tetany. The serum magnesium level usually ranges between 0.15 and 0.5 mmol/L when tetany occurs. In addition, the serum calcium level and blood pH are normal, whereas the serum potassium level is decreased. This type of tetany is treated with MgSO4 to increase the level of serum magnesium, thus alleviating the tetany and convulsions that accompany this disorder.

What effect would a low-salt diet, upright position, and diuretics have on the following test results? A. Renin T, aldosterone T, hypernatremia, hypokalemia B. Renin T, aldosterone I, hypernatremia, hypokalemia C. Renin I, aldosterone I, hyponatremia, hyperkalemia D. Renin I, aldosterone T, hyponatremia, hyperkalemia

A. A low-salt diet, upright position, and diuretics cause a decrease in effective plasma volume. This decrease stimulates the renin-angiotensin system, which increases aldosterone secretion. Aldosterone promotes sodium retention and potassium loss.

Hemoglobin S is an abnormal hemoglobin that is characterized by a substitution of which amino acid? A. Valine for glutamic acid in position 6 on the beta chain B. Valine for glutamic acid in position 6 on the alpha chain C. Lysine for glutamic acid in position 6 on the beta chain D. Lysine for glutamic acid in position 6 on the alpha chain

A. A number of hemoglobinopathies exist where a substitution of one amino acid on either the alpha chain or the beta chain causes the formation of an abnormal hemoglobin molecule. Hemoglobin S is an abnormal hemoglobin that is characterized by the substitution of valine for glutamic acid in position 6 of the beta chain. Hemoglobin C is an abnormal hemoglobin in which lysine replaces glutamic acid in position 6 of the beta chain. The structural changes that are seen in hemoglobins S and C disorders are inherited as autosomal recessive traits.

Which of the following results would be the most consistent with high risk for coronary heart disease? A. 20 mg/dL HDL cholesterol and 250 mg/dL total cholesterol B. 45 mg/dL HDL cholesterol and 210 mg/dL total cholesterol C. 50 mg/dL HDL cholesterol and 180 mg/dL total cholesterol D. 55 mg/dL HDL cholesterol and 170 mg/dL total cholesterol

A. A number of risk factors are associated with developing coronary heart disease. Notable among these factors are increased total cholesterol and decreased HDL cholesterol levels. Although the reference ranges for total cholesterol and HDL cholesterol vary with age and sex, reasonable generalizations can be made: An HDL cholesterol less than 40 mg/dL and a total cholesterol value >240 mg/dL are undesirable and the individual is at greater risk for coronary heart disease. Total cholesterol values between 200 and 239 mg/dL are borderline high.

How is the majority of reverse T3 (rT3) made? A. Peripheral deiodination of T4 B. Peripheral deiodination of T3 C. From T3 in the thyroid gland D. From thyroglobulin in the thyroid gland

A. A small amount of reverse T3 (rT3) is made in the thyroid gland, but the majority is made from peripheral deiodination of T4. rT3 varies from T3 in that rT3 contains one iodine atom in the tyrosyl ring and two iodines in the phenolic ring, whereas T3 has two iodines in the tyrosyl ring and one iodine in the phenolic ring. rT3 does not have any physiological action as it is metabolically inactive. However, increased levels of rT3 are associated with nonthyroidal illness (NTI), which also manifests with decreased levels of total T3.

A 42-year-old male presents with anorexia, nausea, fever, and icterus of the skin and mucous membranes. He noticed that his urine had appeared dark for the past several days. The physician orders a series of biochemical tests. Based on the following test results, what is the most likely diagnosis? Serum alkaline phosphatase—slightly elevated Serum alanine aminotransferase— markedly elevated Serum aspartate aminotransferase— markedly elevated Serum gamma-glutamyltransferase— slightly elevated Serum total bilirubin—moderately elevated Urine bilirubin—positive Fecal urobilinogen—decreased A. Acute hepatitis B. Alcoholic cirrhosis C. Metastatic carcinoma of the pancreas D. Obstructive jaundice

A. Acute hepatitis is characterized by markedly elevated levels of serum alanine aminotransferase and aspartate aminotransferase, which may range from 10- to 100-fold greater than the reference values. Although alkaline phosphatase and gamma-glutamyltransferase are increased, their elevations are less notable than the aminotransferases. Alkaline phosphatase may range up to two times the reference range whereas gamma-glutamyltransferase may go as high as five times the reference range in acute hepatitis. Due to leakage of conjugated bilirubin from the hepatocytes, the urine bilirubin will be positive. With less conjugated bilirubin reaching the intestines, fecal urobilinogen will be less than normal.

Nortriptyline is the active metabolite of which of the following drugs? A. Amitriptyline B. Desipramine C. Imipramine D. Doxepin

A. Amitriptyline, doxepin, and imipramine and their active metabolites nortriptyline, nordoxepin, and desipramine, respectively, are tricyclic compounds particularly useful in the treatment of endogenous depression. These compounds are lipid soluble and, therefore, highly protein bound in the plasma. Although toxic concentrations of these drugs often lead to cardiac arrhythmias, low concentrations have been found to have antiarrhythmic activity. Because of these varying biological effects at differing serum concentrations, there is a need both for monitoring in cases of therapy and screening for toxic effects in cases of overdose.

Which of the following disorders is best characterized by laboratory findings that include increased serum levels of inorganic phosphorus, magnesium, potassium, uric acid, urea, and creatinine and decreased serum calcium and erythropoietin levels? A. Chronic renal failure B. Renal tubular disease C. Nephrotic syndrome D. Acute glomerulonephritis

A. As renal function continues to be lost over time, chronic renal failure develops. Chronic renal failure is manifested by loss of excretory function, inability to regulate water and electrolyte balance, and increased production of parathyroid hormone, all of which contribute to the abnormal laboratory findings. The decreased production of erythropoietin causes anemia to develop.

When performing electrophoresis at pH 8.6, which hemoglobin molecule migrates the fastest on cellulose acetate toward the anode? A. A! B. A2 C. F D. S

A. At pH 8.6, hemoglobins have a net negative charge and migrate from the point of application toward the anode. When hemoglobin electrophoresis is performed on cellulose acetate at pH 8.6, hemoglobin A migrates the fastest toward the anode, followed respectively by hemoglobins F and S. Hemoglobins A2 and C have the same electrophoretic mobility and migrate slightly slower than hemoglobin S. Because hemoglobins A2 and C exhibit nearly the same mobility, they cannot be differentiated on cellulose acetate.

Which of the following may be associated with bioluminescence? A. Light emission produced due to enzymatic oxidation of a substrate B. Less sensitive than direct fluorescent assays C. Electron excitation caused by radiant energy D. Employs a radioactive label

A. Bioluminescence is a type of chemiluminescence in which the excitation energy is supplied by an enzymatic chemical reaction rather than by radiant energy, as in fluorescence and phosphorescence. Bioluminescence assays may employ such systems as NADH:FMN oxidoreductasebacterial luciferase or adenosine triphosphatefirefly luciferase. Bioluminescence assays are nonradioactive, having sensitivity levels in the attomole (1CT18) to zeptomole (10~21) ranges, which makes them more sensitive than direct fluorescence assays. Bioluminescence has been applied in the development of immunoassays.

Which of the following is a sialylated Lewis blood group antigen associated with colorectal carcinoma? A. CA 19-9 B. CA 15-3 C. CA549 D. CEA

A. CA 19-9 is an oncofetal protein that is a sialylated Lewis blood group antigen. It is found in increased levels in colorectal carcinoma as well as in gastric, hepatobiliary, and pancreatic cancers. CA 19-9 is also elevated in several benign disorders, including pancreatitis, extra-hepatic cholestasis, and cirrhosis. The combination use of CA 19-9 and CEA (carcinoembryonic antigen) is helpful in monitoring the recurrence of colorectal cancer.

Which of the following would be elevated in the blood in medullary carcinoma of the thyroid? A. Calcitonin B. Thyroxine C. Catecholamines D. Secretin

A. Calcitonin is a calcium-lowering hormone secreted by the parafollicular or C cells of the thyroid. Calcitonin acts as an antagonist to parathyroid hormone (PTH) action on the bone and kidneys. Medullary carcinoma of the thyroid is a neoplasm of the parafollicular cells that usually results in elevated seaim levels of calcitonin. If the fasting calcitonin level is within the normal reference interval in a patient with suspected medullary carcinoma, a provocative calcium infusion test is often useful in improving the sensitivity of the test.

Which of the following is characterized by a deficiency of glucose-6-phosphatase resulting in hepatomegaly, lactic acidosis, and severe fasting hypoglycemia? A. Type I—von Gierke disease B. Type II—Pompe disease C. Type III—Cori disease D. Type IV—Andersen disease Lipids and Lipoproteins

A. Carbohydrate is stored in the body in the form of glycogen. There are many enzymes involved in the metabolism of glycogen. A deficiency of any one of the enzymes involved will result in what are called glycogen storage diseases, or glycogenoses. There are at least 10 distinct types of glycogen storage diseases, and all of them are rare. All are hereditary. Diagnosis of each type can be made by the assay of the deficient enzyme from the appropriate tissue and by microscopic study of the affected tissues. • Type I—von Gierke disease is clinically characterized by severe fasting hypoglycemia and lactic acidosis. This is due to a deficiency of the enzyme glucose-6-phosphatase. Glucose cannot be transported from the liver as glucose-6-phosphate during the breakdown of glycogen. It is metabolized to lactic acid and thus results in lactic acidosis. • Type II—Pompe disease is caused by a deficiency of lysosomal a-l,4-glucosidase. This results in an increase of glycogen in all organs and abnormally large lysosomes. The glycogen cannot be degraded because of the deficiency of a-1,4-glucosidase. • Type III—Cori disease is caused by the absence of a debrancher enzyme. This disease is characterized by hypoglycemia, hepatomegaly, seizures, and growth retardation. • Type IV—Andersen disease is caused by a deficiency of brancher enzyme. It is a rare disease characterized by progressive liver enlargement or cirrhosis and muscular weakness by the age of 2 months. Storage glycogen is not usually found, but unbranched amylopectin accumulates in this disease.

Which of the following chromatography systems is characterized by a stationary phase of silica gel on a piece of glass and a moving phase of liquid? A. Thin-layer B. Ion-exchange C. Gas-liquid D. Partition

A. Chromatography provides a variety of means of separating mixtures of substances on the basis of their physicochemical properties, primarily their solubility in a variety of solvents. Chromatographic methods always involve a stationary phase and a mobile phase. The sample containing the substances to be separated is carried in the mobile phase; the mobile phase passes over the stationary phase at different rates depending on their relative solubilities in the two phases. The amount of separation depends on (1) the rate of diffusion, (2) the solubility of the substances being separated, and (3) the nature of the solvent. In TLC, the stationary phase is a thin layer of some sorbent such as silica gel uniformly spread on a piece of glass or plastic.

Which of the following is not a colligative property of solutions? A. pH B. Freezing point C. Osmotic pressure D. Vapor pressure

A. Colligative properties of a solution are those properties that depend only on the number of particles in solution, not on the nature of the particles. The colligative properties are boiling point, freezing point, osmotic pressure, and vapor pressure. Terms used to describe the concentration of particles in solution are "osmole" (the number of particles, 6.0224 x 1023, that lowers the freezing point 1.86°C) and "osmolal" (a concentration of 1 Osm of solute per kilogram of water). One mole of an un-ionized solute dissolved in 1 kg of water lowers the freezing point 1.86°C. Thus it is an osmolal solution. For un-ionized substances such as glucose, 1 mol equals 1 Osm. For substances that ionize, such as sodium chloride, wherein each molecule in solution becomes two ions and thus two particles, 1 mol of sodium chloride theoretically equals 2 Osm. In reality, however, this is not always the case; an osmotic activity coefficient factor is used to correct for the deviation. In practice, three types of osmometers are available. They are the freezing point, vapor pressure, and colloid osmotic pressure osmometers.

An increased serum level of which of the following analytes is most commonly associated with decreased glomerular filtration? A. Creatinine B. Uric acid C. Urea D. Ammonia

A. Creatinine is a waste product of muscle metabolism and as such its production is rather constant on a daily basis. Creatinine is freely filtered by the glomerulus, with only a very small amount secreted by the proximal tubule. Thus, measurement of creatinine is a reflection of glomerular filtration. An increase in the serum creatinine level would be indicative of decreased glomerular filtration. Although uric acid, urea, and ammonia levels may be increased with decreased glomerular filtration, increased levels of these analytes are associated with a number of specific metabolic diseases and, therefore, they are not used as indicators of the glomerular filtration rate.

Which of the following statements best describes discrete analysis? A. Each sample-reagent mixture is handled separately in its own reaction vessel. B. Samples are analyzed in a flowing stream of reagent. C. Analyzer must be dedicated to measurement of only one analyte. D. It does not have random access capability.

A. Discrete analyzers are designed so that each specimen-reagent mixture is analyzed separately in its own vessel. Although a discrete analyzer may be designed to measure only one analyte, most discrete analyzers are very versatile and are able to run multiple tests on each sample. Some discrete analyzers also have random access capability that allows STAT samples to be accessed easily.

If elevated, which of the following is associated with increased risk for coronary heart disease? A. Homocysteine B. Vitamin B6 C. Myoglobin D. pro-BNP

A. Elevated homocysteine levels are associated with increased risk for coronary heart disease. Increased homocysteine contributes to the damage of arterial walls preceding formation of plaques. Individuals at risk need to be evaluated for vitamin B levels, because low levels of folic acid, vitamin 65, and vitamin 612 are associated with increased levels of homocysteine.

In what form is glucose stored in muscle and liver? A. Glycogen B. Maltose C. Lactose D. Starch

A. Glycogen is a polysaccharide composed of many glucose molecules. In contrast to the amylopectin molecule, a glycogen molecule is more highly branched and more compact. Glycogen is found in a variety of animal tissues, particularly in the liver, and provides the storage form for carbohydrates in the body. When energy requirements warrant it, glycogen may be broken down to glucose by a series of phosphorylating and related enzymes.

Which glycoprotein binds with hemoglobin to facilitate the removal of hemoglobin by the reticuloendothelial system? A. Haptoglobin B. Ceruloplasmin C. cxpAntitrypsin D. Fibrinogen

A. Haptoglobin is a glycoprotein produced mainly by the liver that migrates electrophoretically as an alpha2-globulin. Increased serum concentrations of haptoglobin are seen in inflammatory conditions and tissue necrosis, whereas decreased levels are seen in hemolytic situations in which there is extensive red blood cell destruction. In the latter situation, haptoglobin binds with free hemoglobin to form a stable complex that may then be removed by the reticuloendothelial system. Because of the size of the haptoglobin-hemoglobin complex, urinary excretion of hemoglobin by the kidney is avoided, thereby preventing the loss of iron by the kidney.

Which of the following is characteristic of hemolytic jaundice? A. Unconjugated serum bilirubin level increased B. Urinary bilirubin level increased C. Urinary urobilinogen level decreased D. Fecal urobilin level decreased

A. Hemolytic jaundice is also referred to as prehepatic jaundice. It is caused by excessive destruction of erythrocytes at a rate that exceeds the conjugating ability of the liver. As a result, increased levels of unconjugated bilirubin appear in the blood. The amount of conjugated bilirubin being formed in the liver is proportionately greater than normal; this is reflected in the increased levels of urobilinogen and urobilin found in the stool. Because of the enterohepatic circulation, the increased urobilinogen levels in the small intestines are reflected by an increase in the circulating blood levels of urobilinogen. Because the liver is unable to pick up all the circulating urobilinogen, the urinary levels of urobilinogen are increased. Urinary bilirubin levels are negative because the blood level of conjugated bilirubin is usually normal.

What technique is used to quantify specific immunoglobulin classes? A. Immunonephelometry B. Serum protein electrophoresis C. Isoelectric focusing D. Immunoelectrophoresis

A. Immunonephelometric and immunoturbidimetric techniques are used to quantify specific immunoglobulin classes. Nephelometric techniques used to quantify the immunoglobulins are based on the measurement of light scatter by the antigen-antibody complexes formed. This method also calls for the comparison of unknowns with standards. Although radial immunodiffusion can be used to quantify the immunoglobulins, it is not a method of choice. Serum protein electrophoresis, immunoelectrophoresis, and isoelectric focusing cannot be used to quantify the immunoglobulins.

Which of the following reagents is not utilized in a coupled enzymatic reaction method to quantify serum creatinine? A. Picric acid B. Chromogenic dye C. Creatinine amidohydrolase D. Sarcosine oxidase

A. In addition to the endpoint and kinetic methods, which use the Jaffe reaction (picric acid), several methods have been developed that use coupled enzymatic reactions for the quantification of creatinine. In one such method, creatinine amidohydrolase (creatininase) catalyzes the conversion of creatinine to creatine and subsequently to sarcosine and urea. Sarcosine oxidase catalyzes the oxidation of sarcosine to glycine, formaldehyde, and hydrogen peroxide. The hydrogen peroxide reacts with the reduced form of a chromogenic dye in the presence of peroxidase to form an oxidized colored dye product that is read spectrophotometrically.

Identification of which of the following is useful in early stages of glomerular dysfunction? A. Microalbuminuria B. Ketonuria C. Hematuria D. Urinary light chains

A. In renal disease, glomerular or tubular malfunction results in proteinuria. In early stages of glomerular dysfunction, small quantities of albumin will appear in the urine. Because the concentration is so low, urine dipstick assays are unable to detect the presence of such a small quantity of albumin; hence the term "microalbuminuria." Annual testing of diabetic individuals for microalbuminuria is recommended, because identification of these low levels of albumin that precede nephropathy would allow for clinical intervention to control blood glucose levels and blood pressure. The reference interval for urinary albumin is less than 30 mg/day. Microalbuminuria may be quantified using immunonephelometry and enzyme immunoassay.

A mpatient has a urine uric acid level of 1575 mg/day. What effect will this have on the measured urine glucose level when the glucose oxidase/peroxidase method is employed? A. Urine glucose level will be falsely low. B. Urine glucose level will be falsely high. C. Urine glucose level will be accurate. D. Urine glucose level will exceed the linearity of the method.

A. In the glucose oxidase/peroxidase method, the second coupled enzyme reaction involves peroxidase catalyzing the reaction between hydrogen peroxide and a chromogenic oxygen acceptor, which is oxidized to its colored form. Several blood constituents, including uric acid, ascorbic acid, bilirubin, tetracycline, hemoglobin, and glutathione, when present in increased concentrations can interfere with the assay by competing for the hydrogen peroxide produced in the first coupled enzyme reaction. This loss of hydrogen peroxide would result in falsely low plasma glucose results. Because of the high levels of uric acid normally found in urine, the glucose oxidase/peroxidase method would not be suitable for measuring urine glucose.

Which of the following results is least consistent with a diagnosis of viral hepatitis? A. Seaim total bilirubin 7.5 mg/dL, direct bilirubin 5.5 mg/dL, indirect bilirubin 2.0 mg/dL B. Urine urobilinogen increased C. AST increased 10 times the upper limit of the reference range D. ALT increased 13 times the upper limit of the reference range

A. In viral hepatitis, hepatocyte injury and necrosis cause the release of cellular contents. ALT is more specific for hepatocyte injury because it is significantly present in liver tissue, whereas AST is less specific because of its significant presence not only in liver but also in many other tissues. In viral hepatitis, both ALT and AST are significantly elevated in serum. Because liver function is compromised in viral hepatitis, the liver will be unable to pick up urobilinogen from the enterohepatic circulation to process, resulting in increased urobilinogen excretion in the urine. Although serum total bilirubin will be elevated, the indirect bilirubin (unconjugated) will comprise the larger fraction and the direct bilirubin (conjugated) will be the lesser fraction. Liver function is compromised, as is the ability of the liver to pick up bilirubin and conjugate it.

Which of the following may be associated with fluorescence polarization? A. Plane-polarized light is used for sample excitation. B. Small molecular complexes show a greater amount of polarization. C. It is a heterogeneous technique employed in fluorophore-ligand immunoassays. D. Polarized light detected is directly proportional to concentration of ligand in sample.

A. Instrumentation employing fluorescence polarization is used for such testing as therapeutic drug levels and fetal lung maturity analysis. In these immunologic assays, plane-polarized light excites fluorophors in the sample cuvet. The free fluorophore-labeled ligands rotate freely because of their small size and primarily emit depolarized light. The labeled ligand-antibody complexes rotate more slowly because of their large size and emit polarized fluorescent light. Because of the differences in emitted light, it is not necessary to separate free from bound fluorophore- labeled ligands, allowing for use of the homogeneous assay technique. The emitted fluorescence intensity is measured by a polarization analyzer in the vertical plane, followed by its 90-degree movement for measurement in the horizontal plane. The amount of polarized light detected is inversely proportional to the concentration of ligand in the serum sample.

Which of the following does not accurately describe properties associated with lactate dehydrogenase? A. Optimum pH for the catalysis of lactate to pyruvate is 7.4-7.8. B. LD is increased in a hemolyzed serum specimen. C. LD catalyzes the oxidation of lactate to pyruvate with mediation of nicotinamide-adenine dinucleotide. D. LD-4 and LD-5 are labile in the cold.

A. Lactate dehydrogenase (LD, also abbreviated LDH) is found in all body tissues and is especially abundant in red and white blood cells. Hence hemolyzed serum will give falsely elevated results for LD. The enzyme catalyzes the conversion of lactate to pyruvate at pH 8.8-9.8 and pyruvate to lactate at pH 7.4-7.8, mediated by nicotinamide adenine dinucleotide (NAD+). Each of these reactions is associated with its own unique reference range. LD exists in five isomeric forms called isoenzymes. The isoenzymes can be separated by electrophoresis. Serum specimens for LD isoenzyme determinations can be stored at room temperature for 2 or 3 days without appreciable loss of activity. Room temperature storage is necessary because LD-4 and LD-5 are labile in the cold. This is in contrast to most enzymes, which are more stable when refrigerated or frozen.

In regard to bichromatic analysis, which of the following is false? A. Absorbance is measured at the spectral absorbance peak for a blank and the sample using the same wavelength. B. Eliminates background interferences C. Sample concentration determined from difference in two measured absorbances D. Functions as a reference blank for each sample

A. Measurement of an assay at two different wavelengths is termed bichromatic. The wavelengths chosen for absorbance readings will represent the peak and base of the spectral absorbance curve for the particular assay. By determining the difference between the two measured absorbances, the sample's concentration can be calculated with elimination of background interference from such substances as bilirubin and hemoglobin. Thus, bichromatic analysis functions as a reference blank for each individual sample.

What is the recommended name for diphenylhydantoin? A. Phenytoin B. Nalorphine C. Primidone D. Carbamazepine

A. Phenytoin is the recommended name for the anticonvulsant diphenylhydantoin. Because of its wide use and toxicity at high concentrations, phenytoin is often the subject of overdose. Thinlayer chromatography or spectrophotometry is used for screening. Quantification usually requires gas- or high-performance liquid chromatography or immunoassay (e.g., EMIT, FPIA).

Which of the following disorders can be classified as a form of prehepatic jaundice? A. Acute hemolytic anemia B. Cirrhosis C. Dubin-Johnson syndrome D. Neoplasm of common bile duct

A. Prehepatic jaundice is also known as hemolytic jaundice, a term that is descriptive of the cause of the disorder. Any disorder that causes the destruction of erythrocytes at a faster rate than the liver is able to conjugate the bilirubin being formed by the reticuloendothelial system will exhibit hyperbilirubinemia. The increased concentration of bilirubin and the ensuing jaundice is not due to any hepatic malfunction but only to the inability of the liver to handle the conjugation of such a bilirubin overload. Therefore, the jaundice is caused by an increased concentration of unconjugated bilirubin. Disorders that follow this type of course are acute hemolytic anemia, chronic hemolytic anemia, and neonatal jaundice. Causes of hemolytic anemia may be genetic or acquired and include hereditary spherocytosis, sickle-cell anemia, and blood transfusion reactions. Neonatal jaundice may be due to an ABO or Rh incompatibility, as seen in erythroblastosis fetalis.

The following laboratory results are obtained on a 60-year-old woman who is complaining of anorexia, constipation, abdominal pain, nausea, and vomiting: Ionized serum calcium—elevated Serum inorganic phosphate— decreased Urine calcium—elevated Urine phosphate—elevated What do these results suggest? A. Primary hyperparathyroidism B. Vitamin D deficiency C. Hypoparathyroidism D. Paget disease

A. Primary hyperparathyroidism is a disorder characterized by increased secretion of PTH into the blood, without the stimulus on the parathyroid gland of a decreased level of ionized calcium. The increase in PTH produces increased blood calcium and vitamin D3 levels, along with a decreased blood phosphorus level. The hypersecretion is most often caused by a single parathyroid adenoma. PTH secretion can usually, but not in all cases, be suppressed by calcium infusion. The decreased blood phosphate level is a result of the action of PTH on the kidneys, which decreases tubular reabsorption of phosphate ions. The increased blood level of 1,25-dihydroxyvitamin D3 is also caused by PTH action on the kidneys in that PTH stimulates increased renal synthesis of this compound.

Because of infertility problems, a physician would like to determine when a woman ovulates. The physician orders serial assays of plasma progesterone. From these assays, how can the physician recognize when ovulation occurs? A. After ovulation, progesterone rapidly increases. B. After ovulation, progesterone rapidly decreases. C. Right before ovulation, progesterone rapidly increases. D. There is a gradual, steady increase in progesterone throughout the menstrual cycle.

A. Progesterone production can be monitored by measuring plasma progesterone or urinary pregnanediol, the major metabolite of progesterone. In the follicular stage of the menstrual cycle, only a small amount of progesterone is secreted. In the luteal stage, or the time from ovulation to menstruation, progesterone levels rapidly increase. Hence, serial assays of plasma progesterone or urinary pregnanediol can be used to identify the time of ovulation. If pregnancy does not occur, progesterone quickly decreases approximately 24 hours before menstruation. If there is no ovulation, then there is no corpus luteum formation and no cyclic rise in progesterone levels.

Using the following data: Na+ = 143 mmol/L; K+ = 4.9 mmol/L; Cl" = 105 mmol/L; and HCO^ = 25 mmol/L, which of the following statements is false? A. Patient results are not acceptable. B. Anion gap is useful in detecting some disease states. C. Anion gap equals 18 mmol/L. D. Anion gap is useful for checking analytical error.

A. The calculation of the anion gap may be used both to assess instrument performance and as a quality assurance tool for electrolyte analyses. The following is one of several equations that may be used to calculate the anion gap: anion gap (mmol/L) = (Na+ + K+) - (CF + HCO^~). The acceptable reference range for this method of calculation is 10-20 mmol/L. If the values of a particular patient fall within this acceptable level, it is presumed that there are no gross problems with the electrolyte measurements. In this case, the anion gap is 18 mmol/L and within the reference range. When using the anion gap it is important to remember that values are affected not only by measurement errors but also by such disease processes as renal failure, ketoacidosis, and salicylate poisoning. Therefore, it is important to differentiate between laboratory errors and true disease states.

To maintain acid-base balance, it is necessary that the blood ammonia level be kept within narrow limits. This is accomplished primarily by which of the following? A. Synthesis of urea from ammonia B. Synthesis of glutamine from ammonia C. Excretion of ammonia in the bile D. Excretion of ammonia in the stools

A. The catabolism of some amino acids involves a transamination reaction in which the a-amino group of the amino acid is enzymatically removed. After its removal, the a-amino group is transferred to an a-keto acid (a-ketoglutarate) with the formation of L-glutamate. Glutamate, which is the common product formed by most transaminase reactions, then may undergo oxidative deamination in the liver mitochondria with the formation of ammonia. The ammonia thus formed leaves the mitochondria as the amino group of citrulline. Citrulline, in turn, condenses with aspartate, which contains the second amino group needed for urea synthesis, forming argininosuccinate, which ultimately leads to the formation of urea. Therefore, the formation of urea and its excretion in the urine provide the principal means by which the body is able to free itself of excess ammonia.

As the red blood cells disintegrate, hemoglobin is released and converted to the pigment bilirubin. Which organ is primarily responsible for this function? A. Spleen B. Kidneys C. Intestines D. Liver

A. The cells of the reticuloendothelial system are responsible for the removal of old red blood cells from the peripheral circulation. As the red blood cells reach the end of their 120-day life span, the specialized cells mainly of the spleen phagocytize the aged cells and convert the released hemoglobin into the excretory pigment bilirubin. The bone marrow is also responsible for the destruction of a small number of red blood cells that have not completed the maturation process. The bilirubin produced by the reticuloendothelial cells is indirect bilirubin, which, as a protein-bound compound, is transported to the liver for conjugation into direct bilirubin.

Which of the following tissues is important in vitamin D metabolism? A. Skin B. Spleen C. Pancreas D. Thyroid

A. The designation "vitamin D" applies to a family of essential fat-soluble sterols that includes vitamin D3 or cholecalciferol. This compound can either be absorbed directly or synthesized in the skin from 7-dehydrocholesterol with the help of ultraviolet irradiation. For physiological functioning, vitamin D3 must be metabolized first by the liver to 25-hydroxyvitamin D3 and then by the kidney to the final hormonal product, 1,25- dihydroxyvitamin D3 (calcitriol). The kidney also synthesizes 24,25-dihydroxyvitamin D3 by an alternate pathway. This compound does not have the hormonal activity of calcitriol, but because of its similar structure and relatively high concentration in the serum, it has complicated the determination of serum calcitriol.

Which of the following defines the term "glycolysis"? A. Conversion of glucose into lactate or pyruvate B. Conversion of glucose to glycogen C. Breakdown of glycogen to form glucose D. Breakdown of lipids to form glucose

A. The level of glucose in the blood is a result of a variety of metabolic processes. Processes that increase the blood glucose include ingestion of sugar, synthesis of glucose from noncarbohydrate sources, and breakdown of glycogen. Processes that decrease blood glucose include metabolizing glucose to produce energy and converting glucose to glycogen or fat. Glycogen is a polysaccharide, which is the storage form of carbohydrates in animals. Glycogenesis refers to the formation of glycogen in the liver from blood glucose. This occurs in response to increased blood glucose levels. In response to decreasing blood glucose levels, glycogen in the liver is broken down to glucose. This process is called glycogenolysis. When glucose is metabolized, for example, to produce energy, it is converted to lactate or pyruvate. This process is called glycolysis. When the body synthesizes glucose from noncarbohydrate sources—that is, amino acids, glycerol, or lactate—the process is called gluconeogenesis. When the body uses glucose to synthesize fat, this process is called lipogenesis.

A 54-year-old male, with a history of type 2 diabetes mellitus for the past 8 years, is seen by his family physician. The patient indicates that during the past week he had experienced what he described as feeling lightheaded and faint. He also indicated that he became out of breath and had experienced mild chest pain when doing heavy yard work, but the chest pain subsided when he sat down and rested. The physician performed an ECG immediately, which was normal, and he ordered blood tests. The patient fasted overnight and had blood drawn the next morning. The laboratory test values follow: Test Patient's Values Reference Ranges Glucose, fasting 175 mg/dL 74-99 mg/dL Hemoglobin Alc 8.1% 4-6% Total cholesterol 272 mg/dL <200mg/dL HDL cholesterol 30mg/dL >40 mg/dL LDL cholesterol 102 mg/dL < 130 mg/dL Triglyceride 250 mg/dL < 150 mg/dL hs-CRP 6.2 mg/L 0.3-8.6 mg/L,< 1.0 mg/L low risk Based on the patient's test results, history, and symptoms, which of the laboratory values in the chart above does not support the patient's diagnosis? A. LDL cholesterol B. HDL cholesterol C. Hemoglobin Aic D. hs-CRP

A. The patient is a known diabetic who has been experiencing chest pain and shortness of breath with activity. The ECG was normal. The most likely diagnosis is angina pectoris. The LDL cholesterol result does not correlate with the other lipid results, and it appears to be less than what would be expected. Using the formula LDL cholesterol = total cholesterol - [HDL cholesterol + triglycerides/5], the calculated LDL cholesterol would be 192 mg/dL. The total cholesterol, HDL cholesterol, and triglyceride results con-elate and indicate hyperlipidemia. The elevated fasting glucose indicates poor carbohydrate metabolism, and the elevated hemoglobin Ajc indicates a lack of glucose control during the previous 2 to 3 months. The elevated glucose and lipid results support an increased risk of coronary artery disease, as does the hs-CRP value, which falls in the high risk range (>3.0 mg/L).

Which of the following will cause a shift of the oxygen dissociation curve to the right, resulting in a decreased affinity of hemoglobin for C"2? A. Low plasma pH level B. Low PCO2 level C. Low concentration of 2,3-bisphosphoglycerate D. Low temperature

A. There is a wide variety of conditions that will cause a shift of the dissociation curve of oxyhemoglobin to the left or to the right. A shift to the left will mean an increase in the affinity of hemoglobin for oxygen. Because of this increased affinity, there is also less oxygen delivered to the tissue for a given percent saturation of hemoglobin. When the curve is shifted to the right, there is a decrease in the affinity of hemoglobin for oxygen. Hence there is increased oxygen delivered to tissues for a given hemoglobin oxygen saturation. Oxyhemoglobin is a stronger acid than deoxyhemoglobin. Both exist in equilibrium in the blood. Increased hydrogen ion concentration shifts the equilibrium toward the deoxygenated form. This shift results in increased oxygen delivery to the tissue. The higher the concentration of 2,3-bisphosphoglycerate in the cell, the greater is the displacement of oxygen, thus facilitating the release of oxygen at the tissue level. Increased /3CO2 and increased temperature will also have this same effect.

What is the predominant form of thyroid hormone in the circulation? A. Thyroxine B. Triiodothyronine C. Diiodotyrosine D. Monoiodotyrosine

A. Thyroglobulin is a glycoprotein in which the thyroid hormones are stored in the thyroid gland. When tyrosine residues of the thyroglobulin are iodinated, monoiodotyrosine (MIT) and diiodotyrosine (DIT) are formed. These iodotyrosine residues are not hormones. Triiodothyronine (T3) and thyroxine (T4) are the hormones produced by the thyroid, being formed by the coupling of either MIT or DIT residues. T4 is the predominant form of the thyroid hormones secreted into the circulation, having a concentration in the plasma significantly greater than T3. However, in terms of physiological activity, T3 must be considered because it is four to five times more potent than T4. Thus the overall contribution of T3 to the total physiological effect of the thyroid hormones on the body is very significant.

In iron-deficiency anemia, what would be the expected percent saturation of transferrin with iron? A. Less than 15 B. Between 30 and 40 C. Between 40 and 50 D. Greater than 55

A. Transferrin is a glycoprotein that reversibly binds serum iron that is not combined with other proteins such as hemoglobin and ferritin. Transferrin concentration in serum is rarely determined directly but, rather, in terms of the serum iron content after saturation with iron. This is the total iron-binding capacity (TIBC). The percent saturation of transferrin is determined by dividing the serum iron level by the serum TIBC and expressing this value as a percentage. Normally in adults the percent saturation of transferrin is in the range of 20-50%, whereas in iron-deficiency anemia, the saturation is expected to be less than 15%. In iron-deficiency anemia complicated by other disorders that either increase serum iron concentration or decrease the TIBC, the percent saturation may remain within the reference

In what disorder would an increased percent saturation of transferrin be expected? A. Hemochromatosis B. Iron-deficiency anemia C. Myocardial infarction D. Malignancy

A. Transferrin is the iron transport protein in serum and is normally saturated with iron to the extent of approximately 20-50%. An increased percent saturation of transferrin is expected in patients with hemochromatosis, an iron overload disease, and iron poisoning. The increased saturation is due to the increased iron concentration in the serum. In patients with chronic infections and malignancies, there is impairment of iron release from body storage sites, leading to a decreased percent saturation of transferrin. In myocardial infarction the serum iron levels are depressed, but the TIBC levels are normal. Irondeficiency anemia because of poor absorption, poor diet, or chronic loss results in decreased serum iron, increased transferrin, and decreased percent saturation of transferrin in most cases.

Which of the following is not associated with vitamin B12? A. Insoluble in water B. Intrinsic factor C. Schilling test D. Pernicious anemia

A. Vitamin BJ2 is a water-soluble vitamin. It is absorbed in the gastrointestinal tract by way of a substance called intrinsic factor. Deficiency of vitamin Bj2 produces a megaloblastic anemia. Anemia caused by a deficiency of vitamin B]2 because of a lack of intrinsic factor (IF) is called pernicious anemia. The Schilling test (with and without IF) is used to diagnose pernicious anemia. It is helpful in distinguishing pernicious anemia from other malabsorption syndromes. A positive Schilling test indicates low absorption of B12 without IF and normal absorption with IF. However, in diseases of the small bowel, low absorption occurs with and without IF.

In serum protein electrophoresis, when a barbital buffer of pH 8.6 is employed, what protein fraction will migrate the fastest toward the anode? A. Albumin B. Alpha!-globulin C. Beta-globulin D. Gamma-globulin

A. When serum proteins are exposed to a buffer solution of pH 8.6, the proteins take on a net negative charge. The negatively charged proteins will migrate toward the anode (+) when exposed to an electrical field. Albumin migrates the fastest toward the anode whereas the gammaglobulins remain close to the point of application and actually move slightly in a cathodic (-) direction because of the effects of endosmosis. The order of migration of the serum proteins, starting at the anode with the fastest-moving fraction, is albumin, alpha)-globulin, alpha2- globulin, beta-globulin, and gamma-globulin.

Which of the following formulas is an expression of the Beer-Lambert law that is routinely applied to spectrophotometric analysis? Cv A. Au X —- = Cu As B. Cu X -f = Au As C. As X -^ = Au D. A = 2 - log %T

A. When the absorbance of a sample in solution varies directly with the concentration of the sample, Beer's law is followed. In turn, when the absorbance increases exponentially with an increase in the light path, the Lambert law is followed. Incorporation of these two laws may be stated as A = abc, where A = absorbance, a = absorptivity of the substance being measured, b = light path in cm, and c = concentration of the measured substance. When the Beer-Lambert law is applied to spectrophotometric analyses of standards and unknown samples that are being measured, the following equation is derived: &u X CS/AS = Cu, where Au = absorbance of unknown, Cu — concentration of unknown, As = absorbance of standard, and Au = absorbance of unknown. This formula is applied to assays that exhibit linear relationships between changes in absorbance with changes in concentration to calculate the concentration of the unknown sample.

What is the sedimentation nomenclature associated with alpha-lipoprotein? A. Very-low-density lipoproteins (VLDLs) B. High-density lipoproteins (HDLs) C. Low-density lipoproteins (LDLs) D. Chylomicrons

B. A double nomenclature exists for the five principal lipoprotein fractions. The nomenclature is such that the various fractions have been named on the basis of both the electrophoretic mobilities and the ultracentrifugal sedimentation rates. The chylomicrons are known as chylomicrons by both methods. The chylomicrons are the least dense fraction, exhibiting a solvent density for isolation of less than 0.95 g/mL, and have the slowest electrophoretic mobility. The HDLs, also known as the alpha-lipoproteins, have the greatest density of 1.063-1.210 g/mL and move the fastest electrophoretically toward the anode. The VLDLs, also known as the prebeta-lipoproteins, move slightly slower electrophoretically than the alpha fraction. The VLDLs have a density of 0.95-1.006 g/mL. The IDLs, intermediate-density lipoproteins, have a density of 1.006-1.019 g/mL and migrate as a broad band between beta- and prebeta-lipoproteins. The LDLs, also known as the beta-lipoproteins, have an electrophoretic mobility that is slightly slower than that of the IDL fraction. The LDLs have an intermediate density of 1.019-1.063 g/mL, which is between the IDLs and the HDLs. To summarize the electrophoretic mobilities, the alpha-lipoprotein fraction migrates the farthest toward the anode from the origin, followed in order of decreasing mobility by the prebeta-lipoprotein, broad band between betaand prebeta-lipoprotein, beta-lipoprotein, and chylomicron fractions. The chylomicrons remain more cathodic near the point of serum application.

An epileptic patient receiving phenytoin develops acute glomerulonephritis. What change, if any, would be expected in the patient's circulating drug level? A. Decrease in free drug B. Increase in free drug C. Increase in protein-bound drug D. No change in circulating drug level

B. Acute glomerulonephritis is characterized by hematuria and albuminuria. The hypoalbuminemia results in less protein-bound drug and an increase in free drug. Thus, more free drug is available in the circulation to enter the tissues. Such a situation may result in severe side effects and even toxic effects. Therefore, to properly regulate drug dosages, it is advisable to measure free drug levels in blood, rather than total drug levels, whenever possible.

An elevated level of which of the following hormones will inhibit pituitary secretion of adrenocorticotropic hormone (ACTH)? A. Aldosterone B. Cortisol C. 17p-Estradiol D. Progesterone

B. Adrenocorticotropic hormone (ACTH) stimulates the adrenal cortex to secrete cortisol and, to a certain extent, aldosterone. However, aldosterone is also regulated by sodium and potassium levels and, more importantly, by the renin-angiotensin system. Cortisol alone has an inhibitory effect or a negative feedback relationship to ACTH secretion by the pituitary. A low level of cortisol stimulates the hypothalamus to secrete corticotropin- releasing hormone (CRH), which in turn stimulates release of ACTH from the pituitary gland and causes the adrenal cortex to secrete more cortisol. Elevated levels of cortisol reverse this process. ACTH secretion is not inhibited by estrogen or progesterone levels.

Which of the following enzymes would not be useful to quantify in the assessment of liver function? A. Alanine aminotransferase B. Creatine kinase C. Alkaline phosphatase D. Gamma-glutamyltransferase

B. Alanine aminotransferase, aspartate aminotransferase, alkaline phosphatase, gammaglutamyltransferase, and lactate dehydrogenase are enzymes for which the serum activities may be assayed to assess liver function. At the cellular level, alkaline phosphatase functions in the membrane border, gamma-glutamyltransferase functions in the cell membrane, and alanine aminotransferase functions both in the cytoplasm and mitochondria. With tissue damage and necrosis, the cells disintegrate and leak their contents into the blood. Because these enzymes are cellular enzymes, any increase in their activity levels in serum is indicative of tissue destruction. It is important to remember that these enzyme levels must be used in conjunction with other clinical data because enzymes generally are not organ specific; they are found in several tissues.

Each lipoprotein fraction is composed of varying amounts of lipid and protein components. The beta-lipoprotein fraction consists primarily of which lipid? A. Fatty acid B. Cholesterol C. Phospholipid D. Triglyceride

B. All the lipoproteins contain some amount of triglyceride, cholesterol, phospholipid, and protein. Each of the lipoprotein fractions is distinguished by its unique concentration of these substances. The beta-lipoprotein fraction is composed of approximately 50% cholesterol, 6% triglycerides, 22% phospholipids, and 22% protein. The beta-lipoproteins, which are also known as the low-density lipoproteins (LDLs), are the principal transport vehicle for cholesterol in the plasma. Both the chylomicrons and the prebeta-lipoproteins are composed primarily of triglycerides. The chylomicrons are considered transport vehicles for exogenous triglycerides. In other words, dietary fat is absorbed through the intestine in the form of chylomicrons. After a meal, the liver will clear the chylomicrons from the blood and use the triglyceride component to form the prebeta-lipoproteins. Therefore, in the fasting state triglycerides are transported in the blood primarily by the prebeta-lipoproteins. The prebeta-lipoproteins are composed of approximately 55% triglycerides.

What is the major carrier protein of the thyroid hormones in the blood? A. Albumin B. Thyroxine-binding globulin C. Thyroxine-binding prealbumin D. Thyroglobulin

B. Almost all the triiodothyronine (T3) and thyroxine (T4) hormones are reversibly bound to the serum proteins, thyroxine-binding globulin (TBG), thyroxine-binding prealbumin (TBPA), and albumin. Most T3 is bound to TBG, whereas 70% of T4 is bound to TBG, 20% to TBPA, and 10% to albumin. T3 has a lower affinity for TBG and TBPA than T4. Thyroglobulin is manufactured and stored in the thyroid follicle and is not released into the circulation.

Which of the following methods utilizes urease and glutamate dehydrogenase for the quantification of serum urea? A. Berthelot B. Coupled enzymatic C. Conductimetric D. Indicator dye

B. An enzymatic method for quantifying urea employs urease and glutamate dehydrogenase (GLDH) in a coupled enzymatic reaction. Urease catalyzes the production of ammonium carbonate from urea. The ammonium ion produced reacts with 2-oxoglutarate and NADH in the presence of GLDH with the formation of NAD+ and glutamate. The decrease in absorbance, as NADH is oxidized to NAD+, is followed kinetically at 340 nm using a spectrophotometer. In the conductimetric method, the formation of ammonium ions and carbonate ions, from the ammonium carbonate, causes a change in conductivity that is related to the amount of urea present in the sample.

The presence of a very high titer for antithyroglobulin antibodies and the detection of antithyroid peroxidase antibodies is highly suggestive of what disorder? A. Pernicious anemia B. Hashimoto thyroiditis C. Multinodular goiter D. Thyroid adenoma

B. Antibodies to thyroglobulin (TgAb) and thyroid cell peroxidase (TPOAb) are produced in several thyroid diseases. Very high antibody titers for antithyroglobulin antibodies and the detection of antithyroid peroxidase antibodies are highly suggestive of Hashimoto thyroiditis (a type of hypothyroidism). These antibodies are also frequently detected in primary myxedema and Graves disease by means of hemagglutination methods. It should be noted that antithyroid antibodies do occur in other thyroid diseases, but their prevalence is less. These antibodies have also been detected in 5-10% of the normal population.

Diabetes insipidus is associated with depressed secretion of which of the following hormones? A. Prolactin B. Antidiuretic hormone C. Growth hormone D. Oxytocin

B. Antidiuretic hormone (ADH), also known as vasopressin, is a peptide hormone secreted by the posterior pituitary gland under the influence of three major stimuli: decreased serum osmolality, increased blood volume, or psychogenic factors. ADH increases the renal reabsoiption of water by increasing the permeability of the collecting ducts, with the result that body water is retained and urine osmolality increases. Diabetes insipidus is the syndrome that results from decreased secretion of ADH from any cause. Serum levels of ADH can be measured, but usually the measurement of serum and urine osmolality is sufficient to indicate the severity of the disease.

Which of the following enzymes catalyzes the transfer of amino groups causing the interconversion of amino acids and a-oxoacids? A. Amylase B. Aspartate transaminase C. Alkaline phosphatase D. Lactate dehydrogenase

B. Aspartate and alanine aminotransferases catalyze the transfer of amino groups between amino acids and a-oxoacids. A prosthetic group, pyridoxal-5'-phosphate (P-5'-P), is required for the transfer of the amino group. In the aspartate aminotransferase (AST) reaction, AST catalyzes the transfer of an amino group from L-aspartate to a-oxoglutarate, with the amino group transfer mediated by P-5'-P, which is bound to the apoenzyme. The products formed are oxaloacetate and L-glutamate. By coupling this reaction with a malate dehydrogenase reaction, the decrease in absorbance of NADH as it is oxidized to NAD+ can be followed at 340 nm. The change in absorbance will be proportional to the AST activity present in the serum specimen.

Which of the following statements best describes "base excess"? A. Primarily refers to carbonic acid concentration B. Positive values reflect metabolic alkalosis. C. Created through metabolism of carbohydrates D. Negative values represent a respiratory imbalance.

B. Base excess is a measure of the nonrespiratory buffers of the blood. They are hemoglobin, serum protein, phosphate, and bicarbonate. Therefore, base excess reflects an abnormality in the buffer base concentration. Bicarbonate has the greatest influence on base excess, which is an indicator of metabolic function. The normal range for base excess is ±2.5 mmol/L. A quick estimation of base excess is to subtract the average "normal" reference bicarbonate level set by the laboratory from the measured bicarbonate level (e.g., if laboratory reference bicarbonate = 25 and patient's bicai'bonate = 30, then base excess = (30 — 25) = + 5; if patient's bicarbonate = 20, then base excess = (20 — 25) = —5). As demonstrated, a positive base excess is associated with metabolic alkalosis, and a negative base excess is associated with metabolic acidosis.

Ketone bodies are formed because of an excessive breakdown of fatty acids. Of the following metabolites, which may be classified as a ketone body? A. Pyruvic acid B. (3-Hydroxybutyric acid C. Lactic acid D. Oxaloacetic acid

B. Beta-hydroxybutyric acid, acetoacetic acid, and acetone are collectively referred to as ketone bodies. They are formed as a result of the process of beta-oxidation in which liver cells degrade fatty acids with a resultant excess accumulation of acetyl-coenzyme A (CoA). The acetyl-CoA is the parent compound from which ketone bodies are synthesized through a series of reactions.

What breakdown product of bilirubin metabolism is produced in the colon from the oxidation of urobilinogen by microorganisms? A. Porphobilinogen B. Urobilin C. Stercobilinogen D. Protoporphyrin

B. Bilirubin that has been secreted through the bile into the small intestine is reduced by anaerobic microorganisms to urobilinogen. One of the possible fates of urobilinogen is its conversion to urobilin. In the colon, a portion of the urobilinogen is oxidized by the action of microorganisms to urobilin, which is excreted in the feces as an orange-brown pigment.

Which total protein method requires copper sulfate, potassium iodide in sodium hydroxide, and potassium sodium tartrate in its reagent system? A. Kjeldahl B. Biuret C. Folin-Ciocalteu D. Ultraviolet absorption

B. Biuret reagent is a combination of copper sulfate, potassium iodide in sodium hydroxide, and potassium sodium tartrate. The copper sulfate is the key to the reaction because it is the cupric ion that complexes with the peptide bonds of protein. To keep the copper in solution until its use, potassium sodium tartrate is employed as a complexing agent, whereas the autoreduction of copper is prevented by potassium iodide.

Which of the following characterizes Crigler-Najjar syndrome? A. Inability to transport bilirubin from the sinusoidal membrane to the microsomal region B. Deficiency of the enzyme system required for conjugation of bilirubin C. Inability to transport bilirubin glucuronides to the bile canaliculi D. Severe liver cell damage accompanied by necrosis

B. Both Crigler-Najjar syndrome and neonatal jaundice, a physiological disorder, are due to a deficiency in the enzyme-conjugating system. With a deficiency in uridine diphosphate glucuronyltransferase, the liver is unable to conjugate bilirubin, and both of these conditions are characterized by increased levels of unconjugated bilirubin. Unlike Crigler-Najjar syndrome, which is a hereditary disorder, neonatal physiological jaundice is a temporary situation that usually corrects itself within a few days after birth.

What is the principle of the "direct" or "homogeneous" HDL cholesterol automated method, which requires no intervention by the laboratorian? The direct HDL method A. Quantifies only the cholesterol in HDL, whereas the precipitation HDL method quantifies the entire lipoprotein B. Utilizes polymers and detergents that make the HDL cholesterol soluble while keeping the other lipoproteins insoluble C. Uses a nonenzymatic method to measure cholesterol, whereas the other methods use enzymes to measure cholesterol D. Uses a column chromatography step to separate HDL from the other lipoproteins, whereas the other methods use a precipitation step

B. Both the direct and the heparin sulfatemanganese chloride precipitation methods measure HDL cholesterol. The direct or homogeneous method for HDL cholesterol uses a mixture of polyanions and polymers that bind to LDL and VLDL and chylomicrons, causing them to become stabilized. The polyanions neutralize ionic charges on the surface of the lipoproteins, and this enhances their binding to the polymer. When a detergent is added, HDL goes into solution, whereas the other lipoproteins remain attached to the polymer/polyanion complexes. The HDL cholesterol then reacts with added cholesterol enzyme reagents while the other lipoproteins remain inactive. The reagents, polymer/polyanions, and detergent can be added to the specimen in an automated way without the need for any manual pretreatment step. Furthermore, the direct HDL cholesterol procedure has the capacity for better precision than the manual precipitation methods. Both the adaptability to automated instalments and the better precision make the direct method a preferred choice for quantifying HDL cholesterol.

Which of the following is an oncofetal antigen that is elevated in nonmucinous epithelial ovarian cancer? A. CA549 B. CA 125 C. CA 19-9 D. CA 15-3

B. CA 125 is an oncofetal antigen, glycoprotein in nature, that is produced by ovarian epithelial cells. The majority of individuals with nonmucinous epithelial ovarian cancer exhibit elevated levels of CA 125. CA 125 is also increased in other malignancies, including endometrial, breast, colon, pancreas, and lung cancers. Several benign disorders also exhibit CA 125 elevated levels. It appears that the primary usefulness of CA 125 is in monitoring the success of therapy in treating ovarian carcinoma.

Which of the following statements is not associated with serum cholinesterase? A. Inhibited by organic insecticides B. Referred to as "true" cholinesterase C. Decreased level causes prolonged apnea after administration of succinyldicholine D. Acts on the substrate propionylthiocholine

B. Cholinesterase is a serum enzyme synthesized by the liver. It is also known as pseudocholinesterase to distinguish it from "true" cholinesterase (acetylcholinesterase) of erythrocytes. Although a number of disease states are associated with abnormal levels of this enzyme, cholinesterase levels are especially important in detecting organic insecticide poisoning of workers in the chemical industry and agriculture. Decreased cholinesterase levels and atypical enzyme forms are associated with prolonged apnea after succinylcholine administration during surgery. Propionylthiocholine is a commonly used substrate for measuring serum cholinesterase activity.

To quantify serum bilirubin levels, it is necessary that bilirubin couples with diazotized sulfanilic acid to form what complex? A. Verdobilirubin B. Azobilirubin C. Azobilirubinogen D. Bilirubin glucuronide

B. Diazo reagent is a mixture of sulfanilic acid, sodium nitrite, and hydrochloric acid. The mixing of sodium nitrite with hydrochloric acid forms nitrous acid, which in turn reacts with sulfanilic acid to form a diazonium salt. This diazotized sulfanilic acid mixture, when mixed with solubilized bilirubin, forms a red azobilirubin complex. The azobilirubin complexes are isomeric structures formed from the splitting of the bilirubin compound in half. Each half then reacts with the diazo reagent to form two isomeric azobilirubin complexes.

Which of the following disorders is not associated with an elevation of serum aj-fetoprotein? A. Testicular germ cell tumors B. Prostatic carcinoma C. Pancreatic carcinoma D. Gastric carcinoma

B. Elevations of serum levels of AFP are found in a number of malignant as well as benign disorders. Although AFP is considered the most specific laboratory test for hepatocellular carcinoma, increased levels are also found in benign liver disease, including viral hepatitis, chronic active hepatitis, and cirrhosis. Other malignant disorders associated with increased levels of AFP include testicular and ovarian germ cell tumors, pancreatic carcinoma, gastric carcinoma, and colonic carcinoma. Thus, AFP is not a tissue-specific tumor marker. AFP is not elevated in prostatic cancer, which is characterized by an elevation in PSA. The use of AFP in conjunction with human chorionic gonadotropin (hCG) is effective in monitoring treatment and identifying recurrence of testicular cancer.

The properties of enzymes are correctly described by which of the following statements? A. Enzymes are stable proteins. B. Enzymes are protein catalysts of biological origin. C. Enzymes affect the rate of a chemical reaction by raising the activation energy needed for the reaction to take place. D. Enzyme activity is not altered by heat denaturation.

B. Enzymes are protein in nature. Like all proteins, they may be denatured with a loss of activity as a result of several factors (e.g., heat, extreme pH, mechanical agitation, strong acids, and organic solvents). Enzymes act as catalysts for the many chemical reactions of the body. Enzymes increase the rate of a specific chemical reaction by lowering the activation energy needed for the reaction to proceed. They do not change the equilibrium constant of the reaction; but rather, enzymes affect the rate at which equilibrium occurs between reactants and products.

In the assay of an enzyme, zero-order kinetics are best described by which of the following statements? A. Enzyme is present in excess; rate of reaction is variable with time and dependent only on the concentration of the enzyme in the system. B. Substrate is present in excess; rate of reaction is constant with time and dependent only on the concentration of enzyme in the system. C. Substrate is present in excess; rate of reaction is constant with enzyme concentration and dependent only on the time in which the reaction is run. D. Enzyme is present in excess; rate of reaction is independent of both time and concentration of the enzyme in the system.

B. Enzymes are proteins that act as catalysts. It is not practical to measure enzyme concentrations in a body fluid specimen, but rather to assay enzymes according to their activity in catalyzing an appropriate reaction; that is, the conversion of substrate to product. An enzyme acts by combining with a specific substrate to form an enzymesubstrate complex, which then breaks down into product plus free enzyme, which is reused. A general form of the reaction is [S]*±[ES]->[P] where [E] = concentration of enzyme, [S] = concentration of substrate, [ES] = concentration of enzyme-substrate complex, and [P] = concentration of product of the reaction. Because the rate of such a reaction is used as a measure of enzyme activity, it is important to consider the effect of substrate concentration on the rate of the reaction. The kinetics of the reaction are initially of the first order (i.e., the rate varies with the concentration of substrate as well as the concentration of enzyme) until there is sufficient substrate present to combine with all enzyme. The reaction rate then becomes zero order (i.e., the rate is independent of concentration of substrate and directly proportional to concentration of enzyme as measured by reaction rate) when substrate is present in excess. Hence it is desirable to use conditions that provide zeroorder kinetics when assaying enzyme activity.

What effect if any would be expected when the secretion of epinephrine is stimulated by physical or emotional stress? A. Decreased blood glucose level B. Increased blood glucose level C. Increased glycogen storage D. No effect on blood glucose or glycogen levels

B. Epinephrine is produced by the adrenal medulla. It promotes glycogenolysis, thus increasing the blood glucose level. Epinephrine also inhibits the secretion of insulin and stimulates the secretion of glucagon.

The presence of only slightly visible hemolysis will significantly increase the serum level of which of the following electrolytes? A. Sodium B. Potassium C. Chloride D. Bicarbonate

B. Hemolysis of blood specimens because of physiological factors is often difficult to differentiate from hemolysis produced by the blood collection itself. In either case, the concentration of potassium will be increased in the serum because of the release of the very high level of intracellular potassium from the erythrocytes into the plasma. When hemolysis is present, the serum concentrations of sodium, bicarbonate, chloride, and calcium will be decreased because their concentrations are lower in erythrocytes than in plasma.

A 24-year-old drug abuser is brought into the emergency department unconscious. He has shallow breaths, looks pale, and is "clammy." Blood gases show the following results: pH = 7.29, PCO2 = 50 mmHg, HCOs = 25 mmol/L .What condition is indicated by these results? A. Metabolic alkalosis, partially compensated B. Respiratory acidosis, uncompensated C. A dual problem of acidosis D. An error in one of the blood gas measurements

B. Here the pH is decreased indicating acidosis. The /3CO2 is increased, which indicates that the problem is respiratory in nature. The HCO^ is unchanged from the reference range, which indicates that there is no compensation; thus the patient has uncompensated respiratory acidosis.

Which of the following chromatography systems may be described as having a stationary phase that is liquid absorbed on particles packed in a column and a liquid moving phase that is pumped through a column? A. Thin-layer B. High-performance liquid C. Ion-exchange D. Gas-liquid

B. High-performance liquid chromatography is also called high-pressure liquid chromatography. It is a form of column chromatography in which a liquid moving phase is actively pumped through the column, thus speeding the separation process considerably. HPLC is used in therapeutic drug monitoring and in assaying vitamin and hormone concentrations.

The measurement of CO2 in blood by means of a PCO2 electrode is dependent on the A. Passage of H+ ions through the membrane that separates the sample and the electrode B. Change in pH because of increased carbonic acid in the electrolyte surrounding the electrodes C. Movement of bicarbonate across the membrane that separates the sample and the electrode D. Linear relationship between PCO2 in the sample and measured pH

B. In a blood gas analyzer, the PCO2 electrode is actually a pH electrode immersed in a bicarbonate solution. The bicarbonate solution is separated from the sample by a membrane that is permeable to gaseous CC>2 but not to ionized substances such as H"1" ions. When CC>2 from the sample diffuses across the membrane, it dissolves, forming carbonic acid and thus lowering the pH. The pH is inversely proportional to the log of the PCO2. Hence the scale of the meter can be calibrated directly in terms of PCC^. It should be noted that whereas pH refers to the negative logarithm of the H+ ion concentration, PCO2 refers to the partial pressure of CO2.

Which of the following is false about myoglobin as it relates to acute myocardial infarction (AMI)? A. Measure serially B. Cardiac specific C. Initial increase occurs in 1-3 hours D. Doubling of initial value within 1-2 hours suggestive of AMI 229. Which of the following disorders is not associated with an elevation of serum creatine kinase? A. Cerebrovascular accidents B. Hypothyroidism C. Bone disease D. Intramuscular injection 230. Which of the following statements concerning creatine kinase is false? A. Rises within 4-6 hours after acute myocardial infarction B. Catalyzes the phosphorylation of creatine by ATP C. Requires Ca2+ for activity D. Found mainly in skeletal and cardiac muscles and in brain tissue 231. Which enzyme is measured by methodologies that use small oligosaccharides and 4-nitrophenyl-glycoside for substrates? A. Lipase B. Amylase C. Creatine kinase D. Cholinesterase 232. Which statement concerning gammaglutamyltransferase is false? A. Present in almost all cells of the body B. Elevated in liver and some pancreatic diseases C. Elevated in chronic alcoholism D. Elevated in bone disease 233. Which of the following statements correctly describes alkaline phosphatase? A. Decreased in Paget disease B. Decreased in third trimester of a normal pregnancy C. Increased in obstructive jaundice D. Primarily found in cardiac muscle 234. Which of the following enzymes would not be useful to quantify in the assessment of liver function? A. Alanine aminotransferase B. Creatine kinase C. Alkaline phosphatase D. Gamma-glutamyltransferase

B. In acute myocardial infarction (AMI), the initial increase in serum myoglobin levels occurs in 1 to 3 hours following onset of symptoms. Serial measurements need to be made because a single value is not diagnostic. When doubling of the initial value occurs within 1 to 2 hours, this is suggestive of AMI. In AMI, the myoglobin level will peak within 5 to 12 hours, with serum levels returning to normal within 18 to 30 hours. Because myoglobin is found in other tissues and is not cardiac specific, it is usually used in conjunction with cardiac troponin and CK-MB to assess the occurrence of AMI.

Which of the following characterizes hepatic dysfunction in the early stage of viral hepatitis? A. Elevation in urobilinogen and urobilin excretion in the feces B. Elevation in the serum unconjugated bilirubin fraction C. Depression in the serum conjugated bilirubin fraction D. Depression in urinary urobilinogen excretion

B. In disorders such as viral hepatitis, toxic hepatitis, and cirrhosis, hepatocellular damage occurs. The damaged parenchymal cells lose their ability either to conjugate bilirubin or to transport the bilirubin that is conjugated into the bile. Because of loss of conjugating ability by some parenchymal cells, the early stage of viral hepatitis is characterized by an increase in the unconjugated bilirubin fraction in the blood. An increase of lesser magnitude in the conjugated fraction is also demonstrated. The increase in conjugated bilirubin is due to the fact that some cells are able to conjugate but are damaged in such a way that there is leakage of conjugated bilirubin into the sinusoids and the general circulation. Because of this increase in the conjugated fraction, urinary bilirubin excretion is positive. Because the amount of conjugated bilirubin reaching the intestines is less than normal, it follows that the fecal urobilinogen and urobilin levels will also be less than normal. However, the urinary urobilinogen levels will be greater than normal because the urobilinogen that does reach the enterohepatic circulation is not efficiently removed by the liver but, rather, is excreted by the urinary system.

Mercury covered by a layer of mercurous chloride in contact with saturated potassium chloride solution is a description of which of the following types of electrodes? A. Sodium B. Calomel C. Calcium D. Silver/silver chloride

B. In practical applications of potentiometry, it is desirable to use one half-cell with a known and constant potential that is not sensitive to the composition of the material to be analyzed. This is called the reference electrode. One type of reference electrode is the calomel electrode, which consists of mercury covered by a layer of mercurous chloride in contact with a saturated solution of potassium chloride. The other half-cell, called the indicator electrode, is selected on the basis of the change in its potential with change in the concentration of the analyte of interest. The silver-silver chloride electrode is a commonly used type of reference electrode. The sodium and calcium electrodes are types of ionselective electrodes.

Which of the following methods employs a reaction where bilirubin is oxidized to colorless biliverdin? A. Bilirubinometer B. Bilirubin oxidase C. High-performance liquid chromatography D. Jendrassik-Grof

B. In the bilirubin oxidase method, the enzyme bilirubin oxidase catalyzes the oxidation of bilirubin to the product biliverdin, which is colorless. This is seen as a decrease in absorbance and is monitored between 405 and 460 nm. This method has an advantage over diazo methods in that hemoglobin does not interfere in the assay and cause falsely low results.

In acute diabetic ketoacidosis, which of the following laboratory findings would be expected? A. Fasting blood glucose elevated, pH elevated, ketone bodies present B. Fasting blood glucose elevated, pH low, ketone bodies present C. Fasting blood glucose elevated, pH normal, ketone bodies absent D. Fasting blood glucose decreased, pH low, ketone bodies absent

B. In the diabetic patient, diabetic ketoacidosis is one of the complications that may require emergency therapy. Blood glucose levels are usually in the range of 500-700 mg/dL but may be higher. The result is severe glycosuria that produces an osmotic diuresis, leading to loss of water and depletion of body electrolytes. Lipolysis is accelerated as a result of insulin deficiency. The free fatty acids produced are metabolized to acetyl-coenzyme A units, which are converted in the liver to ketone bodies. Hydrogen ions are produced with ketone bodies (other than acetone), contributing to a decrease in blood pH. Ketoacids are also excreted in the urine, causing a decrease in urinary pH.

In the diacetyl method, what does diacetyl react with to form a yellow product? A. Ammonia B. Urea C. Uric acid D. Nitrogen

B. In the diacetyl method, acidic diacetyl reacts directly with urea to form a yellow-diazine derivative. Thiosemicarbazide and ferric ions are reagents used to intensify the color of the reaction. Because urea is quantified directly, the method does not suffer from interferences fromammonia contamination, as do some of the urea methods.

As a reduction product of bilirubin catabolism, this compound is partially reabsorbed from the intestine through the portal circulation for reexcretion by the liver. What is this compound? A. Verdohemoglobin B. Urobilinogen C. Urobilin D. Biliverdin

B. In the small intestine, urobilinogen is formed through the enzymatic reduction process of anaerobic bacteria on bilirubin. The fate of urobilinogen is such that some of the urobilinogen will be excreted unchanged in the stool, a portion will be oxidized to urobilin for excretion in the stool, and up to 20% will be absorbed from the intestine into the portal circulation. This circulating urobilinogen is almost completely picked up by the liver, with only a small amount excreted in the urine. The liver oxidizes a small part of the recycled urobilinogen to bilirubin. This newly formed bilirubin and any unchanged urobilinogen are transported through the bile canaliculi into the bile for reexcretion by the intestines. This recycling of urobilinogen is part of the enterohepatic circulation.

Which of the following is not associated with isoelectric focusing? A. Continuous pH gradient B. Migration of proteins with net charge of zero C. Separation dependent on isoelectric point D. Zone electrophoresis

B. Isoelectric focusing is a type of zone electrophoresis. It requires the establishment of a pH gradient, within the agarose or polyacrylamide gel medium, to obtain the separation of charged proteins. Under constant power, the proteins migrate to the pH that corresponds to the isoelectric point of the particular protein.

At what level should a 52-year-old male diagnosed with type 2 diabetes mellitus maintain his hemoglobin A]c? A. <3% B. <7% C. <9% D. <11%

B. It is currently recommended by the ADA that hemoglobin Al c should be lowered to an average of approximately 7% in individuals with diabetes mellitus. When hemoglobin Alc is reduced to this level or less, there is a reduction in microvascular and neuropathic complications of diabetes and to some degree macrovascular disease. Therefore, the ADA recommends that nonpregnant adults be maintained at a hemoglobin A]c level of <7%. There is some discussion that 6% would be better. Hemoglobin AIC is the major component of the glycated hemoglobins. Quantification of hemoglobin Alc may be performed using highperformance liquid chromatography, ionexchange chromatography (manual), isoelectric focusing, and immunoassay techniques.

What is the specimen of choice for analysis of acid-base disturbances involving pulmonary dysfunction in an adult? A. Venous blood B. Arterial blood C. Capillary blood D. Urine

B. It is possible to use arterial, venous, or capillary blood for blood gas analysis. The specimen of choice for determining pulmonary dysfunction in adults is arterial blood. Analysis of arterial blood is the best indicator of pulmonary function, the capacity of the lungs to exchange carbon dioxide for oxygen. PO? and PCO2 measurements from capillary blood are usually confined to infant sampling, and they are dependent on the patient preparation and sampling site. Venous blood should not be used for blood gas studies involving pulmonary problems because venous blood gas values also reflect metabolic processes. Furthermore, the reference range for PO2 in venous blood varies drastically from arterial blood. Urine cannot be used to determine the acid/base status of a patient.

Free erythrocyte protoporphyrin (FEP) levels are useful as a screening method for exposure to which of the following metals? A. Zinc B. Lead C. Iron D. Mercury

B. Lead interferes in heme biosynthesis at several stages, the last of these being the incorporation of iron into the tetrapyrrole ring. This alteration in biosynthesis results in the formation and accumulation of zinc protoporphyrin (ZPP), with zinc replacing the iron in the tetrapyrrole ring. Free erythrocyte protoporphyrin is the extraction product of the zinc metabolite and is a sensitive screening method for determining lead exposure above 25 (xg/dL. The test is not as specific as accurate determination of lead content, however, because iron-deficiency anemia and erythropoietic protoporphyria give false-positive results. Caution must be exercised in monitoring children under 6 years of age, because the Centers for Disease Control and Prevention has defined the acceptable blood level for lead to be less than 10 |xg/dL in young children. At this level, ZPP and erythrocyte protoporphyrin assays are not sufficiently sensitive.

When performing parathyroid surgery for adenoma resection, parathyroid hormone is quantified at three points relative to the surgical procedure: baseline prior to incision, second baseline with gland exposure, and third sample at postexcision. Which of the following is not correct in assessing the PTH values? A. The second baseline value should be higher than the first baseline. B. The first baseline value should be the highest value of the three samples. C. The post-excision value should be at least 50% of or lower than the second baseline. D. The lack of decrease in the PTH value post-excision indicates possible multigland disease.

B. Measurement of PTH during surgery for adenoma resection of the parathyroid glands assists the surgeon in determining completeness of the resection based on the rapid fall of PTH. At least three samples are needed: first, a pre-incision baseline sample as surgery starts; a second baseline sample following exposure of the gland because PTH will increase with any manipulation of the tissue; and a post-excision sample drawn 10 minutes following gland removal (some surgical protocols may require multiple sampling at 5 minutes, 10 minutes, and 20 minutes postexcision). In general, at 10 minutes post-excision, the PTH level should fall to 50% or less of the pre-incision value or the value at the time of gland resection. If the PTH value remains increased and such a decrease does not occur or if the PTH rises again after what initially appeared to be a decrease, multigland disease or ectopic production need to be investigated.

Based on the following graph of velocity of an enzyme reaction versus substrate concentration, you are designing a new method to measure the activity of an enzyme of clinical interest. To formulate the new methodology so that enzyme activity is assessed using zero-order kinetics, which concentration of substrate should you initially determine experimentally? A. Substrate concentration a B. Substrate concentration b C. Substrate concentration c D. Substrate concentration d

B. Michaelis and Menten proposed a basis for the theory of enzyme-substrate complexes and rate reactions. By measuring the velocity of the reaction at varying substrate concentrations, it is possible to determine the Michaelis constant (^m) for any specific enzymatic reaction. Km represents the specific concentration of substrate that is required for a particular reaction to proceed at a velocity that is equal to half of its maximum velocity. The A^m value tells something about the affinity of an enzyme for its substrate. When [S] = Km, the velocity of the reaction is expressed as V = 1/2 Vmax. In the graph shown with this question, the Km of the reaction is represented by b. Because substrate must be present in excess to obtain zero-order kinetics, the substrate concentration necessary would have to be at least 10 times the Km, which is represented by d. Usually substrate concentrations 20-100 times the Km are used to be sure that substrate is present in excess. Thus it is critical that the ^m value be determined experimentally.

Nephelometry is based on the measurement of light that is A. Absorbed by particles in suspension B. Scattered by particles in suspension C. Produced by fluorescence D. Produced by excitation of ground-state atoms

B. Nephelometry is the measurement of the amount of light scattered by particles in suspension. The amount of light scattered depends on the size and shape of the particles and on the wavelength of the incident light. Ultraviolet light should not be used because it might produce some fluorescence, which would lead to erroneously high results.

A patient's total cholesterol is 300 mg/dL, his HDL cholesterol is 50 mg/dL, and his triglyceride is 200 mg/dL. What is this patient's calculated LDL cholesterol? A. 200 B. 210 C. 290 D. 350

B. Once the total cholesterol, triglyceride, and HDL cholesterol are known, LDL cholesterol can be quantified by using the Friedewald equation LDL cholesterol = Total cholesterol — (HDL cholesterol + Triglyceride/5) In this example, all results are in mg/dL: LDL cholesterol = 300 - (50 + 200/5) = 300 - (90) = 210 mg/dL This estimation of LDL cholesterol has been widely accepted in routine clinical laboratories and can be easily programmed into laboratory computers. In addition, LDL methods are available for direct measurement of serum levels. Note: The equation should not be used with triglyceride values exceeding 400 mg/dL because the VLDL composition is abnormal, making the [triglyceride/5] factor inapplicable.

Several malabsorption problems are characterized by a condition known as steatorrhea. Steatorrhea is caused by an abnormal accumulation of what substance in the feces? A. Proteins B. Lipids C. Carbohydrates D. Vitamins

B. Pancreatic insufficiency, Whipple disease, cystic fibrosis, and tropical sprue are diseases characterized by the malabsorption of lipids from the intestines. This malabsorption results in an excess lipid accumulation in the feces that is known as steatorrhea. When steatorrhea is suspected, the amount of lipid material present in the feces may be quantified. A 24- or 72-hour fecal specimen should be collected, the latter being the specimen of choice. The lipids are extracted from the fecal specimen and analyzed by gravimetric or titrimetric methods.

The following laboratory results are determined on a patient with a suggested diagnosis of biliary obstruction: Serum total bilirubin—increased Serum conjugated bilirubin—normal Urine bilirubin—increased Fecal urobilin—decreased Which laboratory result is the least consistent with such a diagnosis? A. Serum total bilirubin B. Serum conjugated bilirubin C. Urine bilirubin D. Fecal urobilin

B. Posthepatic jaundice is caused by an obstruction in the common bile duct, extrahepatic ducts, or the ampulla of Vater. Such an obstruction may be caused by gallstones, neoplasms, or strictures. In this type of jaundice, the liver is functioning properly in its conjugation of bilirubin, but the obstruction causes a blockage so that the conjugated bilirubin is unable to be excreted through the intestines. Therefore, there is a backup of bile into the sinusoids and an overflow into the blood. The circulating blood will characteristically contain excessive amounts of conjugated bilirubin, which will cause increased amounts of bilirubin to be excreted in the urine. Because the blockage prevents proper excretion of bilirubin into the intestines, the formation of urobilinogen and urobilin is impeded. This pattern will continue until the regurgitation of bile causes hepatocellular damage. With destruction of the parenchymal cells, conjugation of bilirubin will be depressed and the blood levels of unconjugated bilirubin will also rise.

Plasma renin activity (PRA) measurements are usually made by measuring which of the following using immunoassay? A. Angiotensinogen B. AngiotensinI C. Angiotensin II D. Angiotensin-converting enzyme

B. Renin is a proteolytic enzyme secreted by the juxtaglomerular cells of the kidneys. In the blood, renin acts on renin substrate (angiotensinogen) to produce angiotensin I. An angiotensin-converting enzyme secreted by endothelial cells then converts angiotensin I to angiotensin II. It is the latter that is responsible for the vasoconstrictive action of renin release. Angiotensin III is a product of aminopeptidase on angiotensin II, and the action of angiotensin II and III is directed at modulating aldosterone secretion. Plasma renin activity, determined by immunoassay, is assessed by quantifying the amount of angiotensin I produced by the action of renin on angiotensinogen using an initial kinetic assay. In addition, renin can be measured directly by an immunometric-mass assay that utilizes a monoclonal antibody.

Which of the following reagents is used to determine the concentration of serum inorganic phosphate? A. Ehrlich's reagent B. Ammonium molybdate C. 8-Hydroxyquinoline D. Bathophenanthroline

B. Serum inorganic phosphate concentrations are determined most commonly by reacting with ammonium molybdate reagent. The molybdenum- phosphate complexes can be quantified at 340 nm. Alternately, treatment of the phosphomolybdate compound formed with a reducing agent leads to the formation of molybdenum blue, which can be measured spectrophotometrically. Use of the anticoagulants EDTA, oxalate, and citrate should be avoided, because they interfere with the formation of phosphomolybdate.

A patient's serum inorganic phosphate level is found to be elevated but the physician cannot determine a physiological basis for this abnormal result. What could possibly have caused an erroneous result to be reported? A. Patient not fasting when blood was drawn B. Specimen was hemolyzed C. Effect of diurnal variation D. Patient receiving intravenous glucose therapy

B. Similarly to potassium, which is a major intracellular cation, phosphate is a major intracellular anion. Therefore, when blood is drawn for serum inorganic phosphate measurement, hemolysis of the specimen must be avoided. Also, serum should be removed from the clot as soon after collection as possible to avoid leakage of phosphate into the serum. Both of these situations would contribute to falsely increased serum phosphate levels. Conversely, serum phosphate levels will be depressed following meals, during the menstrual period, and during intravenous glucose and fructose therapy.

Which of the following is the most common cause of the adrenogenital syndrome called congenital adrenal hyperplasia, and which test is used for its diagnosis? A. 17a-Hydroxylase deficiency; progesterone assay B. 21 -Hydroxylase deficiency; 17ct-hydroxyprogesterone assay C. 3[3-Hydroxysteroid dehydrogenaseisomerase deficiency; 17a-hydroxypregnenolone assay D. 11 (3-Hydroxylase deficiency; 11-deoxycortisol assay

B. The adrenogenital syndrome, congenital adrenal hyperplasia, is due to a deficiency in specific enzymes needed for the synthesis of cortisol and aldosterone. Because cortisol production is blocked, the pituitary increases its secretion of adrenocorticotropic hormone (ACTH), causing adrenal hyperplasia and hypersecretion of cortisol precursors. There are eight recognized types of inherited enzyme defects in cortisol biosynthesis. The most common type of defect is the lack of 21-hydroxylase, occurring in 95% of the cases. Conversion of 17a-hydroxyprogesterone to 11- deoxycortisol is impaired, causing accumulation of 17a-hydroxyprogesterone, which is metabolized to pregnanetriol. An increased plasma 17a-hydroxyprogesterone level is diagnostic and can be determined by radioimmunoassay. Determinations of serum testosterone and urinary pregnanetriol elevations are also diagnostic of this disorder. Virilization takes place in this syndrome because cortisol precursors are shunted to produce weak androgens [e.g., dehydroepiandrosterone (DHEA) and androstenedione]. These androgens are converted peripherally to testosterone in large-enough amounts to create this condition. The second most common defect is 11 p-hydroxylase deficiency with an accumulation of 11-deoxycortisol. 3(3-Hydroxysteroid dehydrogenase- isomerase deficiency and C-17,20-lyase/ 17a-hydroxylase deficiency are examples of other enzyme defects seen in this disorder. A testicular or adrenal tumor may cause symptoms similar to this syndrome; however, these tumors would be acquired in contrast to congenital disorders.

A 4-year-old male child is brought to the pediatrician because the parents are concerned about the child's frequent falling, which results in bruising. The parents indicate that the child has difficulty running, walking, standing up, climbing stairs, and even sitting up straight. The child also appears somewhat weak. Which of the following results is not consistent with the most likely diagnosis? A. Moderately elevated AST B. Moderately elevated ALP C. Moderately elevated LD

B. The child's symptoms are consistent with Duchenne dystrophy, which is an X-linked recessive disorder. It is characterized by muscle weakness, which is caused by destruction of muscle fibers. Symptoms are seen in male children starting at 3 to 7 years of age. The most notable enzyme increase is in creatine kinase, which may increase 50-100 times the reference range. Aspartate transaminase and lactate dehydrogenase would also be increased, because both enzymes are present in skeletal muscle tissue. Alkaline phosphatase is not present in skeletal muscle tissue and is measured to assess hepatobiliary and bone disorders.

transfusion reactions

a decrease in haptoglobin may be an indication of _____________

noncompetitive inhibitor

binds to a site other than the active site; inhibits the enzyme by changing its conformation

uncompetitive inhibitor

binds to the enzyme-substrate complex, preventing the complex from releasing products

live, kidneys

both are forms of Vit D are metabolized in the _____ first and then the ______

Calcium (Ca)

combines with phosphate in bones

Coomassie Brilliant Blue

dye used in urine and CSF total protein testing

Cholinesterase

enzyme that breaks down acetylcholine helpful in determining pesticide poisoning

Cirrhosis & viral hepatitis

hepatic jaundice; plasma/serum--> unconjugated= increased conjugated= increased Urine---> bili- 0/ increased urobili= increased

Hyperchloremia

high levels of chloride in the body

spina bifida

increase in AFP in amniotic fluid and maternal serum is a sign of ____________

bone disease

increase in ALP+ normal 5'NT=

gamma globulin

increase in: chronic inflammation, cirrhosis or viral hepatitis, collagen diseases, papaproteins decrease in : congenital or acquired immuno-deficiency

function of Cl

maintains hydration, osmotic pressure, and the normal anion-cation balance

Chloride (Cl)

major anion in extracellular fluid

obstructive jaundice

posthepatic jaundice; plasma/serum--> unconjugated= N conjugated= increased Urine---> bili- increased urobili= decreased

metabolic alkalosis causes

severe vomiting, excessive GI suctioning, diuretics, excessive NaHCO3

kidneys

urea is produced in the liver and eliminated in the urine by the ______

hypoparathyroidism, chronic renal failure, excess vitamin D

what are some disease states associated with increased levels of PO4?

rickets, osteopenia, hypertension, obesity, diabetes, cancers, multiple myeloma etc.

what are some diseases associated with vitamin D deficiencies.

tired, polyuria, increase thirst & hunger, weight loss, poor healing

what are some symptoms of decreased or no insulin productions in the body?

freezing point depression, vapor pressure depression

what are the 2 colligative properties to measure osmolality?

Albumin, alpha 1, alpha 2, beta, gamma

what are the 5 major bands in migration at a pH of 8.6

Myloglobin, Ck-M, troponin

what are the cardiac markers that are used to test for MI

albumin, sulfate, phosphate

what are the major unmeasured anions

K, Ca, Mg

what are the major unmeasured cations

biliary obstruction

what disease state has increased: ALP, 5'NT, GGT, Bili?

hepatitis

what disease state has increased: AST, ALT, GGT, Bili

storage iron depletion

what disease state(s) would yield these results? serum iron: N transferrin saturation: N TIBC: N serum ferritin: decreased

7.35-7.45

what is the range for the pH of blood?

98-106mM/L

what is the reference range for Cl

3.5-5.0mM/L

what is the reference range for K?

135-145mM/L

what is the reference range for Na?

50-170µg/dL

what is the reference range for serum iron in females?

65-175µg/dL

what is the reference range for serum iron in males?

A1c, fructosamine, urinary albumin, C peptide

what test are done to monitor diabetes

to prevent K from shifting to serum

why is it important to separate cell quickly from serum when testing for K

A 68-year-old male in an unconscious state is transported to the emergency department after being involved in a one-car crash, where he drove off the road and hit a tree. Because he was alone at the time and there was no apparent cause for the accident, it is assumed that he blacked out, which caused him to lose control of the car. He was not wearing a seat belt and has a broken leg, multiple contusions, and cuts. Blood samples were drawn upon arrival to the ED and in 3-hour intervals for 12 hours; all control values were within acceptable range. Selected test results follow: Test Initial Values 3 hours 9 hours Ref Range Myoglobin-57 ng/mL-140 ng/mL-281 ng/mL-30-90 ng/mL Total CK-112U/L-1 70 U/L- 390 U/L-15-1 60 U/L CK-MB-3 ng/mL- 6 ng/mL -8 ng/mL-0-5 ng/mL Troponin I-0.10 ng/mL-0.12 ng/mL-0.11 ng/mL-<0.40 ng/mL What do these test results suggest? A. The man had a myocardial infarction, which caused the accident. B. The elevated results are from the skeletal muscle injuries sustained in the car crash. C. The elevated results are a combination of the car crash injuries and a myocardial infarction. D. The elevated total CK and CK-MB results indicate that the man had a stroke.

B. The controls were within acceptable limits, so it is assumed that all test results are accurate. The elevated myoglobin, total CK, and CK-MB with a troponin I that showed no change and remained in the reference range suggest that the elevated results were due to the skeletal muscle injuries sustained in the car crash. Myoglobin is not tissue specific and may be increased in skeletal muscle injuries, muscular dystrophy, and AMI. The same is true for creatine kinase, which is not tissue specific and may be increased in skeletal muscle disorders as well as cardiac muscle disorders. CK-MB, although it is associated with cardiac muscle and the occurrence of AMI, is not tissue specific and will increase with skeletal muscle injury (but to a lesser degree than CK-MM). Cardiac troponin I is tissue specific and not expressed by skeletal muscle; thus it would remain within the reference range in the absence of an AMI. Total CK and CK-MB do not provide information to assess if a stroke has occurred.

Which of the following is not associated with hypoglycemia? A. Neuroglycopenia B. Symptoms occur with plasma glucose level of 60-70 mg/dL C. Decreased hepatic glucose production D. Diagnostic test is 72-hour fast

B. The diagnostic test for hypoglycemia is the 72-hour fast, which requires the analysis of glucose, insulin, C-peptide, and proinsulin at 6- hour intervals. The test should be concluded when plasma glucose levels drop to <45 mg/dL, when hypoglycemic symptoms appear, or after 72 hours have elapsed. In general, hypoglycemic symptoms occur when the plasma glucose level falls below 55 mg/dL. Such symptoms may include headache, confusion, blurred vision, dizziness, and seizures. The term "neuroglycopenia" has been applied to these central nervous system disorders. Although decreased hepatic glucose production and increased glucose utilization may cause hypoglycemia, there are over 100 causes of this disorder.

As a screening test for Gushing syndrome, the physician wishes to see whether a patient exhibits normal diurnal rhythm in his or her cortisol secretion. At what time should the specimens be drawn for plasma cortisol determination? A. 6 A.M., 2 P.M. B. 8 A.M.,4 P.M. C. 12 noon, 6 P.M. D. 12 noon, 12 midnight

B. The hypothalamus, which secretes CRH, is sensitive not only to cortisol levels and stress but also to sleep-wake patterns. Thus plasma ACTH and cortisol levels exhibit diurnal variation or circadian rhythm. Cortisol secretion peaks at the time of awakening between 6 A.M. and 8 A.M. and then declines to the lowest level between early evening and midnight. After midnight the level again begins to increase. Specimens should be taken at 8 A.M. and 8 P.M. The evening cortisol level should be at least 50% lower than the morning result. In 90% of patients with Gushing syndrome there is no diurnal variation. However, absence of the normal drop in the evening cortisol level is not specific for Gushing syndrome. Other conditions, such as ectopic ACTH syndrome, blindness, hypothalamic tumors, obesity, acute alcoholism, and various drugs, alter normal circadian rhythm in cortisol secretion. To confirm Gushing syndrome, a dexamethasone suppression test may be performed.

To maintain a pH of 7.4 in plasma, it is necessary to maintain a A. 10:1 ratio of bicarbonate to carbonic acid B. 20:1 ratio of bicarbonate to carbonic acid C. 1:20 ratio of bicarbonate to carbonic acid D. 20:1 ratio of carbonic acid to bicarbonate

B. The most important buffer pair in the plasma is bicarbonate with carbonic acid. Use of the Henderson-Hasselbalch equation pH = pK' + log [salt] [acid] shows that the pH changes with the ratio of salt to acid—that is, bicarbonate to carbonic acid— because pK' is a constant. For this buffer pair, apparent pK' = 6.1. When the ratio of the concentrations of bicarbonate to carbonic acid is 20:1 (log of 20 = 1.3), the pH is 7.4; that is, pH = 6.1 + log 20 7.4 = 6.1 + 1.3 The carbonic acid designation represents both the undissociated carbonic acid and the physically dissolved carbon dioxide found in the blood. Because the concentration of the undissociated carbonic acid is negligible compared to the concentration of physically dissolved carbon dioxide, the expression for carbonic acid concentration is usually written (PCO2 x 0.03).

The physician determined that the patient needed an oral glucose tolerance test (OGTT) to assist in diagnosis. The patient had blood drawn for the OGTT, and the following serum glucose results were obtained. These results are indicative of what state? Fasting serum glucose 124 mg/dL 2-hour postload serum glucose 227 mg/dL A. Normal B. Diabetes mellitus C. Addison disease D. Hyperinsulinism

B. The patient presents as having diabetes mellitus. The American Diabetes Association (ADA) published updated standards in 2007 for the classification and diagnosis of diabetes mellitus. Three criteria have been defined, with only one needing to be present to establish the diagnosis of diabetes rnellitus. The three criteria include classic diabetic symptoms and a casual plasma glucose of >200 mg/dL, a fasting plasma glucose of S:126 mg/dL, and a 2-hour postload plasma glucose (part of OGTT) of >200 mg/dL. It is recommended that any positive test be repeated on a subsequent day, if possible, to confirm the diagnosis. It should be noted that the OGTT is not recommended for routine clinical use and would be used only in special circumstances.

Once synthesized, the thyroid hormones are stored as a component of thyroglobulin in what area of the thyroid gland? A. Epithelial cell wall of the follicle B. Colloid in the follicle C. Isthmus of the thyroid gland D. Extracellular space of the thyroid gland

B. The thyroid gland is composed of two lobes connected by a structure called the isthmus. The lobes consist of many follicles. The follicle, in the shape of a sphere, is lined with a single layer of epithelial cells. The epithelial cells produce T3 and T4, which are stored as a component of the thyroglobulin. Within the lumen of the follicle is colloid. Thyroglobulin, secreted by the epithelial cells, makes up 90% of the colloid. As the epithelial cells synthesize the thyroid hormones, the hormones are stored in the thyroglobulin molecule. Thyroglobulin is then secreted into the colloid of the follicular lumen. When the thyroid hormones are needed, they are absorbed by the epithelial cells from their storage site, and through proteolysis, the hormones are released from fragments of the thyroglobulin molecule. T3 and T4 are then secreted by the cells into the blood.

Which of the following is a commonly encountered xanthine that could potentially interfere with the determination of theophylline? A. Nicotine B. Caffeine C. Amphetamine D. Procainamide

B. Theophylline, a xanthine with bronchodilator activity, is widely used in the treatment of asthma. Because of its availability and potential toxicity, it can also be subject to accidental overdose. Chromatographic methods are effective in separating theophylline from caffeine and theobromine, which are two commonly occurring and potentially interfering xanthines. However, most clinical thin-layer chromatographic methods are relatively insensitive to the xanthines, and suspected theophylline overdose should be confirmed by HPLC or immunoassay methods.

What is the end product of purine catabolism in humans? A. Urea B. Uric acid C. Allantoin D. Ammonia

B. Through a sequence of enzymatic reactions, the purine nucleosides, adenosine and guanosine, are catabolized to the waste product uric acid. The catabolism of purines occurs primarily in the liver, with the majority of uric acid being excreted as a urinary waste product. The remaining amount of uric acid is excreted in the biliary, pancreatic, and gastrointestinal secretions through the gastrointestinal tract. In the large intestine, uric acid is further degraded by bacteria and excreted in the stool.

Increased trough levels of aminoglycosides in the serum are often associated with toxic effects to which organ? A. Heart B. Kidney C. Pancreas D. Liver

B. Tobramycin and gentamicin are examples of aminoglycoside antibiotics. Their use has been associated with both nephrotoxicity and ototoxicity. Drug concentration monitoring of patients taking the aminoglycosides requires an analytic system with good precision and accuracy over a wide range because both peak and trough levels are usually monitored. The trough level is used mainly as a measure of nephrotoxicity, whereas the peak level is useful in determining whether adequate therapy is being given to eliminate the causative organism.

In the ultraviolet procedure for quantifying uric acid, what does the reaction between uric acid and uricase cause? A. Production of reduced nicotinamideadenine dinucleotide (NADH) B. The formation of allantoin C. An increase in absorbance D. A reduction of phosphotungstic acid

B. Uric acid absorbs light in the ultraviolet region of 290-293 nm. When uricase is added to a uric acid mixture, uricase destroys uric acid by catalyzing its degradation to allantoin and carbon dioxide. On the basis of these two characteristics, differential spectrophotometry has been applied to the quantification of uric acid. This type of method is used on analyzers that are capable of monitoring the decrease in absorbance as uric acid is destroyed by uricase. The decrease in absorbance is proportional to the concentration of uric acid in the specimen.

Which of the following techniques can be used to quantify apolipoproteins? A. Spectrophotometric endpoint B. Ion-selective electrode C. Immunonephelometric assay D. Refractometry

C. A number of immunochemical assays can be used to quantify the apolipoproteins. Some of the techniques that can be used include immunonephelometric assay, enzyme-linked immunosorbent assay (ELISA), and immunoturbidimetric assay. Commercial kits are available for the quantification of Apo A-I and Apo B-100. Measuring the apolipoproteins can be of use in assessing increased risk for coronary heart disease.

Which of the following isolates light within a narrow region of the spectrum? A. Photomultiplier tube B. Monochromator C. Photovoltaic cell D. Detector

C. A photomultiplier tube (PMT) responds to the radiant energy (light) it absorbs by emitting electrons in a proportional amount to the initial light absorbed. These electrons then go through a series of stages where amplification occurs. The cascade effect, as the electrons go through 10 to 15 stages, results in a final current that may be one million times the initial current. The PMT exhibits rapid response time and sensitivity. These qualities also dictate that this type of detector be shielded from stray light and room light to prevent burnout. The rapid response time of a PMT makes it able to monitor interrupted light beams produced by a chopper.

Proteins may become denatured when subjected to mechanical agitation, heat, or extreme chemical treatment. How are proteins affected by denaturation? A. Alteration in primary structure B. Alteration in secondary structure C. Alteration in tertiary structure D. Increase in solubility

C. A variety of external factors, such as mechanical agitation, application of heat, and extreme chemical treatment with acids or salts, may cause the denaturation of proteins. When proteins are denatured, they undergo a change in their tertiary structure. Tertiary structure describes the appearance of the protein in its folded, globular form. When the covalent, hydrogen, or disulfide bonds are broken, the protein loses its shape as its polypeptide chain unfolds. With the loss of this tertiary structure, there is also a loss in some of the characteristic properties of the protein. In general, proteins will become less soluble, and enzymes will lose catalytic activity. Denaturation by use of chemicals has been a useful laboratory tool. The mixing of serum proteins with sulfosalicylic acid or trichloroacetic acid causes the precipitation of both the albumin and globulin fractions. When albumin is placed in water, dilute salt solutions, or moderately concentrated salt solutions, it remains soluble. However, the globulins are insoluble in water but soluble in weak salt solutions. Both the albumins and globulins are insoluble in concentrated salt solutions. Primary structure refers to the joining of the amino acids through peptide bonds to form polypeptide chains. Secondary structure refers to the twisting of more than one polypeptide chain into coils or helices.

A patient presents with Addison disease. Serum sodium and potassium analyses are performed. What would the results reveal? A. Normal sodium, low potassium levels B. Low sodium, low potassium levels C. Low sodium, high potassium levels D. High sodium, low potassium levels

C. Addison disease is characterized by the hyposecretion of the adrenocortical hormones by the adrenal cortex. Both aldosterone, a mineralocorticoid, and cortisol, a glucocorticoid, are inadequately secreted in this disorder. The decreased secretion of aldosterone will affect body electrolyte balance and extracellular fluid volume. The decrease in sodium reabsorption by the renal tubules will be accompanied by decreased chloride and water retention. This loss of sodium, chloride, and water into the urine will cause the extracellular fluid volume to be decreased. Additionally, the decreased reabsorption of sodium will interfere with the secretion of potassium and hydrogen ions in the renal tubules, causing an increase in the serum potassium ion and hydrogen ion (acidosis) concentrations.

Of the following specimens, which would be appropriate for determining exposure to lead? A. EDTA plasma B. Serum C. Whole blood D. Cerebrospinal fluid

C. After absorption, lead is distributed into an active pool in the blood and soft tissue and a storage pool in bone, teeth, and hair. In blood, the majority is found in erythrocytes, with only minor quantities in plasma or serum. Lead is mainly excreted by the kidney; hence urine or whole blood would be appropriate specimens for determining lead exposure. Provision for leadfree sample containers is a major requirement. Lead analysis can be done accurately by flameless atomic absorption or anodic stripping voltammetry.

The turbid, or milky, appearance of serum after fat ingestion is termed postprandial lipemia, which is caused by the presence of what substance? A. Bilirubin B. Cholesterol C. Chylomicron D. Phospholipid

C. After fat ingestion, lipids are first degraded, then reformed, and finally incorporated by the intestinal mucosal cells into absorbable complexes known as chylomicrons. These chylomicrons enter the blood through the lymphatic system, where they impart a turbid appearance to serum. Such lipemic plasma specimens frequently interfere with absorbance or cause a change in absorbance measurements, leading to invalid results.

Which glucose method is considered to be the reference method? A. Glucose oxidase B. o-Toluidine C. Hexokinase D. Glucose dehydrogenase

C. Although there are several reliable enzymatic glucose methods available, the hexokinase method is the reference method for quantifying glucose. The reference method requires that a protein-free filtrate be made using barium hydroxide and zinc sulfate. The clear supernatant is then used as the sample in the hexokinase/ glucose-6-phosphate dehydrogenase coupled enzyme reactions. For routine clinical use, serum is used directly in the hexokinase method because deproteinization is too time-consuming.

Checking instrument calibration, temperature accuracy, and electronic parameters are part of A. Preventive maintenance B. Quality control C. Function verification D. Precision verification

C. As part of a good quality assurance program, a laboratory should perform function verification, performance verification, and preventive maintenance for all instrument systems. Function verification is the monitoring of specific instrument functions and the correcting of these functions when necessary to assure reliable operation. Function verification includes monitoring temperature, setting electronic parameters, calibrating instruments, and analyzing quality control data. It is important that performance of these activities be properly documented.

Which of the following methods is not used for the quantification of serum bilirubin? A. Bilirubinometer B. Jendrassik and Grof C. Zimmerman D. Bilirubin oxidase

C. Bilirubinometer, bilirubin oxidase, and Jendrassik-Grof are methods that have been used to quantify serum bilirubin concentrations. The bilirubinometer is used for direct spectrophotometric assay in which the bilirubin concentration is read directly at 454 nm. In the bilirubin oxidase method, bilirubin is oxidized to biliverdin and the reaction is followed at 405-460 nm. The Jendrassik-Grof method utilizes a caffeine-sodium benzoate mixture to accelerate the coupling reaction of unconjugated bilirubin with diazo reagent to form an azobilirubin complex. Because of a high recovery rate, the Jendrassik-Grof method is considered to be the method of choice for bilirubin analysis.

Which of the following is an anionic dye that binds selectively with albumin? A. Amido black B. Ponceau S C. Bromcresol green D. Coomassie brilliant blue

C. Bromcresol green (BCG) and bromcresol purple (BCP) are anionic dyes that bind selectively with albumin without preliminary extraction of the globulins. The nature of the dyes is such that the color of the free dye is different from the color of the albumin-dye complex so that the color change is directly proportional to the concentration of albumin in the specimen. Although amido black, Ponceau S, and Coomassie brilliant blue are able to bind albumin, they also react with the globulins, thus prohibiting their use in a direct procedure for quantification of serum albumin.

Which test, if elevated, would provide information about risk for developing coronary artery disease? A. Troponin B. CK-MB C. hs-CRP D. Myoglobin

C. C-reactive protein is an acute-phase reactant that is increased in the presence of inflammation. High-sensitivity C-reactive protein (hs- CRP) refers to a sensitive method that is able to measure low levels of CRP in serum. One theory is that elevated levels of CRP contribute to the damage of arterial walls that precedes plaque formation. hs-CRP is considered a good predictor test for assessing cardiovascular risk. However, it is also elevated in other conditions, including infection, stress, and trauma. CK-MB, troponin, and myoglobin are tests used to assess if a myocardial infarction has occurred.

Which of the following best describes chemiluminescence? A. Electron excitation caused by radiant energy B. Enzymatic oxidation of a substrate produces light emission C. Chemical energy excites electrons that emit light upon return to ground state D. Employs a fluorescent label that produces light

C. Chemiluminescence is a type of luminescence where excitation does not require absorption of radiant energy. Chemiluminescence is the process where the chemical energy of a reaction produces excited atoms, and upon electron return to ground state photons of light are emitted. Chemiluminescence has been applied in the development of immunoassays and has ultrasensitivity in the attomole (10~18) to zeptomole (10~21) ranges.

From what precursor is creatinine formed? A. Urea B. Glucose C. Creatine D. Uric acid

C. Creatine is synthesized from the amino acids arginine, glycine, and methionine. In tissues that include the kidneys, small intestinal mucosa, pancreas, and liver, arginine and glycine form guanidoacetate through a transaminidase reaction. The guanidoacetate is transported in the blood to the liver, where it reacts with S-adenosylmethionine through a transmethylase reaction to form creatine. Creatine is transported in the blood to muscle tissue. Creatine in the form of phosphocreatine is a high-energy storage compound that provides the phosphate needed to produce adenosine triphosphate (ATP) for muscle metabolism. When ATP is formed from phosphocreatine, free creatine is also released. Creatine, through a spontaneous and irreversible reaction, forms creatinine. Creatinine serves no functional metabolic role. It is excreted in the urine as a waste product of creatine.

Which of the following statements concerning creatine kinase is false? A. Rises within 4-6 hours after acute myocardial infarction B. Catalyzes the phosphorylation of creatine by ATP C. Requires Ca2+ for activity D. Found mainly in skeletal and cardiac muscles and in brain tissue

C. Creatine kinase (CK) is found mainly in skeletal muscle, cardiac muscle, and brain tissue. CK catalyzes the following reversible reaction: pH9.0 creatine + adenosine triphosphate (ATP) < > pH 6.7 phosphocreatine+adenosine diphosphate (ADP) Mg2+ is required as an activator. The direction in which the reaction takes place, and hence the equilibrium point, depends on the pH. Measurement of CK activity is valuable in the early diagnosis of acute myocardial infarction. Its level rises 4 to 6 hours after infarction, reaches its peak at 12 to 24 hours, and returns to normal by the third day. In addition to quantifying total CK activity, electrophoresis may be performed to ascertain the presence of an MB band, which represents the heart tissue isoenzyme. Electrophoretically, the MB band moves to an intermediary position between the BB and the MM bands. The BB band travels fastest toward the anode and the MM band travels slowest, remaining in the gamma-globulin region. Electrophoretic separation of CK-MB has been widely replaced by immunologic methods that can be performed on automated instruments.

Why are the total thyroxine (T4) levels increased in pregnant women and women who take oral contraceptives? A. Inappropriate iodine metabolism B. Changes in tissue use C. Changes in concentration of thyroxine-binding globulin (TBG) D. Changes in thyroglobulin synthesis

C. Due to increased protein synthesis, the binding capacity of thyroxine-binding globulin (TBG) is increased in situations such as pregnancy and administration of oral contraceptives. The increased total thyroxine (total T4) levels in these situations do not reflect the functional state of the thyroid gland. It is important when interpreting total T4 levels to take into consideration situations such as these. Free T4 is not affected by variations in thyroxine-binding proteins and better reflects the metabolic state that is euthyroid. However, use of the thyroid hormone binding ratio (THBR), which measures the unoccupied binding sites of TBG, in conjunction with the free and total T4 levels permits a better interpretation of thyroid function. By this process it can be seen where the primary change occurs, whether in the level of T4 or in TBG-binding capacity.

Which of the following reagent systems contains the components sulfanilic acid, hydrochloric acid, and sodium nitrite? A. Jaffe B. Zimmerman C. Diazo D. Lowry

C. Ehrlich's diazo reagent consists of sulfanilic acid, hydrochloric acid, and sodium nitrite. Sulfanilic acid is dissolved in hydrochloric acid and diluted to volume with deionized water. Sodium nitrite is dissolved in deionized water and diluted to volume. Aliquots of these two reagent mixtures are combined to prepare Ehrlich's diazo reagent, which must be prepared fresh before use because of its unstable nature.

Name a commonly used precipitating reagent to separate HDL cholesterol from other lipoprotein cholesterol fractions. A. Zinc sulfate B. Trichloroacetic acid C. Heparin-manganese D. Isopropanol

C. Either a dextran sulfate-magnesium chloride mixture or a heparin sulfate-manganese chloride mixture may be used to precipitate the LDL and VLDL cholesterol fractions. This allows the HDL cholesterol fraction to remain in the supernatant. An aliquot of the supernatant may then be used in a total cholesterol procedure for the quantification of the HDL cholesterol level.

Lactate dehydrogenase (LD) catalyzes the following reaction: Lactate+ NADH+ <=LD => pyruvate + NADH As the reaction is written, which of the following techniques can be used to assess LD activity? A. Measure the colorimetric product pyruvate. B. Measure the colorimetric product NADH. C. Measure the increase in absorbance at 340 nm as NADH is produced. D. Measure the decrease in absorbance at 340 nm as NADH is produced.

C. Enzymes catalyze specific reactions or closely related groups of reactions. Lactate dehydrogenase (LD), with nicotinamide adenine dinucleotide (NAD+) as a hydrogen acceptor, catalyzes the oxidation of L-lactate to pyruvate and the reduction of NAD+ to NADH. Because NAD+ does not absorb light at 340 nm but NADH does, the production of NADH can be monitored as an increase in absorbance at 340 nm and related to the LD activity present in the specimen. Because this reaction is reversible, either the forward or reverse reaction can be used in the laboratory to quantify LD activity. Although the reaction equilibrium favors the formation of lactate from pyruvate, this reaction is less commonly used. It should be noted that the reference ranges for the two reactions are considerably different. Elevation of serum LD is associated with acute myocardial infarction, liver disease, pernicious anemia, malignant disease, and pulmonary embolism. It is also seen in some cases of renal disease, especially where tubular necrosis or pyelonephritis exists.

Which of the following sets of tests would be the most useful in diagnosing an AMI? A. AST, LD, CK-MB B. LD, CK-MB, troponin C. CK-MB, troponin, myoglobin D. LD, troponin, myoglobin

C. For many years, the diagnosis of an AMI was facilitated by assaying serum levels of aspartate aminotransferase (AST), lactate dehydrogenase (LD), creatine kinase (CK), and LD and CK isoenzymes. Today the clinical usefulness of AST and LD has been replaced primarily by cardiac troponin and to a lesser degree by myoglobin, whereas CK isoenzymes continue to play a role. Although myoglobin will increase above the upper reference interval in 1-3 hours following AMI, it is not tissue specific for cardiac muscle and its application has found limited usefulness. Myoglobin will also be increased following skeletal muscle trauma. Troponin I and troponin T have proven to be useful markers, because each has a cardiac-specific isoform, cTnl and cTnT. cTnl appears to be more specific for cardiac muscle, because it has not been identified in regenerating or diseased skeletal muscle, whereas cTnT is made in small amounts by skeletal muscle. Total CK is elevated in AMI and takes 4-6 hours to rise above the upper reference interval. It is the increased level of CK-2 (CKMB) that is more helpful in diagnosing AMI, but caution needs to be exercised here, because skeletal muscle injury can cause a similar increase.

When a pH-sensitive glass electrode is not actively in use, in what type of solution should it be kept? A. Tap water B. Physiologic saline solution C. The medium recommended by the manufacturer D. A buffer solution of alkaline pH

C. For optimum performance, pH-sensitive glass electrodes that are not actively in use should be kept immersed in an aqueous medium. Because the exact composition of the pH-sensitive glass varies from one manufacturer to another, the glass electrode should be maintained in the medium recommended by the manufacturer. Usual media are deionized water, dilute HC1, and buffer with a pH near the pH of the solution to be measured. The functioning of a glass electrode depends on the properties of the pH-sensitive glass. A typical glass electrode is made by sealing a thin piece of pH-sensitive glass at the end of a piece of glass tubing and filling the tube with a solution of hydrochloric acid saturated with silver chloride. A silver wire is immersed in the solution in the tube, with one end extending outside the tube for external connection. This is essentially a silver/silver chloride reference electrode sealed within the tube with the pH-sensitive glass tip. This pH-sensitive glass functions appropriately only when it is saturated with water. Then each surface of the glass develops a hydrated lattice, where exchange of alkaline metal ions in the lattice for hydrogen ions in the test solution can occur.

Measuring the tubular reabsorption of phosphate is useful in diagnosing diseases that affect which of the following organs? A. Liver B. Adrenal gland C. Thyroid gland D. Parathyroid gland

C. Free ionized calcium normally accounts for about 50% of total serum calcium, with the remainder being made up of complexed calcium (about 10%) and calcium bound to proteins (about 40%). The main factors that affect the free ionized calcium fraction are the protein concentration and the pH of the blood. Calcium ions are bound mainly to albumin, but they also bind to globulins. Because the binding is reversible, factors that decrease the protein concentration will increase the free ionized fraction of calcium in the blood. A decrease in blood pH will also increase the fraction of free ionized calcium.

In a patient suspected of having primary myxedema, one would expect the following serum results: free thyroxine (FT4) __________, thyroid hormone binding ratio (THBR) __________, and thyroid-stimulating hormone (TSH) A. Decreased, increased, decreased B. Increased, increased, decreased C. Decreased, decreased, increased D. Increased, decreased, increased

C. Hypothyroidism is a systemic disorder in which the thyroid gland does not secrete sufficient thyroid hormone. Myxedema is commonly used synonymously for hypothyroidism. Hypothyroidism can result from various diseases. If the disease affects the thyroid itself, it is referred to as primary hypothyroidism. If there is TSH deficiency of the pituitary gland, it is termed secondary hypothyroidism. Tertiary hypothyroidism is caused by hypothalamic failure that results in a decreased secretion of thyrotropin-releasing hormone. Thyroid failure in the newborn is termed cretinism. The free T4 level and the thyroid hormone binding ratio (THBR) are decreased because of inadequate secretion of hormones. Since the thyroid hormones are low in concentration, the feedback mechanism to the anterior pituitary gland is triggered to increase production of TSH.

Which type of elution technique may be used in high-performance liquid chromatography? A. Amphoteric B. Isoelectric C. Gradient D. Ion exchange

C. In HPLC, the technique used for the mobile phase may be isocratic or gradient elution. With isocratic elution the strength of the solvent remains constant during the separation. With gradient elution the strength of the solvent is continually increased (percent per minute) during the separation process. The gradient elution technique is sometimes employed to improve HPLC resolution and sensitivity.

What is the primary storage form of iron? A. Apotransferrin B. Myoglobin C. Ferritin D. Hemosiderin

C. In adults the total body iron content averages 3-4 g. The majority of this iron is found in the active pool as an essential constituent of hemoglobin, with a much lesser amount being an integral component of myoglobin and a number of enzymes. Approximately 25% of the body iron is found in inactive storage forms. The major storage form of iron is ferritin, with a lesser amount being stored as hemosiderin. Ferritin may be found in most body cells but especially in reticuloendothelial cells of the liver, spleen, and bone marrow.

When quantifying glucose using an amperometric glucose electrode system, which of the following is not a component of the system? A. Product oxidation produces a current B. Hydrogen peroxide formed C. Hexokinase reacts with glucose D. Platinum electrode

C. In an amperometric glucose electrode system, glucose oxidase reacts with glucose to produce hydrogen peroxide and gluconic acid. The platinum electrode that operates at a positive potential oxidizes the hydrogen peroxide to oxygen. The oxidation of hydrogen peroxide produces a current that is directly proportional to the glucose level in the sample.

Which of the following is not characteristic of multiple myeloma? A. Monoclonal band in the gamma region B. Hypercalcemia C. Hyperalbuminemia D. Hyperglobulinemia

C. In multiple myeloma there is an abnormal proliferation of plasma cells. These plasma cells produce a homogeneous immunoglobulin protein that stains as a well-defined peak in the gamma region. Because of the presence of this monoclonal protein, the serum total protein will be elevated. Bone destruction is commonly seen in this disorder, with the plasma cells forming densely packed groups in the lytic areas. Hypercalcemia is primarily the result of bone destruction.

What enzyme catalyzes the conjugation of bilirubin? A. Leucine aminopeptidase B. Glucose-6-phosphate dehydrogenase C. Uridine diphosphate glucuronyltransferase D. Carbamoyl phosphate synthetase

C. In order for the bilirubin-albumin complex to reach the parenchymal cells of the liver, the complex must be transported from the sinusoids to the sinusoidal microvilli and into the parenchymal cell. The microsomal fraction of the parenchymal cell is responsible for the conjugation of bilirubin. It is here that bilirubin reacts with uridine diphosphate glucuronate in the presence of the enzyme uridine diphosphate glucuronyltransferase to form bilirubin diglucuronide.

Which of the following disorders is best characterized by these laboratory results? Serum iron—decreased Total iron-binding capacity— increased Transferrin saturation—decreased Serum ferritin—decreased Free erythrocyte protoporphyrin— increased A. Anemia of chronic disease B. Thalassemia C. Iron-deficiency anemia D. Hemochromatosis

C. In order to differentiate among diseases, it is necessary to perform several laboratory determinations to properly assess iron metabolism. In iron-deficiency anemia, the serum iron is decreased whereas the TIBC is increased. Thus it follows that the transferrin saturation is decreased. The serum ferritin level, which represents stored body iron, is depressed, and the free erythrocyte protoporphyrin (FEP) level is increased. FEP is not a specific test for iron-deficiency anemia, but it can function as a screening test.

During pregnancy in the second trimester, human chorionic gonadotropin (hCG) levels and progesterone ___________ and estriol levels ________. A. Increase,increase B. Increase, decrease C. Decrease, increase D. Decrease, decrease

C. In pregnant women the level of human chorionic gonadotropin (hCG) is highest during the first trimester, then it stabilizes to a lower level during the rest of the pregnancy. In the first trimester, the level of pregnanediol is slightly higher than that found in nonpregnant women during the luteal phase of the menstrual cycle. As pregnancy progresses, the placenta secretes more progesterone, which peaks midway into the third trimester and then levels off. It should be noted that pregnanediol is a biologically inactive metabolite of progesterone that is sometimes measured in urine. After the second month of pregnancy, estriol levels steadily increase as the placenta takes over estrogen production.

In spectrophotometry, which of the following is a mathematical expression of the relationship between absorbance and transmittance? A. A — abc R Au _ As C~~C ^U ^5 C. A = 2 - log %T D. A = log %T

C. In spectrophotometry, molecules in solution will cause incident light to be absorbed while the remaining light energy will be transmitted. Absorbance is the term used to describe the monochromatic light that is absorbed by the sample, and transmittance describes the light that passes through the sample. The mathematical relationship between absorbance and transmittance is expressed by A = 2 — log %T.

The enzyme activity measured in the EMIT is the result of the reaction between the substrate and coenzyme with A. Free antibody B. Free unlabeled antigen C. Free labeled antigen D. Labeled antigen-antibody complexes

C. In the EMIT assay, antibody specific to the drug being quantified is added to the serum sample that contains the drug. Substrate and coenzyme specific for the enzyme label being used are added. Finally, the enzyme-labeled drug (free labeled antigen) is added to the mixture. The drug in the serum sample and the enzyme-labeled drug compete for the binding sites on the antibody. The binding of the enzyme-labeled drug to the antibody causes a steric alteration that results in decreased enzyme activity. This steric change prevents the substrate from reacting at the active site of the enzyme, leaving only the free enzymelabeled drug able to react with the substrate and coenzyme. The resulting enzyme activity, measured at 340 nm, is directly proportional to the concentration of the drug in the serum sample. The greater the amount of enzyme activity measured, the greater is the concentration of free enzyme-labeled drug and, therefore, the greater is the concentration of drug in the serum sample.

Which of the following may be associated with reflectance spectrophotometry as it relates to the dry reagent slide technique? A. Light projected to the slide at 180-degree angle B. Dye concentration directly proportional to reflectance C. Unabsorbed, reflected light detected by photodetector D. Reflectance values are linearly proportional to transmission values

C. In the dry reagent slide technique, as light from a radiant energy source passes through an interference filter, it is projected to the slide at a 45-degree angle. The light then follows a path through the clear support material and reagent layer and hits a white spreading layer; the unabsorbed light is then reflected back through the reagent and support layers. This reflected light impinges on the photodetector, which is positioned at a 90-degree angle to the slide. Because reflectance values are neither linearly proportional to transmission values nor consequently to dye concentration, the microcomputer utilizes an algorithm as a linearizing transformation of reflectance values to transmission values so that concentration may be calculated.

Enzymatic methods for the determination of total cholesterol in serum utilize a cholesterol oxidase-peroxidase method. In this method, cholesterol oxidase reacts specifically with what? A. Free cholesterol and cholesteryl ester B. Free cholesterol and fatty acid C. Free cholesterol only D. Cholesteryl ester only

C. In the enzymatic method for quantifying total cholesterol in serum, the serum specimen must initially be treated with cholesteryl ester hydrolase. This enzyme hydrolyzes the cholesteryl esters into free cholesterol and fatty acids. Both the free cholesterol, derived from the cholesteryl ester fraction, and any free cholesterol normally present in serum may react in the cholesterol oxidase/peroxidase reactions for total cholesterol. The hydrolysis of the cholesteryl ester fraction is necessary because cholesterol oxidase reacts only with free cholesterol.

Which of the following disorders is not associated with an elevation of serum creatine kinase? A. Cerebrovascular accidents B. Hypothyroidism C. Bone disease D. Intramuscular injection

C. Increased serum creatine kinase (CK), formerly called creatine phosphokinase (CPK), values are caused primarily by lesions of cardiac muscle, skeletal muscle, or brain tissue. CK increases in the early stages of Duchenne-type progressive muscular dystrophy. Assays of total CK and CK isoenzymes are commonly used in the diagnosis of myocardial infarction. Hypothyroidism causes a moderate increase in CK values. Elevation of this enzyme also occurs after vigorous muscular activity, in cases of cerebrovascular accidents (stroke), and after repeated intramuscular injections. In addition to quantifying total CK activity, isoenzymes may be determined by using electrophoretic, immunologic, or ion-exchange chromatography methods. Three isoenzymes have been identified: CK-1 or BB, primarily found in brain and nerve tissues with some in thyroid, kidney, and intestine; CK-2 or MB, primarily found in heart muscle; and CK-3 or MM, primarily found in skeletal muscle but present in all body tissues. CK is not elevated in bone disease.

The measurement of the pressure of dissolved CO2 (PCO2) in the blood is most closely associated with the concentration of what substance? A. pH B. Bicarbonate (HCOs) C. Carbonic acid (H2CO3) D. PO2

C. PCO2 is an indicator of carbonic acid (H2CO3). The PCO2 millimeters of mercury value (mm Hg) multiplied by the constant 0.03 equals the millimoles per liter (mmol/L) concentration of H2CO3 (PCO2 x 0.03 - H2CO3). PCO2 can be measured using a pH/blood gas analyzer.

Which of the following does not need to be done when collecting, handling, and using a specimen for ammonia analysis? A. Avoid using a hemolyzed specimen. B. Collect blood in EDTA or heparin evacuated tubes. C. Place specimen in a 37°C water bath immediately. D. Advise patient not to smoke for 8 hours before blood collection.

C. Plasma is the specimen of choice for ammonia analysis. Ethylenediaminetetra-acetic acid (EDTA) and heparin (not the ammonium salt) are acceptable anticoagulants. Because exposure of blood to air is contraindicated, the evacuated blood collection tube should be filled completely. The blood specimen should be placed on iceimmediately and centrifuged as soon as possible to inhibit deamination of amino acids. Because the concentration of ammonia in red blood cells is approximately three times greater than in plasma, the analysis should be performed on a nonhemolyzed specimen. Because of the false increase in ammonia levels caused by smoking, patients should be instructed to refrain from smoking for 8 hours before blood collection.

Portal cirrhosis is a chronic disease of the liver. As observed on an electrophoretic serum protein pattern, what is a predominant characteristic of this disease? A. Monoclonal band in the gammaglobulin region B. Polyclonal band in the gammaglobulin region C. Bridging effect between the beta- and gamma-globulin bands D. Increase in the alpha2-globulin band

C. Portal cirrhosis is a chronic disease of the liver in which fibrosis occurs as a result of tissue necrosis and diffuse small nodules form as liver cells regenerate, with a concomitant distortion of liver structure. The cause of this disorder may include alcoholism, malnutrition, or submassive hepatic necrosis. When a serum protein electrophoresis is performed, the characteristic pattern seen in portal cirrhosis is an elevation of both the gamma- and beta-globulin regions, with these two regions showing a bridging or fusing appearance. This beta-gamma bridging effect is due to an increased level of IgA, which migrates with beta mobility. It should also be noted that the albumin level is depressed.

Which of the following is an electrophoretic technique employing a pH gradient that separates molecules with similar isoelectric points? A. Zone electrophoresis B. High-resolution electrophoresis C. Isoelectric focusing D. Immunoelectrophoresis

C. Protein molecules can exist as anions, cations, or zwitterions, depending on the pH of the solution in which they are placed. The pH at which they exist in the form of zwitterions and hence have no net charge is called the isoelectric point. The principle of isoelectric focusing is based on the ability to separate proteins because of differences in their isoelectric points. Aliphatic polyamino polycarboxylic acids, known as ampholytes, are used to produce the pH gradient.

Which of the following is not associated with assessment of an AMI? A. Elevated serum cTnl level B. Elevated serum CK-MB level C. Abnormal serum alkaline phosphatase isoenzyme pattern D. Blood collected upon presentation and serially in 3- to 6-hour intervals

C. Quantification of serum total creatine kinase, CK-MB (or CK-2) isoenzyme, and cardiac troponin I (cTnl) or cardiac troponin T (cTnT) is very useful in determining an AMI. Determining the presence and activity level of CK-MB is valuable, because CK-MB levels can increase following an infarct, ranging from 6 to 30% of the total CK. Serial assessment of serum specimens is recommended, with the initial specimen obtained at presentation, followed by blood collection at 3-6 hours, 6-9 hours, and 12-24 hours from the initial time. Because alkaline phosphatase isoenzymes are associated with liver, bone, intestinal, and placental tissues, their analysis would not contribute any significant information to determining the occurrence of an AMI.

Which of the following instruments is used in the clinical laboratory or in reference laboratories to detect beta and gamma emissions? A. Fluorometer B. Nephelometer C. Scintillation counter D. Spectrophotometer

C. Radionuclides are quantified by measuring the amount of energy that they emit. This can be in the form of alpha emission 2He2+, beta emission (electrons ejected from the nucleus of a radioisotope during radioactive decay), or gamma emission (electromagnetic radiation emitted during radioactive decay). Beta and gamma emissions can be detected by scintillation counters. The sensing element of a scintillation counter is a fluor, a substance capable of converting radiation energy to light energy. The light energy is converted to electrical energy and amplified by a photomultiplier tube. A fluor commonly employed in solid scintillation counters is a large crystal of sodium iodide containing a small amount of thallium as an activator; it is used for gamma counting. Beta emission is counted by liquid scintillation counters using fluors dissolved in organic solvents. Alpha emission has very low penetrating power and is not measured in the clinical laboratory. Although radioimmunoassay (RIA) is no longer used for routine analyses and has been replaced by nonradioactive immunoassays, it is still used in a limited manner in some clinical reference laboratories and in research settings.

Of which of the following is 5-hydroxyindoleacetic acid (5-HIAA) the primary metabolite? A. Epinephrine B. Norepinephrine C. Serotonin D. Prolactin

C. Serotonin (5-hydroxytryptamine or 5-HT) is synthesized from tryptophan in a variety of tissues, with the majority found in the argentaffin (enterochromaffin) cells of the intestine. Abdominal carcinoid is a metastasizing tumor of those cells and is associated with excessive production of serotonin. Serotonin in the blood is found almost exclusively in the platelets and is rapidly oxidized in the lungs to 5-hydroxyindoleacetic acid (5-HIAA), its major urinary metabolite. Urinary levels of 5-HIAA may also be increased by eating foods such as bananas and avocados, which are rich in serotonin; by the use of certain drugs such as the phenothiazines; and by carcinoid tumors.

Which of the following is a true statement concerning serum enzymes? A. The presence of hemolyzed red cells is of no significance for an accurate assay of most serum enzymes. B. Serum asparate transaminase (AST), but not serum lactate dehydrogenase (LD), is usually elevated in acute myocardial infarction. C. Increased serum alkaline phosphatase may be found in bone disease. D. Aspartate transaminase was formerly known as glutamate pyruvate transaminase.

C. Serum alkaline phosphatase is elevated in several disorders, including hepatobiliary and bone diseases. For an accurate assay of most serum enzymes, the presence of hemolyzed red blood cells must be avoided because many enzymes are present in red cells. Serum aspartate transaminase (formerly known as glutamate-oxaloacetate transaminase, GOT) and lactate dehydrogenase are both enzymes that are elevated in acute myocardial infarction and liver disease.

Which of the following is not quantified using an immunoassay method? A. Vitamins B. Hormones C. Electrolytes D. Drugs

C. Since the conception of radioimmunoassay (RIA), in the early 1960s, a variety of immunoassay techniques have been developed and applied to measuring a wide variety of substances that are present in the blood in very small concentrations. Categories of ligands for which immunoassay methods have been developed include drugs, hormones, vitamins, tumor markers, and enzymes. Electrolytes are commonly quantified using ion-selective electrodes. Some drugs that are assayed by immunoassay include digoxin, gentamicin, phenobarbital, phenytoin, and theophylline. Immunoassay methods are available for the vitamins B12 and folic acid. Creatine kinase-MB isoenzyme mass analysis uses an immunoassay technique. The list of hormones that are assayed by immunoassay is extensive. Some of these hormones are thyroxine, triiodothyronine, thyroid-stimulating hormone, follicle-stimulating hormone, luteinizing hormone, estradiol, estriol, beta-chorionic gonadotropin, cortisol, prolactin, aldosterone, insulin, gastrin, testosterone, and prostaglandins. The immunoassay methods are generally automated, and enzyme labels and fluorogenic labels are commonly used.

Which of the following drugs is used as an immunosuppressant in organ transplantation, especially in liver transplants? A. Methotrexate B. Amiodarone C. Tacrolimus D. Paroxetine

C. Tacrolimus (Prograf) is an antibiotic that functions as an immunosuppressant in organ transplantation, especially in liver transplants. By inhibiting interleukin production, it blocks lymphocyte proliferation. Adverse reactions to the drug include nephrotoxicity, nausea, vomiting, and headaches. Other immunosuppressant drugs include cyclosporine, mycophenolic acid, and sirolimus. Methotrexate is an antineoplastic drug, amiodarone is an antiarrhythmic drug, and paroxetine is an antidepressant drug.

For what colorimetric determination is the Trinder reaction widely used? A. Acetaminophen B. Propoxyphene C. Salicylate D. Barbiturate

C. The Trinder reaction or modification is used almost routinely in the determination of salicylate and is based on the colorimetric reaction with ferric ions. The availability of rapid quantification in cases of salicylate overdose has been particularly useful because of the necessity of determining the drug's elimination half-life. Most clinically used thin-layer chromatographic methods are insensitive to the presence of salicylate. Because the colorimetric reaction used for determining the presence of phenothiazines with feme perchloricnitric (FPN) reagent is dependent on ferric ions also, false-positive reactions in the ferric ion methods for salicylate may be expected.

The VLDL fraction primarily transports what substance? A. Cholesterol B. Chylomicron C. Triglyceride D. Phospholipid

C. The VLDL fraction is primarily composed of triglycerides and lesser amounts of cholesterol and phospholipids. Protein components of VLDL are mostly apolipoprotein B-100 and apolipoprotein C. VLDL migrates electrophoretically in the prebeta region.

Given the following information, calculate the blood pH. PCO2 = 44 mm Hg Total CO2 = 29 mmol/L A. 6.28 B. 6.76 C. 7.42 D. 7.44

C. The acid-base equilibrium of the blood is expressed by the Henderson-Hasselbalch equation: pH = pK' + log cHCC-3 (PCC-2 X 0.03) For the stated problem, convert PCO2 in mm Hg to dissolved CO2, multiplying by the solubility coefficient of CO2 gas: 44 mm Hg x 0.03 mmol/L/mm Hg = 1.32 mmol/L. Next, determine the bicarbonate concentration by finding the difference between the total CO2 and dissolved CO2 concentrations: 29 mmol/L - 1.32 mmol/L = 27.68 mmol/L. pK' for the bicarbonate buffer system is 6.1. Therefore, 27.68 pH = 6.1 + log t o 1J2 pH = 6.1 + log 20.97 pH = 6.1 + log 21 pH = 6.1 + 1.32 pH = 7.42

The adrenal medulla secretes which of the following in the greatest quantity? A. Metanephrine B. Norepinephrine C. Epinephrine D. Dopamine

C. The adrenal medulla produces 80% epinephrine and 20% norepinephrine (noradrenalin). Metanephrine is a metabolite of epinephrine. Dopamine, a catecholamine, is a precursor of norepinephrine. Norepinephrine is converted to epinephrine by an enzyme, ,/V-methyltransferase, which is present almost exclusively in the adrenal medulla. A tumor of the chromaffin tissue, called a pheochromocytoma, secretes excessive amounts of epinephrine. Ninety percent of pheochromocytomas are in the adrenal medulla. The increased levels of epinephrine from the pheochromocytoma cause hypertension. Although hypertension caused by a pheochromocytoma is rare, a correct diagnosis is very important because pheochromocytoma is one of the few causes of hypertension that is curable by surgery.

What is the function of the flame in atomic absorption spectroscopy? A. Absorb the energy emitted from the metal analyte in returning to ground state B. Supply the thermal energy needed to excite the metal analyte C. Bring the metal analyte to its ground state D. Supply the light that is absorbed by the metal analyte

C. The basis of AAS is the measurement of light, at a specific wavelength, that is absorbed by an element whose atoms are in a ground state. The flame in AAS serves two functions—to accept the sample, thus serving as a cuvet, and to supply heat for converting the element, which is usually present in the sample in molecular form, into its atomic form at ground-state energy level. The hollow-cathode lamp supplies the emission line of light required for the analysis. The metal element of interest is coated on the cathode of the lamp. When the inert gas, either argon or neon, becomes ionized, it is drawn toward the cathode. The impact excites the metal element coated on the cathode, resulting in the emission of spectral lines specific for the element. This light emission is then absorbed by the metal element in the sample. A flameless AAS employs a carbon rod (graphite furnace), tantalum, or platinum to hold the sample in a chamber. The temperature is raised to vaporize the sample being analyzed. The atomized sample then absorbs the light energy from the hollow-cathode lamp. This technique is more sensitive than the flame method. deterioration of the hollow-cathode lamp. AAS may be used to measure such analytes as lead, zinc, copper, aluminum, magnesium, calcium, and lithium.

When is a blood sample for determination of the trough level of a drug appropriately drawn? A. During the absorption phase of the drug B. During the distribution phase of the drug C. Shortly before drug administration D. Two hours after drug administration

C. The collection of blood samples for therapeutic drug monitoring requires both the selection of the proper time for sampling and the recording of that time on the report. It is essential that the drug level be related in time to the time of the previous and/or the next drug administration. Collection of blood samples is generally avoided during the drug's absorption and distribution phases. When peak levels of the drug are required, the blood sample must be drawn at a specified time after drug administration. Trough levels are most reliably determined by collecting the blood sample before the next drug administration.

Which of the following may be associated with the colloid osmotic pressure (COP) osmometer? A. Utilizes a cooling bath set at -7°C B. Measures total serum osmolality C. Negative pressure on reference (saline) side equivalent to COP of sample D. Measures contribution of electrolytes to osmolality

C. The colloid osmotic pressure (COP) osmometer is composed of a semipermeable membrane that separates two chambers, a mercury manometer, a pressure transducer, and a meter. When a serum sample is introduced into the sample chamber, saline solution from the reference chamber moves across the membrane by osmosis. This causes the development of a negative pressure on the saline side that is equivalent to the COP, which represents the amount of protein in the serum sample. COP osmometers measure the serum protein contribution to the total osmolality in terms of millimeters of mercury. COP levels are helpful in monitoring intravenous fluid therapy.

Which of the following glucose methods should not be used during the administration of an oral xylose absorption test? A. Glucose oxidase—colorimetric B. Glucose oxidase—polarographic C. Glucose dehydrogenase D. Hexokinase

C. The glucose dehydrogenase method uses only one enzymatic reaction for the measurement of glucose in a sample. Glucose dehydrogenase catalyzes the oxidation of glucose and the reduction of nicotinamide adenine dinucleotide. The amount of reduced NADH formed is proportional to the glucose concentration in the sample. When measuring blood glucose levels during the administration of an oral xylose tolerance test, the glucose dehydrogenase method should not be used, because the relative rate of reaction of Dxylose as compared to glucose is 15% with this method. In contrast, D-xylose will not react in thehexokinase and glucose oxidase methods, thus allowing glucose to be measured accurately. The D-xylose absorption test is useful in distinguishing two types of malabsorption: intestinal malabsorption and malabsorption resulting from pancreatic insufficiency. When D-xylose is administered orally, it is absorbed by passive diffusion into the portal vein from the proximal portion of the small intestine. Because D-xylose is not metabolized by the liver, it is excreted unchanged by the kidneys. In intestinal malabsorption, the amount of D-xylose excreted, as measured in a 5-hour urine specimen, is less than normal because of decreased absorption of D-xylose. In malabsorption caused by pancreatic insufficiency, the absorption of D-xylose is normal.

Which glucose method catalyzes the phosphorylation of glucose by adenosine triphosphate, forming glucose-6- phosphate and adenosine diphosphate with the absorbance of the NADPH product read at 340 nm? A. o-Toluidine B. Glucose oxidase C. Hexokinase D. Glucose dehydrogenase

C. The hexokinase method for quantifying glucose uses two coupled enzymatic reactions. In the first reaction, which is catalyzed by hexokinase, glucose is phosphorylated by adenosine triphosphate, forming glucose-6-phosphate and adenosine diphosphate. In the second reaction, glucose-6-phosphate dehydrogenase (derived from yeast) catalyzes the oxidation of glucose-6- phosphate and the reduction of nicotinamide adenine dinucleotide phosphate. The amount of reduced NADPH formed is proportional to the glucose concentration in the sample. Thus, the greater the absorbance reading of NADPH at 340 nm, the greater is the glucose concentration. If bacterial G-6-PD is used, the cofactor is NAD+ with the production of NADH.

Secretion of hormones by the anterior pituitary may be controlled by the circulating levels of hormones from the respective target gland, as well as hormones secreted by what organ? A. Posterior lobe of the pituitary gland B. Intermediate lobe of the pituitary gland C. Hypothalamus D. Adrenal medulla

C. The hypothalamus produces releasing factors or hormones that affect the release and synthesis of anterior pituitary hormones. The releasing hormones could have a stimulatory effect, as in the case of luteinizing hormone-releasing hormone (LH-RH), or an inhibitory effect, as in the case of prolactin-inhibiting factor (PIF). The posterior lobe of the pituitary acts only as a storage area for vasopressin and oxytocin, which are manufactured in the hypothalamus. The posterior lobe of the pituitary gland does not affect any feedback control on the anterior lobe. The intermediate lobe secretes beta-melaninophore-stimulating hormone, which acts on the skin. It also does not affect any control over the anterior lobe. The adrenal medulla secretes catecholamines, which are not involved in any feedback mechanism to the pituitary gland.

Which of the following proteins is normally produced by the fetus but is found in increased amounts in the amniotic fluid in cases of spina bifida? A. «|-Antitrypsin B. ai-Acid glycoprotein C. aj-Fetoprotein D. a2-Macroglobulin

C. The liver of a fetus and the yolk sac produce a protein known as arfetoprotein (AFP). The concentration of AFP in the blood of a fetus reaches a maximum concentration at approximately 16 to 18 weeks gestation. Blood levels decline from this point and finally disappear approximately 5 weeks after birth. In cases of open spina bifida or anencephaly, the fetus leaks large amounts of AFP into the amniotic fluid. By means of an amniocentesis, the amount of AFP present in the amniotic fluid may be quantified by enzyme-labeled immunoassay and other immunoassay techniques.

Which of the following statements correctly describes alkaline phosphatase? A. Decreased in Paget disease B. Decreased in third trimester of a normal pregnancy C. Increased in obstructive jaundice D. Primarily found in cardiac muscle

C. The main sources of alkaline phosphatase are liver, bone, intestine, and placenta. Elevated seaim alkaline phosphatase is associated with liver disease and with both obstructive jaundice and intrahepatic jaundice. In most cases, the serum alkaline phosphatase value in obstructive jaundice is higher than in intrahepatic jaundice. Increased serum values are also found in bone diseases such as Paget disease; in pregnant women (placental origin), especially in the third trimester of a normal pregnancy; and in normal growing children. In the presence of the latter conditions, when liver disease is also suspected, a GOT assay may be performed to aid in a differential diagnosis. Serum GOT levels are normal in these conditions but are elevated in liver disease.

The term "lipid storage diseases" is used to denote a group of lipid disorders, the majority of which are inherited as autosomal recessive mutations. What is the cause of these diseases? A. Excessive dietary fat ingestion B. Excessive synthesis of chylomicrons C. A specific enzyme deficiency or nonfunctional enzyme form D. An inability of adipose tissue to store lipid materials

C. The majority of the lipid (lysosomal) storage diseases are inherited as autosomal recessive mutations. This group of diseases is characterized by an accumulation of Sphingolipids in the central nervous system or some other organ. Such lipid accumulation frequently leads to mental retardation or progressive loss of central nervous system functions. The cause of such lipid accumulation has been attributed either to specific enzyme deficiencies or to nonfunctional enzyme forms that inhibit the normal catabolism of the Sphingolipids.

A male patient, 48 years old, mentions during his annual physical that he has been having difficulty urinating. The physician performs a rectal examination, and he orders a total prostate-specific antigen (PSA) and free PSA. The patient has the tests done the following week, and the total PSA result is 3.1 ng/mL and the free PSA is 0.3 ng/mL. What do these results suggest? A. Both are normal, no disease present B. Benign prostatic hypertrophy C. Increased risk of prostate cancer D. Free PSA is low and does not correlate with total PSA

C. The normal range for total PSA is sometimes referenced as less than 4.0 ng/mL. Early detection guidelines endorse a lower cutoff for total PSA up to 2.5 ng/mL and recommend that values >2.5 ng/mL should be followed up by performing a biopsy. Men with prostate cancer tend to have lower % free PSA (free PSA/total PSA) than men with benign disease; thus lower % free PSA is associated with a higher risk of prostate cancer. In the case presented, the patient's total PSA was 3.1 ng/mL with a free PSA of 0.3 ng/mL, which is 10% free PSA. This low percentage is suggestive of a higher probability of cancer, whereas a percentage >25% is associated with lower risk of cancer.

To what class of enzymes does lactate dehydrogenase belong? A. Isomerases B. Ligases C. Oxidoreductases D. Transferases

C. The oxidoreductases are enzymes that catalyze the addition or removal of hydrogen from compounds. These enzymes need a coenzyme, such as nicotinamide adenine dinucleotide (NAD+) or its phosphorylated derivative NADP+, as a hydrogen acceptor or donor in order to function. Lactate dehydrogenase and glucose-6-phosphate dehydrogenase are examples of oxidoreductases. Isomerases are those enzymes that catalyze intramolecular conversions such as the oxidation of a functional group by an adjacent group within the same molecule. Glucose phosphate isomerase is an example of this class of enzymes. Ligases are those enzymes that catalyze the union of twomolecules accompanied by the breakdown of a phosphate bond in adenosine triphosphate (ATP) or a similar triphosphate. An example is glutamine synthetase.

Blood gases are drawn on a 68-year-old asthmatic who was recently admitted for treatment of a kidney infection. Blood gas results are as follows: pH — 7.25, PCO2 — 56mmHg, HCOJ = 16 mmol/L .What condition is indicated by these results? A. Metabolic alkalosis, partially compensated B. Respiratory acidosis, uncompensated C. A dual problem of acidosis D. An error in one of the blood gas measurements

C. The pH clearly indicates acidosis. Both the metabolic (decreased HCO^) and respiratory (increased PCOo) components, however, indicate acidosis. There is no compensation seen in the results. Thus the patient has a double or mixed problem of acidosis.

A 30-year-old woman is admitted to the hospital. She has truncal obesity, buffalo humpback, moon face, purple striae, hypertension, hyperglycemia, increased facial hair, acne, and amenorrhea. The physician orders endocrine testing. The results are as follows: Urine free cortisol—increased Serum cortisol (8 A.M.)—increased Plasma ACTH—decreased Dexamethasone suppression test: Overnight low dose—no suppression of serum cortisol High dose—no suppression of serum cortisol What is the most probable diagnosis? A. Pituitary adenoma B. Ectopic ACTH lung cancer C. Adrenocortical carcinoma D. Addison disease

C. The probable diagnosis is Gushing syndrome caused by adrenocortical carcinoma. In adrenocortical carcinoma, the urinary free cortisol and the serum cortisol levels would be elevated and the plasma ACTH level would be decreased. The carcinoma produces excess cortisol that, because of the feedback loop, turns off pituitary production of ACTH. Neither the lowdose dexamethasone suppression test nor the high-dose test is able to suppress cortisol production. Because dexamethasone is a cortisol analogue, it would normally suppress ACTH and cortisol levels in a healthy individual. All these data support primary adrenal dysfunction caused by an adrenal carcinoma. If the elevated cortisol level was due to a pituitary adenoma or ectopic ACTH lung cancer, the ACTH level would also be increased. Addison disease is caused by hypofunction of the adrenal cortex.

The quantification of the high-density lipoprotein cholesterol level is thought to be significant in the risk assessment of what disease? A. Pancreatitis B. Cirrhosis C. Coronary artery disease D. Hyperlipidemia

C. The quantification of the HDL cholesterol level is thought to contribute in assessing the risk that an individual may develop coronary artery disease (CAD). There appears to be an inverse relationship between HDL cholesterol and CAD. With low levels of HDL cholesterol, the risk of CAD increases. It is thought that the HDL facilitates the removal of cholesterol from the arterial wall, therefore decreasing the risk of atherosclerosis. In addition, LDL cholesterol may be assessed, because increased LDL cholesterol and decreased HDL cholesterol are associated with increased risk of CAD.

In order to maintain electrical neutrality in the red blood cell, bicarbonate leaves the red blood cell and enters the plasma through an exchange mechanism with what electrolyte? A. Sodium B. Potassium C. Chloride D. Phosphate

C. The red blood cell membrane is permeable to both bicarbonate and chloride ions. Chloride ions participate in buffering the blood by diffusing out of or into the red blood cells to compensate for the ionic change that occurs when bicarbonate enters or leaves the red blood cell. This is called the chloride shift.

What is the reference interval for fasting serum glucose in an adult expressed in SI units (International System of Units)? A. 1.7-3.3 mmol/L B. 3.3-5.6 mmol/L C. 4.1-5.5 mmol/L D. 6.7-8.3 mmol/L

C. The reference interval for fasting serum glucose in an adult expressed in conventional units is 74-99 mg/dL. To convert conventional units to SI units (Systeme International d'Unites), multiply the conventional units in mg/dL by the 0.0555 conversion factor to obtain SI units in mmol/L. Thus, 74 mg/dL x 0.0555 = 4.1 mmol/L and 99 mg/dL x 0.0555 = 5.5 mmol/L. Although conventional units are used commonly in the United States, many scientific journals require the use of SI units in their publications and many foreign countries use SI units routinely in clinical practice. To identify additional conversion factors for other analytes, consult the appendix of a clinical chemistry textbook.

An individual has a plasma glucose level of 110 mg/dL. What would be the approximate glucose concentration in this patient's cerebrospinal fluid? A. 33 mg/dL B. 55 mg/dL C. 66 mg/dL D. 110 mg/dL

C. The reference interval for glucose in CSF is 60% of the normal plasma value. For a plasma glucose of 110 mg/dL, the expected CSF glucose level would be 66 mg/dL. The equilibration of CSF with plasma glucose takes several hours. The reference interval for the CSF glucose level is 40-70 mg/dL as compared with a normal fasting plasma glucose level. Low levels of CSF glucose are associated with a number of diseases including bacterial meningitis and tuberculous meningitis, whereas viral disease generally presents with a normal level of CSF glucose.

What is the approximate number of half-life periods required for a serum drug concentration to reach 97-99% of the steady state? A. 1-3 B. 2-4 C. 5-7 D. 7-9

C. The term "half-life" refers to the time required for a 50% decrease in serum drug concentration after absoiption and distribution are complete. The more complete descriptive term is "drug elimination half-life." It requires 5-7 half-life periods for drug concentration to reach steady state. At steady state, the drug concentration is in equilibrium with the dose administered rate and the elimination rate. Knowledge of a drug's half-life is important both for planning therapy and for monitoring drug concentration. In disease states, particularly involving the kidney and liver, half-life may be significantly altered and lead to accumulations of the daig or its metabolites in the blood.

When an AMI occurs, in what order (list first to last) will the enzymes aspartate aminotransferase (AST), creatine kinase (CK), and lactate dehydrogenase (LD) become elevated in the serum? A. AST, LD, CK B. CK,LD,AST C. CK,AST, LD D. LD, CK,AST

C. When an AMI occurs, CK is the first enzyme to become elevated in the blood, rising within 4 to 6 hours following chest pain. AST exhibits a rise in the serum level within 6 to 8 hours. LD shows an increase in 8 to 12 hours following infarction. Measurement of these three enzymes to assess acute myocardial infarction has been replaced by cardiac troponin, myoglobin, and CK-MB. However, awareness of the CK, AST, and LD patterns as well as other biochemical tests is useful in assessing organ complications that may arise during the period of AMI.

Which of the following laboratory results is not characteristic of a complete obstruction of the common bile duct? A. Negative urine urobilinogen B. Negative fecal urobilinogen and urobilin C. Negative urine bilirubin D. Excretion of a pale-colored stool

C. With complete obstruction of the common bile duct, bilirubin diglucuronide would be unable to pass from the bile into the intestines. Such obstruction to the flow of bile will cause the conjugated bilirubin to be regurgitated into the sinusoids and the general circulation. Because conjugated bilirubin is water soluble, it will be excreted in the urine. However, because of the lack of bile flow into the intestines, neither urobilinogen nor urobilin will be present in the feces. The lack of urobilin in the feces will be apparent from the light brown to chalky-white coloration of the stools. Because there is no urobilinogen in the intestines to be picked up by the enterohepatic circulation, the urinary excretion of urobilinogen will be negative. Because the obstruction may sometimes be only partial, this description would be somewhat altered. Provided that some bile was able to flow into the intestines, the fecal urobilinogen and urobilin concentrations would be present but depressed, the urinary urobilinogen excretion would be below normal, and the urinary bilirubin level would be increased.

A 68-year-old female patient tells her physician of being " cold all the time" and recent weight gain, with no change in diet. The doctor orders a TSH level, and the laboratory reports a value of 8.7 uU/mL (8.7 IU/L) (reference range = 0.5 uU/mL [0.5-5.0 IU/L[). This patient most likely has: A. primary hypothyroidism B. Graves disease C. a TSH-secretin tumor D. primary hyperthyroidism

Correct Answer: A

A common cause of a falsely increased LD1 fraction of lactic dehydrogenase is: A. specimen hemolysis B. liver disease C. congestive heart failure D. drug toxicity

Correct Answer: A

A critically ill patient becomes comatose. The physician believes the coma is due to hepatic failure. The assay most helpful in this diagnosis is: A. ammonia B. ALT C. AST D. GGT

Correct Answer: A

A hospitalized patient is experiencing increased neuromuscular irritability (tetany). Which of the following tests should be ordered immediately? A. calcium B. phosphate C. BUN D. glucose

Correct Answer: A

A lipemic serum is separated and frozen at -20C for assay at a later date. One week later, prior to performing an assay for triglycerides, the specimen should be: A. warmed to 37 C and mixed thoroughly B. warmed to 15C and centrifuged C. transferred to a glycerated test tube D. discarded and a new specimen obtained

Correct Answer: A

A stool specimen that appears black and tarry should be tested for the presence of: A. occult blood B. fecal fat C. trypsin D. excess mucus

Correct Answer: A

An automated method for measuring chloride which generates silver ions in the reaction is: A. coulomery B. mass spectroscopy C. chromatography D. polarography

Correct Answer: A

Arterial blood that is collected in a heparinized syringe but exposed to room air would be most consistent with the changes in which of the following specimens? Specimen; PO2; PCO2; pH -A: elevated; decreased; elevated -B: decreased; elevated; decreased -C: unchanged; elevated; unchanged -D: decreased; decreased: decreased A. specimen A B. specimen B C. specimen C D. specimen D

Correct Answer: A

Assay of transketolase activity in blood is used to detect deficiency of: A. thiamine B. folic acid C. ascorbic acid D. riboflavin

Correct Answer: A

Blood specimens for digoxin assays should be obtained between 8 hours or more after drug administration because: A. tissue and serum levels need to reach equilibrium B. serum digoxin concentrations will be falsely low prior to 6 hours C. all of the digoxin is in the cellular fraction prior to 6 hours D. digoxin protein-binding interactions are minimal prior to 6 hours

Correct Answer: A

Blood was collected in a serum separator tube on a patient who has been fasting since midnight. The time of collection was 7 AM. The laboratory test wich should be recollected is: A. triglycerides B. iron C. LD D. sodium

Correct Answer: A

Bromcresol purple at a pH of 5.2 is used in a colorimetric method to measure: A. albumin B. globulin C. Bence Jonce protein D. immunoprotein

Correct Answer: A

Coulometry is often used to measure: A. chlorid ein sweat B. the pH in saliva C. bicarbonate in urine D. ammonia in plasma

Correct Answer: A

Given the following results: -alkaline phsophatase: slight increase -aspartate amino transferase: marked increase -alanine amino transferase: marked increase -gamma-glutamyl transferase: slight increase This is most consistent with: A. acute hepatitis B. chronic hepatitis C. obstructive jaundice D. liver hamangioma

Correct Answer: A

Hemoglobin S can be separated form hemoglobin D by: A. electrophoresis on a different medium and acidic pH B. hemoglobin A2 quantitation C. electrophoresis at hight voltage D. Kleihauer-Betke acid elution

Correct Answer: A

Hemoglobin S can be separated from hemoglobin D by which of which of the following methods? A. agar gel electrophoresis at pH 5.9 B. thin-layer chromatography C. alkali denaturation D. ammonium precipitation

Correct Answer: A

IF the LDL-cholesterol is to be calculate by the Friedewald formula, what are the 2 measurements that need to be carried out by the same chemical procedure? A. total cholesterol and HDL-cholesterol B. total cholesterol and triglyceride C. triglyceride and chylomicrons D. apolipoprotein A and apolipoprotein B

Correct Answer: A

In amniotic fluid, the procedure used to determine fetal lung maturity is: A. lecithin/sphingomyelin ration B. creatinine C. measurement of absorbance at 450 nm D. alpha-fetoprotein

Correct Answer: A

In bilirubin determinations, the purpose of adding a concentrated caffeine solution or methyl alcohol is to: A. allow indirect bilirubin to react with color reagent B. dissolve conjugated bilirubin C. precipitate protein D. prevent any change in pH

Correct Answer: A

In competitive inhibition of an enzyme reaction , the: A. inhibitor binds to the enzyme at the same site as does the substrate B. inhibitor often has a chemical structure different to that of the substrate C. activity of the reaction can be decreased by increasing the concentration of the substrate D. activity of the reaction can be increased by decreasing the temperature

Correct Answer: A

In the assay of lactate dehydrogenase, which of the following products is actually measured? A. NADH B. ATP C. lactic acid D. pyruvic acid

Correct Answer: A

In which of the following disease states is conjugated bilirubin a major serum component? A. biliary obstruction B. hemolysis C. neonatal jaundice D. erythroblastosis fetalis

Correct Answer: A

Refer to the following illustration: See BOC pg 126 Pic 1 The figure above shows the reciprocal of the measured velocity of an enzyme reaction plotted agains the reciprocal of the substrate concentration. True statements about this figure include: A. the intercept of the line on the ordinate (y-axis) can be used to calculate the Vmax B. the straight line indicates that the enzyme reaction proceeds according to zero order kinetics C. the intercept on the ordinate (y-axis) can be used to calculate the Michaelis-Menten constant D. the fact the substrate concentration is plotted on both sides of the zero point indicates that the reaction is reversible

Correct Answer: A

The buffer pH most effective at allowing amphoteric proteins to migrate toward the cathode in an electrophoretic system would be: A. 4.5 B. 7.5 C. 8.6 D. 9.5

Correct Answer: A

The osmolality of a urine or serum specimen is measured by a change in the A. freezing point B. sedimentation point C. midpoint D. osmotic pressure

Correct Answer: A

The protein portion of an enzyme complex is called the: A. apoenzyme B. coenzyme C. holoenzyme D. proenzyme

Correct Answer: A

The screen for adrenal cortical hyperfunction with the greatest sensitivity and specificity is: A. 24-hour urine free cortisol B. plasma cortisol C. urinary 17-hydroxycorticosteroids D. plasma corticosterone

Correct Answer: A

Which of the following applies to cryoscopic osmometry? A. temperature at equilibrium is a function of the number of particles in solution B. temperature plateau for a solution is horizontal C. freezing point of a sample is absolute D. initial freezing of a sample produces an immediate solid state

Correct Answer: A

Which of the following enzymes are used in the diagnosis of acute pancreatitis? A. amylase (AMS) and lipase (LPS B. aspartate aminotransferase (AST) and alanine aminotransferase (ALT) C. 5'-nucleotide (5' N) and gamma-glutamyl transferase (GGT) D. aspartate aminotransferase (AST) and lactate dehydrogenase (LD)

Correct Answer: A

Which of the following factors is not relevant to therapeutic drug monitoring (TDM) of the aminoglycosides, antibiotics and vancomycin? A. intestinal absorption B. nephrotoxicity C. ototoxicity D. renal function

Correct Answer: A

Which of the following is a glycolytic enzyme that catalyzes the cleavage of fructose-1, 6-diphosphate to glyceraldeyde-3-phosphate and dihydroxyacetone phosphate? A. aldolase B. phosphofructokinase C. pyruvate kinase D. glucose-6-phosphate dehydrogenase

Correct Answer: A

Which of the following is most likely to be ordered in addition to serum calcium to determine the cause of tetany? A. magnesium B. phosphate C. sodium D. vitamin D

Correct Answer: A

Creatinine is formed from the: A. Oxidation of creatine B. Oxidation of protein C. Deamination of dibasic amino acids D. Metabolism of purines

Correct Answer: A 1. A Creatinine is produced at a rate of approximately 2% daily from the oxidation of creatine mainly in skeletal muscle. Creatine can be converted to creatinine by addition of strong acid or alkali or by the enzyme creatine hydroxylase.

Which of the following hormones is often decreased by approximately 25% in the serum of pregnant women who have a fetus with Down syndrome? A. Estriol (E3) B. Human chorionic gonadotropin (hCG) C. Progesterone D. Estradiol (E2)

Correct Answer: A 1. A E3 is produced in the placenta and fetal liver from dehydroepiandrosterone derived from the mother and fetal liver. E3 is the major estrogen produced during pregnancy, and levels rise throughout gestation. Serum free E3 is often lower than expected for the gestational age in a pregnancy associated with Down syndrome. The combination of low serum free estriol, low-α fetoprotein, high hCG, and high- inhibin A is used as a screening test to detect Down syndrome. When one of the four markers is abnormal, amniocentesis should be performed for the diagnosis of Down syndrome by karyotyping or FISH. The four markers have a combined sensitivity (detection rate) of approximately 75%.

Plate 1 is a photomicrograph of an antinuclear antibody test using human fibroblasts, fluorescein isothiocyanate (FITC)-conjugated antihuman serum, and transmitted fluorescence microscopy. Which pattern of immunofluorescence is demonstrated in this 400× field? A. Homogenous B. Peripheral C. Nucleolar D. Speckled

Correct Answer: A 1. A Using FITC-conjugated antihuman serum, diffuse apple-green fluorescence seen over the entire nucleus characterizes the homogenous pattern. At a significant titer, this pattern occurs in a variety of systemic autoimmune diseases including systemic lupus erythematosus, rheumatoid arthritis, systemic sclerosis, and Sjögren's syndrome. The antibodies are directed against nucleoprotein; although they are mainly nonpathological, they are useful markers for active disease.

Which of the following effects results from expsosure of a normal arterial blood sample to room air? A. PO2 increased;PCO2 decreased; pH increased B. PO2 decreased;PCO2 increased; pH decreased C. PO2 increased; PCO2 decreased; pH decreased D. PO2 decreased; PCO2 decreased; pH decreased

Correct Answer: A 10. A The PO2 of air at sea level (21% O2) is about 150 mm Hg. The PCO2 of air is only about 0.3 mm Hg. Consequently, blood releases CO2 gas and gains O2 when exposed to air. Loss of CO2 shifts the equilibrium of the bicarbonate buffer system to the right, decreasing hydrogen ion concentration and blood becomes more alkaline.

Which of the following statements is true? A. Cystatin C is measured immunochemically B. The calibrator used for cystatin C is traceable to the National Bureau of Standards calibrator C. Cystatin C assays have a lower coefficient of variation than plasma creatinine D. Enzymatic and rate Jaffe reactions for creatinine give comparable results

Correct Answer: A 11. A Cystatin C can be measured by enzyme immunoassay, immunonephelometry, and immunoturbidimetry. However, there is no standardized calibrator as for creatinine, and therefore, results vary considerably from lab to lab. The coefficient of variation for these methods tends to be slightly higher than for creatinine. Since the enzymatic methods are specific, they give lower plasma creatinine results than the Jaffe method in persons with normal renal function. However, they tend to give higher clearance results than for inulin or iohexol clearance because some creatinine is secreted by the renal tubules.

Which of the following formulas for O2 content is correct? A. O2 content = %O2 saturation/100 × Hgb g/dL × 1.39 mL/g + (0.0031 × PO2) B. O2 content = PO2 × 0.0306 mmol/L/mm C. O2 content = O2 saturation × Hgb g/dL × 0.003 mL/g D. O2 content = O2 capacity × 0.003 mL/g

Correct Answer: A 11. A Oxygen content is the sum of O2 bound to Hgb and O2 dissolved in the plasma. It is dependent upon the Hgb concentration and the percentage of Hgb bound to O2 (O2 saturation). Each gram of Hgb binds 1.39 mL of O2. The dissolved O2 is determined from the solubility coefficient of O2 (0.0031 mL per dL/mm Hg) and the PO2. O2 content = % Sat/100 × Hgb in g/dL × 1.39 mL/g + (0.0031 × PO2).

At pH 8.6, proteins are _________ charged and migrate toward the _________. A. Negatively, anode B. Positively, cathode C. Positively, anode D. Negatively, cathode

Correct Answer: A 11. A Proteins are amphoteric owing to ionization of acidic and basic side chains of amino acids. When the pH of the solution equals the isoelectric point (pI), the protein will have no net charge and is insoluble. When the pH of the solution is above the pI, the protein will have a net negative charge. Anions migrate toward the anode (positive electrode).

At pH 8.6, the cathodal movement of γ globulins is caused by: A. Electroendosmosis B. Wick flow C. A net positive charge D. Cathodal sample application

Correct Answer: A 13. A Agarose and cellulose acetate contain fixed anions (e.g., acetate) that attract counterions when hydrated with buffer. When voltage is applied the cations migrate to the cathode, creating an osmotic force that draws H2O with them. This force, called electroendosmosis, opposes protein migration toward the anode and may cause some γ-globulins to be displaced toward the cathode.

Which set of results for ER and PR is associated with the highest likelihood of a favorable response to treatment with estrogen-suppression therapy (tamoxifen)? A. ER positive, PR positive B. ER positive, PR negative C. ER negative, PR positive D. ER negative, PR negative

Correct Answer: A 13. A Both ER and PR receptor assays are performed on breast tissue biopsies to determine the probability of response to tamoxifen. The PR receptor is produced from the ER receptor and expression of both predicts a positive response to the drug. Less than 15% of persons who are ER negative and PR negative have a favorable response, whereas over 75% of those who are positive for both receptors have a favorable response to tamoxifen.

Which condition produces the highest elevation of serum lactate dehydrogenase? A. Pernicious anemia B. Myocardial infarction C. Acute hepatitis D. Muscular dystrophy

Correct Answer: A 13. A Serum LD levels are highest in pernicious anemia, reaching 10-50 times the upper reference limit (URL) as a result of intramedullary hemolysis. Moderate elevations (5-10 × URL) usually are seen in acute myocardial infarction, necrotic liver disease, and muscular dystrophy. Slight increases (2-3 × URL) are sometimes seen in obstructive liver disease.

Which statement regarding measurement of Hgb A1c is true? A. Levels do not need to be done fasting B. Both the labile and stable Hgb A1c fractions are measured C. Samples should be measured within 2 hours of collection D. The assay must be done by chromatography

Correct Answer: A 13. A Since Hgb A1C represents the average blood glucose 2-3 months prior to blood collection, the dietary status of the patient on the day of the test has no effect upon the results. Refrigerated whole-blood samples are stable for up to 1 week. Hgb A1C is assayed by cation exchange high-performance liquid chromatography or immunoassay (immunoturbidimetric inhibition) because both methods are specific for stable Hgb A1C, and do not demonstrate errors caused by abnormal hemoglobins, temperature of reagents, or fractions other than A1c.

SITUATION: Laboratory results on a patient from the emergency department are -glucose = 1,100 mg/dL -Na = 155 mmol/L -K = 1.2 mmol/L -Cl = 115 mmol/L -TCO2 = 3.0 mmol/L What is the most likely explanation of these results? A. Sample drawn above an IV B. Metabolic acidosis with increased anion gap C. Diabetic ketoacidosis D. Laboratory error measuring electrolytes caused by hyperglycemia

Correct Answer: A 13. A These results are consistent with dilution of venous blood by intravenous fluid containing 5% dextrose and normal saline. The intravenous fluid is free of potassium and bicarbonate, accounting for the low level of these electrolytes (incompatible with life).

Which type of cancer is associated with the highest level of AFP? A. Hepatoma B. Ovarian cancer C. Testicular cancer D. Breast cancer

Correct Answer: A 14. A AFP is increased in all persons with yolk sac tumors and over 80% of those with hepatoma. Levels above 1000 ng/mL are diagnostic of hepatoma. Ectopic AFP-secreting tumors are produced by ovarian, testicular, breast, GI, and bladder cancers, and these sources should be considered when 10-fold or higher elevations are seen in the absence of abnormal liver function. AFP is used along with hCG to increase the diagnostic sensitivity of nonseminoma testicular tumors and to stage the disease. Approximately 42% of persons with nonseminoma testicular cancer are positive for hCG but over 70% are positive for hCG or AFP.

Which of the following statements is correct in assessing GH deficiency? A. Pituitary failure may involve one, several, or all adenohypophyseal hormones; but GH deficiency is usually found B. A normal random serum level of GH in a child under 6 years old rules out GH deficiency C. Administration of arginine, insulin, or glucagon will suppress GH release D. GH levels in the blood show little variation within a 24-hour period

Correct Answer: A 14. A Because GH is the most abundant pituitary hormone, it may be used as a screening test for pituitary failure in adults. Pituitary hormone deficiencies are rare and are evaluated by measuring those hormones associated with the specific type of target organ dysfunction. GH secretion peaks during sleep, and pulsed increases are seen following exercise and meals. In adults, a deficiency of GH can be ruled out by demonstrating normal or high levels on two successive tests. In children, there is extensive overlap between normal and low GH levels, and a stimulation (provocative) test is usually needed to establish a diagnosis of deficiency. Exercise is often used to stimulate GH release. If GH levels are greater than 6 μg/L after vigorous exercise, then deficiency is ruled out. In addition to exercise, drugs such as arginine, insulin, propranolol, and glucagon can be used to stimulate GH release. Deficiency is documented by registering a subnormal response to two stimulating agents.

Urea is produced from: A. The catabolism of proteins and amino acids B. Oxidation of purines C. Oxidation of pyrimidines D. The breakdown of complex carbohydrates

Correct Answer: A 15. A Urea is generated by deamination of amino acids. Most is derived from the hepatic catabolism of proteins. Uric acid is produced by the catabolism of purines. Oxidation of pyrimidines produces orotic acid.

The half-band width of a monochromator is defined by: A. The range of wavelengths passed at 50% maximum transmittance B. One-half the lowest wavelength of optical purity C. The wavelength of peak transmittance D. One-half the wavelength of peak absorbance

Correct Answer: A 16. A Half-band width is a measure of bandpass made using a solution or filter having a narrow natural bandpass (transmittance peak). The wavelength giving maximum transmittance is set to 100%T (or 0 A). Then, the wavelength dial is adjusted downward, until a readout of 50%T (0.301 A) is obtained. Next, the wavelength is adjusted upward until 50%T is obtained. The wavelength difference is the half-band width. The narrower the half-band width, the better the photometric resolution of the instrument.

In which type of liver disease would you expect the greatest elevation of LD? A. Toxic hepatitis B. Alcoholic hepatitis C. Cirrhosis D. Acute viral hepatitis

Correct Answer: A 16. A Liver disease produces an elevated LD-4 and LD-5. Levels may reach up to 10 times the URL in toxic hepatitis and in hepatoma. However, LD levels are lower in viral hepatitis (2-5 × URL), only slightly elevated in cirrhosis (2-3 × URL) and not significantly elevated in alcoholic liver disease.

Blood sample collection time for peak drug levels: A. Varies with the drug, depending on its rate of absorption B. Is independent of drug formulation C. Is independent of the route of administration D. Is 30 minutes after a bolus intravenous injection is completed

Correct Answer: A 16. A The peak concentration of a drug is the highest concentration obtained in the dosing interval. For oral drugs, the time of peak concentration is dependent upon their rates of absorption and elimination and is determined by serial blood measurements. Peak levels for oral drugs are usually drawn 1-2 hours after administration of the dose. For drugs given intravenously, peak levels are measured immediately after the infusion is completed.

Which of the following is one advantage of high-resolution (HR) agarose electrophoresis over lower-current electrophoresis? A. High-resolution procedures detect monoclonal and oligoclonal bands at a lower concentration B. A smaller sample volume is used C. Results are obtained more rapidly D. Densitometric scanning of HR gels is more accurate

Correct Answer: A 17. A HR agarose procedures use higher current and a cooling device to resolve 12 or more bands. Advantages include phenotyping of α1-antitrypsin (detection of Z and S variants), detection of β2 microglobulin in urine indicating tubular proteinuria (often associated with drug-induced nephrosis), and greater sensitivity detecting monoclonal gammopathies, immune complexes, and oligoclonal bands in CSF associated with multiple sclerosis. Its disadvantage is that densitometric scans of HR gels usually underestimate albumin.

When referring to quality control (QC) results, what parameter usually determines the acceptable range? A. The 95% confidence interval for the mean B. The range that includes 50% of the results C. The central 68% of results D. The range encompassed by ±2.5 standard deviations

Correct Answer: A 17. A The acceptable range for quality control results is usually set at the 95% confidence interval. This is defined as the range between -1.96s and +1.96s. This means that we can expect a QC result to fall within this range 95 out of 100 times. For practical purposes, this is the same as ±2 s (95.4 out of 100 results should fall within ±2 s of the mean on the basis of chance).

The reagent blank corrects for absorbance caused by: A. The color of reagents B. Sample turbidity C. Bilirubin and hemolysis D. All of these options

Correct Answer: A 17. A When a spectrophotometer is set to 100%T with the reagent blank instead of water, the absorbance of reagents is automatically subtracted from each unknown reading. The reagent blank does not correct for absorbance caused by interfering chromogens in the sample such as bilirubin, hemolysis, or turbidity.

Select the elimination model that best describes most oral drugs. A. One compartment, linear first-order elimination B. Michaelis-Menton or concentration-dependent elimination C. Two compartment with a biphasic elimination curve D. Logarithmic elimination

Correct Answer: A 18. A Most drugs given orally distribute uniformly through the tissues reaching rapid equilibrium, so both blood and tissues can be viewed as a single compartment. Elimination according to Michaelis-Menton kinetics is nonlinear because at high concentrations, the hepatic enzyme system becomes saturated, reducing the elimination efficiency.

In the Oliver-Rosalki method, the reverse reaction is used to measure CK activity. The enzyme(s) used in the coupling reactions is (are): A. Hexokinase and G-6-PD B. Pyruvate kinase and LD C. Luciferase D. Adenylate kinase

Correct Answer: A 18. A The Oliver-Rosalki method for CK is based upon the formation of ATP from creatine phosphate. Hexokinase catalyzes the phosphorylation of glucose by ATP. This produces glucose-6-PO4 and adenosine diphosphate (ADP). The glucose-6-PO4 is oxidized to 6-phosphogluconate as NADP+ is reduced to NADPH. ATP + glucose Hexokinase --> ADP + glucose-6-PO4 glucose-6-PO4 + NADP+ G-6-PD --> 6-phosphogluconate + NADPH + H+

Which of the following conditions is cause for rejecting an analytical run? A. Two consecutive controls greater than 2 s above or below the mean B. Three consecutive controls greater than 1 s above the mean C. Four controls steadily increasing in value but less than ±1 s from the mean D. One control above +1 s and the other below -1 s from the mean

Correct Answer: A 19. A Rejecting a run when three consecutive controls fall between 1 and 2 s or when a trend of four increasing or decreasing control results occurs would lead to frequent rejection of valid analytical runs. Appropriate control limits are four consecutive controls above or below 1 s (41s ) to detect a significant shift, and a cusum result exceeding the ±2.7 s limit to detect a significant shift or trend. When controls deviate in opposite directions, the difference should exceed 4s before the run is rejected.

In general, in which of the following situations is the analysis of a tumor marker most useful? A. Testing for recurrence B. Prognosis C. Screening D. Diagnosis

Correct Answer: A 2. A Most tumor markers are expressed at very low levels so that the concentration in early malignancy overlaps that seen in normal individuals. This makes them ineffective for screening. Three exceptions are hCG in males for testicular cancer, calcitonin for thyroid medullary cancer, and prostate-specific antigen (PSA) for prostate cancer. Most tumor markers are increased in nonmalignant disease, and this nonspecificity reduces their usefulness for diagnosis of malignancy. In addition to the three markers mentioned, the hormones insulin (insulinoma), gastrin (gastrinoma), and prolactin (prolactinoma), and the catecholamines (pheochromocytoma) have some diagnostic utility. Some tumor markers are useful predictors of disease progression and response to treatment. These include BRCA-1, estrogen and progesterone receptors, cathepsin-D, and the Philadelphia chromosome. The major use of tumor markers is to monitor recurrence and therapy. Successful treatment reduces the concentration of the marker significantly or results in an undetectable level. A rise in level following treatment signals recurrence.

Which fact must be considered when evaluating a patient who displays signs of drug toxicity? A. Drug metabolites (e.g., N-acetylprocainamide) may need to be measured as well as parent drug B. If the concentration of total drug is within therapeutic limits, the concentration of free drug cannot be toxic C. If the drug has a wide therapeutic index, then it will not be toxic D. A drug level cannot be toxic if the trough is within the published therapeutic range

Correct Answer: A 20. A Altered drug pharmacokinetics may result in toxicity even when the dose of drug is within the accepted therapeutic range. Two common causes of this are the presence of unmeasured metabolites that are physiologically active, and the presence of a higher than expected concentration of free drug. Because only free drug is physiologically active, decreased binding protein or factors that shift the equilibrium favoring more unbound drug can result in toxicity when the total drug concentration is within the therapeutic range. Some drugs with a wide therapeutic index are potentially toxic because they may be ingested in great excess with little or no initial toxicity. For example, acetaminophen overdose does not usually become apparent until 3-5 days after the overdose. This creates the potential for hepatic damage to occur from continued use, especially in patients who have decreased hepatic or renal function because the drug half-life is extended.

A 55-year-old male with early stage prostate cancer diagnosed by biopsy had his prostate gland removed (simple prostatectomy). His PSA prior to surgery was 10.0 ng/mL. If the surgery was successful in completely removing the tumor cells, what would the PSA result be 1 month after surgery? A. Undetectable B. 1-3 ng/mL C. Less than 4 ng/mL D. Less than 10 ng/mL

Correct Answer: A 20. A If the tumor were confined to the prostate, the PSA would be undetectable 1 month following successful surgery, since there is no other tissue source of PSA. The half-life of PSA is 2.2-3.2 days, and the minimum detection limit of most assays is 0.2 ng/mL or lower. Therefore, it would require at least 2 weeks before the PSA level would be undetectable. The low minimum detection limit of the PSA assay, combined with the high tissue specificity of PSA, makes the test very sensitive in detecting recurrence.

One of two controls within a run is above +2s and the other control is below -2s from the mean. What do these results indicate? A. Poor precision has led to random error (RE) B. A systematic error (SE) is present C. Proportional error is present D. QC material is contaminated

Correct Answer: A 20. A When control results deviate from the mean in opposite directions, the run is affected by RE, which results from imprecision. An analytical run is rejected when two controls within the same run have an algebraic difference in excess of 4s (R4s ). The R4S rule is applied only to controls within a run (Level 1 - Level 2), never across runs or days.

Bichromatic measurement of absorbance can correct for interfering substances if: A. The contribution of the interferent to absorbance is the same at both wavelengths B. Both wavelengths pass through the sample simultaneously C. The side band is a harmonic of the primary wavelength D. The chromogen has the same absorbance at both wavelengths

Correct Answer: A 21. A In bichromatic photometry, the absorbance of sample is measured at two different wavelengths. The primary wavelength is at or near the absorbance maximum. An interfering substance having the same absorbance at both primary and secondary (side band) wavelengths does not affect the absorbance difference (Ad).

n which circumstance will the reporting of calculated oxygen saturation of hemoglobin based on PO2, PCO2, pH, temperature, and hemoglobin be in error? A. Carbon monoxide poisoning B. Diabetic ketoacidosis C. Patient receiving oxygen therapy D. Assisted ventilation for respiratory failure

Correct Answer: A 22. A CO has about 200 times the affinity as O2 for hemoglobin and will displace O2 from hemoglobin at concentrations that have no significant effect on the PAO2. Consequently, calculated oxygen saturation will be erroneously high. Other cases in which the calculated O2Sat should not be used include any hemoglobinopathy that affects oxygen affinity and methemoglobinemia. The other situations above affect the O2 saturation of hemoglobin in a manner that can be predicted by the effect of pH, PO2, and PCO2 on the oxyhemoglobin dissociation curve.

Which of the following statements about cortisol in Cushing's syndrome is true? A. Twenty-four-hour urinary free cortisol is a more sensitive test than plasma total cortisol B. Patients with Cushing's disease show pronounced diurnal variation in serum cortisol C. Free cortisol is increased by a high-serum cortisol-binding protein concentration D. An elevated serum total cortisol level is diagnostic of Cushing's syndrome

Correct Answer: A 22. A Serum cortisol can be increased by factors such as stress, medications, and cortisol-binding protein, and the cortisol level of normal patients will overlap those seen in Cushing's syndrome because of pulse variation. When cortisol levels become elevated, cortisol-binding protein becomes saturated, and free (unbound) cortisol is filtered by the glomeruli. Most is reabsorbed, but a significant amount reaches the urine as free cortisol. Twenty-four-hour urinary free cortisol avoids the diurnal variation that may affect plasma free cortisol levels and is a more sensitive test than serum total or free cortisol.

The electrophoretic pattern shown in the following densitometric tracing most likely indicates: See Harr pg 250 A. α1-Antitrypsin deficiency B. Infection C. Nephrosis D. Systemic sclerosis

Correct Answer: A 23. A This pattern shows a marked decrease in the α1-globulin (slightly less than one-fifth of the expected peak area). Staining of the α1-globulin fraction is predominately determined by the α1-antitrypsin level. A value of less than 20% of normal (0.2-0.4 g/dL) is usually caused by homozygous α1-antitrypsin deficiency. There is a slight decrease in albumin and increase in the α2-fraction. Patients with α1-antitrypsin deficiency often display elevations in the α2-globulin and γ-globulin fraction because the condition is associated with chronic emphysema and hepatic cirrhosis.

Which of the following components is not needed in a chemiluminescent immunoassay analyzer? A. Source lamp B. Monochromator C. Photodetector D. Wash station

Correct Answer: A 24. A Chemiluminescence is the production of light following a chemical reaction. Immunoassays based upon chemiluminescence generate light when the chemiluminescent molecule becomes excited; therefore, a light source is not used. In immunoassay platforms, chemiluminescent molecules such as acridinium can be used to label antigens or antibodies. Alternatively, chemiluminescent substrates such as luminol or dioxetane phosphate may be used. Light will be emitted when the enzyme-labeled molecule reacts with the substrate. In such assays, free and bound antigen separation is required and is usually accomplished using paramagnetic particles bound to either antibody or reagent antigen.

Which of the following conditions is classified as a renal-type aminoaciduria? A. Fanconi syndrome B. Wilson's disease C. Hepatitis D. Homocystinuria

Correct Answer: A 24. A Fanconi syndrome is an inherited disorder characterized by anemia, mental retardation, rickets, and aminoaciduria. Because the aminoaciduria results from a defect in the renal tubule, it is classified as a (secondary-inherited) renal-type aminoaciduria. Wilson's disease (inherited ceruloplasmin deficiency) causes hepatic failure. It is classified as a secondary-inherited overflow-type aminoaciduria because the aminoaciduria results from urea cycle failure. Hepatitis is classified as a secondary-acquired overflow-type aminoaciduria. Homocystinuria is a primary-inherited overflow-type aminoaciduria, and is caused by a deficiency of cystathionine synthase.

In most circumstances, when two controls within a run are both greater than ±2s from the mean, what action should be taken first? A. Recalibrate, then repeat controls followed by selected patient samples if quality control is acceptable B. Repeat the controls before taking any corrective action C. Change the reagent lot, then recalibrate D. Prepare fresh standards and recalibrate

Correct Answer: A 24. A When a 22s rule is broken an SE is present and corrective action is required (repeating just the QC will not correct the problem). If recalibration yields acceptable QC results, both sets of QC results and the corrective action taken are documented in the QC log. If the magnitude of the error is large enough to be medically significant, then all patient samples since the last previously acceptable QC should be repeated. If in question, the magnitude of the error can be evaluated by repeating abnormal patient samples. If the average difference between results before and after recalibration is > 2s, then all samples should be repeated since the last acceptable QC.

Which of the following is the mechanism causing Cushing's disease? A. Excess secretion of pituitary ACTH B. Adrenal adenoma C. Treatment with corticosteroids D. Ectopic ACTH production by tumors

Correct Answer: A 25. A Cushing's disease refers to adrenal hyperplasia resulting from misregulation of the hypothalamic-pituitary axis. It is usually caused by small pituitary adenomas. Cushing's syndrome may be caused by Cushing's disease, adrenal adenoma or carcinoma, ectopic ACTH-producing tumors, or excessive corticosteroid administration. The cause of Cushing's syndrome can be differentiated using the ACTH and dexamethasone suppression tests.

Which of the following mechanisms is responsible for metabolic acidosis? A. Bicarbonate deficiency B. Excessive retention of dissolved CO2 C. Accumulation of volatile acids D. Hyperaldosteronism

Correct Answer: A 25. A Metabolic acidosis is caused by bicarbonate deficiency and metabolic alkalosis by bicarbonate excess. Respiratory acidosis is caused by PCO2 retention (defective ventilation), and respiratory alkalosis is caused by PCO2 loss (hyperventilation). Important causes of metabolic acidosis include renal failure, diabetic ketoacidosis, lactate acidosis, and diarrhea.

A patient with hemolytic-uremic syndrome associated with septicemia has a haptoglobin level that is normal, although the plasma free hemoglobin is elevated and hemoglobinuria is present. Which test would be more appropriate than haptoglobin to measure this patient's hemolytic episode? A. Hemopexin B. Alpha-1 antitrypsin C. C-reactive protein D. Transferrin

Correct Answer: A 26. A Hemopexin is a small β globulin that binds to free heme. Haptoglobin is an α-2 globulin that binds to free hemoglobin and disappears from the serum when intravascular hemolysis produces more than 3 grams of free plasma hemoglobin. However, haptoglobin is an acute phase protein, and hepatic production and release are increased in response to acute infections. The normal serum haptoglobin is most likely the result of increased synthesis and would not accurately estimate the hemolytic episode in this patient.

Light scattering when the wavelength is greater than 10 times the particle diameter is described by: A. Rayleigh's law B. The Beer-Lambert law C. Mie's law D. The Rayleigh-Debye law

Correct Answer: A 26. A Rayleigh's law states that when the incident wavelength is much longer than the particle diameter, there is maximum backscatter and minimum right-angle scatter. The Rayleigh-Debye law predicts maximum right-angle scatter when wavelength and particle diameter approach equality. In nephelometry, the relationship between wavelength and diameter determines the angle at which the detector is located.

Of the methods used to measure amino acids, which is capable of measuring fatty acids simultaneously? A. Tandem-mass spectroscopy B. High-performance liquid chromatography C. Capillary electrophoresis D. Two-dimensional thin-layer chromatography

Correct Answer: A 27. A All four methods are able to separate each amino acid (up to 40 species); however, tandem-mass spectroscopy with electrospray ionization can measure amino acids, organic acids such as methylmalonic acid, and fatty acids. The acids are eluted from the dried blood spot with methanol after addition of internal standards and then derivatized with butanol-hydrochloric acid. Soft ionization of the butyl esters of the amino acids and butyl acylcarnitines of organic and fatty acids yields parent ions, and these are fragmented by collision with argon in the second mass filter to produce daughter ions. A process called multiple reaction monitoring identifies both parent ions and neutral fragments that identify the acids. Carnitines are quarternary ammonium compounds that carry the acids across the mitochondrial membrane.

The purpose of the nebulizer in an atomic absorption spectrophotometer that uses a flame is to: A. Convert ions to atoms B. Cause ejection of an outer shell electron C. Reduce evaporation of the sample D. Burn off organic impurities

Correct Answer: A 28. A The atomizer of the atomic absorption spectrophotometer consists of either a nebulizer and flame or a graphite furnace. The nebulizer disperses the sample into a fine aerosol, distributing it evenly into the flame. Heat from the flame is used to evaporate water and break the ionic bonds of salts, forming ground state atoms. The flame also excites a small percentage of the atoms, which release a characteristic emission line.

Enzymatic measurement of ammonia requires which of the following substrates and coenzymes? Substrate; Coenzyme A. α-Ketoglutarate; NADH B. Glutamate; NADH C. Glutamine; ATP D. Glutamine; NAD+

Correct Answer: A 29. A Enzymatic assays of ammonia utilize glutamate dehydrogenase (GLD). This enzyme forms glutamate from α-ketoglutarate (2-oxoglutarate) and ammonia, resulting in oxidation of NADH. The rate of absorbance decrease at 340 nm is proportional to ammonia concentration when the reaction rate is maintained under first-order conditions.

A flameless atomic absorption spectrophotometer dehydrates and atomizes a sample using: A. A graphite capillary furnace B. An electron gun C. A thermoelectric semiconductor D. A thermospray platform

Correct Answer: A 29. A Flameless atomic absorption uses a hollow tube of graphite with quartz ends. The tube is heated in stages by an electric current to successively dry, ash, and atomize the sample. During the ash and atomization steps, argon is injected into the tube to distribute the atoms. The furnace is more sensitive than a flame atomizer and more efficient in atomizing thermostable salts. However, it is prone to greater matrix interference and is slower than the flame atomizer because it must cool down before introduction of the next sample.

Which of the following statements about blood alcohol measurement is correct? A. Symptoms of intoxication usually begin when the level exceeds 0.05% w/v B. The skin puncture site should be disinfected with isopropanol C. The reference method is based upon enzymatic oxidation of ethanol by alcohol dehydrogenase D. Gas chromatography methods require extraction of ethanol from serum

Correct Answer: A 33. A Alcohol dehydrogenase is not specific for ethanol, and in vitro interference can occur with some ADH methods when skin is disinfected with other alcohols. For this reason, and to avoid interference with the interpretation of chromatograms for volatiles, blood samples are collected after disinfecting the skin site with benzalkonium chloride or other nonalcohol antiseptic. GLC is the legally accepted method of ethanol analysis. The low boiling point of ethanol permits direct analysis on blood or plasma diluted with water containing 1-propanol or other suitable internal standard.

Which of the following conditions is associated with hyperuricemia? A. Renal failure B. Chronic liver disease C. Xanthine oxidase deficiency D. Paget's disease of the bone

Correct Answer: A 33. A Excessive retention of uric acid results from renal failure and diuretics (or other drugs) that block uric acid excretion. Hyperuricemia may result from overproduction of uric acid in primary essential gout or excessive cell turnover associated with malignancy and chemotherapy. Overproduction may also result from an enzyme deficiency in the pathway forming guanosine triphosphate (GTP) or adenosine monophosphate (AMP) (purine salvage). Hyperuricemia is also associated with ketoacidosis and lactate acidosis, hypertension, and hyperlipidemia. Xanthine oxidase converts xanthine to uric acid; therefore, a deficiency of this enzyme results in low serum levels of uric acid. Paget's disease of bone causes cyclic episodes of bone degeneration and regeneration and is associated with very high serum ALP and urinary calcium levels.

Which statement best describes immunofixation electrophoresis (IEF)? A. Proteins are separated by electrophoresis followed by overlay of monospecific anti-immunoglobulins B. Proteins react with monospecific antisera followed by electrophoresis C. Antisera are electrophoresed, then diffused against patient's serum D. Serum is electrophoresed; the separated immunoglobulins diffuse against specific antisera placed into troughs

Correct Answer: A 33. A Immunofixation electrophoresis (IFE) is used to identify monoclonal bands in serum or urine. Electrophoresis is performed on the serum or urine sample in the same manner as for protein electrophoresis, except that six lanes are used for the same sample. After the proteins are separated, a different monospecific antiserum is applied across the surface of each lane. After incubating, the gel is washed and blotted to remove uncomplexed proteins and salts. The immune complexes that remain are stained. Monoclonal bands will be seen only in those lanes where the monoclonal immunoglobulins were recognized by the corresponding antiserum.

Identify the enzyme deficiency responsible for type 1 glycogen storage disease (von Gierke's disease). A. Glucose-6-phosphatase B. Glycogen phosphorylase C. Glycogen synthetase D. β-Glucosidase

Correct Answer: A 33. A Type 1 glycogen storage disease (von Gierke's disease) is an autosomal recessive deficiency of glucose-6-phosphatase. Glycogen accumulates in tissues, causing hypoglycemia, ketosis, and fatty liver. There are seven types of glycogen storage disease, designated type 1 through type 7, involving deficiency of an enzyme that acts on glycogen. Types 1, 4, and 6 cause deficient glycogen breakdown in the liver. Types 2, 5, and 7 involve skeletal muscle and are less severe. Type 3 usually involves both liver and muscle, although an uncommon subtype (3B) involves only the liver.

Why are three levels used for quality control of pH and blood gases? A. Systematic errors can be detected earlier than with two controls B. Analytical accuracy needs to be greater than for other analytes C. High, normal, and low ranges must always be evaluated D. A different level is needed for pH, PCO2, and PO2

Correct Answer: A 34. A Error detection occurs sooner when more controls are used. Some errors, such as those resulting from temperature error and protein coating of electrodes, are not as pronounced near the calibration point, as in the acidosis and alkalosis range. The minimum requirement for blood gas QC is one sample every 8 hours and three levels (acidosis, normal, alkalosis) every 24 hours. Three levels of control are also used commonly for therapeutic drug monitoring and hormone assays because precision differs significantly in the high and low ranges.

Which specimen is the sample of choice for lead screening? A. Whole blood B. Hair C. Serum D. Urine

Correct Answer: A 34. A Lead accumulates in RBCs, bones, and neural tissues, and whole blood, hair, and urine are suitable for demonstrating lead toxicity. Greatest sensitivity is obtained by using whole blood, which can detect exposure over time. Because lead is rapidly eliminated from plasma, serum or plasma should not be used to test for lead exposure. Lead binds to sulfhydryl groups of proteins such as delta-aminolevulinic acid (Δ-ALA) dehydratase and ferrochelatase and interferes with heme synthesis. This results in increased free erythrocyte protoporphyrin, erythrocyte zinc protoporphyrin, urinary coproporphyrin III, and δ aminolevulinic acid, which are also useful markers for lead poisoning. When screening for lead poisoning in children, the method of choice is graphite furnace atomic absorption spectrophotometry or inductively coupled plasma mass spectroscopy because they offer the best analytical sensitivity. The CDC cutoff for normal lead in children is less than 5.0 μg/dL.

A patient has a plasma CK-MB of 14 μg/L at admission and a total CK of 170 IU/L. Serum myoglobin is 130 μg/L and TnI is 1.6 μg/L. Three hours later, the TnI is 3.0 μg/L. Which statement best describes this situation? A. This patient has had an AMI and further testing is unnecessary B. A second CK-MB and myoglobin test should have been performed at 3 hours postadmission to confirm AMI C. These results are consistent with skeletal muscle damage associated with a crush injury that elevated the CK-MB D. Further testing 6-12 hours postadmission is required to establish a diagnosis of AMI

Correct Answer: A 34. A Results on admission indicate strongly that the patient has suffered an MI. The 3-hour TnI confirms this and rules out the possibility of a sample collection or transcription error for the admission sample. Repeat testing of other cardiac markers at 3 hours was not necessary because admission results were significantly increased for all three markers. Skeletal muscle damage or crush injury does not cause an increase in cardiac TnI.

The following results are reported on an adult male patient being evaluated for chest pain: Myoglobin (Cutoff = 100 μg/L); Troponin I (Cutoff = 0.03 μg/L); CK-MB (Cutoff = 4 μg/L) -Admission: 12 μg/L; 1.1 μg/L; 18 μg/L -3 hours: 360 μg/L; 1.8 μg/L; 26 μg/L -6 hours postadmission: 300 μg/L; 2.4 μg/L; 40 μg/L What is the most likely cause of these results? A. The wrong sample was assayed for the first myoglobin B. The patient did not suffer an MI until after admission C. Hemolysis caused interference with the 3-hour and 6-hour myoglobin result D. The patient is experiencing unstable angina

Correct Answer: A 35. A Myoglobin is the first cardiac marker to rise outside the URL following an MI (2-3 hours) followed by TnI (4-6 hours) and CK-MB (4-8 hours). The admission TnI and CK-MB are both elevated, and they continue to rise in all three samples. Because TnI and CK-MB peak before 24 hours post-AMI, the infarction likely occurred within the last 12-24 hours. The myoglobin can remain elevated for up to 36 hours post-AMI and should have been elevated in the admission sample.

Which uric acid method is associated with negative bias caused by reducing agents? A. Uricase coupled to the Trinder reaction B. Ultraviolet uricase reaction coupled to catalase and alcohol dehydrogenase reactions C. Measurement of the rate of absorbance decrease at 290 nm after addition of uricase D. Phosphotungstic acid using a protein-free filtrate

Correct Answer: A 35. A The peroxidase-coupled uricase reaction is the most common method for measuring uric acid in serum or plasma. Uricase methods form allantoin, carbon dioxide, and hydrogen peroxide from the oxidation of uric acid. When peroxide is used to oxidize a Trinder dye (e.g., a phenol derivative and 4-aminoantipyrine), some negative bias may occur when high levels of ascorbate or other reducing agents are present. Rate UV methods are free from this interference. Reduction of phosphotungstic acid by uric acid forms tungsten blue. This colorimetric reaction is nonspecific, resulting in falsely elevated uric acid caused by proteins and many other reducing substances.

The D-xylose absorption test is used for the differential diagnosis of which two diseases? A. Pancreatic insufficiency from malabsorption B. Primary from secondary disorders of glycogen synthesis C. Type 1 and type 2 diabetes mellitus D. Generalized from specific carbohydrate intolerance

Correct Answer: A 35. A Xylose is a pentose that is absorbed without the help of pancreatic enzymes and is not metabolized. In normal adults, more than 25% of the dose is excreted into the urine after 5 hours. Low blood or urine levels are seen in malabsorption syndrome, sprue, Crohn's disease, and other intestinal disorders, but not pancreatitis.

Which of the following statements about carbohydrate intolerance is true? A. Galactosemia results from deficiency of galactose-1-phosphate (galactose-1-PO4) uridine diphosphate transferase B. Galactosemia results in a positive glucose oxidase test for glucose in urine C. Urinary galactose is seen in both galactosemia and lactase deficiency D. A galactose tolerance test is used to confirm a diagnosis of galactosemia

Correct Answer: A 36. A Galactose is metabolized to galactose-1-PO4 by the action of galactokinase. Galactose-1-PO4 uridine diphosphate (UDP) transferase converts galactose-1-PO4 to glucose. Deficiency of either enzyme causes elevated blood and urine galactose. Lactase deficiency results in the presence of urinary lactose because it is not broken down to glucose and galactose. Tests for reducing sugars employing copper sulfate are used to screen for galactose, lactose, and fructose in urine. Nonglucose-reducing sugars are not detected by the glucose oxidase reaction. A positive test is followed by TLC to identify the sugar, and demonstration of the enzyme deficiency in RBCs. The galactose tolerance test is used (rarely) to evaluate the extent of liver failure since the liver is the site of galactose metabolism.

Which of the following statements regarding paraproteins is true? A. Oligoclonal banding is seen in the CSF of greater than 90% of multiple sclerosis cases B. The Bence-Jones protein heat test is confirmatory for monoclonal light chains C. Light chains found in urine are always derived from monoclonal protein D. The IgA band is usually cathodal to the IgG precipitin band

Correct Answer: A 36. A The α heavy chain is more acidic than γ or μ chains, giving IgA a greater net negative charge at alkaline pH. The IgA precipitin band is anodal to the IgG or IgM band. In hepatic cirrhosis, the β-γ bridging observed on serum protein electrophoresis results from increased IgA. Light chains in the form of Fab fragments are often found in increased amounts in the urine of patients with polyclonal gammopathies, especially from patients with an autoimmune disease. These can cause a positive Bence-Jones test and will produce a polyclonal (spread-out) appearance on IFE gels.

When measuring trace metals in blood other than lead, what type of tube should be used? A. Navy blue top B. Green top C. Purple top D. Red top

Correct Answer: A 38. A In order to avoid trace contamination by metals present in the stopper lubricants, a tube with a navy blue top is used for measuring trace metals. These tubes are validated for most but not all trace metals. Such tubes are available with or without EDTA for serum or whole-blood analysis, respectively. Tubes with tan stoppers containing EDTA are used for lead assay because they are certified to contain no more than 0.25 μg/dL lead. In addition, type 1 purity water (10 Mohm, 10 or less CFU/mL) and analytical reagent grade chemicals are always used to prepare reagents such as matrix modifiers. Although most trace metals are measured in whole blood or serum, arsenic is usually measured in urine because it is metabolized and excreted within hours of ingestion.

Which of the following statistical tests is used to compare the means of two methods? A. Student's t test B. F distribution C. Correlation coefficient (r) D. Linear regression analysis

Correct Answer: A 38. A Student's t test is the ratio of mean difference to the standard error of the mean difference (bias/random error) and tests for a significant difference in means. The F test is the ratio of variances and determines if one method is significantly less precise. The correlation coefficient is a measure of the association between two variables and should be high in any method comparison. An r value less than 0.90 in method comparisons usually occurs when the range of results is too narrow.

Which of the following is associated with low serum iron and high total iron-binding capacity (TIBC)? A. Iron deficiency anemia B. Hepatitis C. Nephrosis D. Noniron deficiency anemias

Correct Answer: A 39. A Iron-deficiency anemia is the principal cause of low serum iron and high TIBC because it promotes increased transferrin. Pregnancy without iron supplementation depletes maternal iron stores and also results in low serum iron and high TIBC. Iron-supplemented pregnancy and use of contraceptives increase both iron and TIBC. Nephrosis causes low iron and TIBC due to loss of both iron and transferrin by the kidneys. Hepatitis causes increased release of storage iron, resulting in high levels of iron and transferrin. Noniron deficiency anemias may cause high iron and usually show low TIBC and normal or high ferritin.

Alcoholism, liver failure, and hypoxia induce acidosis by causing: + A. Depletion of cellular NAD B. Increased excretion of bicarbonate C. Increased retention of PCO2 D. Loss of carbonic anhydrase

Correct Answer: A 39. A Oxygen debt and liver failure block oxidative phosphorylation, preventing NADH from being oxidized back to NAD+. Oxidation of ethanol to acetate results in accumulation of NADH. When NAD+ is depleted, glycolysis cannot proceed. It is regenerated by reduction of pyruvate to lactate, causing lactate acidosis.

Two freezing point osmometers are compared by running 40 paired patient samples one time on each instrument, and the following results are obtained: If the critical value for F = 2.8, then what conclusion can be drawn regarding the precision of the two instruments? Standard Instrument; Mean; Standard Deviation -Osmometer A; 280 mOsm/kg; 3.1 -Osmometer B; 294 mOsm/kg; 2.8 A. There is no statistically significant difference in precision B. Osmometer A demonstrates better precision that is statistically significant C. Osmometer B demonstrates better precision that is statistically significant D. Precision cannot be evaluated statistically when single measurements are made on samples

Correct Answer: A 39. A The F test determines whether there is a statistically significant difference in the variance of the two sampling distributions. Assuming the samples are collected and stored in the same way and the analysis is done by a technologist who is familiar with the instrument, then differences in variance can be attributed to a difference in instrument precision. The F test is calculated by dividing the variance (s1)2 of the instrument having the higher standard deviation by the variance (s2)2 of the instrument having the smaller standard deviation. F = (s1)2 ÷ (s2)2 = (3.1)2 ÷ (2.8)2 = 9.61 ÷7.84 = 1.22 If the value of F is smaller than the critical value at the 0.10 level of significance, then the hypothesis (there is no significant difference in the variance of the two instruments) is accepted.

Which of the following formulas is the correct expression for creatinine clearance? A. Creatinine clearance = U/P X V X 1.73/A B. Creatinine clearance = P/V X U X A/1.73 C. Creatinine clearance = P/V X U X 1.73/A D. Creatinine clearance = U/V X P X 1.73/A

Correct Answer: A 4. A Clearance is the volume of plasma that contains the same quantity of substance that is excreted in the urine in 1 minute. Creatinine clearance is calculated as the ratio of urine creatinine to plasma creatinine in milligrams per deciliter. This is multiplied by the volume of urine produced per minute and corrected for lean body mass by multiplying by 1.73/A, where A is the patient's body surface area in square meters. Separate reference ranges are needed for males, females, and children because each has a different percentage of lean muscle mass.

Zollinger-Ellison (Z-E) syndrome is characterized by great (e.g., 20-fold) elevation of: A. Gastrin B. Cholecystokinin C. Pepsin D. Glucagon

Correct Answer: A 4. A Z-E syndrome is caused by a pancreatic or intestinal tumor secreting gastrin (gastrinoma), and results in greatly increased gastric acid production. A serum gastrin level 10-fold greater than the URL in a person with hyperacidity and stomach or duodenal ulcers is diagnostic. Confirmation of gastric hyperacidity is demonstrated using the basal acid output (BAO) test.

Which of the following is the primary mechanism causing respiratory alkalosis? A. Hyperventilation B. Deficient alveolar diffusion C. Deficient pulmonary perfusion D. Parasympathetic inhibition

Correct Answer: A 40. A Hyperventilation via stimulation of the respiratory center (or induced by a respirator) is the mechanism of respiratory alkalosis. Causes include low PO2, anxiety, fever, and drugs that stimulate the respiratory center. Acute respiratory alkalosis is often uncompensated because renal compensation is not rapid. Uncompensated respiratory alkalosis is characterized by an elevated pH and a low PCO2 with normal bicarbonate.

Which of the electrodes below is a current- producing (amperometric) rather than a voltage-producing (potentiometric) electrode? A. Clark electrode B. Severinghaus electrode C. pH electrode D. Ionized calcium electrode

Correct Answer: A 40. A The Clark electrode is composed of two half cells that generate current, not voltage. It is used to measure partial pressure of oxygen (PO2), and is based upon an amperometric method called polarography. When -0.8 V is applied to the cathode, O2 is reduced, causing current to flow. Current is proportional to the PO2 of the sample.

Capillary electrophoresis differs from agarose gel electrophoresis in which respect? A. A stationary support is not used B. An acidic buffer is used C. A low voltage is used D. Electroendosmosis does not occur

Correct Answer: A 41. A Capillary electrophoresis is a rapid automated procedure for separating serum or body fluid proteins. Instead of a stationary support, the proteins migrate based upon their charge/mass ratio inside a small-bore silica capillary tube (20-200 μm). The cations in the buffer are attracted to the negatively charged silicates and migrate to the cathode rapidly when voltage is applied. The electroendosmotic force created moves the proteins toward the cathode and they are detected by an in-line UV photometer that measures their absorbance. High voltage (e.g., 9,000 volts) is used to effect separation of serum proteins in an 8-10 minute run, giving resolution equal to or greater than HR agarose gel electrophoresis.

Following ultracentrifugation of plasma, which fraction correlates with pre-β lipoprotein? A. Very low-density lipoprotein (VLDL) B. Low-density lipoprotein (LDL) C. High-density lipoprotein (HDL) D. Chylomicrons

Correct Answer: A 43. A The VLDL (very low-density lipoprotein) migrates in the pre-β zone. The VLDL is about 50% triglyceride, whereas LDL is only 10% triglyceride by weight. LDL is formed from VLDL in the circulation. The process is initiated by apoC-II on VLDL activating peripheral lipoprotein lipase. Hydrolysis of triglycerides and transfer of apoproteins from VLDL to HDL result in formation of IDL. Larger IDLs are returned to the liver as remnant lipoproteins. Further hydrolysis of triglycerides, transfer cholesterol esters from HDL, and transfer of apoproteins to HDL convert IDL to LDL.

Serum samples collected from hospitalized patients over a 2-week period are split into two aliquots and analyzed for prostate specific antigen (PSA) by two methods. Each sample was assayed by both methods within 30 minutes of collection by a technologist familiar with both methods. The reference method is method × (upper reference limit = 4.0 μg/L). Linear regression analysis was performed by the least-squares method, and results are as follows: -Linear Regression: ŷ = 2.10 + 1.01x -Correlation Coefficient (r): 0.984 -Standard Error of Regression Estimate (sy/x): 0.23 Which statement best characterizes the relationship between the methods? A. There is a significant bias caused by constant error B. There is a significant proportional error C. There is no disagreement between the methods because the correlation coefficient approaches 1.0 D. There is no systematic error, but the random error of the new method is unacceptable

Correct Answer: A 43. A The linear regression analysis is the most useful statistic to compare paired patient results because it estimates the magnitude of specific errors. The y intercept of the regression line is a measure of constant error, and the slope is a measure of proportional error. Together, these represent the bias or SE of the new method. The correlation coefficient is influenced by the range of the sample and the RE. Two methods that measure the same analyte will have a high correlation coefficient, provided the concentrations are measured over a wide range, and this statistic should not be used to judge the acceptability of the new method. The standard error of estimate is a measure of the closeness of data points to the regression line and is an expression of RE.

The serum TSH level is almost absent in: A. Primary hyperthyroidism B. Primary hypothyroidism C. Secondary hyperthyroidism D. Euthyroid sick syndrome

Correct Answer: A 44. A Low TSH and a high T3 (and usually T4) occur in primary hyperthyroidism, but may also occur in systemic nonthyroid illnesses where T4 has been converted to T3. A 2-fold increase in free hormone can produce a 100-fold decrease in TSH. In primary hyperthyroidism, the TSH will be within a range of 0-0.02 mU/mL, while in nonthyroid illnesses it will be 0.03 mU/mL or higher. A high TSH and low T4 occur in primary hypothyroidism but can also occur in an acutely ill patient without thyroid disease, the euthyroid sick syndrome. Secondary hyperthyroidism is caused by pituitary hyperfunction, resulting in increased serum TSH.

Which of the statements below regarding the methods of Henry for AST and ALT is correct? A. Hemolysis will cause positive interference in both AST and ALT assays B. Loss of activity occurs if samples are frozen at -20°C C. The absorbance at the start of the reaction should not exceed 1.0 A D. Reaction rates are unaffected by addition of P-5 ́-P to the substrate

Correct Answer: A 44. A RBCs are rich in AST and to a lesser extent in ALT. Hemolysis causes positive interference in both assays, although the effect on AST is greater. Samples are stable for up to 24 hours at room temperature and up to 3 days at 4°C, and should be frozen if kept longer. The starting absorbance should be at least 1.5 A for both assays. Substrates with lower concentrations of NADH are subject to NADH depletion during the lag phase due to side reactions or high transaminase activity. When P-5'-P is added, a significant increase in activity sometimes occurs because some of the enzyme in the serum is in the inactive apoenzyme form.

An AFP measured on a 30-year-old pregnant woman at approximately 12 weeks gestation is 2.5 multiples of the median (MOM). What course of action is most appropriate? A. Repeat the serum AFP in 2 weeks B. Recommend AFP assay on amniotic fluid C. Repeat the AFP using the same sample by another method D. Repeat the AFP using the sample by the same method

Correct Answer: A 45. A The analytical sensitivity of immunochemical AFP tests is approximately 5 ng/mL. The maternal serum AFP at 12 weeks' gestation is barely above the analytical detection limit. Therefore, to achieve the needed sensitivity, the test should be repeated at 14 weeks. If the result is still equal to or greater than 2.5 MOM, then ultrasound should be performed to verify last menstrual period dating. AFP normally first becomes detectable in maternal serum at week 12 and increases by 15% per week through the 26th week. Levels of 2.5 MOM or greater are associated with spina bifida but also occur in ventral wall and abdominal wall defects, fetal death, Turner's syndrome, trisomy 13, congenital hypothyroidism, tyrosinemia, and several other fetal conditions. A positive serum test should always be repeated, and if positive again, followed by ultrasound. If ultrasound does not explain the elevation, amniotic fluid testing including AFP and acetylcholinesterase is usually recommended.

Which of the following statements regarding the TIBC assay is correct? A. All TIBC methods require addition of excess iron to saturate transferrin B. All methods require the removal of unbound iron C. Measurement of TIBC is specific for transferrin- bound iron D. The chromogen used must be different from the one used for measuring serum iron

Correct Answer: A 46. A All TIBC methods require addition of excess iron to saturate transferrin. Excess iron is removed by ion exchange or alumina gel columns or precipitation with MgCO3 and the bound iron is measured by the same procedure as is used for serum iron. Alternatively, excess iron in the reduced state can be added at an alkaline pH. Under these conditions, transferrin will bind Fe2+ and the unbound Fe2+ can be measured directly.

Which apoprotein is inversely related to risk of coronary heart disease? A. Apoprotein A-I B. Apoprotein B100 C. Apoprotein C-II D. Apoprotein E4

Correct Answer: A 46. A Apoprotein A-I and apo A-II are the principal apoproteins of HDL, and low apo A-I has a high correlation with atherosclerosis. Conversely, apo-B100 is the principal apoprotein of LDL, and an elevated level is a major risk factor in developing coronary heart disease. Apoprotein assays are not recommended as screening tests because they are not as well standardized as LDL cholesterol assays. However, apo-B100 assay is more sensitive than LDL cholesterol in predicting coronary artery disease risk. Apo-B100 may be abnormal in persons with increased small dense LDL. Small dense LDL is more atherogenic than large LDL molecules. In addition, persons with hyperapobetalipoproteinemia overproduce apo-B100 without having significantly elevated LDL cholesterol.

Which of the following is a marker for bone formation? A. Osteocalcin B. Tartrate resistant acid phosphatase (TRAP) C. Urinary pyridinoline and deoxypyridinoline D. Urinary C-telopeptide and N-telopeptide crosslinks (CTx and NTx)

Correct Answer: A 46. A Biochemical markers for osteoporosis are classified as either markers for bone formation or resorption. Osteocalcin is a protein hormone that stimulates osteoblasts and increases bone mineralization. Pyridinoline is formed when hydroxylysine groups on adjacent fibrils are joined together, and deoxypyridinoline when hydroxylysine and lysine groups are joined. These form crosslinks between the C and N terminal ends of one fibril (which are nonhelical) and the helical portion of an adjacent fibril. The resulting products are called C- and N-telopeptide crosslinks of type 1 collagen. Osteoclasts cause cleavage of these bonds, resulting in loss of both telopeptides—deoxypyridinoline and pyridinoline—in the urine. TRAP is an enzyme (produced by osteoclasts) that hydrolyzes phosphate in the hydroxyapatite matrix of the bone.

Which of the following compounds can interfere with the coulometric chloride assay? A. Bromide B. Ascorbate C. Acetoacetate D. Nitrate

Correct Answer: A 47. A Chloride assays based upon either coulometric or chemical titration are subject to positive interference from other anions and electronegative radicals that may be titrated instead of chloride ions. These include other halogens such as bromide, cyanide, and cysteine.

Bilirubin is transported from reticuloendothelial cells to the liver by: A. Albumin B. Bilirubin-binding globulin C. Haptoglobin D. Transferrin

Correct Answer: A 48. A Albumin transports bilirubin, haptoglobin transports free Hgb, and transferrin transports ferric iron. When albumin binding is exceeded, unbound bilirubin, called free bilirubin, increases. This may cross the blood-brain barrier, resulting in kernicterus.

A patient presents to the emergency department with symptoms of intoxication including impaired speech and movement. The plasma osmolality was measured and found to be 330 mOs/kg. The osmolal gap was 40 mOsm/Kg. A blood alcohol was measured by the alcohol dehydrogenase method and found to be 0.15% w/v (150 mg/dL). Electrolyte results showed an increased anion gap. Ethylene glycol intoxication was suspected because the osmolal gap was greater than could be explained by ethanol alone, but gas chromatography was not available. Which of the following would be abnormal if this suspicion proved correct? A. Arterial blood gases B. Lactic acid C. Urinary ketones D. Glucose

Correct Answer: A 48. A Ethylene glycol is sometimes used as a substitute for ethanol by alcoholics. It is metabolized to formic acid and glycolic acid by the liver, resulting in metabolic acidosis and an increased anion gap. Lactic acid, glucose, and urinary ketones would be useful in ruling out other causes of metabolic acidosis, but would not be abnormal as a result of ethylene glycol intoxication.

All of the following compounds contribute to the osmolality of plasma except: A. Lipids B. Creatinine C. Drug metabolites D. Glucose

Correct Answer: A 48. A Osmolality is the concentration (in moles) of dissolved solute per kilogram solvent. Proteins and lipids are not in solution, and do not contribute to osmolality. The nonionized solutes such as glucose and urea contribute 1 osmole per mole per kilogram water, whereas dissociated salts contribute 1 osmole per mole of each dissociated ion or radical.

What measurement in addition to true negatives and prevalence is required to calculate the predictive value of a negative test result (PV-)? A. False negatives B. Variance C. True positives D. False positives

Correct Answer: A 49. A The PV- is defined as the probability that a person with a negative test result is free of disease. A high PV- is a characteristic of a good screening test. The predictive value of a negative test is calculated by multiplying the true negatives by 100, then dividing by the sum of the true negatives and false negatives. %PV- = TN × 100 / TN + FN

Which substrate concentration is needed to achieve zero-order conditions? A. Greater than 99 × Km B. [S] = Km C. Less than 10 × Km D. [S] = 0

Correct Answer: A 5. A A zero-order reaction rate is independent of substrate concentration because there is sufficient substrate to saturate the enzyme. V = Vmax × [S]/Km + [S] where V = velocity, Vmax = maximum velocity, [S] = substrate concentration, and Km = substrate concentration required to give 1/2 Vmax. If [S] >>> Km, then the Km can be ignored. V = Vmax × [S]/[S] = Vmax × [S]° or velocity approaches maximum and is independent of substrate concentration.

Which statement about multiple endocrine neoplasia (MEN) is true? A. It is associated with hyperplasia or neoplasia of at least two endocrine organs B. Insulinoma is always present when the pituitary is involved C. It is inherited as an autosomal recessive disorder D. Plasma hormone levels from affected organs are elevated at least 10-fold

Correct Answer: A 5. A Multiple-endocrine neoplasia syndrome is inherited as an autosomal dominant disease involving excess production of hormones from several endocrine glands. MEN I results from adenomas (usually benign) of at least two glands, including the pituitary, adrenal cortex, parathyroid, and pancreas. The parathyroid gland is the organ most commonly involved, and in those patients an elevated Cai is an early sign. The pancreas is the next most frequently involved organ, but the hormone most commonly oversecreted is gastrin (not insulin). MEN II is characterized by pheochromocytoma and thyroid carcinoma. MEN II-B is a variant of MEN II showing the addition of neurofibroma.

Which of the following statements regarding proteins is true? A. Total protein and albumin are about 10% higher in ambulatory patients B. Plasma total protein is about 20% higher than serum levels C. Albumin normally accounts for about one-third of the cerebrospinal fluid total protein D. Transudative serous fluid protein is about two-thirds of the serum total protein

Correct Answer: A 5. A Water pools in the vascular bed in nonambulatory patients, lowering the total protein, albumin, hematocrit, and calcium. Plasma levels of total protein are 0.2-0.4 g/dL higher than serum (about 5%) owing to fibrinogen. Cerebrospinal fluid albumin levels are normally 10-30 mg/dL, which is approximately two-thirds of the CSF total protein. Transudates have a total protein below 3.0 g/dL and less than 50% of the serum total protein.

Which of the following conditions is most consistently associated with secondary hypercholesterolemia? A. Hypothyroidism B. Pancreatitis C. Oral contraceptive therapy D. Diabetes mellitus

Correct Answer: A 50. A The conditions listed are very commonly encountered causes of secondary hyperlipoproteinemia. Oral contraceptives, pregnancy, and estrogens may cause secondary hypertriglyceridemia owing to increased VLDL and endogenous triglycerides. Hypothyroidism and obstructive hepatobiliary diseases are usually associated with secondary hypercholesterolemia owing to high LDL. Diabetes mellitus and chronic pancreatitis may produce hypertriglyceridemia, chylomicronemia, or mixed hyperlipidemia.

Which of the following is a characteristic of conjugated bilirubin? A. It is water soluble B. It reacts more slowly than unconjugated bilirubin C. It is more stable than unconjugated bilirubin D. It has the same absorbance properties as unconjugated bilirubin

Correct Answer: A 50. B UDP-glucuronyl transferase esterifies glucuronic acid to unconjugated bilirubin, making it water soluble. Most conjugated bilirubin is diglucuronide; however, the liver makes a small amount of monoglucuronide and other glycosides. β-Glucuronidase hydrolyzes glucuronide from bilirubin, hormones, or drugs. It is used prior to organic extraction to deconjugate urinary metabolites (e.g., total cortisol). Biliverdin reductase forms bilirubin from biliverdin (and heme oxygenase forms biliverdin from heme). Bilirubin oxidase is used in an enzymatic bilirubin assay in which bilirubin is oxidized back to biliverdin and the rate of biliverdin formation is measured at 410 nm.

Which of the following statements regarding urobilinogen is true? A. It is formed in the intestines by bacterial reduction of bilirubin B. It consists of a single water-soluble bile pigment C. It is measured by its reaction with p-aminosalicylate D. In hemolytic anemia, it is decreased in urine and feces

Correct Answer: A 50. B UDP-glucuronyl transferase esterifies glucuronic acid to unconjugated bilirubin, making it water soluble. Most conjugated bilirubin is diglucuronide; however, the liver makes a small amount of monoglucuronide and other glycosides. β-Glucuronidase hydrolyzes glucuronide from bilirubin, hormones, or drugs. It is used prior to organic extraction to deconjugate urinary metabolites (e.g., total cortisol). Biliverdin reductase forms bilirubin from biliverdin (and heme oxygenase forms biliverdin from heme). Bilirubin oxidase is used in an enzymatic bilirubin assay in which bilirubin is oxidized back to biliverdin and the rate of biliverdin formation is measured at 410 nm.

Which of the following is the most accurate measurement of Pi in serum? A. Rate of unreduced phosphomolybdate formation at 340 nm B. Measurement of phosphomolybdenum blue at 680 nm C. Use of aminonaptholsulfonic acid to reduce phosphomolybdate D. Formation of a complex with malachite green dye

Correct Answer: A 51. A The colorimetric method (Fiske and SubbaRow) used previously for Pi reacted ammonium molybdate with Pi, forming ammonium phosphomolybdate (NH4)3[PO4(MoO3)12]. A reducing agent, aminonaptholsulfonic acid (ANS), was added, forming phosphomolybdenum blue. The product was unstable and required sulfuric acid, making precipitation of protein a potential source of error. These problems are avoided by measuring the rate of formation of unreduced phosphomolybdate at 340 nm.

Which statement regarding bilirubin metabolism is true? A. Bilirubin undergoes rapid photo-oxidation when exposed to daylight B. Bilirubin excretion is inhibited by barbiturates C. Bilirubin excretion is increased by chlorpromazine D. Bilirubin is excreted only as the diglucuronide

Correct Answer: A 55. A Samples for bilirubin analysis must be protected from direct sunlight. Drugs may have a significant in vivo effect on bilirubin levels. Barbiturates lower serum bilirubin by increasing excretion. Other drugs that cause cholestasis, such as chlorpromazine, increase the serum bilirubin. Although most conjugated bilirubin is in the form of diglucuronide, some monoglucuronide and other glycosides are excreted. In glucuronyl transferase deficiency, some bilirubin is excreted as sulfatides.

Which of the following is a characteristic of obstructive jaundice? A. The ratio of direct to total bilirubin is greater than 1:2 B. Conjugated bilirubin is elevated, but unconjugated bilirubin is normal C. Urinary urobilinogen is increased D. Urinary bilirubin is normal

Correct Answer: A 58. A Obstruction prevents conjugated bilirubin from reaching the intestine, resulting in decreased production, excretion, and absorption of urobilinogen. Conjugated bilirubin regurgitates into sinusoidal blood and enters the general circulation via the hepatic vein. The level of serum direct (conjugated) bilirubin becomes greater than unconjugated bilirubin. The unconjugated form is also increased because of accompanying necrosis, deconjugation, and inhibition of UDP-glucuronyl transferase

Which of the following would cause an increase in only the unconjugated bilirubin? A. Hemolytic anemia B. Obstructive jaundice C. Hepatitis D. Hepatic cirrhosis

Correct Answer: A 59. A Conjugated bilirubin increases as a result of obstructive processes within the liver or biliary system or from failure of the enterohepatic circulation. Hemolytic anemia (prehepatic jaundice) presents a greater bilirubin load to a normal liver, resulting in increased bilirubin excretion. When the rate of bilirubin formation exceeds the rate of excretion, the unconjugated bilirubin rises.

SITUATION: A technologist is performing an enzyme assay at 340 nm using a visible-range spectrophotometer. After setting the wavelength and adjusting the readout to zero %T with the light path blocked, a cuvette with deionized water is inserted. With the light path fully open and the 100%T control at maximum, the instrument readout will not rise above 90%T. What is the most appropriate first course of action? A. Replace the source lamp B. Insert a wider cuvette into the light path C. Measure the voltage across the lamp terminals D. Replace the instrument fuse

Correct Answer: A 6. A Visible spectrophotometers are usually supplied with a tungsten or quartz halogen source lamp. Tungsten lamps produce a continuous range of wavelengths from about 320-2,000 nm. Output increases as wavelength becomes longer peaking at around 1,000 nm, and is poor below 400 nm. As the lamp envelope darkens with age, the amount of light reaching the photodetector at 340 nm becomes insufficient to set the blank reading to 100%T. Quartz halogen lamps produce light from 300 nm through the infrared region. Deuterium or hydrogen lamps produce ultraviolet-rich spectra optimal for ultraviolet (UV) work. Mercury vapor lamps produce a discontinuous spectrum that includes a high output at around 365 nm that is useful for fluorescent applications. Xenon lamps generate a continuous spectrum of fairly uniform intensity from 300-2,000 nm, making them useful for both visible and UV applications.

In which condition would an elevated serum alkaline phosphatase be likely to occur? A. Small cell lung carcinoma B. Hemolytic anemia C. Prostate cancer D. Acute myocardial infarction

Correct Answer: A 61. A The primary diagnostic utility of ALP is to help differentiate necrotic jaundice (↑ALT) from obstructive jaundice (↑ ALP). ALP is also increased in several bone diseases. Large increases are seen in Paget's disease, moderate increases in bone cancer, and slight increases in rickets. Total ALP may be slightly increased in osteoporosis but often it is not. In addition to obstructive jaundice and bone diseases, alkaline phosphatase is a tumor marker. In most cases, the alkaline phosphatase is the product of fetal gene activation, and resembles placental ALP (e.g., hepatoma, small cell carcinoma of the lung, ovarian cancer). Leukemia and Hodgkin's disease may cause an elevated leukocyte or bone-derived ALP.

Which substrate is used in the Bowers-McComb method for ALP? A. p-Nitrophenyl phosphate B. β-Glycerophosphate C. Phenylphosphate D. α-Naphthylphosphate

Correct Answer: A 63. A The method of Bowers-McComb (Szasz modification) is the IFCC-recommended method for ALP. This method uses 2-amino-2-methyl-1-propanol, pH 10.15, and measures the increase in absorbance at 405 nm as p-nitrophenyl phosphate is hydrolyzed to p-nitrophenol.

What is the purpose of an internal standard in HPLC and GC methods? A. To compensate for variation in extraction and injection B. To correct for background absorbance C. To compensate for changes in flow rate D. To correct for coelution of solutes

Correct Answer: A 64. A Internal standards should have the same affinity as the analyte for the extraction reagents. Dividing peak height (or area) of all samples (standards and unknowns) by the peak height (or area) of the internal standard reduces error caused by variation in extraction recovery and injection volume.

Which of the following conditions is associated with total body sodium excess? A. Renal failure B. Hyperthyroidism C. Hypoparathyroidism D. Diabetic ketoacidosis

Correct Answer: A 64. A Total body sodium excess often occurs in persons with renal failure, congestive heart failure, and cirrhosis of the liver. When water is retained along with sodium, total body sodium excess results rather than hypernatremia. Heart failure causes sodium and water retention by reducing blood flow to the kidneys. Cirrhosis causes obstruction of hepatic lymphatics and portal veins, leading to local hypertension and accumulation of ascites fluid. Renal failure results in poor glomerular filtration and isosmotic equilibration of salt and water.

Which of the following conditions is associated with hyponatremia? A. Diuretic therapy B. Cushing's syndrome C. Diabetes insipidus D. Nephrotic syndrome

Correct Answer: A 65. A Diuretics lower blood pressure by promoting water loss. This is accomplished by causing sodium loss from the proximal tubule and/or loop. Addison's disease, syndrome of inappropriate ADH release, burns, diabetic ketoacidosis, hypopituitarism, vomiting, diarrhea, and cystic fibrosis also cause hyponatremia. Cushing's syndrome causes hypernatremia by promoting sodium reabsorption in the collecting tubule in exchange for potassium. Diabetes insipidus and nephrotic syndrome promote hypernatremia by causing water loss.

Which definition best describes the catalytic activity of amylase? A. Hydrolyzes second α 1-4 glycosidic linkages of starch, glycogen, and other polyglucans B. Hydrolyzes all polyglucans completely to produce glucose C. Oxidatively degrades polysaccharides containing glucose D. Splits polysaccharides and disaccharides by addition of water

Correct Answer: A 67. A Amylase in humans is a hydrolase that splits the second α 1-4 glycosidic bonds of polyglucans forming maltose. There are two major types of amylase: P-type derived from the pancreas and S-type derived from the salivary glands. These can be differentiated by both electrophoresis and immunoassay. In healthy persons, the principal form in plasma is the salivary isoenzyme. There are several genetic variants of the salivary isoenzyme, which in part accounts for the broad reference range.

Which of the following laboratory results is usually associated with cystic fibrosis? A. Sweat chloride greater than 60 mmol/L B. Elevated serum sodium and chloride C. Elevated fecal trypsin activity D. Low glucose

Correct Answer: A 67. A Cystic fibrosis causes obstruction of the exocrine glands including the sweat glands, mucus glands, and pancreas. Newborns with pancreatic involvement demonstrate fecal trypsin deficiency, which may be detected by a low fecal chymotrypsin or immunoreactive trypsin result. However, these tests require confirmation. Serum sodium and chloride levels are low. More than 98% of affected infants have elevated sweat sodium and chloride and low serum levels. Sweat chloride in excess of 60 mmol/L confirms the clinical diagnosis. Some persons with the disease have insulin deficiency and elevated blood glucose. Genetic tests are available to detect several mutations that occur at the cystic fibrosis transmembrane conductance regulator (CFTR) locus on chromosome 7.

What process is most often used in LC-MS to introduce the sample into the mass filter? A. Electrospray ionization B. Chemical ionization C. Electron impact ionization D. Fast atom bombardment

Correct Answer: A 68. A HPLC instruments use solvent rather than gas to separate molecules. The sample is converted into a gaseous state by electrospray ionization before it enters the mass filter. Electrospray ionization uses a small-bore tube that forms a 1-4 μ nozzle at the mass filter inlet and which is charged by several kilovolts. The sample enters the tube along with inert drying gas. The tube is heated to help evaporate solvent, but unlike electron impact used in GC-MS, the ionizer is not under vacuum. When a droplet of the sample reaches the nozzle, it becomes highly charged. The size of the droplet is decreased owing to evaporation. This causes the charge density to become excessive, and the droplets break apart. The tiny charged droplets repel each other and break apart again, forming a plume. These particles are drawn into the mass filter by "ion optics" (a system of repeller plates, counter electrode, and magnets). ESI does not result in extensive fragmentation, producing mostly the parent or "molecular" ion, a process called soft ionization.

A neonatal bilirubin assay performed at the nursery by bichromatic direct spectrophotometry is 4.0 mg/dL. Four hours later, a second sample assayed for total bilirubin by the Jendrassik-Grof method gives a result of 3.0 mg/dL. Both samples are reported to be hemolyzed. What is the most likely explanation of these results? A. Hgb interference in the second assay B. δ-Bilirubin contributing to the result of the first assay C. Falsely high results from the first assay caused by direct bilirubin D. Physiological variation owing to premature hepatic microsomal enzymes

Correct Answer: A 68. A The Jendrassik-Grof method is based upon a diazo reaction that may be suppressed by Hgb. Because serum blanking and measurement at 600 nm correct for positive interference from Hgb, the results may be falsely low when significant hemolysis is present. Direct spectrophometric bilirubin methods employing bichromatic optics correct for the presence of Hgb. These are often called "neonatal bilirubin" tests. A commonly used approach is to measure absorbance at 454 nm and 540 nm. The absorbance contributed by Hgb at 540 nm is equal to the absorbance contributed by Hgb at 454 nm. Therefore, the absorbance difference will correct for free Hgb. Neonatal samples contain little or no direct δ-bilirubin. They also lack carotene pigments that could interfere with the direct spectrophotometric measurement of bilirubin.

High serum total protein but low albumin is usually seen in: A. Multiple myeloma B. Hepatic cirrhosis C. Glomerulonephritis D. Nephrotic syndrome

Correct Answer: A 7. A In multiple myeloma, synthesis of large quantities of monoclonal immunoglobulin by plasma cells often results in decreased synthesis of albumin. In glomerulonephritis and nephrotic syndrome, both total protein and albumin are low owing to loss of proteins through the glomeruli. In hepatic cirrhosis, decreased hepatic production of protein results in low total protein and albumin.

Which of the following statements about enzymatic reactions is true? A. NADH has absorbance maximas at 340 and 366 nm B. Enzyme concentration must be in excess to achieve zero-order kinetics C. Rate is proportional to substrate concentration in a zero-order reaction D. Accumulation of the product increases the reaction rate

Correct Answer: A 7. A Most enzymes are measured by monitoring the rate of absorbance change at 340 nm as NADH is produced or consumed. This rate will be proportional to enzyme activity when substrate is in excess. When the enzyme is present in excess, the initial reaction rate will be proportional to substrate concentration. This condition, called a first-order reaction, is needed when the enzyme is used as a reagent to measure a specific analyte.

In the Henderson-Hasselbalch expression pH = 6.1 + log HCO3-/dCO2, the 6.1 represents: A. The combined hydration and dissociation constants for CO2 in blood at 37°C B. The solubility constant for CO2 gas C. The dissociation constant of H2O D. The ionization constant of sodium bicarbonate (NaHCO3)

Correct Answer: A 7. A The equilibrium constant, Kh, for the hydration of CO2 (dCO2 + H2O → H2CO3) is only about 2.3 × 10-3M, making dCO2 far more prevalent than carbonic acid. The dissociation constant, Kd, for the reaction H2CO3 →H+ + HCO3- is about 2 × 10-4 M. The product of these constants is the combined equilibrium constant, K ́. The negative logarithm of K ́ is the pK ́, which is 6.103 in blood at 37°C.

The reference method for lipase uses olive oil as the substrate because: A. Other esterases can hydrolyze triglyceride and synthetic diglycerides B. The reaction product can be coupled to NADH generating reactions C. Synthetic substrates are less soluble than olive oil in aqueous reagents D. Triglyceride substrates cause product inhibition

Correct Answer: A 74. A Triglycerides may be hydrolyzed by nonspecific esterases in serum as well as lipase. Lipase acts only at an interface of oil and H2O and requires bile salts and colipase for activity. Colipase is a protein secreted by the pancreas.

Which statement about the clinical utility of plasma or serum lipase is true? A. Lipase is not increased in mumps, malignancy, or ectopic pregnancy B. Lipase is not increased as dramatically as amylase in acute pancreatitis C. Increased plasma or serum lipase is specific for pancreatitis D. Lipase levels are elevated in both acute and chronic pancreatitis

Correct Answer: A 75. A Lipase elevation is of greater magnitude (2-50 × N) and duration than amylase in acute pancreatitis. When the lipase method is optimized by inclusion of colipase and bile salts, the test is more sensitive and specific than serum amylase for detection of acute pancreatitis. However, lipase is also increased in peptic ulcers, renal insufficiency, and intestinal obstruction. Lipase levels are often low in chronic pancreatitis, and are low in cystic fibrosis.

Which component is needed for a thermal cycler to amplify DNA? A. Programmable heating and cooling unit B. Vacuum chamber with zero head space C. Sealed airtight constant-temperature chamber D. Temperature-controlled ionization chamber

Correct Answer: A 75. A The polymerase chain reaction for DNA amplification consists of three phases. Denaturation requires a temperature of 90°C-94°C and separates the double-stranded DNA. Annealing requires a temperature between 40°C-65°C and allows the primers to bind to the target base sequence. Extension requires a temperature of 72°C and allows the heat-stable polymerase to add complementary bases to the primer in the 5' to 3' direction. A cycle consists of each temperature stage for a specific number of minutes and most procedures require 30 or more cycles to generate a detectable quantity of target DNA. Rapid heating and cooling is usually achieved using a thermoelectric block that is cooled by forced air flow.

In real-time PCR, what value is needed in order to determine the threshold? A. Background signal B. Melt temperature C. Maximum fluorescence D. Threshold cycle

Correct Answer: A 76. A In real-time PCR, the fluorescence of the reporter probe is proportional to the concentration of PCR products. For quantitation of PCR products, a well factor and background fluorescence must be determined. Well-factor values are analogous to cuvette blanks. They are used to correct the measurements from each well so that the same concentration of fluorescent dye gives the same signal intensity regardless of the well. The threshold is the lowest signal that indicates the presence of product. It can be calculated manually from a real-time amplification curve by finding the average standard deviation of the fluorescent signal (RFU) from cycles 2-10. This is multiplied by 10 to give the threshold value in RFUs.

In addition to velocity, what variable is also needed to calculate the relative centrifugal force (g force) of a centrifuge? A. Head radius B. Angular velocity coefficient C. Diameter of the centrifuge tube D. Ambient temperature in degrees Centigrade

Correct Answer: A 78. A The relative centrifugal force (number times the force of gravity) is proportional to the square of the rotor speed in revolutions per minute and the radius in centimeters of the head (distance from the shaft to the end of the tube). RCF = s2 x r x 1.118 x 10-5 where s is the speed in RPM, r is the radius in cM, and 1.118 x 10-5 is a conversion constant.

Which formula can be used to estimate dosage needed to give a desired steady-state blood level? A. Dose per hour = clearance (milligrams per hour) × average concentration at steady state ÷ f B. Dose per day = fraction absorbed - fraction excreted C. Dose = fraction absorbed × (1/protein-bound fraction) D. Dose per day = half-life × log Vd (volume distribution)

Correct Answer: A 8. A After a patient receives a loading dose to rapidly bring the drug level up to the desired therapeutic range, a maintenance dose must be given at consistent intervals to maintain the blood drug level at the desired concentration. The dose per hour is determined by multiplying the clearance per hour by the desired average steady-state concentration, then dividing by f (bioavailable fraction).

Plate 8 shows the electrophoresis of Hgb samples on acid agar gel, pH 6.0. The sample order is the same as for plate 7 with the A, S, C control hemolysate in lanes 2 and 10. Based upon the electrophoretic mobility of sample 7 as seen in both plate 7 and plate 8, what is the patient's Hgb phenotype? A. SS B. AS C. AD D. AG

Correct Answer: A 8. A Sample 7 demonstrates one major band on plate 7 in the Hgb S position. Because Hgb A is not present, there is no normal β-gene, and the patient can be classified as a homozygote for Hgb S, D, or G which migrate to the same position on agarose gel at a pH between 8.4 and 9.2. Hgb S can be differentiated from Hgbs D and G by performing electrophoresis on agar gel at pH 6.0-6.2. On agar at acid pH, Hgb C migrates furthest toward the anode. Hgb S migrates toward the anode, but not as far as Hgb C. Hgb F migrates furthest toward the cathode, while Hgbs A, D, G, and E migrate to the same position, slightly cathodal to the point of application. On plate 8, sample 7 shows a single large band that migrated toward the anode at the same position as the S band in the control sample.

In addition to sodium bicarbonate, what other substance contributes most to the amount of base in the blood? A. Hemoglobin concentration B. Dissolved O2 concentration C. Inorganic phosphorus D. Organic phosphate

Correct Answer: A 9. A The primary blood buffer bases preventing acidosis in order of concentration are bicarbonate, deoxyhemoglobin, albumin, and monohydrogen phosphate. At physiological pH, there is significantly more H2PO4-1 than HPO4-2, and phosphate is a more efficient buffer system at preventing alkalosis than acidosis. Since all of the blood buffer systems are in equilibrium, the pH can be calculated accurately from the concentration of bicarbonate and dissolved CO2 using the Henderson-Hasselbalch equation.

A reciprocal relationship exists between: A. sodium and potassium B. calcium and phosphate C. chloride and CO2 D. calcium and magnesium

Correct Answer: B

Amperometry is the principle of the: A. PCO2 electrode B. PO2 electrode C. pH electrode D. Ionized calcium electrode

Correct Answer: B

Hydrogen ion concentration (pH) in blood is usually determined by means of which of the following electrodes? A. silver B. glass C. platinum D. platinum-lactate

Correct Answer: B

In a pleural effusion caused by Streptococcus pneumoniae, the protein value of the pleural fluid as compared to the serum value would probably be: A. decreased by 2 B. decreased by 1/2 C. increased by 1/2 D. equal

Correct Answer: B

In the fasting state, the arterial and capillary blood glucose concentration varies form the venous glucose concentration by approximately how many mg/dL (mmol/L)? A. 1 mg/dL (0.05 mmol/L) higher B. 5 mg/dL (0.27 mmol/L) higher C. 10 mg/dL (0.55 mmol/L) lower D. 15 mg/dL (0.82 mmol/L) lower

Correct Answer: B

In the hexokinase method for glucose determination, the actual end product measured is the: A. amount of hydrogenperoxide produced B. NADH produced from the reduction of NAD C. amount of glucose combined with bromcresol purple D. condensation of glucose with an aromatic amine

Correct Answer: B

Increased serum lactic dehydrogenase activity due to elevation of fast fractions (1 and 2) on electrophoretic separation is cause by: A. nephrotic syndrome B. hemolytic anemia C. pancreatitis D. hepatic damage

Correct Answer: B

On electrophoresis, transient bisalbuminemia or a grossly widened albumin zone is associated with: A. dirty applicators B. presence of therapeutic drugs in serum sample C. endomosis D. prestaining with tracer dye

Correct Answer: B

Reverse phase high-performance liquid chromatography is being increasingly utilized in therapeutic drug monitoring. The term reverse phase implies that the column eluant is: A. pumped up the column B. more polar than the stationary phase C. always nonpolar D. less polar than the stationary phase

Correct Answer: B

Serial bilirubin determination are charted below. Day; Collected; Assayed; Result 1: 7 am; 8 am; 14.0 mg/dL (239.4 umol/L) 2; 7 am; 6 pm; 9.0 mg/dL (153.9 umol/L) 3: 6 am; 8 am; 15.0 mg/dL (256.5 umol/L) The best explanation for the results is: A. sample hemolysis and hemoglobin deterioration B. sample exposure to light C. sample left in warm location D. reagent deterioration

Correct Answer: B

The regulation of calcium and phosphorous metabolism is accomplished by which of the following glands? A. thyroid B. parathyroid C. adrenal glands D. pituitary

Correct Answer: B

The solute that contributes the most to the total serum osmolality is: A. glucose B. sodium C. chloride D. urea

Correct Answer: B

Which of the following is secreted by the placenta and used for the early detection of pregnancy? A. follicle-stimulating hormone (FSH) B. human chorionic gonadotropin (HCG) C. luteinizing hormone D. progesterone

Correct Answer: B

Which of the following sets of results would be consistent with macroamylasemia? A. normal serum amylase and elevated urine amylase values B. increased serum amylase and normal urine amylase values C. increased serum and urine amylase values D. normal serum and urine amylase values

Correct Answer: B

Which of the following substances is the biologically active precursor of a fat soluble vitamin? A. biotin B. retinol C. folic acid D. ascorbic acid

Correct Answer: B

Which of the following would be an example of a glucose-specific colorimetric method? A. alkaline ferricyanide B. glucose oxidase C. hexokinase D. o-toluidine

Correct Answer: B

How many significant figures should be reported when the pH of a 0.060 M solution of nitric acid is calculated? A. 1 B. 2 C. 3 D. 4

Correct Answer: B 10. B When zeros appear by themselves to the left of the decimal point, they are not significant. When they are to the left of the decimal point and are preceded by a number, they are significant. Zeros after the decimal point preceding a number are not significant. However, they are significant if they follow another number or are between two numbers. Therefore, 0.060 M has only two significant figures (the underlined digits). In laboratory practice, most analytes are reported with two significant figures. Routine analytes that are exceptions are pH, which includes three significant figures, and analytes with whole numbers above 100 such as sodium, cholesterol, triglycerides, and glucose.

Which of the following enzymes is a transferase? A. ALP B. CK C. Amylase D. LD

Correct Answer: B 11. B Enzymes are identified by a numeric system called the EC (Enzyme Commission) number. The first number refers to the class of the enzyme. There are six classes; in order, these are oxidoreductases, transferases, hydrolases, lyases, isomerases, and ligases. Dehydrogenases are oxidoreductases, whereas kinases and transaminases are transferases. CK is EC number 2.7.3.2, which distinguishes it from other kinases.

A person is suspected of having testicular cancer. Which type of hCG test would be most useful? A. Plasma immunoassay for intact hCG only B. Plasma immunoassay for intact hCG and the β-hCG subunit C. Plasma immunoassay for the free alpha and β-hCG subunits D. Urine assay for hCG β core

Correct Answer: B 11. B In addition to testicular cancer, hCG is produced by trophoblastic tumors and choriocarcinomas. Some of these tumors secrete the β-subunit without intact hCG. This is especially true after treatment when hCG is used to monitor for recurrence. The use of an immunoassay that measures both the intact and free β hCG will have greater sensitivity than an assay for intact hCG or an assay for only free subunits. Free α hCG subunits may be produced in persons with testicular and urinary bladder (urothelial) cancer, but the incidence of α hCG subunit secretion only is relatively low. Urinary β core (urinary gonadotropin peptide) is a metabolic product of the β subunit and has been used to monitor for persistence of trophoblastic disease and recurrence of some hCG-producing tumors.

Which statement about methods for measuring LD is true? A. The formation of pyruvate from lactate (forward reaction) generates NAD+ B. The pyruvate-to-lactate reaction proceeds at about twice the rate as the forward reaction C. The lactate-to-pyruvate reaction is optimized at pH 7.4 D. The negative-rate reaction is preferred

Correct Answer: B 12. B Although the rate of the reverse reaction (P → L) is faster, the L → P reaction is more popular because it produces a positive rate (generates NADH), is not subject to product inhibition, and is highly linear. The pH optimum for the forward reaction is approximately 8.8.

What is the American Diabetes Association recommended cutoff value for adequate control of blood glucose in diabetics as measured by glycated hemoglobin? A. 5% B. 6.5% C. 9.5% D. 11%

Correct Answer: B 12. B The ADA recommends that 6.5% be used as the cutoff for determining the adequacy of treatment for diabetes. A glycated hemoglobin test should be performed at the time of diagnosis and every 6 months thereafter if the result is < 6.5%. If the result is 6.5% or more, the treatment plan should be adjusted to achieve a lower level, and the test performed every 3 months until control is improved.

A solvent needed for HPL C requires a 20.0mmol/L phosphoric acid buffer, pH 3.50, made by mixing KH2PO4 and H3PO4. How many grams of KH2PO4 are required to make 1.0 L of this buffer? Formula weights: KH2PO4 = 136.1; H3PO4 = 98.0; pKa H3PO4 = 2.12 A. 1.96 g B. 2.61 g C. 2.72 g D. 19.2 g

Correct Answer: B 13. B The Henderson-Hasselbalch equation is used to calculate the ratio of salt to acid needed to give a pH of 3.50. pH = pKa + log(salt/acid) 3.50 = 2.12 + log(KH2PO4/H3PO4) 1.38 = log(KH2PO4/H3PO4) antilog 1.38 = KH2PO4/H3PO4 KH2PO4/H3PO4 = 23.99 Rearranging gives KH2PO4 = 23.99 × H3PO4. Because the phosphate in the buffer is 20.0 mmol/L, then H3PO4 + KH2PO4 must equal 20. Because KH2PO4 = 23.99 × H3PO4 then: H3PO4 + (23.99 × H3PO4) = 20.0 mmol/L 24.99 × H3PO4 = 20.0 mmol/L H3PO4 = 20.0/24.99 = 0.800 mmol/L KH2PO4 = 20.0-0.800 = 19.2 mmol/L (0.0192 M) To determine the grams required, multiply the moles of KH2PO4 by the formula weight. 0.0192 mol/L × 136.1 g/mol = 2.613 g

Why do many optical systems in chemistry analyzers utilize a reference light path? A. To increase the sensitivity of the measurement B. To minimize error caused by source lamp fluctuation C. To obviate the need for wavelength adjustment D. To reduce stray light effects

Correct Answer: B 14. B A reference beam is used to produce an electrical signal at the detector to which the measurement of light absorption by the sample is compared. This safeguards against measurement errors caused power fluctuations that change the source lamp intensity. Although reference beams increase the accuracy of measurements, they do so at the expense of optical sensitivity since some of the incident light must be used to produce the reference beam.

A procedure for cholesterol is calibrated with a serum-based cholesterol standard that was determined by the Abell-Kendall method to be 200.0 mg/dL. Assuming the same volume of sample and reagent are used, calculate the cholesterol concentration in the patient's sample from the following results. -Standard Concentration: 200 mg/dL -Absorbance of Reagent Blank: 0.00 -Absorbance of Standard: 0.860 -Absorbance of Patient Serum: 0.740 A. 123 mg/dL B. 172 mg/dL C. 232 mg/dL D. 314 mg/dL

Correct Answer: B 14. B Cu = Au/As × Cs where Cu = concentration of unknown, Au = absorbance of unknown, As = absorbance of standard, and Cs = concentration of standard. Cu = 0.740/0.860 × 200 mg/dL = 172 mg/dL

Which stationary phase is used for the measurement of hemoglobin A1c by high performance liquid chromatography? A. Octadecylsilane (C18) B. Cation exchanger C. Anion exchanger D. Polystyrene divinylbenzene

Correct Answer: B 14. B HPLC methods for measuring Hgb A1c are performed by diluting whole blood with an acid buffer that hemolyzes the sample. Normal hemoglobin A has a weak positive charge at an acidic pH and binds weakly to the resin. Glycated hemoglobin has an even weaker positive charge and is eluted before hemoglobin A. Abnormal hemoglobin molecules S, D, E, and C have a higher positive charge than hemoglobin A and are retained longer on the column. Elution is accomplished by increasing the ionic strength of the mobile phase. Cations in the buffer displace the hemoglobin pigments from the column.

A glycerol kinase method for triglyceride calls for a serum blank in which normal saline is substituted for lipase in order to measure endogenous glycerol. Given the following results, and assuming the same volume of sample and reagent are used for each test, calculate the triglyceride concentration in the patient's sample. -Standard Concentration: 125 mg/dL -Absorbance of Reagent Blank: 0.000 -Absorbance of Standard: 0.62 -Absorbance of Patient Serum: 0.750 -Absorbance of Serum Blank: 0.100 A. 119 mg/dL B. 131 mg/dL C. 156 mg/dL D. 180 mg/dL

Correct Answer: B 15. B The serum blank absorbance is subtracted from the result for the patient's serum before applying the ratiometric formula to calculate concentration. Cu = [(Au - ASB)/As ] × Cs where ASB = absorbance of serum blank = (0.750-0.100)/0.620 × 125 mg/dL = 131 mg/dL

Select the anticoagulant of choice for blood gas studies. A. Sodium citrate 3.2% B. Lithium heparin 100 U/mL blood C. Sodium citrate 3.8% D. Ammonium oxalate 5.0%

Correct Answer: B 16. B Heparin is the only anticoagulant that does not alter the pH of blood; heparin salts must be used for pH and blood gases. Solutions of heparin are air equilibrated and must be used sparingly to prevent contamination of the sample by gas in the solution.

A patient's blood gas results are: -pH = 7.50 -PCO2 = 55 mm Hg -HCO3- = 40 mmol/L These results indicate: A. Respiratory acidosis B. Metabolic alkalosis C. Respiratory alkalosis D. Metabolic acidosis

Correct Answer: B 19. B A pH above 7.45 corresponds with alkalosis. Both bicarbonate and PCO2 are elevated. Bicarbonate is the conjugate base and is under metabolic (renal) control, while PCO2 is an acid and is under respiratory control. Increased bicarbonate (but not increased CO2) results in alkalosis; therefore, the classification is metabolic alkalosis, partially compensated by increased PCO2.

Which of the following statements regarding PSA is true? A. Complexed PSA in plasma is normally less than free PSA B. Free PSA below 25% is associated with malignant disease C. A total PSA below 4 ng/mL rules out malignant disease D. A total PSA above 10 ng/mL is diagnostic of malignant disease

Correct Answer: B 19. B In normal plasma, 55%-95% of the PSA is bound to protease inhibitors, primarily α1-antichymotrypsin, and the remainder is called free PSA. At a cutoff of 4 ng/mL commonly used for the upper reference limit, total PSA has a sensitivity of approximately 60%, and 22% of men with a PSA below 4 ng/mL have evidence of early prostate cancer on biopsy. For this reason, some laboratories prefer a cutoff of 2.5 ng/mL for total PSA. However, based upon this cutoff alone, the number of false-positive findings (unnecessary biopsies) would be extremely high. A PSA of 2.6 ng/mL that was 2.6 ng/mL the previous year would not likely be significant; however, a PSA of 2.6 ng/mL that was only 1.6 ng/mL the previous year would warrant further testing. In persons with a total PSA between 2.6 and 10.0 ng/mL, a low ratio of free PSA:total PSA (<25% fPSA) or a high level of complexed PSA increases the diagnostic sensitivity and specificity. Persons with a PSA between 2.6 and 10.0 ng/mL are selected for biopsy if either the fPSA is low or the cPSA is high. Initial studies also indicate that the incomplete cleavage of the proenzyme of PSA (proPSA) in persons with cancer results in a high ratio of proPSA to fPSA. This ratio was reported to have better diagnostic sensitivity and specificity than the percentage of fPSA alone. The probability of cancer when the total PSA is higher than 10 ng/mL is approximately 50%, and this necessitates a biopsy to determine if the prostate is malignant.

Which support medium can be used to determine the molecular weight of a protein? A. Cellulose acetate B. Polyacrylamide gel C. Agar gel D. Agarose gel

Correct Answer: B 19. B Polyacrylamide gels separate by molecular sieving as well as charge. Sodium dodecyl sulfate (SDS) is a nonionic detergent that binds to proteins, neutralizing their charge. Polyacrylamide gel electrophoresis (PAGE) after treating with SDS separates proteins on the basis of molecular size. The smaller proteins become trapped in the pores of the gel and migrate more slowly.

A solution that has a transmittance of 1.0 %T would have an absorbance of: A. 1.0 B. 2.0 C. 1% D. 99%

Correct Answer: B 2. B A = 2.0 - log %T A = 2.0 - log 1.0 The log of 1.0 = 0 A = 2.0

Creatinine is considered the substance of choice to measure endogenous renal clearance because: A. The rate of formation per day is independent of body size B. It is completely filtered by the glomeruli C. Plasma levels are highly dependent upon diet D. Clearance is the same for both men and women

Correct Answer: B 2. B Creatinine concentration is dependent upon muscle mass, but varies by less than 15% per day. Creatinine is not metabolized by the liver, or dependent on diet, and is 100% filtered by the glomeruli. It is not reabsorbed significantly but is secreted slightly, especially when filtrate flow is slow. Plasma creatinine and cystatin C are the two substances of choice for evaluating the glomerular filtration rate (GFR).

Which of the following hormones promotes hyperglycemia? A. Calcitonin B. Growth hormone C. Aldosterone D. Renin

Correct Answer: B 2. B Growth hormone and cortisol promote gluconeogenesis and epinephrine stimulates glycogenolysis. Excess thyroid hormone causes hyperglycemia by increasing glucagon and inactivation of insulin, thereby promoting both gluconeogenesis and glycogenolysis. An increase in any of these hormones can cause hyperglycemia. Calcitonin opposes the action of parathyroid hormone. Aldosterone is the primary mineralocorticoid hormone and stimulates sodium reabsorption and potassium secretion by the kidneys. Renin is released from the kidney due to ineffective arterial pressure and promotes activation of angiotensinogen and aldosterone secretion.

BUN is determined electrochemically by coupling the urease reaction to measurement of: A. Potential with a urea-selective electrode B. The timed rate of increase in conductivity C. The oxidation of ammonia D. Carbon dioxide

Correct Answer: B 20. B A conductivity electrode is used to measure the increase in conductance of the solution as urea is hydrolyzed by urease in the presence of sodium carbonate. Urea + H2O → 2NH3 + CO2 2NH3 + 2H2O + Na2CO3 → 2NH4 + + CO3 -2 + 2NaOH Ammonium ions increase the conductance of the solution. The timed rate of current increase is proportional to the BUN concentration. Alternatively, the ammonium ions produced can be measured using an ion-selective electrode.

A blood sample is left on a phlebotomy tray for 4.5 hours before it is delivered to the laboratory. Which group of tests could be performed? A. Glucose, Na, K, Cl, TCO2 B. Uric acid, BUN, creatinine C. Total and direct bilirubin D. CK, ALT, ALP, AST

Correct Answer: B 20. B Glucose in serum is metabolized by cells at a rate of about 7% per hour. Bilirubin levels will fall if the sample is exposed to sunlight. Transaminases should be measured within 4 hours and ALP within 2 hours if the sample is stored at room temperature. Uric acid, BUN, and creatinine are least likely to be affected.

Which set of results is consistent with uncompensated respiratory alkalosis? A. pH 7.70; HCO3 30 mmol/L; PCO2 25 mm Hg B. pH 7.66; HCO3 22 mmol/L; PCO2 20 mm Hg C. pH 7.46; HCO3 38 mmol/L; PCO2 55 mm Hg D. pH 7.36; HCO3 22 mmol/L; PCO2 38 mm Hg

Correct Answer: B 20. B Respiratory alkalosis is caused by hyperventilation, inducing low PCO2. Very often, in the early phase of an acute respiratory disturbance, the kidneys have not had time to compensate, and the bicarbonate is within normal limits. In answer A, the bicarbonate is high and PCO2 low; thus, both are contributing to alkalosis and this would be classified as a combined acid-base disturbance. In answer C, the pH is almost normal, and both bicarbonate and PCO2 are increased. This can occur in the early stage of a metabolic acid- base disturbance when full respiratory compensation occurs or in a combined acid-base disorder. In answer D, both bicarbonate and PCO2 are within normal limits (22-26 mmol/L, 35-45 mm Hg, respectively) as is the pH.

In the ultraviolet enzymatic method for BUN, the urease reaction is coupled to a second enzymatic reaction using: A. AST B. Glutamate dehydrogenase C. Glutamine synthetase D. Alanine aminotransferase (ALT)

Correct Answer: B 21. B BUN is most frequently measured by the urease-UV method in which the urease reaction is coupled to the glutamate dehydrogenase reaction, generating NAD+. Urea + H2O Urease ---> 2NH3 + CO2 2-Oxoglutarate + NH3 + NADH + H+ ---> GLD Glutamate + NAD+ + H2O When the urease reaction is performed under first-order conditions, the decrease in absorbance at 340 nm is proportional to the urea concentration.

Which of the following statements regarding adrenal cortical dysfunction is true? A. Patients with Cushing's syndrome usually have hyperkalemia B. Cushing's syndrome is associated with glucose intolerance C. Addison's disease is associated with hypernatremia D. Addison's disease is caused by elevated levels of cortisol

Correct Answer: B 21. B Patients with Cushing's syndrome have elevated levels of cortisol and other adrenal corticosteroids. This causes the characteristic cushingoid appearance that includes obesity, acne, and humpback posture. Osteoporosis, hypertension, hypokalemia, and glycosuria are characteristics. Addison's disease results from adrenal hypoplasia and produces the opposite symptoms including hypotension, hyperkalemia, and hypoglycemia.

Polarographic methods for glucose analysis are based upon which principle of measurement? A. Nonenzymatic oxidation of glucose B. The rate of O2 depletion C. Chemiluminescence caused by formation of adenosine triphosphate (ATP) D. The change in electrical potential as glucose is oxidized

Correct Answer: B 22. B Polarographic glucose electrodes measure the consumption of O2 as glucose is oxidized. Glucose oxidase in the reagent catalyzes the oxidation of glucose by O2 under first-order conditions, forming hydrogen peroxide (H2O2). As the dissolved O2 decreases, less is reduced at the cathode, resulting in a decrease in current proportional to glucose concentration. It is important that the H2O2 not breakdown to re-form O2. This is prevented by adding molybdate and iodide that react with H2O2, forming iodine and water, and by adding catalase and ethanol that react with H2O2, forming acetaldehyde and water.

The term R4S means that: A. Four consecutive controls are greater than ±1 standard deviation from the mean B. Two controls in the same run are greater than 4s units apart C. Two consecutive controls in the same run are each greater than ±4s from the mean D. There is a shift above the mean for four consecutive controls

Correct Answer: B 22. B The R4s rule is applied to two control levels within the same run. The rule is violated when the algebraic difference between them (level 1 - level 2) exceeds 4s. The rule is never applied across different runs. The R4s rule detects random error (error due to poor precision).

Which of the following statements about TLC for drug screening is true? A. Acidic drugs are extracted in an alkaline nonpolar solvent B. A drug is identified by comparing its Rf value and staining to standards C. Testing must be performed using a urine sample D. Opiates and other alkaloids are extracted at an acid pH

Correct Answer: B 24. B TLC can be performed on urine, serum, or gastric fluid and qualitatively identifies most drugs. Each has a characteristic Rf , which is the ratio of the distance migrated by the drug to the solvent. The Rf of the sample must match the Rf of the drug standard. Extraction of drugs for TLC is highly pH dependent. The pH must be adjusted to reduce the solubility (ionization) of the drug in the aqueous phase. Usually, alkaline drugs (e.g., opiates) are extracted at pH 9.0 and acidic drugs (e.g., barbiturates) at pH 4.5.

A patient's CK-MB is reported as 18 μg/L and the total CK as 560 IU/L. What is the CK relative index (CKI)? A. 0.10% B. 3.2% C. 10.0% D. 30.0%

Correct Answer: B 25. B The CKI is an expression of the percentage of the total CK that is attributed to CK-MB. CKI = (CK-MB in μg/L / Total CK in IU/L) x value by 100% The reference range is 0%-2.5%. Values above 2.5% point to an increase in CK-MB from cardiac muscle.

Which statement about EMIT is true? A. Enzyme activity is inversely proportional to drug level B. Formation of NADH is monitored at 340 nm C. ALP is the commonly used conjugate D. Assay use is restricted to serum

Correct Answer: B 26. B EMIT is a homogenous immunoassay, meaning that free antigen does not have to be separated from bound antigen. Most EMIT assays use a two-reagent system. Reagent A contains substrate (usually glucose-6-PO4), coenzyme (NAD+), and antibody to the drug. Reagent B contains enzyme-labeled drug (usually G-6-PD-drug) and buffer. The rate of NADH production is proportional to the drug concentration. EMIT assays are commonly used to test for drugs of abuse in urine. In such cases, the enzyme activity of the low calibrator (drug concentration equal to U.S. Substance Abuse and Mental Health Services Administration minimum for a positive test) is used as the cutoff.

Which statement regarding cloned enzyme donor immunoassay (CEDIA) is true? A. The enzyme used is glucose-6-phosphate dehydrogenase B. The enzyme donor and acceptor molecules are fragments of β-galactosidase C. Drug concentration is inversely related to fluorescence D. The antibody is covalently linked to the enzyme donor

Correct Answer: B 27. B CEDIA is a homogenous enzyme immunoassay that is commonly used to measure drugs of abuse. Drug conjugated to a fragment of β-galactosidase that is catalytically inactive competes with drug in the sample for a limited number of antibodies to the drug. The fragment, called the enzyme donor (ED), and substrate (chlorophenol red-β-D-galactopyranose) are mixed with the sample. A second reagent containing monoclonal antibody and a second fragment of β-galactosidase called the enzyme acceptor (EA) is added. If the antibody is neutralized by drug from the sample, the ED and EA combine forming an active enzyme. The concentration of drug in the sample is directly proportional to the amount of chlorophenol red formed.

Which statement best describes the clinical utility of plasma or serum myoglobin? A. Levels greater than 100 μg/L are diagnostic of AMI B. Levels below 100 μg/L on admission and 2-4 hours postadmission help to exclude a diagnosis of AMI C. Myoglobin peaks after the cardiac troponins but is more sensitive D. The persistence of myoglobin > 110 μg/L for 3 days following chest pain favors a diagnosis of AMI

Correct Answer: B 27. B Myoglobin is a heme-containing pigment in both skeletal and cardiac muscle cells. The upper limit of normal is approximately 90 μg/L for males and 75 μg/L for females. The plasma myoglobin is a sensitive marker for AMI. Over 95% of affected persons have a value higher than the cutoff (typically >110 μg/L). However, specificity is approximately 75%-85% owing to skeletal muscle injury or renal insufficiency. For this reason, a plasma myoglobin below the cutoff on admission, and within the first 3 hours following chest pain helps to rule out AMI. A value above the cutoff must be confirmed using a cardiac specific assay such as TnI or TnT.

Which of the following abnormal types of Hgb migrates to the same position as Hgb S on agarose or cellulose acetate at pH 8.6? A. Hgb C B. Hgb DPunjab C. Hgb OArab D. Hgb E

Correct Answer: B 29. B Hgb DPunjab migrates with Hgb S on cellulose acetate or agarose at pH 8.6-9.2. Hgb C, E, OArab, and CHarlem migrate to the same position as Hgb A2 on cellulose acetate or agarose at pH 8.6-9.2. Hgb S may be differentiated from Hgb DPunjab using citrate (acid) agar at pH 6.2. Using this technique, Hgb S migrates further toward the anode than Hgb DPunjab.

Quantitationofadrugbygaschromatography-mass spectroscopy (GC-MS) is usually performed in which mode? A. Total ion chromatography B. Selective ion monitoring C. Ion subtraction D. Selective reaction monitoring

Correct Answer: B 29. B Most GC-MS instruments use an electron beam to split the drug emerging from the column into its component ions. These are drawn into the mass analyzer, usually a vacuum chamber containing two pairs of charged rods (a positive pair and a negative pair) called a quadrupole analyzer. By changing the potential and radio frequency applied to the rods, the travel of ions will vary depending upon their mass to charge (m/z) ratio. As ions emerge from the mass filter, they are detected by an electron multiplier tube. CG-MS instruments can be operated in two modes, total ion chromatography and selective ion monitoring. A total ion chromatograph displays the retention time of all ions detected and their abundance. It is primarily used for identification of unknown compounds. SIM mode measures the abundance of one or more principal ions that provides sufficient specificity to eliminate potential interfering substances and greater quantitative sensitivity. For example, tetrahydrocannabinol (THC) can be identified by ions m/z 371.3, 372.3, and 473.3.

Which of the following plots is best for comparison of precision and accuracy among laboratories? A. Levy-Jennings B. Tonks-Youden C. Cusum D. Linear regression

Correct Answer: B 29. B The Tonks-Youden plot is used for interlaboratory comparison of monthly means. The method mean for level 1 is at the center of the y axis and mean for level 2 at the center of the x axis. Lines are drawn from the means of both levels across the graph, dividing it into four equal quadrants. If a laboratory's monthly means both plot in the lower left or upper right, then systematic error (SE) exists in its method.

The term pharmacodynamics is an expression of the relationship between: A. Dose and physiological effect B. Drug concentration at target sites and physiological effect C. Time and serum drug concentration D. Blood and tissue drug levels

Correct Answer: B 3. B Pharmacodynamics is the relationship between the drug concentration at the receptor site (tissue concentration) and the response of the tissue to that drug. For example, the relationship between lidocaine concentration in the heart muscle and the duration of the action potential of Purkinje fibers.

Which statement regarding creatinine is true? A. Serum levels are elevated in early renal disease B. High serum levels result from reduced glomerular filtration C. Serum creatine has the same diagnostic utility as serum creatinine D. Serum creatinine is a more sensitive measure of renal function than creatinine clearance

Correct Answer: B 3. B Serum creatinine is a specific but not a sensitive measure of glomerular function. About 60% of the filtration capacity of the kidneys is lost when serum creatinine becomes elevated. Because urine creatinine diminishes as serum creatinine increases in renal disease, the creatinine clearance is more sensitive than serum creatinine in detecting glomerular disease. A creatinine clearance below 60 mL/min indicates loss of about 50% functional nephron capacity and is classified as moderate (stage 3) chronic kidney disease.

Which of the following statements about the diagnosis of Addison's disease is true? A. Patients with primary Addison's disease show a normal response to ACTH stimulation B. Primary and secondary Addison's disease can often be differentiated by plasma ACTH C. Twenty-four-hour urinary free cortisol is normal in Addison's disease D. Pituitary ACTH reserves are normal in secondary Addison's disease

Correct Answer: B 30. B ACTH (Cortrosyn) stimulation is used as a screening test for Addison's disease. A 250-μg dose of Cortrosyn is given intravenously. Normal patients show a 2-5 times increase in serum cortisol. A subnormal response occurs in both primary and secondary Addison's disease. Plasma ACTH is high in primary but is low in secondary Addison's disease. Patients with secondary Addison's disease (pituitary failure) do not respond to metyrapone because their ACTH reserve is diminished.

Point-of-care-tests (POCTs) for whole-blood glucose monitoring are based mainly on the use of: A. Glucose oxidase as the enzyme B. Amperometric detection C. Immunochromatography D. Peroxidase coupling reactions

Correct Answer: B 30. B All POCT devices for monitoring blood glucose use either glucose dehydrogenase (GDH) or glucose oxidase and are amperometric. For glucose oxidase methods, the electrons derive from the oxidation of hydrogen peroxide. For GDH, the electrons are transferred from one of several coenzymes that are reduced when glucose is oxidized, FAD+, NAD+, or PQQ (pyrroloquinoline quinone). Interferences depend upon which enzyme/coenzyme pair are used. For example, maltose and xylose interference can be pronounced with GDH/PQQ-based strips, but not with other GDH or glucose oxidase strips. Uric acid depresses glucose oxidase reactions but has no effect on GDH reactions.

Which Hgb is a β-δ chain hybrid and migrates to the same position as Hgb S at pH 8.6? A. Hgb CHarlem B. HgbLepore C. Hgb GPhiladelphia D. Hgb DPunjab

Correct Answer: B 30. B HgbLepore results from translocation of β and δ globin genes, resulting in a polypeptide chain that migrates midway between Hgb A2 and Hgb A. The chain is transcribed more slowly than the β polypeptide chain, causing the quantity of HgbLepore to be less than 15%. HgbLepore is suspected when Hgb migrating in the "S" zone comprises less than 20% of the total Hgb. In Hgb S trait, the AS phenotype produces 20%-40% Hgb S.

SITUATION: Results of biochemistry tests are: -Na = 138 mmol/L -Cl = 94 mmol/L -glucose = 100 mg/dL -BUN = 6.8 mg/dL -albumin = 4.8 g/dL -K = 4.2 mmol/L -TCO2 = 20 mmol/L -T bili = 1.2 mg/dL -creat = 1.0 mg/dL -T protein = 5.1 g/dL What should be done next? A. Request a new specimen B. Repeat the total protein C. Repeat all tests D. Perform a protein electrophoresis

Correct Answer: B 31. B All results are normal except total protein. The albumin level cannot be 94% of the total protein, and a random error in total protein measurement should be assumed.

What is the recommended troponin T and I cutoff (upper limit of normal) for detecting myocardial infarction? A. The cutoff varies with the method of assay but should be no lower than 0.2 ng/mL B. The upper 99th percentile or lowest level that can be measured with 10% CV C. The concentration corresponding to the lowest level of calibrator used D. The highest value fitting under the area of the curve for the 95% confidence interval

Correct Answer: B 31. B The American College of Cardiology recommends the cutoff for an abnormal troponin test be set at the 99th percentile of the normal population, 0.013 ng/mL, or if the assay precision at this level is >10% then the cutoff should be the lowest value measurable with a CV of 10% (typically 0.03 ng/mL). An abnormal result (0.04 ng/mL or higher) in a patient with other evidence of ischemic changes indicates cardiac damage. This typically occurs when a pattern of increasing troponin concentration is seen over the first 6 hours after initial testing.

The following chart compares the monthly total bilirubin mean of Laboratory A to the monthly mean of Laboratory B, which uses the same control materials, analyzer, and method. L1 Mean; L1 CV; L2 Mean, L2 CV -Lab A: 1.1 mg/dL; 2.1%; 6.7 mg/dL; 3.2% -Lab B: 1.4 mg/dL; 2.2%; 7.0 mg/dL; 3.6% Both laboratories performed controls at the beginning of each shift using commercially prepared liquid QC serum stored at -20°C. Which of the following conditions would explain these differences? A. Improper handling of the control material by Laboratory A resulted in loss of bilirubin due to photodegradation B. The laboratories used a different source of bilirubin calibrator C. Laboratory B obtained higher results because its precision was poorer D. Carryover from another reagent falsely elevated the results of Laboratory B

Correct Answer: B 32. B Interlaboratory variation in bilirubin results is often caused by differences in the assigned value of the calibrator used. Bilirubin calibrators are either serum-based material that have been reference assayed or unconjugated bilirubin stabilized by addition of alkali and albumin. Calibrator differences result in bias and should be suspected when the laboratory's mean differs significantly from the peer group's mean. The bias in this example is due to constant rather than proportional error. When bilirubin calibrator error is suspected, the molar absorptivity of the calibrator should be measured and the bilirubin concentration calculated. Photodegradation generally results in a greater loss of bilirubin at higher concentration and also contributes to random error.

Which of the following cardiac markers is consistently increased in persons who exhibit unstable angina? A. Troponin C B. Troponin T C. CK-MB D. Myoglobin

Correct Answer: B 32. B Persons with unstable angina (angina at rest) who have an elevated TnT or TnI are at eight times greater risk of having an MI within the next 6 months. This property is being used to identify short-term risk patients who should be considered for coronary angioplasty. The reference range for troponin is very low (0-0.03 ng/mL); persons with unstable angina usually have values between 0.04 and 0.1 ng/mL without clinical evidence of AMI. CK-MB and myoglobin have not been useful in identifying persons with unstable angina.

In double immunodiffusion reactions, the precipitin band is: A. Invisible before the equivalence point is reached B. Concave to the protein of greatest molecular weight C. Closest to the well containing the highest level of antigen D. Located in an area of antibody excess

Correct Answer: B 34. B In double immunodiffusion (Ouchterlony), the molecules of lower molecular weight move fastest through the gel, causing a visible precipitin arc when antigen and antibody approach equivalence. At equivalence the precipitin arc remains stationary. If the concentration of antisera is constant, the distance of the precipitin arc from the antigen well is proportional to antigen concentration.

Ion selective analyzers using undiluted samples have what advantage over analyzers that use a diluted sample? A. Can measure over a wider range of concentration B. Are not subject to pseudohyponatremia caused by high lipids C. Do not require temperature equilibration D. Require less maintenance

Correct Answer: B 34. B Ion-selective analyzers measure the electrolyte dissolved in the fluid phase of the sample in millimoles per liter of plasma water. When undiluted blood is assayed, the measurement is independent of colloids such as protein and lipid. Hyperlipemic samples cause falsely low sodium measurements when assayed by flame photometry and ion-selective analyzers requiring dilution because lipids displace plasma water containing the electrolytes. One drawback to undiluted or direct measuring systems is that the electrodes require more frequent deproteinization and usually have a shorter duty cycle.

Which statement regarding the measurement of urinary catecholamines is true? A. An increased excretion of total urinary catecholamines is specific for pheochromocytoma B. Twenty-four-hour urinary catecholamine assay avoids pulse variations associated with measurement of plasma catecholamines C. Total urinary catecholamine measurement provides greater specificity than measurement of urinary free catecholamines D. Total urinary catecholamines are not affected by exercise

Correct Answer: B 34. B Measurement of total urinary catecholamines is not a specific test for pheochromocytoma. Urine levels may be increased by exercise and in muscular diseases. Catecholamines in urine may also be derived from dietary sources rather than endogenous production. Most catecholamines are excreted as the glucuronide, and the urinary free catecholamines increase only when there is increased secretion. Measurement of free hormone in urine is equal in clinical sensitivity and specificity to measurement of metanephrines. Twenty-four-hour urine is the sample of choice because plasma levels are subject to pulse variation and affected by the patient's psychological and metabolic condition at the time of sampling.

Which method is most often used to measure fractionated catecholamines (epinephrine, norepinephrine, and dopamine)? A. Measurement of fluorescence following oxidation by potassium ferricyanide B. Measurement by HPLC with electrochemical detection C. Measure of radioactivity after conversion by catechol-O-methyltransferase (COMT) to tritiated metanephrines D. Measurement by HPLC with fluorescence detection

Correct Answer: B 35. B HPLC-ECD separates catecholamines by reverse- phase chromatography, then detects them by oxidizing the aromatic ring at +0.8 V to a quinone ring. Current is proportional to epinephrine and norepinephrine concentration. Fluorescent methods employing ferricyanide (trihydroxyindole method) or ethylenediamine (EDA method) show interference by Aldomet and several other drugs and are obsolete. The radioenzymatic assay of catecholamines is a specific alternative to HPLC but requires a liquid scintillation counter. The method uses the enzyme COMT to transfer a tritiated methyl group from S-adenosyl methionine to the catecholamines. This results in formation of radiolabeled metanephrines that are measured. HPLC with fluorescence detection is not as sensitive as HPLC-ECD. Electrospray ionization tandem-mass spectroscopy is an alternative to HPLC-EDC for measurement of fractionated catecholamines and metanephrines.

Which of the following statements regarding the identification of monoclonal proteins by IFE is true? A. The monoclonal band must be present in the γ region B. When testing for a monoclonal gammopathy, both serum and urine must be examined C. A diagnosis of monoclonal gammopathy is based upon quantitation of IgG, IgA, and IgM D. A monoclonal band always indicates a malignant disorder

Correct Answer: B 35. B Quantitation of IgG, IgA, IgM, or IgD indicates the concentration of each class of immunoglobulin but does not distinguish monoclonal from polyclonal gammopathies. Monoclonal characteristics are determined by demonstrating restricted electrophoretic mobility, indicating that all immunoglobulins in the band are of the same amino acid sequence. Monoclonal light chains can be demonstrated in about 60% of monoclonal gammopathies. In up to 25% of multiple myeloma patients, a heavy chain gene deletion results in production of monoclonal light chains only. Because these are filtered by the glomerulus, the procedure must be performed on urine as well as serum. Some patients with a monoclonal protein fail to develop malignant plasma cell proliferation. This state is called a monoclonal gammopathy of undetermined significance (MGUS). Within 10-15 years, 15%-20% of persons with MGUS develop some form of lymphoproliferative disease.

A biochemical profile routinely performed bimonthly on a renal dialysis patient showed a decreased serum calcium and decreased PTH level. Such a lab result may be explained by which of the following circumstances? A. Malignancy B. Aluminum toxicity C. Hypervitaminosis D D. Acidosis

Correct Answer: B 37. B Aluminum present in medications and dialysis bath fluid can cause aluminum toxicity in patients receiving dialysis. Renal failure patients often display high PTH levels owing to poor retention of calcium, and are at risk of developing osteitis fibrosa (soft bones) as a result. Excess aluminum causes osteomalacia by inhibiting release of parathyroid hormone. The finding of low PTH would not be expected with low serum calcium unless aluminum poisoning was present. Malignancy, hypervitaminosis D, and acidosis are associated with high serum calcium.

The term RT/nF in the Nernst equation defines the: A. Potential at the ion-selective membrane B. Slope of the electrode C. Decomposition potential D. Isopotential point of the electrode

Correct Answer: B 37. B In the term RT/nF, R = the molar gas constant, T = temperature in degrees Kelvin, F = Faraday's constant, and n = the number of electrons donated per atom of reductant. The slope depends upon the temperature of the solution and the valence of the reductant. At room temperature, the slope is 59.2 mV for a univalent ion and 29.6 mV for a divalent ion.

Given the following serum electrolyte data, determine the anion gap. -Na = 132 mmol/L -Cl =90 mmol/L -HCO3 = 22 mmol/L A. 12 mmol/L B. 20 mmol/L C. 64 mmol/L D. Cannot be determined form the information provided

Correct Answer: B 37. B The anion gap is defined as unmeasured anions minus unmeasured cations. It is calculated by subtracting the measured anions (bicarbonate and chloride) from the serum sodium (or sodium plus potassium). A normal anion gap is approximately 8-16 mmol/L (12-20 mmol/L when potassium is used). Anion gap = Na - (HCO3 + Cl) Anion gap = 132 - (90 + 22) = 20 mmol/L

Which of the following processes occurs when iron is in the oxidized (Fe3+) state? A. Absorption by intestinal epithelium B. Binding to transferrin and incorporation into ferritin C. Incorporation into protoporphyrin IX to form functional heme D. Reaction with chromogens in colorimetric assays

Correct Answer: B 38. B Intestinal absorption occurs only if the iron is in the reduced (Fe+2) state. After absorption, Fe+2 is oxidized to Fe+3 by gut mucosal cells. Transferrin and ferritin bind iron efficiently only when in the oxidized state. Iron within Hgb binds to O2 by coordinate bonding, which occurs only if the iron is in the reduced state. Likewise, in colorimetric methods, Fe+2 forms coordinate bonds with carbon and nitrogen atoms of the chromogen.

Which set of the following laboratory results is most likely from a patient who has suffered an AMI? Reference intervals are in parenthesis. Total CK (10-110 U/L); CK-MB (1-4 μg/L); CK index (1%-2.5%) A. 760 U/L 16 μg/L 2.1% B. 170 U/L 14 μg/L 8.2% C. 160 U/L 4 μg/L 2.5% D. 80 U/L 2 μg/L 2.5%

Correct Answer: B 38. B Results shown in C and D can be excluded because the CK-MB is not increased. Results shown in A and B have CK-MB levels above the URL. However, patient A has a CK index under 2.5% and a 5- to 10-fold elevation of total CK. These results indicate release of a small of amount of CK-MB from skeletal muscle rather than from cardiac muscle. To maximize the sensitivity of CK-MB, laboratories use an URL of 4 or 5 μg/L. This cutoff can detect about two-thirds of AMI cases within 3 hours of the infarct, but requires the use of a conservative CK index and other cardiac markers to avoid a high number of false positives.

Which whole-blood level is suggestive of excessive exposure to lead in children but not adults? A. 4 μg/dL B. 14 μg/dL C. 28 μg/dL D. 32 μg/dL

Correct Answer: B 39. B Because lead exposure in children leads to learning impairment, the cutoff for exposure recommended by the Centers for Disease Control is 5 μg/dL in venous whole blood. Values of 5 μg/dL or more should be monitored closely with follow-up testing, and if they increase, steps should be taken to remove lead contamination from the home and environment. For adults the recommended cutoff is 25 μg/dL. Because lead readily enters the red blood cells, and passes from plasma to urine quickly, whole blood is a more sensitive measure of exposure than plasma. Because lead from the fingers may contaminate the specimen, a venous sample is preferred over a capillary sample collected by finger stick.

Which statement best describes the clinical utility of plasma homocysteine? A. Levels are directly related to the quantity of LDL cholesterol in plasma B. High plasma levels are associated with atherosclerosis and increased risk of thrombosis C. Persons who have an elevated plasma homocysteine will also have an increased plasma Lp(a) D. Plasma levels are increased only when there is an inborn error of amino acid metabolism

Correct Answer: B 39. B Homocysteine includes the monomeric amino acid as well as the dimers such as homocystine that contain homocysteine. Plasma levels are measured as an independent risk factor for coronary artery disease. High levels of homocysteine are toxic to vascular endothelium and promote inflammation and plaque formation. Plasma levels are independent of LDL and other cholesterol fractions and help explain why approximately 35% of people with first-time AMI have LDL cholesterol levels < 130 mg/dL.

The response of a sodium electrode to a 10-fold increase in sodium concentration should be: A. A 10-fold drop in potential B. An increase in potential of approximately 60 mV C. An increase in potential of approximately 10 mV D. A decrease in potential of approximately 10 mV

Correct Answer: B 39. B The Nernst equation predicts an increase of approximately 60 mV per 10-fold increase in sodium activity. For sodium: E = E° + RT/nF × 2.3 log10[Na+] RT/nF × 2.3 = 60 mV at 37°C. Therefore: E = E° + 60 mV × log10[Na+]. If sodium concentration is 10 mmol/L, then: E = E° + 60 mV × log10[10] = E° + 60 mV. If sodium concentration increases from 10 mmol/L to 100 mmol/L, then: E = E° + 60 mV × log10[100] = E° + 60 mV × 2 = E° + 120 mV.

SITUATION: A 22S QC error occurs for serum calcium by atomic absorption. Fresh standards prepared in 5.0% w/v albumin are found to be linear, but repeating the controls with fresh material does not improve the QC results. Select the most likely cause of this problem. A. Matrix effect caused by a viscosity difference between the standards and QC sera B. Chemical interference caused incomplete atomization C. Incomplete deconjugation of protein-bound calcium D. Ionization interference caused by excessive heat

Correct Answer: B 4. B Poor recovery of calcium by atomic absorption is often caused by failure to break thermostable bonds between calcium and phosphate (a form of chemical interference). This may be caused by failure to add lanthanum to the diluent or by low atomizer temperature. The use of 5.0 % w/v albumin in the calibrator produces viscosity and protein-binding characteristics similar to plasma, helping to eliminate matrix interference.

Which of the following protein methods has the highest analytical sensitivity? A. Refractometry B. Folin-Lowry C. Turbidimetry D. Direct ultraviolet absorption

Correct Answer: B 4. B The Folin-Lowry (Lowry's) method uses both biuret reagent and phosphotungstic and molybdic acids to oxidize the aromatic side groups on proteins. The acids oxidize the phenolic rings of tyrosine and tryptophan. These, in turn, reduce the Cu+2 in the biuret reagent, increasing sensitivity about 100-fold.

Which of the following statements about the aminotransferases (AST and ALT) is true? A. Isoenzymes of AST and ALT are not found in humans B. Both transfer an amino group to α-ketoglutarate C. Both require NADP+ as a coenzyme D. Both utilize four carbon amino acids as substrates

Correct Answer: B 41. B ALT catalyzes the transfer of an amino group from alanine, a three-carbon amino acid, to α-ketoglutarate (2-oxoglutarate), forming pyruvate. AST catalyzes the transfer of an amino group from aspartate (four carbons) to α-ketoglutarate, forming oxaloacetate. The reactions are highly reversible and regulate the flow of aspartate into the urea cycle. Both transaminases require P-5'-P as an intermediate amino acceptor (coenzyme). Cytoplasmic and mitochondrial isoenzymes are produced but are not differentiated in clinical practice.

When the magnitude of error increases with increasing sample concentration, it is called: A. Constant error B. Proportional error C. Random error D. Bias

Correct Answer: B 41. B Proportional error (slope or percent error) results in greater absolute error (deviation from the target value) at higher sample concentration. Constant error refers to a difference between the target value and the result, which is independent of sample concentration. For example, if both level 1 and level 2 controls for laboratory A average 5 mg/dL below the cumulative mean reported by all other laboratories using the same method, then laboratory A has a constant error of -5 mg/dL for that method.

Which of the following would cause a "response" error from an ion-selective electrode for sodium when measuring serum but not the calibrator? A. Interference from other electrolytes B. Protein coating the ion-selective membrane C. An overrange in sodium concentration D. Protein binding to sodium ions

Correct Answer: B 41. B Response is the time required for an electrode to reach maximum potential. Ion-selective analyzers use a microprocessor to monitor electrode response, slope, drift, and noise. When an electrode gives an acceptable response time when measuring an aqueous calibrator, but not when measuring serum, the cause is often protein buildup on the membrane.

Which of the following statements regarding thyroid hormones is true? A. Both protein-bound and free T3 and T4 are physiologically active B. Total T3 and T4 are influenced by the level of thyroxine-binding globulin C. Variation in thyroxine-binding protein levels affects both free T3 and T4 D. An elevated serum total T4 and T3 is diagnostic of hyperthyroidism

Correct Answer: B 41. B Total serum T4 and T3 are dependent upon both thyroid function and the amount of thyroxine-binding proteins such as thyroxine-binding globulin (TBG). Total T4 or T3 may be abnormal in a patient with normal thyroid function, if the TBG level is abnormal. For this reason, free T3 and T4 are more specific indicators of thyroid function than are measurements of total hormone. Only free hormone is physiologically active.

Select the products formed from the forward reaction of AST. A. Alanine and α-ketoglutarate B. Oxaloacetate and glutamate C. Aspartate and glutamine D. Glutamate and NADH

Correct Answer: B 42. B AST forms oxaloacetate and glutamate from aspartate and α-ketoglutarate (2-oxoglutarate). Both transaminases use α-ketoglutarate and glutamate as a common substrate and product pair. Both aspartate and alanine can be used to generate glutamate in the central nervous system, where it acts as a neurotransmitter.

Which formula provides the best estimate of serum TIBC? A. Serum transferrin in mg/dL × 0.70 = TIBC (μg/dL) B. Serum transferrin in mg/dL × 1.43 = TIBC (μg/dL) C. Serum iron (μg/dL)/1.2 + 0.06 = TIBC (μg/dL) D. Serum Fe (μg/dL) × 1.25 = TIBC (μg/dL)

Correct Answer: B 43. B Transferrin, a β-globulin, has a molecular size of about 77,000. Transferrin is the principal iron transport protein, and TIBC is determined by the serum transferrin concentration. One mole of transferrin binds two moles of Fe+3, so the transferrin concentration can be used to predict the TIBC. Since the direct measurement of TIBC requires manual pretreatment to remove the excess iron added and is prone to overestimation if all of the unbound iron is not removed, some labs prefer to measure transferrin immunochemically and calculate TIBC. This formula may underestimate TIBC because albumin and other proteins will bind iron when the percent iron saturation of transferrin is abnormally high.

Which element is reduced at the cathode of a Clark polarographic electrode? A. Silver B. Oxygen C. Chloride D. Potassium

Correct Answer: B 44. B The Clark electrode is designed to measure oxygen. O2 diffuses through a gas-permeable membrane covering the electrode. It is reduced at the cathode, which is made of platinum or other inert metal. Electrons are supplied by the anode, which is made of silver. The net reaction is: 4 KCl + 2 H2O + O2 + 4 Ag° →4 AgCl + 4 KOH

Which statement best summarizes the relationship between the new BUN method and reference method based upon the following linear regression scatterplot? -See Harr pg 244 A. The methods agree very well but show a high standard error of estimate B. There is little or no constant error, but some proportional error C. There will be a significant degree of uncertainty in the regression equation D. There is significant constant and proportional error but little random error

Correct Answer: B 44. B The scatterplot shows that each sample produces a coordinate (x corresponds to the reference result and y to the candidate method result) that is very close to the regression line. This means that the variance of regression is low and there is a high degree of certainty that the predicted value of y will be close to its measured value. Near-zero concentration there is good agreement between methods; however, the higher the result, the greater the difference between x and y. The regression equation for this scatterplot is y = -0.01 + 0.90 x, indicating a proportional error of -10%.

Which of the following statements accurately characterizes the coulometric titration of chloride? A. The indicator electrodes generate voltage B. Constant current must be present across the generator electrodes C. Silver ions are formed at the generator cathode D. Chloride concentration is inversely proportional to titration time

Correct Answer: B 45. B The Cotlove chloridometer is based upon the principle of coulometric titration with amperometric detection. Charge in the form of silver ions is generated by oxidation of silver wire at the generator anode. Silver ions react with chloride ions, forming insoluble silver chloride (AgCl). When all of the chloride is titrated, free silver ions are detected by reduction back to elemental silver, which causes an increase in current across the indicator electrodes (a pair of silver electrodes with a voltage difference of about 1.0 V DC). Charge or titration time is directly proportional to chloride concentration as long as the rate of oxidation remains constant at the generator anode.

What is the purpose of LD in the kinetic method of Henry for AST? A. Forms NADH, enabling the reaction to be monitored at 340 nm B. Rapidly exhausts endogenous pyruvate in the lag phase C. Reduces oxaloacetate, preventing product inhibition D. Generates lactate, which activates AST

Correct Answer: B 46. B Patients with liver disease often have high levels of pyruvate and LD. The LD can catalyze the reaction of pyruvate with NADH in the substrate, forming NAD+ and lactate. This would give a falsely high rate for AST because NAD+ is the product measured. Adding LD to the substrate causes pyruvate to be depleted in the first 30 seconds, before AST and MD reactions reach steady state.

In the coulometric chloride titration: A. Acetic acid in the titrating solution furnishes the counter ion for reduction B. The endpoint is detected by amperometry C. The titrating reagent contains a phosphate buffer to keep pH constant D. Nitric acid (HNO3) is used to lower the solubility of AgCl

Correct Answer: B 46. B Reduction of Ag+ back to Ag° generates the current, which signals the endpoint. The titrating reagent contains HNO3, acetic acid, H2O, and either gelatin or polyvinyl alcohol. The HNO3 furnishes nitrate, which is reduced at the generator cathode, forming ammonium ions. The ammonium becomes oxidized back to nitrate at the indicator anode. Gelatin or polyvinyl alcohol is needed to prevent pitting of the generator anode. Acetic acid lowers the solubility of AgCl, preventing dissociation back to Ag+.

SITUATION: Biochemistry tests are performed 24 hours apart on a patient and delta-check flag is reported for inorganic phosphorus by the laboratory information system. Given the results shown in the table above, identify the most likely cause. A. Results suggest altered metabolic status caused by poor insulin control B. The patient was not fasting when the sample was collected on day 2 C. The samples were drawn from two different patients D. The delta-check limit is invalid when samples are collected 24 or more hours apart

Correct Answer: B 46. B The delta check compares the difference of the patient's two most recent laboratory results within a 3-day period to a delta limit usually determined as a percentage difference. The purpose of the delta check is to detect sample identification errors. A delta-check flag can also be caused by random analytical errors and interfering substances such as hemolysis, icterus, and lipemia, and by metabolic changes associated with disease or treatment. Therefore, results should be carefully considered before determining the cause. In this case, hemolysis and icterus can be ruled out because enzymes sensitive to hemolysis interference (AST, ALT, and LD) and bilirubin are within normal limits. Tests showing a significant difference are inorganic phosphorus, ALP, triglycerides, and glucose. These four tests are elevated by diet (the ALP from postprandial secretion of intestinal ALP). All other tests show a high level of agreement between days, and the differences are attributable to normal physiological and analytical variation.

One mole per kilogram H2O of any solute will cause all of the following except: A. Lower the freezing point by 1.86°C B. Raise vapor pressure by 0.3 mm Hg C. Raise the boiling point by 0.52°C D. Raise osmotic pressure by 22.4 atm

Correct Answer: B 49. B Both freezing point and vapor pressure are lowered by increasing solute concentration. Boiling point and osmotic pressure are raised. Increasing solute concentration of a solution opposes a change in its physical state and lowers the concentration of H2O molecules.

Which enzyme deficiency is most commonly associated with familial hypertriglyceridemia associated with fasting plasma cholomicrons (formerly type I hyperlipoproteinemia)? A. β Glucocerebrosidase deficiency B. Post-heparin-activated lipoprotein lipase deficiency C. Apo-B deficiency D. Apo-C-III deficiency

Correct Answer: B 49. B Deficiency of capillary endothelial lipase is the most common cause of fasting chylomicronemia. This lipase is also known as post-heparin- activated lipase and apo C-II-activated lipase. β Glucocerebrosidase deficiency results in accumulation of glucocerebrosides and is the cause of Gaucher's disease. ApoC-II deficiency results in decreased activity of peripheral and hepatic lipases and is associated with hypertriglyceridemia. Apo-B deficiency resulting from a point mutation in the apo-B gene, is responsible for hypobetalipoproteinemia, and is inherited as an autosomal dominant trait. LDL levels are about half normal in heterozygotes, and this reduces their risk of coronary artery disease.

Which statement about TSH and T4 in early pregnancy is correct? A. TSH and thyroid hormones fall B. TSH falls and thyroid hormones rise C. TSH and thyroid hormones both rise D. TSH rises and thyroid hormones fall

Correct Answer: B 49. B Estrogens released in pregnancy cause an increase in TBG, which causes an increase in total T4 and T3. In early pregnancy, the hCG produced by the placenta stimulates the thyroid, causing an increase in free thyroid hormones. This suppresses TSH production. In the second trimester, as hCG diminishes, free T4 levels fall, and may be lower than 0.8 ng/dL, the lower limit of the adult reference range due to expansion of the blood volume. Therefore, both TSH and free T4 should be evaluated during pregnancy using trimester-specific reference ranges. In early pregnancy, a TSH above the first- trimester reference range should be followed up with free T4 and thyroid peroxidase antibody levels to assess the need for thyroid treatment.

A green-colored solution would show highest transmittance at: A. 475 nm B. 525 nm C. 585 nm D. 620 nm

Correct Answer: B 5. B Green light consists of wavelengths from 500-550 nm. A green-colored solution with a transmittance maximum of 525 nm and a 50-nm bandpass transmits light of 525 nm and absorbs light below 475 nm and above 575 nm. A solution that is green would be quantitated using a wavelength that it absorbs strongly, such as 450 nm.

SITUATION: A serum osmolality measured in the emergency department is 326 mOsm/kg. Two hours later, chemistry results are: -Na = 135 mmol/L -Blucose = 72 mg/dL -BUN = 18 mg/dL -Measured osmolality = 318 mOsm/kg What do these results suggest? A. Laboratory error in electrolyte or glucose measurement B. Drug or alcohol intoxication C. Specimen misidentification D. Successful rehydration of the patient

Correct Answer: B 5. B The osmolal gap is the difference between calculated and measured osmolality. Here, the osmolal gap is 38 mOsm/kg. When the osmolal gap is greater than 10 mOsm/kg, an unmeasured solute is present or an analytical error occurred when measuring the osmolality, electrolytes, urea, or glucose. The reference range for serum osmolality is 280-295 mOsm/kg. Both osmolality measurements are above the URL. These results point to the presence of an unmeasured solute. A significant osmolal gap in samples from emergency department patients usually results from alcohol or drug consumption. The difference in osmolality between the two samples is 8 mOsm/kg and can be explained by alcohol metabolism during the 2 hours between samples.

A laboratory is establishing a reference range for a new analyte and wants the range to be determined by the regional population of adults age 18 and older. The analyte concentration is known to be independent of race and gender. Which is the most appropriate process to follow? A. Determine the mean and standard deviation of the analyte from 40 healthy adults and calculate the ±2s limit B. Measure the analyte in 120 healthy adults and calculate the central 95th percentile C. Measure the analyte in 120 healthy adults and use the lowest and highest as the reference range limits D. Measure the analyte in 60 healthy adults and 60 adults with conditions that affect the analyte concentration; calculate the concentration of least overlap

Correct Answer: B 50. B Since the concentration of an analyte may not be normally distributed in a population, the reference range should not be determined from the standard deviation. It is more appropriate to determine the central 95th percentile (the range that encompasses 95% of the results). A minimum of 120 samples is needed for statistical significance. Results are rank ordered from lowest to highest. The 3rd result is the lowest value and the 118th is the highest value in the reference range. The laboratory can verify a preexisting reference range (e.g., as determined by the manufacturer's study) by testing 20 healthy persons. If no more than 10% fall outside the range, it can be considered valid for the patient population.

Which of the following processes is part of the normal metabolism of bilirubin? A. Both conjugated and unconjugated bilirubin are excreted into the bile B. Methene bridges of bilirubin are reduced by intestinal bacteria forming urobilinogens C. Most of the bilirubin delivered into the intestine is reabsorbed D. Bilirubin and urobilinogen reabsorbed from the intestine are mainly excreted by the kidneys

Correct Answer: B 50. B UDP-glucuronyl transferase esterifies glucuronic acid to unconjugated bilirubin, making it water soluble. Most conjugated bilirubin is diglucuronide; however, the liver makes a small amount of monoglucuronide and other glycosides. β-Glucuronidase hydrolyzes glucuronide from bilirubin, hormones, or drugs. It is used prior to organic extraction to deconjugate urinary metabolites (e.g., total cortisol). Biliverdin reductase forms bilirubin from biliverdin (and heme oxygenase forms biliverdin from heme). Bilirubin oxidase is used in an enzymatic bilirubin assay in which bilirubin is oxidized back to biliverdin and the rate of biliverdin formation is measured at 410 nm.

Which enzyme is responsible for the conjugation of bilirubin? A. β-Glucuronidase B. UDP-glucuronyl transferase C. Bilirubin oxidase D. Biliverdin reductase

Correct Answer: B 50. B UDP-glucuronyl transferase esterifies glucuronic acid to unconjugated bilirubin, making it water soluble. Most conjugated bilirubin is diglucuronide; however, the liver makes a small amount of monoglucuronide and other glycosides. β-Glucuronidase hydrolyzes glucuronide from bilirubin, hormones, or drugs. It is used prior to organic extraction to deconjugate urinary metabolites (e.g., total cortisol). Biliverdin reductase forms bilirubin from biliverdin (and heme oxygenase forms biliverdin from heme). Bilirubin oxidase is used in an enzymatic bilirubin assay in which bilirubin is oxidized back to biliverdin and the rate of biliverdin formation is measured at 410 nm.

Which of the following conditions will cause erroneous Cai results? (Assume that the samples are collected and stored anaerobically, kept at 4°C until measurement, and stored for no longer than 1 hour.) A. Slight hemolysis during venipuncture B. Assay of whole blood collected in sodium oxalate C. Analysis of serum in a barrier gel tube stored at 4°C until the clot has formed D. Analysis of whole blood collected in sodium heparin, 20 U/mL (low-heparin tube)

Correct Answer: B 53. B Unlike Pi, the intracellular calcium level is not significantly different from plasma calcium, and calcium is not greatly affected by diet. Whole blood collected with 5-20 U/mL heparin and stored on ice no longer than 2 hours is the sample of choice for Cai. Blood gas syringes prefilled with 100 U/mL heparin should not be used because the high heparin concentration will cause low results. Citrate, oxalate, and ethylenediaminetetraacetic acid (EDTA) must not be used because they chelate calcium. Serum may be used provided that the sample is iced, kept capped while clotting, and assayed within 2 hours (barrier gel tubes may be stored longer).

When measuring calcium with the complexometric dye o-cresolphthalein complexone, magnesium is kept from interfering by: A. Using an alkaline pH B. Adding 8-hydroxyquinoline C. Measuring at 450 nm D. Complexing to EDTA

Correct Answer: B 55. B o-Cresolphthalein complexone can be used to measure either magnesium or calcium. Interference in calcium assays is prevented by addition of 8-hydroxyquinoline, which chelates magnesium. When magnesium is measured, ethyleneglycol bistetraacetic acid (EGTA) or EDTA is used to chelate calcium. Two other dyes that can be used for both magnesium and calcium assays are calmagite and methylthymol blue. Arsenazo III dye is commonly used to measure calcium. It is more specific for Ca+2 than the others, and does not require addition of a Mg+2 chelator.

In hepatitis, the rise in serum conjugated bilirubin can be caused by: A. Secondary renal insufficiency B. Failure of the enterohepatic circulation C. Enzymatic conversion of urobilinogen to bilirubin D. Extrahepatic conjugation

Correct Answer: B 57. B Conjugated bilirubin is increased in hepatitis and other causes of hepatic necrosis due to failure to re-excrete conjugated bilirubin reabsorbed from the intestine. Increased direct bilirubin can also be attributed to accompanying intrahepatic obstruction, which blocks the flow of bile.

The most commonly used detector for clinical gas-liquid chromatography (GLC) is based upon: A. Ultraviolet light absorbance at 254 nm B. Flame ionization C. Refractive index D. Thermal conductance

Correct Answer: B 58. B Volatile solutes can be detected in GLC using flame ionization, thermal conductivity, electron capture, and mass spectroscopy. In flame ionization, energy from a flame is used to excite the analytes as they elute from the column. The flame is made by igniting a mixture of hydrogen, carrier gas, and air. Current is produced when an outer shell electron is ejected from the excited analyte.

Which method is considered the candidate reference method for triglyceride measurement? A. Glycerol kinase-ultraviolet B. CDC modification of van Handel and Zilversmit C. Hantzsch condensation D. Glycerol kinase coupled to peroxidase

Correct Answer: B 59. B Enzymatic methods for triglyceride measurement are widely used because they eliminate the need for extraction and saponification. However, they are subject to positive interference from endogenous glycerol and variations in the efficiency of lipase, which can result in under- or overestimation of triglycerides. The most accurate method for triglyceride assay is the nonenzymatic method based upon reaction of formaldehyde with chromotropic acid. In this method, extraction with silicic acid and chloroform separates triglycerides from lipoproteins, phospholipids, and glycerol. Saponification with alcoholic potassium hydroxide (KOH) produces glycerol, which is oxidized to formaldehyde by periodate. The formaldehyde reacts with chromotropic acid to form a pink product.

Select the main estrogen produced by the ovaries and used to evaluate ovarian function. A. Estriol (E3 ) B. Estradiol (E2 ) C. Epiestriol D. Hydroxyestrone

Correct Answer: B 6. B E2 is the major estrogen produced by the ovaries and gives rise to both estrone (E1) and E3. E2 is used to evaluate both ovarian function and menstrual cycle dysfunction.

Which of the following is the primary mechanism for vasopressin (ADH) release? A. Hypovolemia B. Hyperosmolar plasma C. Renin release D. Reduced renal blood flow

Correct Answer: B 61. B ADH is released by the posterior pituitary in response to increased plasma osmolality. Normally, this is triggered by release of aldosterone caused by ineffective arterial pressure in the kidney. Aldosterone causes sodium reabsorption, which raises plasma osmolality; release of ADH causes reabsorption of water, which increases blood volume and restores normal osmolality. A deficiency of ADH (diabetes insipidus) results in dehydration and hypernatremia. An excess of ADH (syndrome of inappropriate ADH release [SIADH]) results in dilutional hyponatremia. This may be caused by regional hypovolemia, hypothyroidism, central nervous system injury, drugs, and malignancy.

Which statement best characterizes serum bilirubin levels in the first week following delivery? A. Serum bilirubin 24 hours after delivery should not exceed the upper reference limit for adults B. Jaundice is usually first seen 48-72 hours postpartum in neonatal hyperbilirubinemia C. Serum bilirubin above 5.0 mg/dL occurring 2-5 days after delivery indicates hemolytic or hepatic disease D. Conjugated bilirubin accounts for about 50% of the total bilirubin in neonates

Correct Answer: B 62. B Bilirubin levels may reach as high as 2-3 mg/dL in the first 24 hours after birth owing to the trauma of delivery, such as resorption of a subdural hematoma. Neonatal hyperbilirubinemia occurs 2-3 days after birth due to increased hemolysis at birth and transient deficiency of the microsomal enzyme, UDP-glucuronyl transferase. Normally, levels rise to about 5-10 mg/dL but may be greater than 15 mg/dL, requiring therapy with UV light to photo-oxidize the bilirubin. Neonatal jaundice can last up to 1 week in a mature neonate and up to 2 weeks in prematures babies. Neonatal bilirubin is almost exclusively unconjugated.

Which form of jaundice occurs within days of delivery and usually lasts 1-3 weeks, but is not due to normal neonatal hyperbilirubinemia or hemolytic disease of the newborn? A. Gilbert syndrome B. Lucey -Driscoll syndrome C. Rotor syndrome D. Dubin-Johnson syndrome

Correct Answer: B 63. B Lucey-Driscoll syndrome is a rare form of jaundice caused by unconjugated bilirubin that presents within 2-4 days of birth and can last several weeks. It is caused by an inhibitor of UDP-glucuronyl transferase in maternal plasma that crosses the placenta. Jaundice is usually severe enough to require treatment.

Which of the following values is the threshold critical value (alert or action level) for high plasma sodium? A. 150 mmol/L B. 160 mmol/L C. 170 mmol/L D. 180 mmol/L

Correct Answer: B 63. B The adult reference range for plasma sodium is approximately 135-145 mmol/L. Levels in excess of 160 mmol/L are associated with severe dehydration, hypovolemia, and circulatory and heart failure. The threshold for the low critical value for sodium is 120 mmol/L. This is associated with edema, hypervolemia, and circulatory overload. Alert levels must also be established for potassium, bicarbonate, calcium, pH, PO2, glucose, bilirubin, hemoglobin, platelet count, and prothrombin time. When a sample result is below or above the low or high alert level, respectively, the physician must be notified immediately.

Cholesterol esterase is used in enzymatic assays to: A. Oxidize cholesterol to form peroxide B. Hydrolyze fatty acids bound to the third carbon atom of cholesterol C. Separate cholesterol from apoproteins A-I and A-II by hydrolysis D. Reduce NAD+ to NADH

Correct Answer: B 64. B Approximately two-thirds of the serum cholesterol has a fatty acid esterified to the hydroxyl group of the third carbon atom of the cholesterol molecule. Cholesterol esterase hydrolyzes fatty acids and is required because cholesterol oxidase cannot utilize esterified cholesterol as a substrate.

The fragments typically produced and analyzed in methods employing mass spectroscopy are typically: A. Of low molecular size ranging from 10-100 daltons B. Cations caused by electron loss or proton attachment C. Anions caused by bombarding the molecule with an electron source D. Neutral species formed after excited molecules form a stable resonance structure

Correct Answer: B 66. B In almost all MS applications, cations of the molecule are measured. Cations can be formed by various methods, the most common of which is electron bombardment (electron ionization). The energy transferred to the molecule causes ejection of an outer shell electron. MS can analyze sizes from trace metals through macromolecules. Proteins are measured following conversion to cations by ionization procedures such as matrix-assisted laser desorption ionization (MALDI) in which energy from a nitrogen laser causes transfer of a proton from the matrix (an acid) to the protein.

What do "direct" or homogenous methods for LDL cholesterol assay have in common? A. They are inaccurate when plasma triglyceride is above 250 mg/dL B. All use a detergent to facilitate selective reactivity with reagent enzymes C. All use monoclonal antibodies to apo A1 and C D. All are free of interference from abnormal lipoproteins

Correct Answer: B 66. B The direct LDL cholesterol assays are all detergent based methods. One commonly used method employs a polyanionic detergent to release cholesterol from HDL, chylomicrons, and VLDL. The detergent binds to LDL and blocks its reaction with the esterase and oxidase enzymes in the reagent. Cholesterol oxidase oxidizes the non-LDL cholesterol, forming H2O2, and peroxidase catalyzes the oxidation of an electron donor by the H2O2,which does not result in color formation. A second nonionic detergent and chromogen is added. The second detergent removes the first from the LDL, allowing it to react with the enzymes. The resulting H2O2 reacts with the chromogen, forming a colored product.

SITUATION: A lipemic specimen collected from an adult after a 12-hour fast was assayed for total cholesterol, triglycerides, and HDL cholesterol using a direct HDL method. Following are the results: -Total cholesterol = 220 mg/dL -HDL cholesterol = 40 mg/dL -Triglyceride = 420 mg/dL The physician requests an LDL cholesterol assay after receiving the results. How should the LDL cholesterol be determined? A. Dilute the specimen 1:10 and repeat all tests; calculate LDL cholesterol using the Friedewald equation B. Perform a direct LDL cholesterol assay C. Ultracentrifuge the sample and repeat the HDL cholesterol on the infranate. Use the new result to calculate the LDL cholesterol D. Repeat the HDL cholesterol using the manganese heparin precipitation method. Use the new result to calculate the LDL cholesterol

Correct Answer: B 69. B An accurate LDL cholesterol can be reported, if the direct (detergent) method for LDL cholesterol is employed. These methods are not subject to interference by triglycerides at a concentration below 700 mg/dL.

In mass spectroscopy, the term base peak typically refers to: A. The peak with the lowest mass B. The peak with the most abundance C. A natural isotope of the molecular ion D. The first peak to reach the mass detector

Correct Answer: B 69. B The base peak is typically the "molecular ion" or parent ion, meaning that it is the initial fragment made by releasing an electron. The cation thus formed has a charge of +1, and therefore, its m/z ratio is equal to its mass. The base peak is used for selective ion monitoring (SIM). It is the most abundant and most stable ion, and gives the best sensitivity for quantitative analysis.

What substance may be measured as an alternative to creatinine for evaluating GFR? A. Plasma urea B. Cystatin C C. Uric acid D. Potassium

Correct Answer: B 7. B Although all of the analytes listed are increased in chronic kidney disease as a result of low GFR, potassium, urea, and uric acid may be increased by other mechanisms and therefore, they are not specific for glomerular function. Cystatin C is an inhibitor of cysteine proteases. Being only 13 kilodaltons, it is completely filtered by the glomerulus then reabsorbed by the tubules. The plasma level is highly correlated to GFR because little is eliminated by nonrenal routes. Plasma levels are not influenced by diet, age, gender, or nutritional status. Low GFR causes retention of cystatin C in plasma and levels become abnormally high at clearance rates below 90 mL/min, making the test more sensitive than creatinine.

When preparing a patient for an oral glucose tolerance test (OGTT), which of the following conditions will lead to erroneous results? A. The patient remains ambulatory for 3 days prior to the test B. Carbohydrate intake is restricted to below 150 g/day for 3 days prior to test C. No food, coffee, tea, or smoking is allowed 8 hours before and during the test D. Administration of 75 g of glucose is given to an adult patient following a 10-12-hour fast

Correct Answer: B 7. B Standardized OGTTs require that patients receive at least 150 grams of carbohydrate per day for 3 days prior to the test in order to stabilize the synthesis of inducible glycolytic enzymes. The 2-hour OGTT test is no longer recommended for screening and should be reserved for confirmation of diabetes in cases that are difficult to diagnose, such as persons who lack symptoms and signs of fasting hyperglycemia.

How many milliliters of a 2,000.0 mg/dL glucose stock solution are needed to prepare 100.0 mL of a 150.0 mg/dL glucose working standard? A. 1.5 mL B. 7.5 mL C. 15.0 mL D. 25.0 mL

Correct Answer: B 7. B To calculate the volume of stock solution needed, divide the concentration of working standard by the concentration of stock standard, then multiply by the volume of working standard that is needed. C1 × V1 = C2 × V2, where C1 = concentration of stock standard V1 = volume of stock standard C2 = concentration of working standard V2 = volume of working standard 2000.0 mg/dL × V1 = 150.0 mg/dL × 100.0 mL V1 = (150.0 ÷ 2000.0) × 100.0 mL V1 = 7.5 mL

Which of the following statements regarding the diagnosis of pancreatitis is correct? A. Amylase and lipase are as predictive in chronic as in acute pancreatitis B. Diagnostic sensitivity is increased by assaying both amylase and lipase C. Measuring the urinary amylase:creatinine ratio is useful only when patients have renal failure D. Serum lipase peaks several hours before amylase after an episode of acute pancreatitis

Correct Answer: B 70. B Amylase is not increased in all persons with pancreatitis and can be increased in several nonpancreatic conditions. Lipase adds both sensitivity and specificity to the diagnosis of acute pancreatitis. Plasma or serum lipase becomes abnormal within 6 hours, peaks at approximately 24 hours, and remains abnormal for about 1 week following an episode of acute pancreatitis. In acute pancreatitis, the rate of urinary amylase excretion increases, and the amylase:creatinine clearance ratio is helpful in diagnosing some cases of pancreatitis. The normal A:C clearance ratio is 1%-4%. In acute pancreatitis, the ratio is usually above 4% and can be as high as 15%. In chronic pancreatitis, acinar cell degeneration often occurs, resulting in loss of amylase and lipase production. This lowers the sensitivity of amylase and lipase in detecting chronic disease to below 50%. Patients with chronic disease have pancreatic insufficiency giving rise to increased fecal fat, and decreased pancreatic digestive enzymes such as trypsin, chymotrypsin, and elastin.

In tandem-mass spectroscopy, the first mass filter performs the same function as: A. The ion source B. The chromatography column C. Extraction D. The vacuum system

Correct Answer: B 71. B A tandem mass spectrometer uses two or more mass filters in sequence. The first filter functions as an ion trap. Once the sample is ionized, the filter selects molecular or parent ions of interest by excluding ions outside a specified size range. Therefore, it effectively separates the analyte(s) of interest from unwanted compounds. Tandem MS uses ESI to introduce the sample into the first mass filter, usually a quadrapole. The RF and DC voltages of the quadrapole are set to optimize the trajectory of the parent ions of interest and cause ejection of unwanted ions. The parent ions are drawn into a second mass filter where they are bombarded by argon atoms. The collisions result in the formation of mass fragments called daughter ions. This process is called collision-induced dissociation and the second filter is called a collision chamber. The process can be repeated in a third mass filter that generates granddaughter ions. A total-ion chromatogram is produced from these, enabling the compound of interest to be identified and quantified. Tandem MS is used to screen for inborn errors of fatty acid, amino acid, and organic acid metabolism.

Which of the following statements regarding amylase methods is true? A. Requires sulfhydryl compounds for full activity B. Activity will vary depending on the method used C. Amyloclastic methods measure the production of glucose D. Overrange samples are diluted in deionized water

Correct Answer: B 72. B Chloride and Ca2+ ions are required for amylase activity. Samples with high activity should be diluted with NaCl to prevent inactivation. Lipase and CK require sulfhydryl activators. Saccharogenic methods measure the production of glucose, while amyloclastic methods measure the degradation of starch. Starch is a polymer of α-D glucose subunits linked together by both α 1-4 and α 1-6 glycosidic bonds. Different lots may have more or less branching owing to the number of α 1-6 bonds. Since amylase hydrolyzes at the α 1-4 sites only, the amount of product measured is influenced by the extent of branching.

Why is vacuum necessary in the mass filter of a mass spectrometer? A. Ionization does not occur at atmospheric pressure B. It prevents collision between fragments C. It removes electrons from the ion source D. It prevents contamination

Correct Answer: B 73. B Vacuum is needed in the mass filter of the MS to prevent random collisions between ions that would alter their trajectory or time of flight. It is also needed in CG-MS instruments that use electron ionization. The vacuum prevents collision between the carrier gas molecules and the ions. In spectrometers that use electrospray ionization, chemical ionization, and laser desorption ionization (MALDI and SELDI TOF), the ion source is not under vacuum.

Which of the following situations is likely to cause an error when weighing with an electronic analytical balance? A. Failure to keep the knife edge clean B. Failure to close the doors of the balance before reading the weight C. Oxidation on the surface of the substitution weights D. Using the balance without allowing it to warm up for at least 10 minutes

Correct Answer: B 79. B Electronic balances do not use substitution weights or knife edges to balance the weight on the pan. Instead, they measure the displacement of the pan by the weight on it using electromagnetic force to return it to its reference position. Regardless of the type of balance used, all need to be located on a firm weighing table free of vibration. Doors must be closed to prevent air currents from influencing the weighing, and the pan and platform must be clean and free of dust and chemical residue.

Which of the following statements regarding the Philadelphia chromosome is true? A. It is seen exclusively in chronic myelogenous leukemia B. It results from a translocation C. It appears as a short-arm deletion of chromosome 21 D. It is associated with a poor prognosis

Correct Answer: B 8. B The Philadelphia chromosome (Ph1) is formed by translocation of the long arms of chromosomes 9 and 22. The result is that part of the ABL gene of chromosome 9 becomes inserted into the BCR gene of chromosome 22. The ABL gene is an oncogene and the product of the hybrid gene is a tyrosine kinase that signals cell proliferation. The Ph1 chromosome appears on karyotyping as a long-arm deletion of chromosome 22 because only the terminal end of the long arm of chromosome 9 is exchanged for most of the long arm of chromosome 22. The BCR/ABL translocation can be detected using FISH hybridization probes. Approximately 95% of persons with chronic myelogenous leukemia have the Ph1 chromosome. Those patients who do not demonstrate Ph1 have a poorer prognosis. It is also present in the lymphocytes of up to 25% of adults with acute lymphocytic leukemia (ALL) and in a small number of children with ALL and persons with acute myelogenous leukemia.

SITUATION: A patient who has a positive urinalysis for glucose and ketones has a glycated Hgb of 4.0%. A fasting glucose performed the previous day was 180 mg/dL. Assuming acceptable QC, you would: A. Report the glycosylated Hgb B. Request a new specimen and repeat the glycosylated Hgb C. Perform a Hgb electrophoresis on the sample D. Perform a glucose measurement on the sample

Correct Answer: B 8. B The glycated Hgb is at the lowest normal limit (4%-5.5%), but the fasting glucose indicates frank diabetes mellitus. Although the glycosylated Hgb reflects the average blood glucose 2-3 months earlier, the value reported is inconsistent with the other laboratory results. A high probability of sample misidentification or analytical error necessitates that the test be repeated.

Select the primary reagent used in the Jaffe method for creatinine. A. Alkaline copper II sulfate B. Saturated picric acid and NaOH C. Sodium nitroprusside and phenol D. Phosphotungstic acid

Correct Answer: B 9. B The Jaffe method uses saturated picric acid, which oxidizes creatinine in alkali, forming creatinine picrate. The reaction is nonspecific; ketones, ascorbate, proteins, and other reducing agents contribute to the final color. Alkaline CuSO4 is used in the biuret method for protein.

Which statement is true regarding the volume distribution (Vd) of a drug? A. Vd is equal to the peak blood concentration divided by the dose given B. Vd is the theoretical volume in liters into which the drug distributes C. The higher the Vd, the lower the dose needed to reach the desired blood level of drug D. The Vd is the principal determinant of the dosing interval

Correct Answer: B 9. B The Vd of a drug represents the dilution of the drug after it has been distributed in the body. The Vd is used to estimate the peak drug blood level expected after a loading dose is given. The peak blood level equals the dose multiplied by f ÷ Vd. The Vd can be calculated by dividing the dose, Xo, by the initial plasma drug concentration, Co, (Vd = Xo/Co) or by dividing the clearance rate by K, the elimination rate constant (K = 0.693 divided by drug half-life). The greater the Vd, the higher the dose that will be needed to achieve the desired blood concentration of drug. The Vd is the principal determinant of the dose, and the clearance rate is the principal determinant of the dosing interval.

Which statement best describes the use of the Hgb A1C test? Peak;Calibrated % Area; % Area; Retention Time; Peak Area -Alb 0.60; 0.25; 12500 -F 0.50; 0.50; 11300 -LA1c 0.75; 0.70; 15545 -A1c 6.2; 0.90; 45112 -P3 2.6; 1.60; 57489 -Ao 48.0; 1.8; 994813 -C 43.0; 2.00; 926745 A. Should be used for monitoring glucose control only B. May be used for both diagnosis and monitoring C. Should be used only to monitor persons with type 1 diabetes D. May be used only to monitor persons with type 2 diabetes

Correct Answer: B B The ADA now recommends that the hemoglobin A1c test be used for both diagnosis and monitoring blood glucose levels. The cutpoint for diabetes is an A1c of 6.5. Persons with an A1c of 5.7%-6.4% are classified as being at high risk for diabetes within 5 years. An A1c between 4.0%-5.5% is defined as within normal limits.

A diagnosis of primary adrenal insufficiency requires demostration of: A. decreased urinary 17-keto- and 17-hydroxysteroids B. decreased cortisol production C. impaired response to ACTH stimulation D. increased urinary cortisol excretion after metyrapone

Correct Answer: C

A fresh urine sample is received for analysis for "porphyrins" or porphyria: without further information of specifications. Initial analysis should include: A. porphyrin screen and quantitative total porphyrin B. quantitative total porphyrin and porphobilinogen screen C. porphyrin and porphobilinogen screen D. porphobilinogen screen and ion-exchange analysis for porphobilinogen

Correct Answer: C

A patient had the following serum results; -Na+: 140 mEq/L (140 mmol/L) -K+: 4.0 mEq/L (4.0 mmol/L) -glucose: 95 mg/dL (5.2 mmol/L) -BUN: 10 mg/dL (3.57 mmol/L) Which osmolality is consisitent with these results? A. 188 B. 204 C. 270 D. 390

Correct Answer: C

A patient has signs and symptoms suggestive of acromegaly. The diagnosis would be confirmed if the patient had which of the following? A. an elevated serum phosphate concentration B. a decrease serum growth hormone releasing factor concentration C. no decrease in serum growth hormone concentration 90 minutes after oral glucose administration D. an increased serum somatostatin concentration

Correct Answer: C

A patient with myeloproliferative disorder has the following values: -Hgb: 13 g/dL (130 mmol/L) -Hct: 38% -WBC: 30x 10^3/uL (30 x 10^9/L) -platelets: 1000 x 10^3/uL (1000x 10^9L) -serum Na+: 140 mEq/L (140 mmol/L) -serum K+: 7 mEq/L (7 mmol/L) The serum K+ should be confirmed by: A. repeat testing of the original serum B. testing freshly drawn serum C. testing heparinized plasma D. atomic absorption spectrometry

Correct Answer: C

After a difficult venipuncture requireing prolonged application of the tourniquet, the serum K+ was found to be 6.9 mEq/L (6.9 mmol/L). The best course of action is to: A. repeat the test using the same specimen B. adjust the value based on the current serum Na+ C. repeat the test using freshly drawn serum D. cancel the test

Correct Answer: C

An adult diabetic with renal complications has the following results: -sodium: 133 mEq/L (133 mmol/L) -glucose: 487 mg/dL (26.8 mmol/L) -BUN: 84 mg/dL (30.0 mmol/L) -creatinine: 5 mg/dL (442.0 umol/L) On the basis of these results, the calculated serum osmolality is: A. 266 mOsm/kg B. 290 mOsm/kg C. 323 mOsm/kg D. 709 mOsm/kg

Correct Answer: C

An electrode has a silver/silver chloride anode and a platinum wire cathode. It is suspended in KCl solution and separated form the blood to be analyzed by a selectively permeable membrane. Such an electrode is used to measure which of the following? A. pH B. PCO2 C. PO2 D. HCO3

Correct Answer: C

Biochemical profile: Test; Patient values: reference range: -total protein: 7.3 g/dL (73 g/L); 6.0-8.0 g/dL (60-80 g/L) -albumin: 4.1 g/dL (41 g/L); 3.5-5.0 g/dL (35-50 g/L) -calcium: 9.6 mg/dL ( 2.4 mmol/L); 8.5-10.5 mg/dL (2.1-2.6 mmol/L) -phosphorus: 3.3 mg/dL (1.06 mmol/L); 2.5-4.5 mg/dL (0.80-1.45 mmol/L) -glucose: 95 mg/dL (5.2 mmol/L); 65-110 mg/dL (3.6-6.1 mmol/L) -BUN: 16 mg/dL (5.71 mmol/L); 10-20 mg/dL (3.57-7.1 mmol/L) -uric acid: 6.0 mg/dL (356.9 umol/L); 2.5-8.0 mg/dL (148.70475.8 umool/L) -creatinine: 1.2 mg/dL (106.1 umol/L); 0.7-1.4 mg/dL (61.9-123. 8 umol/L) -total bilirubin: 3.7 mg/dL (63.3 umol/L); 0.2-0.9 mg/dL (3.4-15.4 umol/L) -alkaline phosphatase: 275 U/L; 30-80 U/L -lactate dehydrogenase: 185 U/L; 100-225 U/L -AST: 75 U/L, 10-40 U/L The results the biochemical profile are most consistent with: A. viral hepatitis B. hemolytic anemia C. common bile duct stone D. chronic active hepatitis

Correct Answer: C

Blood received in the laboratory for blood gas analysis must meet which of the following requirements? A. on ice, thin fibrin strands only, no air bubbles B. on ice, no clots, fewer than 4 air bubbles C. on ice, no clots, no air bubbles D. room temperature, not clots, no air bubbles

Correct Answer: C

Fasting serum phosphate concentration is controlled primarily by the: A. pancreas B. skeleton C. parathyroid glands D. small intestine

Correct Answer: C

In addition to carcinoma of the prostate, elevated prostate-specific antigen (PSA) can occur due to: A. aspirin therapy B. exogenous steroid use C. benign prostatic hyperplasia D. statin therapy (cholesterol lowering drug)

Correct Answer: C

In amniotic fluid, the procedure used to detect Rh isosensitization is: A. human amniotic placental lactogen (HPL) B. alpha-fetoprotein C. measurement of absorbance at 450 nm D. creatinine

Correct Answer: C

In monitoring glomerular function, which of the following test has the highest sensitivity? A. urine sodium B. BUN/creatinine ration C. creatine clearance D. urea clearance

Correct Answer: C

In spectrophotometric determination, which of the following is the formula for calculating the absorbance of a solution? A. (absorptivity x light path)/ concentration B. (absorptivity x concentration)/ light path C. absorptivity x light path x concentration D. (light path x concentration)/ absorptivity

Correct Answer: C

In the International System of Units, serum urea is expressed in millimoles per liter. urea; NH2CONH2 atomic weight: N=14 C=12 O=16 H=1 A serum urea nitrogen concentration of 28 mg/dL would be equivalent to what concentrations of urea? A. 4.7 mEq/L B. 5.0 mEq/L C. 10.0 mEq/L D. 20.0 mEq/L

Correct Answer: C

Increased concentrations of ascorbic acid inhibit chromogen production in which of the following glucose methods? A. ferricyanide B. ortho-toluidine C. glucose oxidase (peroxidase) D. hexokinase

Correct Answer: C

Lactate dehydrogenase, malate dehydrogenase, isocitrate dehydrogenase, and hydroxybutyrate dehydrogenase all: A. are liver enzymes B. are cardiac enzymes C. catalyze oxidation-reduction reactions D. are class III enzymes

Correct Answer: C

Maple syrup urine disease is characterized by an increase in which of the following urinary amino acids? A. phenylalanine B. tyrosine C. valine, leucine and isoleucine D. cystine and cysteine

Correct Answer: C

Most of the carbo dioxide present in blood is in the form of: A. dissolved CO2 B. carbonate C. bicarbonate ion D. carbonic acid

Correct Answer: C

Normally the bicarbonate concetration is about 24 mEq/L and the carbonic acid concentration is about 1.2, pK=6.1, log 20= 1.3. Using the equation pH= pK + log [salt]/[acid], calculate the pH. A. 7.28 B. 7.38 C. 7.40 D. 7.42

Correct Answer: C

On electrophoresis at alkaline pH, which of the following is the slowest migrating hemoglobin? A. Hgb A B. Hgb S C. Hgb C D. Hgb F

Correct Answer: C

One international unit of enzyme activity is the amount of enzyme that will, under specified reaction conditions of substrate concentration, pH and temperature, cause utilization of substrate at the rate of: A. 1 mol/min B. 1 mmol/min C. 1 umol/min D. 1 nmol/min

Correct Answer: C

Refer to the following illustration: See BOC pg 120 Pic 1 The above figure shows the reciprocal of the measured velocity of an enzyme reaction plotted against the reciprocal of the substrate concentration. True statement about this figure include: A. the intercept of the line on the abscissa (x-axis) can be used to calculate the Vmax B. the straight line indicates that the enzyme reaction proceeds according to zero order kinetics C. the intercept on the abscissa (x-axis) can be used to calculate the Michaelis-Menten constant D. the fact that the substrate concentration is plotted on both sides of the zero point indicates that the reaction is reversible

Correct Answer: C

Serum and urine copper levels are assayed on a hospital patient with the following results: Patient values; Reference values -serum Cu: 20 ug/dL (3.1 umol/L); 70-140 ug/dL (11.0-22.0 umol/L) -urine Cu: 83 ug/dL (13.0 umol/L); < 40 ug/dL (<63 umol/L) This is most consistent with: A. normal copper levels B. Wilms tumor C. Wilson disease D. Addison disease

Correct Answer: C

Serum haptoglobin: A. is decreased in patients with tissue injury and neoplasia B. is increased in patients with prosthetic heart valves C. can be separated into distinct phenotypes by starch-gel electrophoresis D. binds heme

Correct Answer: C

Testing for the diagnosis of lead poisoning should include: A. erythrocyte protoporphyrin (EPP) B. urine delta-aminolevulinic acid C. whole blood lead D. zinc protoporphyrin (ZPP)

Correct Answer: C

The anion gap is useful for quality control of laboratory results for: A. amino acids and proteins B. blood gas analyses C. sodium, potassium, chloride, and total CO2 D. calcium, phosphorus and magnesium

Correct Answer: C

The chemical composition of HDL-cholesterol corresponds to: Triglyceride; Cholesterol; Protein A. 60%; 15%; 10% B. 10%; 45%; 25% C. 5%; 15%; 50% D. 85%; 5%; 2%

Correct Answer: C

The drug procainamide is prescribed to treat cardiac arrhythmia. What biologically active liver metabolite of procainamide is often measured simultaneously? A. phenobarbitol B. quinidine C. N-acetyl procainamide (NAPA) D. lidocaine

Correct Answer: C

The followign bilirubin result are obtained on a patient: -day 1: 4.3 mg/dL (73. 5 umol/L) -day 2: 4.6 mg/dL (78.7 umol/L) -day 3: 4.5 mg/dL (77.0 umol/L) -day 4: 2.2 mg/dL (37.6 umol/L) -day 5: 4.4 mg/dL (75.2 umol/L) -day 6: 4.5 mg/dL (77.0 umol/L) Given that the controls were within range each day, what is a probable explanation for the result on day 4? A. no explanation necessary B. serum, not plasma, was used for testing C. specimen had prolonged exposure to light D. specimen was hemolyzed

Correct Answer: C

The following blood gas results were obtained: -pH: 7.18 -PO2: 86 mm Hg -PCO2: 60 mm Hg -O2 saturation: 92% -HCO3: 7921 mEq/L (21 mmol/L) -TCO2: 23 mEq/L (23 mmol/L) -base excess: 8 mEq/L (-8.0 mmol/L) The patient's results are compatible with which of the following? A. fever B. uremia C. emphysema D. dehydration

Correct Answer: C

The following results are form a 21-year-old patient with a back injury who appears otherwise healthy: -Whole blood glucose: 77 mg/dL (4.2 mmol/L) -Serum glucose: 88 mg/dL (4.8 mmol/L) -CSF glucose: 56 mg/dL (3.1 mmol/L) THe best interpretation of these results is that: A. the whole blood and serum values are expected both the CSF value is elevated B. the whole blood glucose value should be higher than the serum value C. all values are consistent with a normal healthy individual D. the serum and whole blood values should be identical

Correct Answer: C

The identification of Bence Jones protein is best accomplished by: A. a sulfosalicylic acid test B. urine reagent strips C. immunofixation D. electrophoresis

Correct Answer: C

The majority of thyroxine (T4) is converted into the more biologically active hormone: A. thyroglobulin B. thyroid-stimulating hormone (TSH) C. triiodothyronine (T3) D. thyrotropin-releasing hormone

Correct Answer: C

The most common form (95%) of congenital adrenal hyperplasia is 21-hydroxylase deficiency, which is detected by elevate plasma: A. cortisol B. aldosterone C. 17-OH-progesterone D. 11-deoxycortisol

Correct Answer: C

The most important buffer pair in plasma is the: A. phosphate/biphosphate pair B. hemoglobin/imidazole pair C. bicarbonate/carbonic acid pair D. sulfate/bisulfate pair

Correct Answer: C

The most sensitive enzymatic indicator for liver damage form ethanol intake is: A. alanine aminotransferase (ALT) B. aspartate aminotransferase (AST) C. gamma-glutamyl transferase (GGT) D. alkaline phosphatase

Correct Answer: C

The presence of C-reactive protein in the blood is an indication of: A. a recent streptococcal infection B. recovery from a pneumococcal infection C. an inflammatory process D. a state of hypersensitivity

Correct Answer: C

The presence of increased CKMB activity on a CK electrophoresis pattern is most likely found in a patient suffering from: A. actue muscular stress following strenuous exercise B. malignant liver disease C. myocardial infarction D. severe head injury

Correct Answer: C

The reason carbon monoxide is so toxic is because it: A. is a protoplasmic poison B. combines with cytochrome oxidase C. has 200 times the affinity of oxygen for hemoglobin binding sites D. sensitizes the myocardium

Correct Answer: C

To assure an accurate ammonia level result, the specimen should be: A. incubated at 37 C prior to testing B. spun and separated immediately, tested as routine C. spun, separated, iced, and tested immediately D. stored at room temperature until tested

Correct Answer: C

Transportation of 60%-75% of the plasma cholesterol is performed by: A. chylomicrons B. very low-density lipoproteins C. low-density lipoproteins D. high-density lipoproteins

Correct Answer: C

Valinomycin enhances the selectivity of the electrode used to quantitate: A. sodium B. chloride C. potassium D. calcium

Correct Answer: C

What is the proper pH for buffered solution used to perform serum protein electrophoresis? A. 5.6 B. 7.6 C. 8.6 D. 9.6

Correct Answer: C

What substance gives feces its normal color? A. uroerythrin B. urochrome C. urobilin D. urobilinogen

Correct Answer: C

When separating serum proteins by cellulose acetate electrophoresis, using Veronal (TM) buffer at pH 8.6, beta globulin migrates: A. faster than albumin B. slower than gamma globulin C. faster than gamma globulin D. faster than alpha-2 globulin

Correct Answer: C

Which blood gas electrode is composed of silver/silver chloride reference electrode and glass? A. PO2 B. pH C. PCO2 D. HCO3

Correct Answer: C

Which of the following diseases results from a familial absence of high density lipoprotein? A. Krabble disease B. Gaucher disease C. Tangier disease D. Tay-Sachs disease

Correct Answer: C

Which of the following hormones regulates normal blood calcium levels? A. thyroxine B. estriol C. parathyroid hormone D. growth hormone

Correct Answer: C

Which of the following sample collections would give an accurate assessment of potential excess cortisol production (hypercortisolism)? A. collect a plasma sample as a baseline, and another one-hour after administration of metyrapone B. collect a plasma sample at 8 AM only C. collect a 24-hour urine free cortisol D. collect a plasma sample at 8 AM and at 8 AM the next day

Correct Answer: C

Which of the following serum protein fractions is most likely to be elevated in patients with nephrotic syndrome? A. alpha-1 globulin B. albumin C. alpha-2 globulin D. beta globulin and gamma globulin

Correct Answer: C

Which percentage of total serum calcium is nondiffusible protein bound? A. 80%-90% B. 51%-60% C. 40%-50% D. 10%-30%

Correct Answer: C

How many grams of sodium hydroxide (NaOH) are required to prepare 150.0 mL of a 5.0% w/v solution? A. 1.5 g B. 4.0 g C. 7.5 g D. 15.0 g

Correct Answer: C 1. C A percent solution expressed in w/v (weight/volume) refers to grams of solute per 100.0 mL of solution. To calculate, multiply the percentage (as grams) by the volume needed (mL), then divide by 100.0 (mL). (5.0 g × 150.0 mL) ÷ 100.0 mL = 7.5 g To prepare the solution, weigh 7.5 g of NaOH pellets and add to a 150.0-mL volumetric flask. Add sufficient deionized H2O to dissolve the NaOH. After the solution cools, add deionized H2O to the 150.0-mL line on the flask and mix again.

Kjeldahl's procedure for total protein is based upon the premise that: A. Proteins are negatively charged B. The pKa of proteins is the same C. The nitrogen content of proteins is constant D. Proteins have similar tyrosine and tryptophan content

Correct Answer: C 1. C Kjeldahl's method measures the nitrogen content of proteins as ammonium ion by back titration following oxidation of proteins by sulfuric acid and heat. It assumes that proteins average 16% nitrogen by weight. Protein in grams per deciliter is calculated by multiplying protein nitrogen by 6.25. The Kjeldahl method is a reference method for total protein that is used to assign a protein assay value to calibrators.

Which of the following findings is characteristic of all forms of clinical hypoglycemia? A. A fasting blood glucose value below 55 mg/dL B. High fasting insulin levels C. Neuroglycopenic symptoms at the time of low blood sugar D. Decreased serum C peptide

Correct Answer: C 10. C Clinical hypoglycemia can be caused by insulinoma, drugs, alcoholism, and reactive hypoglycemia. Reactive hypoglycemia is characterized by delayed or excessive insulin output after eating and is very rare. Fasting insulin is normal but postprandial levels are increased. High fasting insulin levels (usually > 6 μg/L) are seen in insulinoma, and patients with insulinoma almost always display fasting hypoglycemia, especially when the fast is extended to 48-72 hours. C peptide is a subunit of proinsulin that is hydrolyzed when insulin is released. In hypoglycemia, low levels indicate an exogenous insulin source, whereas high levels indicate overproduction of insulin.

When should progesterone be measured when evaluating an adult female for anovulation? A. At the onset of menses B. During the first 7 days of the menstrual cycle C. At the midcycle just after LH peaks D. At the end of the menstrual cycle

Correct Answer: C 10. C Progesterone is often measured along with LH, FSH, estrogen, and prolactin to evaluate female infertility and dysmenorrhea. Progesterone is produced by the corpus luteum and levels are very low during the early follicular phase of the cycle. Progesterone is released by the corpus luteum following the LH surge that occurs 1-2 days prior to ovulation and is an indication that ovulation occurred. Low progesterone at midcycle indicates that ovulation did not occur. This is often the case in polyovarian cyst syndrome.

Interference from other reducing substances can be partially eliminated in the Jaffe reaction by: A. Measuring the product at 340 nm B. Measuring the product with an electrode C. Measuring the timed rate of product formation D. Performing a sample blank

Correct Answer: C 10. C The Jaffe reaction is nonspecific; proteins and other reducing substances such as pyruvate, protein, and ascorbate cause positive interference. Much of this interference is reduced by using a timed rate reaction. Ketoacids react with alkaline picrate almost immediately, and proteins react slowly. Therefore, reading the absorbance at 20 and 80 seconds and using the absorbance difference minimizes the effects of those compounds. Creatinine can be measured using an amperometric electrode. However, this requires the enzymes creatininase, creatinase, and sarcosine oxidase. The last enzyme produces hydrogen peroxide from sarcosine, which is oxidized. This produces current in proportion to creatinine concentration. Performing a sample blank does not correct for interfering substances that react with alkaline picrate.

Which statement regarding glycated (glycosylated) Hgb (G-Hgb) is true? A. Has a sugar attached to the C-terminal end of the β chain B. Is a highly reversible aminoglycan C. Reflects the extent of glucose regulation in the 8- to 12-week interval prior to sampling D. Will be abnormal within 4 days following an episode of hyperglycemia

Correct Answer: C 11. C G-Hgb results from the nonenzymatic attachment of a sugar such as glucose to the N-terminal valine of the β chain. The reaction is nonreversible and is related to the time-averaged blood glucose concentration over the life span of the RBCs. There are three G-Hgb fractions designated A1a, A1b, and Alc. Hemoglobin A1c makes up about 80% of glycated hemoglobin, and is used to determine the adequacy of insulin therapy. The time-averaged blood glucose is approximated by the formula (G-Hgb × 33.3) - 86 mg/dL, and insulin adjustments can be made to bring this level to within reference limits. Also, glycated protein assay (called fructosamine) provides similar data for the period between 2 and 4 weeks before sampling.

If too small a peak-trough difference is seen for a drug given orally, then: A. The dose should be decreased B. Time between doses should be decreased C. Dose interval should be increased D. Dose per day and time between doses should be decreased

Correct Answer: C 12. C Increasing the dosing interval will reduce the trough concentration of the drug, and increasing the dose will increase the peak concentration of the drug, resulting in a greater peak-trough difference. The peak-trough ratio is usually adjusted to 2 with the dose interval set to equal the drug half-life. Under these conditions, both peak and trough levels often fall within the therapeutic range.

The onset of menopause is usually associated with what hormone changes? A. Decreased estrogen, testosterone, and androgens B. Decreased estrogen, FSH, LH, and progesterone C. Decreased estrogen and progesterone, and increased LH and FSH D. Decreased estrogen and progesterone, normal LH and FSH

Correct Answer: C 13. C In menopause, estrogen production decreases to where the menstrual cycle and ovarian follicle maturation stop. The decreased estrogen causes the pituitary release of LH and FSH. In menopause, the FSH at midcycle is higher than LH. The increased LH causes the ovaries to secrete testosterone and androgens.

A decreased PAO2-PaO2 difference is found in: A. A/V (arteriovenous) shunting B. V/Q (ventilation/perfusion) inequality C. Ventilation defects D. All of these options

Correct Answer: C 13. C Patients with A/V shunts, V/Q inequalities, and cardiac failure will have an increased PAO2-PaO2 difference. However, patients with ventilation problems have low alveolar PO2 owing to retention of CO2 in the airway. This reduces the PAO2-PaO2 difference.

Which of the following conditions will prevent any migration of proteins across an electrophoretic support medium such as agarose? A. Using too high a voltage B. Excessive current during the procedure C. Loss of contact between a buffer chamber and the medium D. Evaporation of solvent from the surface of the medium

Correct Answer: C 14. C Movement of proteins is dependent upon the presence of a salt bridge that allows current to flow via transport of ions to the electrodes across the support medium. If the salt bridge is not intact, there will be no migration, even if voltage is maintained across the electrodes. For agarose and cellulose acetate, heat causes evaporation of solvent from the buffer. This increases the ionic strength, causing current to rise during the run. Excessive heat can damage the support medium and denature proteins. Power = E (voltage) × I (current) × t (time); since E = I × R (resistance), heat is proportional to the square of current (P = I^2 × R × t). Constant current or power mode is used for long runs to prevent heat damage.

Hyperparathyroidism is most consistently associated with: A. Hypocalcemia B. Hypocalciuria C. Hypophosphatemia D. Metabolic alkalosis

Correct Answer: C 16. C Hyperparathyroidism causes increased resorption of calcium and decreased renal retention of phosphate. Increased serum calcium leads to increased urinary excretion. The distal collecting tubule of the nephron reabsorbs less bicarbonate as well as phosphate, resulting in acidosis.

A person has an elevated 24-hour urinary homovanillic acid (HVA) and vanillymandelic acid (VMA). Urinary metanephrines, chromogranin A, and neuron-specific enolase are also elevated but 5-hydroxyindoleacetic acid is within the reference range. What is the most likely diagnosis? A. Carcinoid tumors of the intestine B. Pheochromocytoma C. Neuroblastoma D. Pancreatic cancer

Correct Answer: C 17. C Neuron-specific enolase is an isoenzyme containing two gamma polypeptides that are specific for nervous tissue and are found in neuroendocrine cells. Plasma levels are increased in neuroblastomas, carcinoid tumors, thyroid medullary carcinomas, and in some lung cancers and seminomas. Urinary VMA, catecholamines, and metanephrines are increased in both pheochromocytoma (a tumor of chromaffin cells) and neuroblastoma (also a tumor of neuroectodermal cells derived from the neural crest neuroblasts of the sympathetic ganglia). Urinary HVA is increased in about 75% of persons with neuroblastoma but is not usually increased in pheochromocytoma. Chromogranin A is a protein that inhibits release of catecholamines and is increased in pheochromocytoma, neuroblastoma, and carcinoid tumors. Urinary 5-hydroxyindoleacetic acid is increased in carcinoid tumors (enterochromaffin tumors).

Which statement regarding the use of PTH is true? A. Determination of serum PTH level is the best screening test for disorders of calcium metabolism B. PTH levels differentiate primary and secondary causes of hypoparathyroidism C. PTH levels differentiate primary and secondary causes of hypocalcemia D. PTH levels are low in patients with pseudohypoparathyroidism

Correct Answer: C 17. C Serum Cai is the best screening test to determine if a disorder of calcium metabolism is present, and will distinguish primary hyperparathyroidism (high Cai ) and secondary hyperparathyroidism (low Cai ). PTH levels are used to distinguish primary and secondary causes of hypocalcemia. Serum PTH is low in primary hypocalcemia (which results from parathyroid gland disease), but is high in secondary hypocalcemia (e.g., renal failure). Serum PTH is also used for the early diagnosis of secondary hypocalcemia because PTH levels rise prior to a decrease in the serum Cai . Serum PTH is used to help distinguish primary hyperparathyroidism (high PTH) and hypercalcemia of malignancy (usually low PTH), and pseudohypoparathyroidism from primary hypoparathyroidism. Pseudohypoparathyroidism results from a deficient response to PTH and is associated with normal or elevated serum PTH.

A plasma sample is hemolyzed and turbid. What is required to perform a sample blank in order to correct the measurement for the intrinsic absorbance of the sample when performing a spectrophotometric assay? A. Substitute deionized water for the sample B. Dilute the sample 1:2 with a standard of known concentration C. Substitute saline for the reagent D. Use a larger volume of the sample

Correct Answer: C 18. C A sample blank is used to subtract the intrinsic absorbance of the sample usually caused by hemolysis, icterus, turbidity, or drug interference. On automated analyzers, this is accomplished by measuring the absorbance after the addition of sample and a first reagent, usually a diluent. For tests using a single reagent, sample blanking can be done prior to the incubation phase before any color develops. Substituting deionized water for sample is done to subtract the absorbance of the reagent (reagent blanking). Diluting the sample with a standard (standard addition) may be done when the absorbance is below the minimum detection limit for the assay. Using a larger volume of sample will make the interference worse.

Which of the following conditions is associated with "β-γ bridging"? A. Multiple myeloma B. Malignancy C. Hepatic cirrhosis D. Rheumatoid arthritis

Correct Answer: C 18. C Hepatic cirrhosis produces a polyclonal gammopathy associated with a high IgA level. This obliterates the valley between β and γ zones. Malignancy and rheumatoid arthritis produce polyclonal gammopathies classified as chronic inflammatory or delayed response patterns. Multiple myeloma produces a zone of restricted mobility usually in the γ, but sometimes in the β- or α2-region.

A patient's blood gas results are as follows: pH = 7.26 dCO2 = 2.0 mmol/L HCO3- = 29 mmol/L These results would be classified as: A. Metabolic acidosis B. Metabolic alkalosis C. Respiratory acidosis D. Respiratory alkalosis

Correct Answer: C 18. C Imbalances are classified as respiratory when the primary disturbance is with PCO2 because PCO2 is regulated by ventilation. PCO2 = dCO2/0.03 or 60 mm Hg (normal 35-45 mm Hg). Increased dCO2 will increase hydrogen ion concentration, causing acidosis. Bicarbonate is moderately increased, but a primary increase in NaHCO3 causes alkalosis. Thus, the cause of this acidosis is CO2 retention (respiratory acidosis), and it is partially compensated by renal retention of bicarbonate.

The best method of analysis for serum PTH involves using antibodies that detect: A. The amino-terminal fragment of PTH B. The carboxy-terminal end of PTH C. Both the amino-terminal fragment and intact PTH D. All fragments of PTH as well as intact hormone

Correct Answer: C 18. C PTH is a polypeptide comprised of 84 amino acids. The biological activity of the hormone resides in the N-terminal portion of the polypeptide, but the hormone is rapidly degraded and produces N-terminal, middle, and C-terminal fragments. Fragments lacking the N-terminal portion are inactive. Immunoassays for PTH using antibodies to different portions of the polypeptide will give different results. The assay of choice is a two-site double-antibody sandwich method that measures only intact PTH and active fragments. Methods that use single antibodies may detect inactive as well as active PTH fragments and are not as specific for parathyroid disease.

Urinary urea measurements may be used for calculation of: A. Glomerular filtration B. Renal blood flow C. Nitrogen balance D. All of these options

Correct Answer: C 19. C Because BUN is handled by the tubules, serum levels are not specific for glomerular filtration rate. Urea clearance is influenced by diet and liver function as well as renal function. Protein intake minus excretion determines nitrogen balance. A negative balance (excretion exceeds intake) occurs in stress, starvation, fever, cachexia, and chronic illness. Nitrogen balance = (Protein intake in grams per day ÷ 6.25) - (Urine urea nitrogen in grams per day + 4), where 4 estimates the protein nitrogen lost in the feces per day and dividing by 6.25 converts protein to protein nitrogen.

Drugs rapidly infused intravenously usually follow which elimination model? A. One compartment, first order B. One compartment, logarithmic C. Biphasic or two compartment with serum level rapidly falling in the first phase D. Michaelis-Menton or concentration-dependent elimination

Correct Answer: C 19. C Drugs rapidly infused intravenously follow a two-compartment model of elimination. The central compartment is the blood and tissues that are well perfused. The second consists of tissues for which distribution of drug is time dependent. In determining the loading dose, the desired serum concentration should be multiplied by the volume of the central compartment to avoid toxic levels.

Which of the following is most often elevated in hypercalcemia associated with malignancy? A. Parathyroid-derived PTH B. Ectopic PTH C. Parathyroid hormone-related protein (PTHRP) D. Calcitonin

Correct Answer: C 19. C PTHRP is a peptide produced by many tissues and normally present in the blood at a very low level. The peptide has an N-terminal sequence of eight amino acids that are the same as found in PTH and that will stimulate the PTH receptors of bone. Some malignancies (e.g., squamous, renal, bladder, and ovarian cancers) secrete PTHRP, causing hypercalcemia-associated malignancy. Because the region shared with PTH is small and poorly immunoreactive, the peptide does not cross-react in most assays for PTH. For this reason, and because tumors producing ectopic PTH are rare, almost all patients who have an elevated Cai and elevated PTH have primary hyperparathyroidism. The immunoassay for PTHRP will frequently be elevated in patients who have not yet been diagnosed with malignancy but have an elevated Cai , without an elevated serum PTH. Calcitonin is a hormone produced in the medulla of the thyroid that opposes the action of PTH. However, calcitonin levels do not greatly influence the serum calcium. Assay of calcitonin is used exclusively to diagnose medullary thyroid cancer, which produces very high serum levels.

Which instrument requires a highly regulated DC power supply? A. A spectrophotometer with a barrier layer cell B. A colorimeter with multilayer interference filters C. A spectrophotometer with a photomultiplier tube D. A densitometer with a photodiode detector

Correct Answer: C 19. C When AC voltage regulators are used to isolate source lamp power, light output fluctuates as the voltage changes. Because this occurs at 60 Hz, it is not detected by eyesight or slow-responding detectors. Photomultiplier tubes are sensitive enough to respond to the AC frequency and require a DC-regulated power supply.

Plate 2 shows the electrophoresis of serum proteins on a high-resolution agarose gel at pH 8.6. Sample 1 (in lane 1) is a normal serum control. Which sample can be presumptively classified as a monoclonal gammopathy? A. Sample 2 B. Sample 4 C. Sample 6 D. Sample 8

Correct Answer: C 2. C A monoclonal gammopathy causes a band showing restricted electrophoretic mobility usually located in the γ or the β region. The band represents the accumulation of identical immunoglobulin molecules or fragments secreted by a malignant or benign clone of plasma cells. Confirmation of the band as immunoglobulin is required because other homogenous proteins (such as fibrinogen or carcinoembryonic antigen) can occur in the same regions.

Which testing situation is appropriate for the use of point-of-care whole-blood glucose methods? A. Screening for type 2 diabetes mellitus B. Diagnosis of diabetes mellitus C. Monitoring of blood glucose control in type 1 and type 2 diabetics D. Monitoring diabetics for hyperglycemic episodes only

Correct Answer: C 20. C The ADA does not recommend the use of whole-blood glucose monitors for establishing a diagnosis of diabetes or screening persons for diabetes. The analytical measurement range of these devices varies greatly, and whole blood glucose is approximately 10% lower than serum or plasma glucose. In addition, analytical variance is greater and accuracy less than for laboratory instruments. Whole blood glucose meters should be used by diabetics and caregivers to monitor glucose control and can detect both hyper- and hypoglycemic states that result from too little or too much insulin replacement. Therefore, postprandial monitoring with such a device is recommended for all persons who receive insulin therapy.

SITUATION: A specimen for CK performed on an automated analyzer using an optimized Oliver-Rosalki method gives an error flag indicating substrate depletion. The sample is diluted 1:2 and 1:4 by the serial dilution technique and reassayed. After correcting for the dilution, the results are as follows: -1:2 Dilution = 3,000 IU/L -1:4 Dilution = 3,600 IU/L Dilutions are made a second time and assayed again but give identical results. What is the most likely explanation? A. The serum became contaminated prior to making the 1:4 dilution B. The wrong pipet was used to make one of the dilutions C. An endogenous competitive inhibitor is present in the serum D. An error has been made in calculating the enzyme activity of one of the two dilutions

Correct Answer: C 21. C When a competitive inhibitor is present in the serum, a dilution of the sample will cause an increase in the reaction rate by reducing the concentration of the inhibitor. Dilution of serum frequently increases the activity of CK and amylase. The same effect will occur when a smaller volume of serum is used in the assay because less inhibitor will be present in the reaction mixture.

After staining a silica gel plate to determine the L/S ratio, the technologist notes that the lipid standards both migrated 1 cm faster than usual. The technologist should: A. Repeat the separation on a new silica gel plate B. Check the pH of the developing solvent C. Prepare fresh developing solvent and repeat the assay D. Reduce solvent migration time for all subsequent runs

Correct Answer: C 22. C TLC plates migrate in solvent until the front comes to 1 cm of the top of the plate. Separation of lipids on silica gel is based upon adsorption. Higher Rf values indicate greater solubility of lipids in the developing solvent. This may be caused by evaporation of H2O, lowering the polarity of the solvent.

Which of the following conditions is characterized by primary hyperaldosteronism caused by adrenal adenoma, carcinoma, or hyperplasia? A. Cushing's syndrome B. Addison's disease C. Conn's syndrome D. Pheochromocytoma

Correct Answer: C 23. C Conn's syndrome is characterized by hypertension, hypokalemia, and hypernatremia with increased plasma and urine aldosterone and decreased renin. Cushing's syndrome results from excessive production of cortisol, and Addison's disease from deficient production of adrenal corticosteroids. Pheochromocytoma is a tumor of chromaffin cells (usually adrenal) that produces catecholamines.

Which of the following statements about fluorometry is accurate? A. Fluorometry is less sensitive than spectrophotometry B. Fluorometry is less specific than spectrophotometry C. Unsaturated cyclic molecules are often fluorescent D. Fluorescence is directly proportional to temperature

Correct Answer: C 23. C Increasing temperature results in more random collision between molecules by increasing their motion. This causes energy to be dissipated as heat instead of fluorescence. Temperature is inversely proportional to fluorescence. Fluorescence is more sensitive than spectrophotometry because the detector signal can be amplified when dilute solutions are measured. It is also more specific than spectrophotometry because both the excitation and emission wavelengths are characteristics of the compound being measured.

A quantitative urine glucose was determined to be 160 mg/dL by the Trinder glucose oxidase method. The sample was refrigerated overnight. The next day, the glucose is repeated and found to be 240 mg/dL using a polarographic method. What is the most likely cause of this discrepancy? A. Poor precision when performing one of the methods B. Contamination resulting from overnight storage C. High levels of reducing substances interfering with the Trinder reaction D. Positive interference in the polarographic method caused by hematuria

Correct Answer: C 23. C Urine often contains high levels of ascorbate and other reducing substances. These may cause significant negative bias when measuring glucose using a peroxidase-coupled method. The reductants compete with chromogen for H2O2.

Which condition results in metabolic acidosis with severe hypokalemia and chronic alkaline urine? A. Diabetic ketoacidosis B. Phenformin-induced acidosis C. Renal tubular acidosis D. Acidosis caused by starvation

Correct Answer: C 24. C Metabolic acidosis can be caused by any condition that lowers bicarbonate. In nonrenal causes, the kidneys will attempt to compensate by increased acid excretion. However, in renal tubular acidosis (RTA), an intrinsic defect in the tubules prevents bicarbonate reabsorption. This causes alkaline instead of acidic urine. Excretion of bicarbonate as potassium bicarbonate (KHCO3) results in severe hypokalemia.

What is the clinical utility of testing for serum prealbumin? A. Low levels are associated with increased free cortisol B. High levels are an indicator of acute inflammation C. Serial low levels indicate compromised nutritional status D. Levels correlate with glomerular injury in patients with diabetes mellitus

Correct Answer: C 24. C Prealbumin (also called transthyretin) is a small protein with a half-life of only 2 days. Serum levels fall rapidly in patients with deficient protein nutrition. As a result, prealbumin is used to detect malnutrition and to measure the patient's response to dietary supplementation. The cutpoint used to identify nutritional deficiency in elderly patients is usually 11 mg/dL. Prealbumin is usually measured by immunonephelometry.

Which of the following statements regarding the clinical use of CK-MB (CK-2) is true? A. CK-MB becomes elevated before myoglobin after an AMI B. CK-MB levels are usually increased in cases of cardiac ischemia C. CK-MB is more specific than myoglobin D. An elevated CK-MB is always accompanied by an elevated total CK

Correct Answer: C 24. C Serum myoglobin becomes abnormal within 1-2 hours after an acute myocardial infarction (AMI) before troponin and CK-MB. CK-MB becomes abnormal shortly after troponin I (TnI) or troponin T (TnT) when a URL of 4 μg/L is used, and peaks at around the same time following AMI. However, cardiac troponins remain elevated significantly longer than CK-MB after AMI, are not increased in crush injuries. There is less than 5 μg/L CK-MB in the serum of healthy adults, while the total CK ranges from 10-110 U/L. Consequently, an abnormal CK-MB can occur in the absence of an elevated total CK.

Which of the following is the most common cause of Cushing's syndrome? A. Pituitary adenoma B. Adrenal hyperplasia C. Overuse of corticosteroids D. Ectopic adrenocorticotropic hormone (ACTH) production by tumors

Correct Answer: C 24. C The most common cause of Cushing's syndrome is the administration of medications with cortisol or glucocorticoid activity. Excluding iatrogenic causes, approximately 60%-70% of Cushing's syndrome results from hypothalamic-pituitary misregulation and is called Cushing's disease. Adrenal adenoma or carcinoma (non-ACTH-mediated Cushing's syndrome) comprise about 20% of cases, and ectopic ACTH production accounts for 10%-20%.

Which glucose method is subject to falsely low results caused by ascorbate? A. Hexokinase B. Glucose dehydrogenase C. Trinder glucose oxidase D. Polarography

Correct Answer: C 26. C Although glucose oxidase is specific for β-D-glucose, the coupling (indicator) reaction is prone to negative interference from ascorbate, uric acid, acetoacetic acid, and other reducing agents. These compete with the chromogen (e.g., o-dianisidine) for peroxide, resulting in less dye being oxidized to chromophore. The choice of chromogen determines the specificity and linearity. 4-aminophenazone and phenol is more resistant to interference from azo compounds and proteins than is o-dianisidine.

In which situation is the plasma or 24-hour urinary cortisol not consistent with the clinical picture? A. In pregnant patients B. In patients with a positive overnight dexamethasone suppression test C. In congenital adrenal hyperplasia D. In Cushing's syndrome caused by ectopic ACTH producing tumors

Correct Answer: C 26. C Congenital adrenal hyperplasia (adrenogenital syndrome) results from a deficiency of an enzyme required for synthesis of cortisol. Approximately 90% of cases are caused by a deficiency of 21-hydroxylase blocking conversion of 17-α hydroxyprogesterone to 11-deoxycortisol. Most other cases are caused by 11-hydroxylase deficiency, which blocks conversion of 11-deoxycortisol to cortisol. Precursors of cortisol, usually either 17-α hydroxyprogesterone or 11-deoxycortisol are increased. This results in low serum cortisol levels, but high levels of these intermediates (mainly 17-ketogenic steroids). The two most common features of CAH are salt wasting caused by increased mineral corticoid activity and virilization due to increased androgens.

In addition to phenylketonuria, maple syrup urine disease, and homocystinuria, what other aminoaciduria can be detected by tandem MS? A. Alkaptonuria B. Hartnup disease C. Citrullinemia D. Cystinuria

Correct Answer: C 26. C Most states use electrospray ionization tandem-mass spectroscopy (MS/MS), which can detect over 20 inborn errors of metabolism from a single blood spot. Typically, this includes phenylketonuria, tyrosinemia, maple syrup urine disease, homocystinuria, citrullinemia, and argininosuccinate acidemia. The latter two are errors of the urea cycle.

Which test is used to distinguish Cushing's disease (pituitary Cushing's) from Cushing's syndrome caused by adrenal tumors? A. Low-dose overnight dexamethasone suppression B. Petrosal sinus sampling C. Serum ACTH D. Twenty-four-hour urinary free cortisol

Correct Answer: C 27. C Serum ACTH assays are very helpful in distinguishing the cause of Cushing's syndrome. Patients with adrenal tumors have values approaching zero. Patients with ectopic ACTH tumors have values greater than 200 pg/dL. Fifty percent of patients with Cushing's disease have high 8 a.m. ACTH levels (between 100-200 pg/dL). The high-dose dexamethasone suppression test is also used. Patients with Cushing's disease show more than 50% suppression of cortisol release after receiving an 8-mg dose of dexamethasone, but patients with adrenal tumors or ACTH producing tumors do not. Inferior petrosal sinus sampling (the petrosal sinuses drain the pituitary) is used to determine if a high ACTH is from the pituitary glands, or from an ectopic source.

What is the typical time course for plasma myoglobin following an AMI? A. Abnormal before 1 hour; peaks within 3 hours; returns to normal in 8 hours B. Abnormal within 3 hours; peaks within 6 hours; returns to normal in 18 hours C. Abnormal within 2 hours; peaks within 12 hours; returns to normal in 36 hours D. Abnormal within 6 hours; peaks within 24 hours; returns to normal in 72 hours

Correct Answer: C 28. C After AMI, myoglobin usually rises above the cutoff within 1-2 hours, peaks within 8-12 hours and returns to normal within 36 hours. Typically, levels reach a peak concentration that is 10-fold the upper reference limit. Since myoglobin is the first marker to become abnormal after an AMI, it should be measured on admission and if negative, again 2 hours later. If both samples are below the cutoff, the probability of an AMI having occurred is low. If the myoglobin is above the cutoff, a cardiac specific marker such as TnI, or TnT must be performed at some point to confirm the diagnosis.

Whichstatementistrueregardingparticle-enhanced turbidimetric inhibition immunoassay methods for therapeutic drugs? A. Drug concentration is proportional to light scatter B. Magnetic separation is needed to remove unbound conjugate C. When particle-bound drug binds to antibody, light scattering is increased D. Two antibodies to the drug are needed

Correct Answer: C 28. C Particle-enhanced turbidimetric inhibition immunoassays are homogenous immunoassays frequently used to measure proteins and therapeutic drugs in serum or plasma. Polystyrene-modified latex particles conjugated to the drug (particle-bound drug) compete with drug in the sample for a limited number of antibodies. If drug concentration is low, more of the antibody binds to the particle-bound drug, increasing the turbidity of the reaction. Therefore, light scattering is inversely proportional to the drug concentration.

SITUATION: A patient's biochemistry results are: -Na = 125 mmol/L -Cl = 106 mmol/L -K = 4.5 mmol/L -TCO2 = 19 mmol/L -chol = 240 mg/dL -triglyceride = 640 mg/dL -glucose = 107 mg/dL -AST = 16 IU/L -ALT = 11 IU/L -amylase = 200 U/L Select the most likely cause of these results. A. The sample is hemolyzed B. Serum was not separated from cells in sufficient time C. Lipemia is causing in vitro interference D. The specimen is contaminated

Correct Answer: C 29. C The triglyceride level is about five times normal, causing the sample to be lipemic. This will cause pseudohyponatremia (unbalanced electrolytes). Lipemia may cause a falsely high rate reaction when amylase is measured by turbidimetry; however, the high amylase may be associated with pancreatitis, which results in hyperlipidemia.

What is the typical time course for plasma TnI or TnT following an AMI? A. Abnormal within 3 hours; peaks within 12 hours; returns to normal in 24 hours B. Abnormal within 4 hours; peaks within 18 hours; returns to normal in 48 hours C. Abnormal within 4 hours; peaks within 24 hours; returns to normal in 1 week D. Abnormal within 6 hours; peaks within 36 hours; returns to normal in 5 days

Correct Answer: C 29. C Troponin is a complex of three polypeptides that function as a regulator of actin and tropomyosin. The three subunits are designated TnC, TnI, and TnT. All are present in both cardiac and some skeletal muscles, but cardiac and skeletal isoforms of TnI and TnT can be differentiated by specific antiseras. TnI and TnT cardiac isoforms in plasma will at least double within 4-6 hours after MI, peak within 24 hours and usually remain elevated for 7-10 days. TnT and TnI have the same sensitivity, but TnT is more commonly elevated in renal failure patients. Both are increased slightly in unstable angina (chest pain while at rest) and cardiac ischemia.

A biuret reagent requires preparation of a stock solution containing 9.6 g of copper II sulfate (CuSO4) per liter. How many grams of CuSO4 • 5H2O are needed to prepare 1.0 L of the stock solution? Atomic weights: H = 1.0; Cu = 63.6; O = 16.0; S = 32.1 A. 5.4 g B. 6.1 g C. 15.0 g D. 17.0 g

Correct Answer: C 3. C Determine the mass of CuSO4 • 5H2O containing 9.6 g of anhydrous CuSO4. First, calculate the percentage of CuSO4 in the hydrate, then divide the amount needed (9.6 g) by the percentage. % CuSO4 = molecular weight CuSO4 ÷ molecular weight CuSO4 • 5H2O × 100 = (159.7 ÷ 249.7) × 100 = 63.96% Grams CuSO4 • 5H2O = 9.6 g ÷ 0.6396 = 15.0 g A convenient formula to use is: g hydrate = (MW hydrate ÷ MW anhydrous salt) × g anhydrous salt

Which statement about the biuret reaction for total protein is true? A. It is sensitive to protein levels below 0.1 mg/dL B. It is suitable for urine, exudates, and transudates C. Polypeptides and compounds with repeating imine groups react D. Hemolysis will not interfere

Correct Answer: C 3. C The biuret reaction is not sensitive to protein levels below 0.1 g/dL and, therefore, is not sensitive enough for assays of total protein in CSF, urine, or transudates. Slight hemolysis does not cause falsely high results, if the absorbance of the Cu+2-protein complexes is measured bichromatically. However, frankly hemolyzed samples contain sufficient globin to cause positive interference. The reagent reacts with peptides containing at least two peptide bonds, but due to the high concentration of proteins in plasma relative to peptides present this reactivity causes insignificant bias.

Plate 3 shows a densitometric scan of a control serum for protein electrophoresis. The percentages of each fraction are shown below the scan. Given these results, what is the most appropriate initial corrective action? A. Repeat the electrophoresis run using fresh control serum B. Report the results, provided that the previous run was in control C. Move the fourth fraction mark to the right and redraw the scan D. Calculate the concentration of each fraction in grams per deciliter

Correct Answer: C 3. C The fraction marker between the α2- and β-fractions is marked improperly. High-resolution gels produce individual peaks for haptoglobin and α2- macroglobulin, which partially splits the α2- band into two subfractions. In addition, the β-band may contain three subfractions corresponding to β-lipoprotein, transferrin, and complement. In this scan, the valley between the α2-subfractions was selected incorrectly as the boundary between the α2- and β-fractions. This fraction marker should be placed at the next valley to the right and the scan redrawn to determine the area under the α2- and β-fractions correctly.

When measuring lead in whole blood using atomic absorption spectrophotometry, what reagent is required to obtain the needed sensitivity and precision? A. Lanthanum B. Lithium C. Triton X-100 D. Chloride

Correct Answer: C 30. C A graphite furnace is preferred over a flame for measuring lead because it is sufficiently sensitive to detect levels below 5 μg/dL, the cutoff needed for lead screening of children. The matrix modifier consists of Triton X -100, ammonium phosphate and nitric acid. This allows for release of Pb from the RBCs, and solubilization of cell stroma. The matrix modifier also prevents loss of Pb caused by formation of lead halides and promotes interaction between Pb and the tube wall, preventing its loss during the ashing cycle.

Which statement about ammonia is true? A. Normally, most of the plasma ammonia is derived from peripheral blood deamination of amino acids B. Ammonia-induced coma can result from salicylate poisoning C. Hepatic coma can result from Reye's syndrome D. High plasma ammonia is usually caused by respiratory alkalosis

Correct Answer: C 30. C Ammonia produced in the intestines from the breakdown of proteins by bacterial enzymes is the primary source of plasma ammonia. Most of the ammonia absorbed from the intestines is transported to the liver via the portal vein and converted to urea. Blood ammonia levels will rise in any necrotic liver disease including hepatitis, Reye's syndrome, and drug-induced injury such as acetaminophen poisoning. In hepatic cirrhosis, shunting of portal blood to the general circulation causes blood ammonia levels to rise. Ammonia crosses the blood- brain barrier, which accounts for the frequency of central nervous system complications and, if severe, hepatic coma.

Which plot will give the earliest indication of a shift or trend? A. Levy-Jennings B. Tonks-Youden C. Cusum D. Histogram

Correct Answer: C 30. C Cusum points are the algebraic sum of the difference between each QC result and the mean. The y axis is the sum of differences and the x axis is the run number. The center of the y axis is 0. Because QC results follow a random distribution, the points should distribute about the zero line. Results are out of control when the slope exceeds 45° or a decision limit (e.g., ±2.7s) is exceeded.

Which of the following is the most effective serial sampling time for ruling out AMI using both myoglobin and a cardiac specific marker in an emergency department environment? A. Admission and every hour for the next 3 hours or until positive B. Admission, 2 hours, 4 hours, and 6 hours or until positive C. Admission, 3 hours, 6 hours, and a final sample within 12 hours D. Admission and one sample every 8 hours for 48 hours

Correct Answer: C 30. C Since the time between the onset of symptoms and arrival in the emergency department is often speculative, serial measurement of cardiac markers is required in order to rule out AMI. Since myoglobin is the first marker to rise after AMI, if used it should be measured on admission. Since TnI, TnT, and CK-MB are more cardiac specific, at least one should be measured starting at 3 hours postadmission, and again at 6 hours postadmission. If all results are negative to this point, a final assay should be performed 6-12 hours postadmission to conclusively rule out the possibility of AMI and evaluate the short-term risk of AMI.

Which statement about the measurement of carboxyhemoglobin is true? A. Treatment with alkaline dithionite is used to convert carboxyhemoglobin to oxyhemoglobin B. Oxyhemoglobin has no absorbance at 540 nm, but carboxyhemoglobin does C. Bichromatic analysis is required in order to eliminate interference by oxyhemoglobin D. Carboxyhemoglobin can be measured by potentiometry

Correct Answer: C 32. C The absorbance spectras of oxy- and carboxyhemoglobin pigments overlap, and bichromatic or multichromatic analysis is required in order to accurately measure carboxyhemoglobin. In bichromatic analysis, oxyhemoglobin and methemoglobin are converted to deoxyhemoglobin by the addition of alkaline sodium dithionite. The ratio of absorbance at 541:555 nm is directly proportional to carboxyhemoglobin concentration. Percent carboxyhemoglobin is commonly determined from simultaneous absorbance measurements at 548, 568, and 578 nm, or other wavelength combinations, a process called oximetry.

Which of the following enzymes can be used to measure plasma or serum salicylate? A. Peroxidase B. Salicylate esterase C. Salicylate hydroxylase D. p-Aminosalicylate oxidase

Correct Answer: C 35. C The enzymatic assay of salicylate uses salicylate hydroxylase, which reduces salicylate with NADH and forms catechol and NAD+. Salicylate can also be measured by HPLC and various immunoassays including EMIT. Salicylate toxicity causes an initial respiratory alkalosis because the drug stimulates the respiratory center. However, this is followed by metabolic acidosis as the drug is metabolized. Therefore, it is imperative to identify salicylate as the cause of toxicity before treatment of an acid-base imbalance caused by aspirin overdose.

Which of the following laboratory tests is a marker for ischemic heart disease? A. CK-MB isoforms B. Myosin light chain 1 C. Albumin cobalt binding D. Free fatty acid binding protein

Correct Answer: C 36. C When heart muscle suffers reversible damage as a result of oxygen deprivation, free radicals are released from the cells and bind to circulating albumin. The albumin is modified at the N-terminus, causing a reduced ability to bind certain metals. This ischemia-modified albumin can be measured by its inability to bind cobalt. An excess of cobalt is incubated with plasma followed by addition of dithiothreitol. The sulfhydryl compound complexes with the free cobalt, forming a colored complex. The absorbance of the reaction mixture is directly proportional to the ischemia-modified albumin concentration. In addition to ischemia-modified albumin, glycogen phosphorylase-BB is a marker for ischemia because it is released from heart muscle during an ischemic episode. Myosin light chains and fatty acid-binding protein are released from necrotic heart tissue in the early stages of AMI.

Which statement regarding IFE is true? A. Serum containing a monoclonal protein should have a κ:λ ratio of 0.5 B. A monoclonal band seen with monospecific antiserum should not be visible in the lane where polyvalent antiserum or sulfosalicylic acid was added C. CSF should be concentrated 50- to100-fold before performing IFE D. When oligoclonal bands are seen in the CSF, they must also be present in serum to indicate multiple sclerosis

Correct Answer: C 37. C Any monoclonal precipitin band formed when heavy- or light-chain-specific antiserum reacts with a sample should also be found in the same position when sample is fixed with sulfosalicylic acid or reacted with polyvalent antihuman Ig. The normal free κ:λ ratio can vary between 0.26 and 1.65. In a monoclonal gammopathy, this ratio always heavily favors the light chain type of M protein. A diagnosis of multiple sclerosis is usually confirmed by demonstration of oligoclonal banding in the CSF, which is not present in the serum. CSF is usually concentrated 50-100 times to increase sensitivity.

What is the minimum requirement for performing QC for a total protein assay? A. One level assayed every 8 hours B. Two levels assayed within 8 hours C. Two levels assayed within 24 hours D. Three levels assayed within 24 hours

Correct Answer: C 37. C The minimum requirement for frequency of quality control for a general chemistry analyte (based upon the Clinical Laboratory Improvement Act, 1988) is two levels of control assayed every 24 hours. Some laboratories prefer to assay two control levels every 8 hours to increase the opportunity for error detection. Two controls every 8 hours are required for blood gases, automated hematology, and point-of-care glucose testing to comply with College of American Pathology requirements. Analytes that display different CVs at the low, normal, and high ranges require 3 levels of control in 24 hours. These include blood gases, therapeutic drugs, and hormones.

Which statement best describes the clinical utility of B-type natriuretic peptide (BNP)? A. Abnormal levels may be caused by obstructive lung disease B. A positive test indicates prior myocardial damage caused by AMI that occurred within the last 3 months C. A normal test result (<100 pg/mL) helps rule out congestive heart failure in persons with symptoms associated with coronary insufficiency D. A level above 100 pg/mL is not significant if evidence of congestive heart failure is absent

Correct Answer: C 38. C B-type natriuretic peptide is a hormone produced by the ventricles in response to increased intracardiac blood volume and hydrostatic pressure. It is formed in the heart from a precursor peptide (preproBNP) by enzymatic hydrolysis, first forming proBNP followed by BNP and NT (N-terminal) proBNP which is not physiologically active. Both BNP and NT-proBNP are increased in persons with congestive heart failure (CHF). Levels are not increased in pulmonary obstruction, hypertension, edema associated with renal insufficiency, and other conditions that cause physical limitation and symptoms that overlap CHF. At a cutoff of <100 pg/mL the BNP test is effective in ruling out CHF. Diagnostic accuracy in distinguishing CHF from nonCHF ranges from 83%-95%. In addition, persons with ischemia who have an increased BNP are at greater risk for MI. The NTpro-BNP assay is similar in clinical value, and can be used for persons being treated with nesiritide, a recombinant form of BNP used to treat CHF.

Hemoglobin electrophoresis performed on agarose at pH 8.8 gives the following results: A2 Position: 35% S Position: 30% F Position: 5% A Position: 30% All components of the Hgb C, S, F, A control hemolysate were within the acceptable range. What is the most likely cause of this patient's result? A. HgbLepore + B. Hgb S-β-thalassemia (Hgb S/β ) C. Hgb SC disease post-transfusion D. Specimen contamination

Correct Answer: C 39. C HemoglobinLepore results from a hybridization of the β and δ genes and produces a pattern that is similar to Hgb S trait (AS), except that the quantity of HgbLepore at the Hgb S position is below 20%. Hemoglobin S-β-thalassemia minor results in an increase in Hgb A2 (and possibly Hgb F) because there is reduced transcription of the structurally normal β chain. However, the Hgb S should be greater than the Hgb A, and the amount at the Hgb A2 is far too high. The concentration of Hgb at the A2 position is too high to result from contamination or to be considered as Hgb A2. This pattern appears to express two abnormal Hgbs (Hgb S and C) as well as the normal adult Hgb A. Because inheritance of two abnormal β genes prohibits formation of normal Hgb A, this pattern would occur only if the patient has been transfused with normal RBCs. Hemoglobin SC disease usually produces almost equal amounts of Hgb C and S (and usually a slight increase in Hgb F), and is the most likely cause of these results. This could be confirmed by acid agar electrophoresis or isofocusing to identify the abnormal Hgbs, and review of the patient's medical record for evidence of recent blood transfusion.

Which of the following is the best analyte to monitor for recurrence of ovarian cancer? A. CA-15-3 B. CA-19-9 C. CA-125 D. CEA

Correct Answer: C 4. C CA-125 is an oncofetal antigen, meaning that it is produced by genes that are active during fetal development but minimally active after birth except in malignant tissues. This group includes α-fetoprotein (AFP), CEA, PSA, and the carbohydrate-associated antigens (CA). CA-15-3 (which shares the same antigenic determinant as CA-27.29) is used mainly to monitor breast cancer treatment and recurrence. CA-19-9 (which shares the same antigenic determinant as CA-50) is a glycoprotein shed from the surface of gastric, pancreatic, and colorectal cancer cells.

How many milliliters of HNO3 (purity 68.0%, specific gravity 1.42) are needed to prepare 1.0 L of a 2.0 N solution? Atomic weights: H = 1.0; N = 14.0; O = 16.0 A. 89.5 mL B. 126.0 mL C. 130.5 mL D. 180.0 mL

Correct Answer: C 4. C The molecular weight of HNO3 is 63.0 g. Because the valance of the acid is 1 (1 mol of hydrogen is produced per mole of acid), the equivalent weight is also 63.0 g. The mass is calculated by multiplying the normality (2.0 N) by the equivalent weight (63.0 g) and volume (1.0 L); therefore, 126.0 g of acid are required. Because the purity is 68.0% and the specific gravity 1.42, the amount of HNO3 in grams per milliliter is 0.68 × 1.42 g/mL or 0.9656 g/mL. The volume required to give 126.0 g is calculated by dividing the mass needed (grams) by the grams per milliliter. mL HNO3 = 126.0 g ÷ 0.9656 g/mL = 126.0 g × 1.0 mL/0.9656 g = 130.5 mL

Which of the following is characteristic of type 2 diabetes mellitus? A. Insulin levels are consistently low B. Most cases require a 3-hour oral glucose tolerance test to diagnose C. Hyperglycemia is often controlled without insulin replacement D. The condition is associated with unexplained weight loss

Correct Answer: C 4. C Type 2, or late-onset diabetes, is associated with a defect in the receptor site for insulin. Insulin levels may be low, normal, or high. Patients are usually obese and over 40 years of age, although the incidence is increasing in both children and young adults. The American Diabetes Association (ADA) recommends screening all adults for diabetes who are overweight and have one additional risk factor and all adults over age 45, and to retest them every 3 years, if negative. Patients do not require insulin to prevent ketosis and hyperglycemia can be controlled in most patients by diet and drugs that promote insulin release. Type 2 accounts for 80%-90% of all diabetes mellitus.

Plate 4 shows the electrophoresis of serum proteins on a high-resolution agarose gel at pH 8.6. Which band represents the β lipoprotein? A. A B. B C. C D. D

Correct Answer: C 4. C Using high current, β lipoprotein can be separated from transferrin and complement (C3). β Lipoprotein migrates anodal to the transferrin (band labeled D), and appears as a thin wavy band. C3 migrates cathodal to the transferrin band. The band labeled A is α-1 antitrypsin and the band labeled B contains α-2 macroglobulin and haptoglobin. The α-2 macroglobulin is usually anodal to the haptoglobin.

Two consecutive serum samples give the results shown in the table above (at the top of this page) for a metabolic function profile. The instrument is a random access analyzer that uses two sample probes. The first probe aspirates a variable amount of serum for the spectrophotometric chemistry tests, and the second probe makes a 1:50 dilution of serum for electrolyte measurements. What is the most likely cause of these results? A. Both patients have renal failure B. There is an insufficient amount of sample in both serum tubes C. There is a fibrin strand in the probe used for the spectrophotometric chemistry tests D. The same patient's sample was accidentally run twice

Correct Answer: C 40. C Electrolyte results for both patients are within the physiological range but are distinctly different. The first results indicate a high potassium and increased anion gap, and one would expect the BUN, uric acid, and creatinine to be elevated. However, the results for BUN and glucose are unlikely for any patient, and the creatinine and uric acid signals are below the detection limit of the analyzer, indicating that little or no sample was added. This could be caused by a partially obstructed sample probe, or insufficient sample volume. The results for the second sample are below detection limits for all spectrophotometric tests, which may be the result of complete probe obstruction or the inability to generate a detectable signal with the trace quantity of serum that was added. Because all of the low or undetectable signals are for tests sampled by the first probe, the only explanation is that the probe is obstructed or malfunctioning.

In polarography, the voltage needed to cause depolarization of the cathode is called the: A. Half-wave potential B. Isopotential point C. Decomposition potential D. Polarization potential

Correct Answer: C 42. C In polarography, a minimum negative voltage must be applied to the cathode to cause reduction of metal ions (or O2) in solution. This is called the decomposition potential. It is concentration dependent (dilute solutions require greater negative voltage), and can be determined using the Nernst equation.

Select the products formed from the forward reaction of ALT. A. Aspartate and alanine B. Alanine and α-ketoglutarate C. Pyruvate and glutamate D. Glutamine and NAD+

Correct Answer: C 43. C Because glutamate is a common product for transaminases, pyruvate (a three-carbon ketoacid) and glutamate would be generated from the transamination reaction between alanine and α-ketoglutarate.

SITUATION: A 6-year-old child being treated with phenytoin was recently placed on valproic acid for better control of seizures. After displaying signs of phenytoin toxicity including ataxia, a stat phenytoin is determined to be 15.0 mg/L (reference range 10-20 mg/L). A peak blood level drawn 5 hours after the last dose is 18.0 mg/L. The valproic acid measured at the same time is within therapeutic limits. Quality control is within acceptable limits for all tests, but the physician questions the accuracy of the results. What is the most appropriate next course of action? A. Repeat the valproic acid level using the last specimen B. Repeat the phenytoin on both trough and peak samples using a different method C. Recommend measurement of free phenytoin using the last specimen D. Recommend a second trough level be measured

Correct Answer: C 43. C Phenytoin levels must be monitored closely because toxic drug levels can occur unexpectedly due to changing pharmacokinetics. Phenytoin follows a nonlinear rate of elimination, which means that clearance decreases as blood levels increase. At high blood levels, saturation of the hepatic hydroxylating enzymes can occur, causing an abrupt increase in the blood level from a small increase in dose. The drug half-life estimated from the two drug levels is approximately 15 hours, which is within the range expected for children, so decreased clearance is not likely the problem. Valproic acid competes with phenytoin for binding sites on albumin. Free phenytoin is the physiologically active fraction and is normally very low, so small changes in protein binding can cause a large change in free drug. For example, a 5% fall in protein binding caused by valproic acid can increase the free phenytoin level by 50%. This patient's free phenytoin level should be measured, and the dose of phenytoin reduced to produce a free drug level that is within the therapeutic range.

Which assay is used to confirm difficult cases of hypothyroidism? A. Free T3 assay B. Free thyroxine index C. Thyrotropin-releasing hormone (TRH) stimulation test D. TBG assay

Correct Answer: C 45. C The TRH stimulation test is used to confirm borderline cases of abnormal thyroid function. In normal patients, intravenous injection of 500 μg of TRH causes a peak TSH response within 30 minutes. In patients with primary hypothyroidism, there is an exaggerated response (>30 mU/L). Patients with hyperthyroidism do not show the expected rise in TSH after TRH stimulation.

In familial β dyslipoproteinemia (formerly type III hyperlipoproteinemia), which lipoprotein accumulates? A. Chylomicrons B. VLDL C. IDL D. VLDL

Correct Answer: C 47. C IDLs have roughly equal amounts of cholesterol and triglyceride. The IDL has a density of about 1.006-1.020, causing it to float on the 1.063 density potassium bromide solution used to recover LDL by ultracentrifugation. IDL has faster electrophoretic mobility on agarose than beta lipoprotein. These observations gave rise to the terms "floating beta" and "broad beta," respectively. Familial dysbetalipoproteinemia is in part caused by a polymorphism of apoE (apo-E2) that has poor affinity for the apo-E receptor on hepatocytes. Not all persons with the homozygous polymorphism develop the disease; thus, other factors are necessary for the accumulation of IDL.

What role do CTx and NTx play in the management of osteoporosis? A. Increased urinary excretion is diagnostic of early stage disease B. Increased levels indicate a low risk of developing osteoporosis C. Decreased urinary excretion indicates a positive response to treatment D. The rate of urinary excretion correlates with the stage of the disease

Correct Answer: C 47. C Markers for both bone formation and resorption are used to monitor treatment for osteoporosis. Serum and urinary measurements of CTx and NTx and urinary deoxypyridinoline are used to monitor medications such as biphosphonates that inhibit bone resorption. Levels fall with successful treatment. DEXA scan, an x-ray procedure based on subtraction of surrounding tissue, is the most sensitive diagnostic test for osteoporosis and can show bone loss as small as 1%. However, it takes months before a DEXA scan shows increased bone remodeling following treatment.

Which of the following statements regarding the naming of transaminases is true? A. Serum glutamic oxaloacetic transaminase (SGOT) is the older abbreviation for ALT B. Serum glutamic pyruvic transaminase (SGPT) is the older abbreviation for AST C. SGPT is the older abbreviation for ALT D. SGOT is the newer abbreviation for AST

Correct Answer: C 47. C SGOT refers to the products measured in the in vitro reaction, and is more correctly named AST for the four-carbon amino acid substrate aspartate. SGPT is the older name referring to the products of the reaction for ALT. SGPT is more correctly named ALT for the three-carbon amino acid substrate alanine.

A serum thyroid panel reveals an increase in total T4, normal TSH, and normal free T4. What is the most likely cause of these results? A. Primary hyperthyroidism B. Secondary hyperthyroidism C. Euthyroid with increased thyroxine-binding protein D. Subclinical hypothyroidism

Correct Answer: C 48. C Patients with a normal TSH are euthyroid, and most commonly an increase in total T4 in these patients is caused by an increase in TBG. An increase in TBG causes an increase in total T4 but not free T4. Subclinical hypothyroidism is usually associated with a high TSH, but normal free T3 and free T4. When TSH is indeterminate, the diagnosis is made by demonstrating an exaggerated response to the TRH stimulation test.

Which statement best describes testing recommendations for vitamin D? A. Vitamin D testing should be reserved only for those persons who demonstrate hypercalcemia of an undetermined cause B. Vitamin D testing should be specific for the 1,25(OH)D3 form C. Testing should be for total vitamin D when screening for deficiency D. Vitamin D testing should not be performed if the patient is receiving a vitamin D supplement

Correct Answer: C 49. C Vitamin D deficiency is far more common than vitamin D excess, and screening for vitamin D deficiency is advocated especially for dark-skinned persons and people who do not get adequate sunlight. Provitamin D is a steroid, and vitamin D is now considered a hormone rather than a vitamin. The hormone regulates transcription of over 200 genes and has pronounced effects on both dendritic cells and T lymphocytes. Deficiency is associated with many chronic diseases including autoimmune diseases, cancers, hypertension, and heart disease. There are two forms of the vitamin, ergocalciferol (D2) and cholecalciferol (D3). Active D2 and D3 are formed when two hydroxyl groups are added, the first being at the 25 position by the liver and the second at the α-1 position by the kidney. The majority of the circulating vitamin D is in the 25-hydroxylated form of D2 and D3, called 25(OH)D. The plasma 25(OH)D concentration is an expression of both dietary and endogenous vitamin D and is the most appropriate test for detecting nutritional vitamin D deficiency. Since the effect on calcium is derived from the active 1,25 form of the vitamin, plasma 1,25(OH)D concentration is a more specific test for hypervitaminosis D.

Select the five pharmacological parameters that determine serum drug concentration. A. Absorption, anabolism, perfusion, bioactivation, excretion B. Liberation, equilibration, biotransformation, reabsorption, elimination C. Liberation, absorption, distribution, metabolism, excretion D. Ingestion, conjugation, integration, metabolism, elimination

Correct Answer: C 5. C Liberation is the release of the drug and absorption is the transport of drug from the site of administration to the blood. The percent of drug absorption and the rate of absorption determine the bioavailable fraction, f. This is the fraction of the dose that reaches the blood. Distribution refers to the delivery of the drug to the tissues. It involves dilution and equilibration of the drug in various fluid compartments including the blood, and is influenced by binding to proteins and blood cells. Metabolism is the process of chemical modification of the drug by cells. This results in production of metabolites with altered activity and solubility. Excretion is the process by which the drug and its metabolites are removed from the body.

Which tumor marker is associated with cancer of the urinary bladder? A. CA-19-9 B. CA-72-4 C. Nuclear matrix protein D. Cathepsin-D

Correct Answer: C 5. C Nuclear matrix proteins (NMPs) are RNA-protein complexes. NMP-22 is shed into the urine in persons with bladder carcinoma and is about 25-fold higher than normal in this condition. It has a clinical sensitivity of about 70% but is likely to be negative when the tumor is low grade. Other markers used for detection of bladder cancer include bladder tumor associated analytes (BTAs), a variant of the complement factor H protein; cytokeratin-20, a variant cytokeratin (fibrous protein) in the cytoplasm of malignant bladder epithelium; and telomerase, an enzyme that adds nucleotides to the ends of chromosomes, preventing telomere degradation. The specificity of these tests varies from approximately 75%-80%. Bladder cancer can also be detected by FISH because it is associated with a high incidence of ploidy and other chromosomal abnormalities that can be detected by fluorescent-labeled DNA probes. FISH specificity is over 94%, and like the immunoassays its sensitivity is higher for high-grade tumors (approximately 78% for grade 2 and 94% for grade 3 cancers).

Convert10.0mg/dLcalcium(atomicweight=40.1) to International System of Units (SI). A. 0.25 B. 0.40 C. 2.5 D. 0.4

Correct Answer: C 5. C The SI unit is the recommended method of reporting clinical laboratory results. The SI unit for all electrolytes is millimole per liter. To convert from milligrams per deciliter to millimoles per liter, multiply by 10 to convert to milligrams per liter, then divide by the atomic mass expressed in milligrams. 10.0 mg/dL × 10.0 dL/1.0 L = 100.0 mg/L 100.0 mg/L × 1.0 mmol/40.1 mg = 2.5 mmol/L

Plate 5 is a densitometric scan of a serum protein electrophoresis sample. The relative and absolute concentration of each fraction and reference limits are shown below the scan. What is the correct classification of this densitometric pattern? A. Polyclonal gammopathy associated with chronic inflammation B. Nephrotic syndrome C. Acute inflammation D. Hepatic cirrhosis

Correct Answer: C 5. C This pattern is characterized by significant relative increases in the α1- and α2-fractions and a decrease in serum albumin concentration. This pattern is most often caused by increased production of acute phase reactants such as α1-antitrypsin and haptoglobin that are associated with acute inflammation. This pattern is seen in myocardial infarction and other forms of acute tissue injury, the early stage of acute infection, and pregnancy.

Which of the following liver diseases produces the highest levels of transaminases? A. Hepatic cirrhosis B. Obstructive jaundice C. Hepatic cancer D. Alcoholic hepatitis

Correct Answer: C 51. C Elevation of transaminases is greatest in acute hepatitis (20-50 × URL). Levels are moderately elevated (5-10 × URL) in hepatic cancer. They are slightly elevated (2-5 × URL) in chronic hepatitis, hepatic cirrhosis, alcoholic hepatitis, and obstructive jaundice.

What type of measuring circuit is used in a freezing point osmometer? A. Electrometer B. Potentiometer C. Wheatstone bridge D. Thermal conductivity bridge

Correct Answer: C 51. C The resistance of the thermistor is measured using a network of resistors called a Wheatstone bridge. When the sample is frozen, the bridge is balanced using a calibrated variable resistor, so that no current flows to the readout. The resistance required to balance the meter is equal to the resistance of the thermistor.

Which of the following methods is most useful in order to detect sample misidentification? A. Cumulative summation B. Critical limit C. Delta limit D. Significant change limit

Correct Answer: C 52. C Cumulative summation is a statistical method used in quality assurance to detect a trend in QC results. Critical limits are used to define when medical intervention is likely needed to prevent injury or death. The significant change limit is the difference in test results that is medically significant, or that which cannot be attributed to the sum of normal physiological and analytical variation. The delta limit (delta check) determines whether the difference between two measurements usually 24-48 hours apart exceeds the expected. Although this can result from an abrupt change in the patient's status, other causes are sample misidentification, contamination, and random error. Delta limits are expressed in percent and vary depending on analyte stability.

The method for measuring iron or lead by plating the metal and then oxidizing it is called: A. Polarography B. Coulometry C. Anodic stripping voltometry D. Amperometry

Correct Answer: C 54. C Anodic stripping voltometry is used to measure lead and iron. The cation of the metal is plated onto a mercury cathode by applying a negative charge. The voltage of this electrode is reversed until the plated metal is oxidized back to a cation. Current produced by oxidation of the metal is proportional to concentration.

What is the most appropriate fasting procedure when a lipid study of triglyceride, total cholesterol, HDL cholesterol, and LDL cholesterol tests are ordered? A. 8 hours; nothing but water allowed B. 10 hours; water, smoking, coffee, tea (no sugar or cream) allowed C. 12 hours; nothing but water allowed D. 16 hours; water, smoking, coffee, tea (no sugar or cream) allowed

Correct Answer: C 54. C Lipid orders that include triglyceride and LDL cholesterol should always be performed using a plasma or serum specimen collected after a 12-14 hour fast. The patient should be instructed to drink nothing but water during this period. Fasting specimens are preferred for total and HDL cholesterol as well, but nonfasting specimens may be used for initial screening purposes.

The term isocratic is used in high-performance liquid chromatography (HPLC) to mean the: A. Mobile phase is at constant temperature B. Stationary phase is equilibrated with the mobile phase C. Mobile phase consists of a constant solvent composition D. Flow rate of the mobile phase is regulated

Correct Answer: C 55. C An isocratic separation uses a single mobile phase of constant composition, pH, and polarity, and requires a single pump. Some HPLC separations use a gradient mobile phase to increase distance between peaks. Gradients are made by mixing two or more solvents using a controller to change the proportions of solvent components.

An EDTA blood sample is collected from a nonfasting person for a CBC. The physician collected the sample from the femoral vein because venipuncture from the arm was unsuccessful. He called the lab 15 minutes after the sample arrived and requested a lipid study including triglyceride, total cholesterol, HDL cholesterol, and LDL cholesterol. Which test results should be used to evaluate the patient's risk for coronary artery disease? A. Total cholesterol and LDL cholesterol B. LDL cholesterol and triglyceride C. Total cholesterol and HDL cholesterol D. Total cholesterol and triglyceride

Correct Answer: C 57. C NCEP recommends a 12-hour fasting sample when screening persons for risk of coronary artery disease. However, if a fasting sample is unavailable, NCEP recommends performing the total cholesterol and HDL cholesterol because these tests are least affected by recent ingestion of food. If the total cholesterol is ≥ 200 mg/dL or the HDL cholesterol is < 40 mg/dL, then testing for LDL cholesterol and triglycerides should be performed when a fasting sample can be obtained. An EDTA plasma sample is acceptable for most enzymatic cholesterol and triglyceride assays.

What type of detector is used in high-performance liquid chromatography with electrochemical detection (HPLC-ECD)? A. Calomel electrode B. Conductivity electrode C. Glassy carbon electrode D. Polarographic electrode

Correct Answer: C 59. C HPLC-ECD uses a glassy carbon measuring electrode and a silver-silver chloride reference. The analyte is oxidized or reduced by holding the glassy carbon electrode at a positive voltage (oxidization) or negative voltage (reduction). The resulting current flow is directly proportional to concentration. Phenolic groups such as catecholamines can be measured by HPLC-ECD.

A person presents with a cushingoid appearance and an elevated 24-hour urinary cortisol level. The plasma adrenocotropic hormone (ACTH) is very elevated, and the physician suspects the cause is ectopic ACTH production. Which test would be most useful in substantiating this diagnosis? A. Plasma cortisol B. CA-50 C. Alkaline phosphatase isoenzymes D. AFP

Correct Answer: C 6. C Most often, ectopic ACTH production occurs in lung cancer. Tumors of the lung are often associated with the production of placental-like alkaline phosphatase, and a positive finding would support the diagnosis of an ectopic (nonpituitary) source of ACTH. Many other tumor markers, including neuron-specific enolase and parathyroid hormone-related protein, are also increased in lung cancers. CA-50 (along with CA-19-9) shares the same antigenic determinant as Lewis A and is a marker for recurrence and treatment of gastrointestinal and pancreatic cancers. AFP is the predominant protein produced by the fetus, and plasma levels are increased primarily in yolk sac, liver, and testicular tumors.

When calibrating a pH meter, unstable readings occur for both pH 7.00 and 4.00 calibrators, although both can be set to within 0.1 pH unit. Select the most appropriate course of action. A. Measure the pH of the sample and report to the nearest 0.1 pH B. Replace both calibrators with unopened buffers and recalibrate C. Examine the reference electrode junction for salt crystals D. Move the electrodes to another pH meter and calibrate

Correct Answer: C 6. C Noise in pH measurements often results from a blocked junction between the reservoir of the reference electrode and test solution. This occurs when salt crystals collect at the junction or when KCl concentration in the reservoir increases due to evaporation of water. The fluid in the reference electrode should be replaced with warm deionized water. After the crystals have dissolved, the water is replaced with fresh reference electrolyte solution.

Convert 2.0 mEq/L magnesium (atomic weight = 24.3) to milligrams per deciliter. A. 0.8 mg/dL B. 1.2 mg/dL C. 2.4 mg/dL D. 4.9 mg/dL

Correct Answer: C 6. C To convert from milliequivalent per liter to milligrams per deciliter, first calculate the milliequivalent weight (equivalent weight expressed in milligrams), which is the atomic mass divided by the valence. Because magnesium is divalent, each mole has the charge equivalent of 2 mol of hydrogen. Then, multiply the milliequivalent per liter by the milliequivalent weight to convert to milligrams per liter. Next, divide by 10 to convert milligrams per liter to milligrams per deciliter. Milliequivalent weight Mg = 24.3 ÷ 2 = 12.15 mg/mEq 2.0 mEq/L × 12.15 mg/mEq = 24.3 mg/L 24.3 mg/L × 1.0 L/10.0 dL = 2.4 mg/dL

Which of the following statements regarding ALP is true? A. All isoenzymes of ALP are antigenically distinct and can be identified by specific antibodies B. Highest serum levels are seen in intrahepatic obstruction C. Elevated serum ALP seen with elevated GGT suggests a hepatic source D. When jaundice is present, an elevated ALP suggests acute hepatitis

Correct Answer: C 60. C ALP isoenzymes can result from different genes or from modification of a common gene product in the tissues. Some differ mainly in carbohydrate content and cannot be identified by immunologic methods. Highest levels of ALP are seen in Paget's disease of bone, where ALP can be as high as 25 times the URL. GGT in serum is derived from the hepatobiliary system and is increased in alcoholic hepatitis and hepatobiliary obstruction. It is not increased in diseases of bone or in pregnancy. When the increase in GGT is twofold higher than the increase in ALP, the liver is assumed to be the source of the elevated ALP. Serum ALP is a sensitive marker for extrahepatic obstruction, which causes an increase of approximately 10 times the URL. A lesser increase is seen in intrahepatic obstruction. ALP is only mildly elevated in acute hepatitis as a result of accompanying obstruction.

Which form of hyperbilirubinemia is caused by an inherited absence of UDP-glucuronyl transferase? A. Gilbert's syndrome B. Rotor syndrome C. Crigler-Najjar syndrome D. Dubin-Johnson syndrome

Correct Answer: C 60. C Crigler-Najjar syndrome is a rare condition that occurs in two forms. Type 1 is inherited as an autosomal recessive trait and causes a total deficiency of UDP-glucuronyl transferase. Life expectancy is less than 1 year. Type 2 is an autosomal dominant trait and is characterized by lesser jaundice and usually the absence of kernicterus. Bilirubin levels can be controlled with phenobarbital, which promotes bilirubin excretion. Gilbert's syndrome is an autosomal recessive condition characterized by decreased bilirubin uptake and decreased formation of bilirubin diglucuronide. It is the most common form of inherited jaundice. UDP glucuronyl transferase activity is reduced owing to an increase in the number of AT repeats in the promoter region of the gene. Dubin-Johnson and Rotor syndromes are autosomal recessive disorders associated with defective delivery of bilirubin into the biliary system.

In thin-layer chromatography (TLC), the distance the solute migrates divided by the distance the solvent migrates is the: A. tR B. Kd C. Rf D. pK

Correct Answer: C 62. C Rf is the distance migrated by the solute divided by the distance migrated by the solvent. The tR refers to the retention time of the solute in HPLC or gas chromatography (GC). The Kd is the partition coefficient, and is a measure of the relative affinity of solutes for the stationary phase. The solute with the greater Kd will be retained longer. The pK is the negative logarithm of K, the ionization constant, and is a measure of ionization.

What is the purpose of the saponification step used in the Abell-Kendall method for cholesterol measurement? A. Remove phospholipids B. Reduce sterol molecules structurally similar to cholesterol C. Convert cholesterol esters to free cholesterol D. Remove proteins that can interfere with color formation

Correct Answer: C 62. C The Abell-Kendall method is the reference method for cholesterol assay because differences in esterase activity and interference in the peroxidase step are potential sources of error in enzymatic assays. Saponification is performed to hydrolyze the fatty acid esters of cholesterol, forming free cholesterol. This is required because the reagents react more intensely with cholesterol esters than with free cholesterol. Saponification is followed by extraction of cholesterol in petroleum ether to separate it from proteins and interfering substances. The extract is reacted with sulfuric acid, acetic anhydride, and acetic acid (Liebermann-Burchard reagent), which oxidizes the cholesterol and forms a colored product.

A lab measures total bilirubin by the Jendrassik-Grof bilirubin method with sample blanking. What would be the effect of moderate hemolysis on the test result? A. Falsely increased due to optical interference B. Falsely increased due to release of bilirubin from RBCs C. Falsely low due to inhibition of the diazo reaction by hemoglobin D. No effect due to correction of positive interference by sample blanking

Correct Answer: C 64. C The sample blank measures the absorbance of the sample and reagent in the absence of azobilirubin formation and corrects the measurement for optical interference caused by hemoglobin absorbing the wavelength of measurement. However, hemoglobin is an inhibitor of the diazo reaction and will cause falsely low results in a blank corrected sample. For this reason, direct bichromatic spectrophotometric methods are preferred when measuring bilirubin in neonatal samples, which are often hemolyzed.

What is the confirmatory method for measuring drugs of abuse? A. HPLC B. Enzyme-multiplied immunoassay technique (EMIT) C. Gas chromatography with mass spectroscopy (GC-MS) D. TLC

Correct Answer: C 65. C GC-MS determines the mass spectrum of the compounds eluting from the analytic column. Each substance has a unique and characteristic spectrum of mass fragments. This spectrum is compared to spectra in a library of standards to determine the percent match. A match of greater than 95% is considered confirmatory.

Which of the following conditions involving electrolytes is described correctly? A. Pseudohyponatremia occurs only when undiluted samples are measured B. Potassium levels are slightly higher in heparinized plasma than in serum C. Hypoalbuminemia causes low total calcium but does not affect Cai D. Hypercalcemia may be induced by low serum magnesium

Correct Answer: C 66. C When serum albumin is low, the equilibrium between bound and Cai is shifted, producing increased Cai. This inhibits release of PTH by negative feedback until the Cai level returns to normal. Potassium is released from platelets and leukocytes during coagulation, causing serum levels to be higher than plasma. Pseudohyponatremia is a measurement error caused by diluting samples containing excessive fat or protein. The colloids displace plasma water, resulting in less electrolytes being delivered into the diluent. Only ion-selective electrodes that measure whole blood or undiluted serum are unaffected. Magnesium is needed for release of PTH, and PTH causes release of calcium and magnesium from bone. Therefore, hypocalcemia can be associated with either magnesium deficiency or magnesium excess.

Which type of dietary fatty acid is not associated with an increase in serum LDL cholesterol production? A. Monounsaturated trans fatty acids B. Saturated fatty acids C. Monounsaturated cis fatty acids D. Monounsaturated trans Ω-9 fatty acids

Correct Answer: C 68. C Polyunsaturated and cis monounsaturated fatty acids are not associated with increased production of LDL cholesterol. On the other hand, saturated and trans monounsaturated fatty acids are both associated with increased LDL. Cis fatty acids are those in which the H atoms belonging to the double-bonded carbons are on the same side of the molecule. Ω-9 (n-9) fatty acids are those with a double bond located 9 carbons from the terminal methyl group. Ω Fatty acids are associated with increased cholesterol, if the hydrogens attached to the double-bonded carbons are in the trans position.

When performing a sweat chloride collection, which of the following steps will result in analytical error? A. Using unweighed gauze soaked in pilocarpine nitrate on the inner surface of the forearm to stimulate sweating B. Collecting more than 75 mg of sweat in 30 minutes C. Leaving the preweighed gauze on the inside of the arm exposed to air during collection D. Rinsing the collected sweat from the gauze pad using chloride titrating solution

Correct Answer: C 68. C The sweat chloride procedure requires the application of pilocarpine to stimulate sweating, and the use of iontophoresis (application of 0.16-mA current for 5 minutes) to bring the sweat to the surface. After iontophoresis, the skin on the inner surface of the forearm is washed with deionized water and dried, and a preweighed pair of 2-in.2 pads is taped to the skin. During the 30-minute collection of sweat, the gauze must be completely covered to prevent contamination and loss of sweat by evaporation. The Gibson-Cooke reference method for sweat chloride uses the Schales and Schales method (titration by Hg[NO3]2 with diphenylcarbazone indicator) to assay 1.0 mL of sweat eluted from the gauze with 5 mL of water. A Cotlove chloridometer is often used to measure sweat chloride. The sweat is eluted from the gauze with the titrating solution to facilitate measurement. Alternatively, a macroduct collection system may be used that does not require weighing. A minimum mass of 75 mg sweat is required for collection in gauze and 15 μL sweat for collection in macroduct tubing.

The phrase "first-pass hepatic metabolism" means that: A. One hundred percent of a drug is excreted by the liver B. All drug is inactivated by hepatic enzymes after one pass through the liver C. Some drug is metabolized from the portal circulation, reducing bioavailability D. The drug must be metabolized in the liver to an active form

Correct Answer: C 7. C Drugs given orally enter the blood via the portal circulation and are transported directly to the liver. Some drugs are excreted by the liver, and a fraction will be lost by hepatic excretion before the drug reaches the general circulation. An example is propranolol, a β-blocker that reduces heart rate and hypertension. The bioavailable fraction is 0.2-0.4 when given orally because much of the drug is removed by first-pass hepatic metabolism.

Which statement best describes the relationship between luteinizing hormone (LH) and follicle- stimulating hormone (FSH) in cases of dysmenorrhea? A. Both are usually increased when there is pituitary adenoma B. Increases in both hormones and a decrease in estrogen signal a pituitary cause of ovarian failure C. Both hormones normally peak 1-2 days before ovulation D. In menopause, the LH level at the midcycle peak is higher than the level of FSH

Correct Answer: C 7. C In women, serum or urine LH and FSH are measured along with estrogen and progesterone to evaluate the cause of menstrual cycle abnormalities and anovulation. Both hormones show a pronounced serum peak 1-2 days prior to ovulation and urine peak 20-44 hours before ovulation. Normally, the LH peak is sharper and greater than the FSH peak; however, in menopause, the FSH usually becomes higher than LH. In patients with primary ovarian failure, the LH and FSH are elevated because low estrogen levels stimulate release of luteinizing hormone-releasing hormone (LHRH) from the hypothalamus. Conversely, in pituitary failure, levels of FSH and LH are reduced, and this reduction causes a deficiency of estrogen production by the ovaries.

What is the principle of the transcutaneous bilirubin assay? A. Conductivity B. Amperometric inhibition C. Multiwavelength reflectance photometry D. Infrared spectroscopy

Correct Answer: C 70. C Measurement of bilirubin concentration through the skin requires the use of multiple wavelengths to correct for absorbance by melanin and other light-absorbing constituents of skin and blood. More than 100 wavelengths and multiple reflectance measurements at various sites may be used to derive the venous bilirubin concentration in mg/dL. Such devices have been shown to have a high specificity. They can be used to identify neonates with hyperbilirubinemia, and to monitor treatment.

Which method is the most useful when screening for errors of amino and organic acid metabolism? A. Two-dimensional thin-layer chromatography B. Gas chromatography C. Electrospray ionization tandem-mass spectroscopy D. Inductively charged coupled-mass spectroscopy

Correct Answer: C 70. C While two-dimensional thin-layer chromatography can separate both amino and organic acids, it is not sufficiently sensitive for newborn screening. Electrospray ionization allows a small alcohol-extracted whole-blood sample to be analyzed by two mass spectrometers without prior separation by liquid or gas chromatography. Disorders of both organic and fatty acid metabolism are identified by the specific pattern of acylcarnitine ions produced. Amino acids are detected as amino species that have lost a carboxyl group during ionization, a process called neutral loss.

Which of the following statements regarding amylase methods is true? A. Dilution of serum may result in lower than expected activity B. Methods generating NADH are preferred because they have higher sensitivity C. Synthetic substrates can be conjugated to p-nitrophenol (PNP) for a kinetic assay D. The reference range is consistent from method to method

Correct Answer: C 73. C Many endogenous inhibitors of amylase, such as wheat germ, are found in serum. Diluted samples often show higher than expected activity caused by dilution of the inhibitor. Units of amylase activity vary widely depending upon the method of assay and calibration. Synthetic substrates such as maltotetrose or 4-nitrophenyl maltohepatoside can be used for kinetic assays. Maltotetrose is hydrolyzed to maltose by amylase, and the maltose hydrolyzed by α-glucosidase or maltose phosphorylase, forming glucose or glucose-1-phosphate, respectively. These can be measured by coupling to NADH-generating reactions. Antibodies to the salivary isoenzyme can be added to synthetic substrate assays to inhibit S-type amylase.

Given the following real-time PCR amplification curve, what is the threshold cycle? A. 15 B. 20 C. 25 D. 30

Correct Answer: C 77. C The maximum curvature of the plot approximates the threshold cycle. A line is drawn from the threshold value on the y-axis through the curve, and a perpendicular dropped to the x-axis. The Ct is determined by the intersection point on the x-axis. The threshold is usually determined by an algorithm but can be calculated manually as 10 times the average standard deviation of the RFUs for cycles 2-10.

Which of the following enzymes allows creatinine to be measured by coupling the creatinine amidohydrolase (creatininase) reaction to the peroxidase reaction? A. Glucose-6-phosphate dehydrogenase B. Creatinine iminohydrolase C. Sarcosine oxidase D. Creatine kinase

Correct Answer: C 8. C The peroxidase-coupled enzymatic assay of creatinine is based upon the conversion of creatinine to creatine by creatinine amidohydrolase (creatininase). The enzyme creatinine amidinohydrolase (creatinase) then hydrolyzes creatine to produce sarcosine and urea. The enzyme sarcosine oxidase converts sarcosine to glycine producing formaldehyde and hydrogen peroxide. Peroxidase then catalyzes the oxidation of a dye (4-aminophenazone and phenol) by the peroxide forming a red-colored product. This method is more specific than the Jaffe reaction, which tends to overestimate creatinine by about 5% in persons with normal renal function.

Which of the following contributes the most to the serum total CO2? A. PCO2 B. dCO2 C. HCO3- D. Carbonium ion

Correct Answer: C 8. C The total CO2 is the sum of the dCO2, H2CO3 (carbonic acid or hydrated CO2), and bicarbonate (as mainly NaHCO3). When serum is used to measure total CO2, the dCO2 is insignificant because all the CO2 gas has escaped into the air. Therefore, serum total CO2 is equivalent to the bicarbonate concentration. Total CO2 is commonly measured by potentiometry. An organic acid is used to release CO2 gas from bicarbonate and pCO2 is measured with a Severinghaus electrode. Alternately, bicarbonate can be measured by an enzymatic reaction using phosphoenol pyruvate carboxylase. The enzyme forms oxaloacetate and phosphate from phosphoenol pyruvate and bicarbonate. The oxaloacetate is reduced to malate by malate dehydrogenase and NADH is oxidized to NAD+. The negative reaction rate is proportional to plasma bicarbonate concentration.

Calculate the pH of a solution of 1.5 x 10^-5M NH4OH. A. 4.2 B. 7.2 C. 9.2 D. 11.2

Correct Answer: C 9. C First, calculate the pOH of the solution. pOH = -Log [OH-] pOH = - Log 1.5 x 10-5 = 4.82 pH = 14 - pOH pH = 14 - 4.8 = 9.2

A 45-year-old woman complains of fatigue, heat intolerance and hair loss. Total and ree T4 are abnormally low. If the TSH showed marked elevation, this would be consistent with: A. Grave disease B. an adenoma of the thyroid C. thyrotoxicosis D. primary hypothyroidism

Correct Answer: D

A chemiluminescent EIA: A. measures absorption of light B. is less sensitive than radioisotopic reactions C. is monitored by the use of a gamma counter D. is quantitated by the amount of light produced by the reaction

Correct Answer: D

A patient's blood was drawn at 8 am for a serum iron determination. The result was 85 ug/dL (15.2 umol/L). A repeat specimen was drawn at 8 PM; the serum was stored at 4 C and run the next morning. The result was 40 ug/dL (7.2 umol/L). These result are most likely due to: A. iron deficiency anemia B. improper storage of the specimen C. possible liver damage D. the time of day the second specimen was drawn

Correct Answer: D

An antiepileptic (or anticonvulsant) used to control seizure disorders is: A. digozin B. acetaminophen C. lithium D. phenytoin

Correct Answer: D

An arterial blood specimen submitted for blood gas analysis was obtained at 8:30 AM but was not received in the laboratory until 11 AM. The technologist should: A. perform the test immediately upon reciept B. perform the test only if the specimen was submitted in ice water C. requires a venous blood specimen D. request a new arterial specimen be obtained

Correct Answer: D

Before unconjugated bilirubin can react with Ehrlich diazo reagent, which of the following must be added? A. acetone B. ether C. distilled water D. caffeine

Correct Answer: D

Cancer antigen 125 (CA 125) is a tumor marker associate with: A. breast carcinoma B. colon cancer C. lung cancer D. ovarian and endometrial carcinoma

Correct Answer: D

Cocaine is metabolized to: A. carbomazepine B. codeine C. hydrocodone D. benzoylecgonine

Correct Answer: D

Erroneous ammonia levels can be eliminated by all of the following except: A. assuring water and reagents are ammonia-free B. separating plasma form cells and performing test analysis as soon as possible C. drawing the specimen in a pre-chilled tube and immersing the tube in ice D. storing the specimen protected form light until the analysis is done

Correct Answer: D

Given the following results: -alkaline phosphatase: marked increase -aspartate amino transferase: slight increase -alanine amino transferase: slight increase -gamma-glutamyl transferase: marked increase This is most consistent with: A. acute hepatitis B. osteitis fibrosa C. chronic hepatitis D. obstructive jaundice

Correct Answer: D

High levels of which lipoprotein class are associated with decreased risk of accelerate atherosclerosis: A. Chylomicrons B. VLDL C. LDL D. HDL

Correct Answer: D

In respiratory acidosis, a compensatory mechanism is the increase in: A. chloride B. calcium C. potassium D. sodium

Correct Answer: D

In the Malloy and Evelyn method for the determination of bilirubin, the reagent that is reacted with bilirubin to form a purple azobilirubin is: A. dilue sulfuric acid B. diazonium sulfate C. sulfobromophthalein D. diazotized sulfanilic acid

Correct Answer: D

In the atomic absorption method for calcium, lanthanum is used: A. as an internal standard B. to bind calcium C. to eliminate protein interference D. to prevent phosphate interference

Correct Answer: D

Increased total serum lactidehydrogensase (LD) activity, confined to fractions 4 and 5 is most likely to be associated with: A. pulmonary infarciton B. hemolytic anemia C. myocardial infarction D. acute viral hepatitis

Correct Answer: D

Most chemical methods for determining total protein utilize which of the following reactions? A. molybdenum blue B. ferri-ferrocyanide C. resorcinol-HCL D. biuret

Correct Answer: D

Oligoclonal bands are present on electrophoresis of concentrated CSF and also on concurrently tested serum of the same patient. The proper interpretation is: A. diagnostic for primary CNS tumor B. diagnostic for multiple sclerosis C. CNS involvement by acute leukemia D. nondiagnostic for multiple sclerosis

Correct Answer: D

Regan isoenzyme has the sam properties as alkaline phosphatase that originates in the: A. skeleton B. kidney C. intestine D. placenta

Correct Answer: D

Rickets is associated with deficiency of which of the following vitamins? A. B1 B. C C. niacin D. D

Correct Answer: D

Stray light can be detected in a spectrophotometer by utilizing a: A. mercury vapor lamp B. homium oxide glass C. potassium dichromate solution D. sharp cutoff filter

Correct Answer: D

The biologically most active, naturally occurring androgen is: A. androstenedione B. cortisol C. epiandrosterone D. testosterone

Correct Answer: D

The enzyme present in almost all tissues that may be separated by electrophoresis into 5 components is: A. lipase B. transaminase C. creatine kinase D. lactate dehydrogenase

Correct Answer: D

The enzyme, which exists chiefly in skeletal muscle, heart, and brain, is grossly elevate in active muscular dystrophy, and rises early in myocardial infarction is: A. lipase B. transaminase C. lactate dehydrogenase D. creatine kinase

Correct Answer: D

The most consistent analytical error involved in the routine determination of HDL-cholesterol is caused by: A. incomplete precipitation of LDL-cholesterol B. coprecipitation of HDL- and LDL-cholesterol C. inaccurate protein estimation of HDL-cholesterol D. a small concentration of apoB-containing lipoproteins after precipitation

Correct Answer: D

The most widely used methods for bilirubin measurement are those based on the: A. Jaffe reaction B. Schale and Schale method C. 8-hyroxyquinoline reaction D. Jendrassik-Grof method

Correct Answer: D

The principle of the occult blood test depends upon the: A. coagulase ability of blood B. oxidative power of atmospheric oxygen C. hydrogen peroxide in hemoglobin D. peroxidase-like activity of hemoglobin

Correct Answer: D

The procedure used to determine the presence of neural tube defects is: A. lecithin/sphingomyelin ration B. amniotic fluid creatinine C. measurement of absorbance at 450 nm D. alpha-fetoprotein

Correct Answer: D

Which of the following is the Henderson-Hasselbalch equation? A. pKa = pH + log ([acid]/[salt]) B. pKa = pH + log ([salt]/[acid]) C. pH=pKa + log ([acid]/[salt]) D. pH=pKa + log ([salt]/[acid])

Correct Answer: D

Which of the following represents the end product of purine metabolism in humans? A. AMP and GMP B. DNA and RNA C. allantion D. uric acid

Correct Answer: D

Which of the following serum constituents is unstable if a blood specimen is left standing at room temperature for 8 hours before processing? A. cholesterol B. triglyceride C. creatinine D. glucose

Correct Answer: D

A procedure for aspartate aminotransferase (AST) is performed manually because of a repeating error code for nonlinearity obtained on the laboratory's automated chemistry analyzer; 0.05 mL of serum and 1.0 mL of substrate are used. The reaction rate is measured at 30°C at 340 nm using a 1.0 cM light path, and the delta absorbance (-∆A) per minute is determined to be 0.382. Based upon a molar absorptivity coefficient for NADH at 340 nm of 6.22 X 103 M-1 cM-1 L-1, calculate the enzyme activity in international units (IUs) per liter. A. 26 IU/L B. 326 IU/L C. 1228 IU/L D. 1290 IU/L

Correct Answer: D 16. D An IU is defined as 1 μmol of substrate consumed or product produced per minute. The micromoles of NADH consumed in this reaction are determined by dividing the change in absorbance per minute by the absorbance of 1 μmol of NADH. Because 1 mol/L/cm would have an absorbance of 6.22 X 103 absorbance units, then 1 μmol/mL/cm would produce an absorbance of 6.22. Therefore, dividing the δA per minute by 6.22 gives the micromoles of NADH consumed in the reaction. This is multiplied by the dilution of serum to determine the micromoles per milliliter, and multiplied by 1,000 to convert to micromoles per liter. IU/L = ΔA/min x TV(mL) × 1,000 mL/L / 6.22(A/μmol/mL/cM) x 1 cm x SV(mL) = ΔA/min × 1.05 x 1,000 6.22 X 0.05 = ΔA/min × 1,050 0.311 = ΔA/min × 3,376 = 0.382 × 3376 = 1,290 IU/L

Which formula correctly describes the relationship between absorbance and %T ? A. A = 2 - log %T B. A = log 1/T C. A = -log T D. All of these options

Correct Answer: D 1. D Absorbance is proportional to the inverse log of transmittance. A = -log T = log 1/T Multiplying the numerator and denominator by 100 gives: A = log (100/100 X T) 100 X T = %T, substituting %T for 100 X T gives: A = log 100/%T A = log 100 - log %T A = 2.0 - log %T For example, if %T = 10.0, then: A = 2.0 - log 10.0 log 10.0 = 1.0 A = 2.0-1.0 = 1.0

In which of the following cases is qualitative analysis of the drug usually adequate? A. To determine whether the dose of a drug with a low therapeutic index is likely to be toxic B. To determine whether a patient is complying with the physician's instructions C. To adjust dose if individual differences or disease alter expected response D. To determine whether the patient has been taking amphetamines

Correct Answer: D 1. D The purpose of therapeutic drug monitoring is to achieve a therapeutic blood drug level rapidly and minimize the risk of drug toxicity caused by overdose. Therapeutic drug monitoring is a quantitative procedure performed for drugs with a narrow therapeutic index (ratio of the concentration producing the desired effect to the concentration producing toxicity). Drug groups that require monitoring because of high risk of toxicity include aminoglycoside antibiotics, anticonvulsants, antiarrhythmics, antiasthmatics, immunosuppressive agents used for transplant rejection, and psychoactive drugs. When testing for abuse substances, the goal is usually to determine whether the drug is present or absent. The most common approach is to compare the result to a cutoff determined by measuring a standard containing the lowest level of drug that is considered significant.

SITUATION: A patient breathing room air has the following arterial blood gas and electrolyte results -pH = 7.54 -HCO3 = 18 mmol/L -Cl = 98 mmol/L -PCO2 = 18.5 mm Hg -Na = 135 mmol/L -TCO2 = 20 mmol/L -PO2 = 145 mm Hg -K = 4.6 mmol/L The best explanation for these results is: A. Blood for electrolytes was drawn above an IV B. Serum sample was hemolyzed C. Venous blood was sampled for arterial blood gases D. Blood gas sample was exposed to air

Correct Answer: D 11. D A patient breathing room air cannot have an arterial PO2 greater than 105 mm Hg because alveolar PO2 is 110 mm Hg when breathing 20% O2. Exposure to air caused loss of CO2 gas and increased pH.

A female with severe excessive pubic and facial hair growth (hirsutism) should be tested for which of the following hormones? A. Estrogen and progesterone B. Chorionic gonadotropin C. Growth hormone D. Testosterone and dehydroepiandrosterone sulfate

Correct Answer: D 11. D Excessive hair grown in females results from excessive androgen production, and is most commonly seen in polycystic ovarian syndrome, which produces high levels of ovarian-derived testosterone. It will also occur as a consequence of Cushing's syndrome and mild congenital adrenal hyperplasia. Therefore, cortisol and 17 α-hydroxyprogesterone can help identify those causes. Rapid onset of hirsutism can result from an ovarian or adrenal tumor. Dehydroepiandrosterone sulfate is produced only by the adrenals and would be useful in identifying those rare cases where the cause is an androgen-secreting adrenal tumor.

A linearity study is performed on a visible spectrophotometer at 650 nm and the following absorbance readings are obtained: See Harr pg 175 number 11 The study was repeated using freshly prepared standards and reagents, but results were identical to those shown. What is the most likely cause of these results? A. Wrong wavelength used B. Insufficient chromophore concentration C. Matrix interference D. Stray light

Correct Answer: D 11. D Stray light is the most common cause of loss of linearity at high-analyte concentrations. Light transmitted through the cuvette is lowest when absorption is highest. Therefore, stray light is a greater percentage of the detector response when sample concentration is high. Stray light is usually most significant when measurements are made at the extremes of the visible spectrum because lamp output and detector response are low.

A patient treated for a germ cell tumor has a total and free β-hCG assay performed prior to surgery. The result is 40,000 mIU/mL. One week following surgery, the hCG is 5,000 mIU/mL. Chemotherapy is started, and the hCG is measured 1 week later and found to be 10,000 mIU/mL. What does this indicate? A. Recurrence of the tumor B. Falsely increased hCG owing to drug interference with the assay C. Analytical error with the test reported as 5,000 mIU/mL D. Transient hCG increase caused by chemotherapy

Correct Answer: D 12. D Treatment of tumors with chemotherapy often causes a transient increase in the production of tumor markers as the drugs destroy tumor cells. The half-life of hCG is 24-36 hours; therefore, the expected decline 1 week postsurgery was consistent with the result of 5,000 mIU/mL. Initiation of chemotherapy probably caused the hCG to double in the following week. The hCG assay should be monitored at regular intervals for several months, since a failure for it to decline or an increased level would suggest recurrence.

A sample of amniotic fluid collected for fetal lung maturity studies from a woman with a pregnancy compromised by hemolytic disease of the newborn (HDN) has a creatinine of 88 mg/dL. What is the most likely cause of this result? A. The specimen is contaminated with blood B. Bilirubin has interfered with the measurement of creatinine C. A random error occurred when the absorbance signal was being processed by the analyzer D. The fluid is urine from accidental puncture of the urinary bladder

Correct Answer: D 13. D Creatinine levels in this range are found only in urine specimens. Adults usually excrete between 1.2 and 1.5 g of creatinine per day. For this reason, creatinine is routinely measured in 24-hour urine samples to determine the completeness of collection. A 24-hour urine with less than 0.8 g/day indicates that some of the urine was probably discarded. Creatinine is also used to evaluate fetal maturity. As gestation progresses, more creatinine is excreted into the amniotic fluid by the fetus. Although a level above 2 mg/dL is not a specific indicator of maturity, a level below 2 mg/dL indicates immaturity.

Which of the following materials is best suited for verifying the wavelength calibration of a spectrophotometer? A. Neutral density filters B. Potassium dichromate solutions traceable to the National Bureau of Standards reference C. Wratten filters D. Holmium oxide glass

Correct Answer: D 13. D Wavelength accuracy is verified by determining the wavelength reading that gives the highest absorbance (or transmittance) when a substance with a narrow natural bandpass (sharp absorbance or transmittance peak) is scanned. For example, didymium glass has a sharp absorbance peak at 585 nm. Therefore, an instrument should give its highest absorbance reading when the wavelength dial is set at 585 nm. Holmium oxide produces a very narrow absorbance peak at 361 nm; likewise, the hydrogen lamp of a UV spectrophotometer produces a 656-nm emission line that can be used to verify wavelength. Neutral density filters and dichromate solutions are used to verify absorbance accuracy or linearity. A Wratten filter is a wide- bandpass filter made by placing a thin layer of colored gelatin between two glass plates and is unsuitable for spectrophotometric calibration.

Which of the following proteins has the highest pI? A. Albumin B. Transferrin C. Ceruloplasmin D. IgG

Correct Answer: D 15. D Albumin is the fastest migrating protein toward the anode at pH 8.6 followed by α1-, α2-, β-, and γ-globulins. Because albumin is fastest, it has the greatest net negative charge and lowest pI (about 4.6). γ-Globulins are predominantly immunoglobulins and have the highest pI (about 7.2).

Evaluate the following chromatogram of a whole-blood hemolysate, and identify the cause and best course of action. A. Result is not reportable because hemoglobin F is present and interferes B. The result is not reportable because hemoglobin C is present and interferes C. The result is not reportable because labile hemoglobin A1c is present D. The result is reportable; neither hemoglobin F or C interfere

Correct Answer: D 15. D The chromatogram is from a person with hemoglobin AC; however, hemoglobin C is completely separated from Hgb A1c and does not interfere. Hgb F is also present, but does not interfere unless its concentration is > 30%. Labile hemoglobin is formed initially when the aldehyde of glucose reacts with the N-terminal valine of the β globin chain. This Shiff base is reversible but is converted to Hgb A1c by rearrangement to a ketoamine. It is called labile A1c and produces a peak (LA1c) after HgF and before Hgb A1c. Therefore, it does not interfere.

The LD pleural fluid:serum ratio for a transudative fluid is usually: A. 3:1 or higher B. 2:1 C. 1:1 D. 1:2 or less

Correct Answer: D 15. D The lactate dehydrogenase activity of body fluids is normally less than serum, and a fluid to serum LD ratio greater than 1:2 is highly suggestive of an exudative process. Elevated lactate dehydrogenase in chest fluid is often caused by lung malignancy, metastatic carcinoma, Hodgkin's disease, and leukemia.

When should blood samples for trough drug levels be collected? A. 30 minutes after peak levels B. 45 minutes before the next dose C. 1-2 hours after the last dose D. Immediately before the next dose is given

Correct Answer: D 15. D The trough concentration of a drug is the lowest concentration obtained in the dosing interval. This occurs immediately before the absorption of the next dose given. Trough levels are usually collected just before the next dose is given.

Which of the following assays is used to determine the risk of developing cancer? A. Epidermal growth factor receptor (EGF-R) B. Squamous cell carcinoma antigen (SCC) C. c-erb B-2 gene expression D. p53 gene mutation

Correct Answer: D 16. D The p53 gene (tumor suppressor gene) is located on chromosome 17 and produces a protein that down-regulates the cell cycle. A mutation of p53 is associated with an increased incidence of many cancers. The c-erb B-2 gene is the same as HER-2/neu; it codes for a growth factor receptor with tyrosine kinase activity on the cell membrane. EGF-R is a receptor for epidermal growth factor and its overexpression in breast tissue is associated with a poorer prognosis. SCC is a glycoprotein antigen found in the cytoplasm of tumors of squamous origin and is secreted in the plasma of persons with uterine cancer.

Which of the following proteins migrates in the β region at pH 8.6? A. Haptoglobin B. Orosomucoprotein C. Antichymotrypsin D. Transferrin

Correct Answer: D 16. D Transferrin, β lipoprotein, C3, and C4 are the dominant proteins in the β-globulin region. Haptoglobin and α2- macroglobulin are the principal proteins in the α2-fraction. α1-Antitrypsin, α1-lipoprotein, and α1-acid glycoprotein (orosomucoprotein) make up most of the α1-fraction. Immunoglobulins dominate the γ region. Plasma is not used for protein electrophoresis because fibrinogen will produce a band resembling a small monoclonal protein in the beta region.

Which of the statements below about serum urea is true? A. Levels are independent of diet B. Urea is not reabsorbed by the renal tubules C. High BUN levels can result from necrotic liver disease D. BUN is elevated in prerenal as well as renal failure

Correct Answer: D 17. D Urea is completely filtered by the glomeruli but reabsorbed by the renal tubules at a rate dependent upon filtrate flow and tubular status. Urea levels are a sensitive indicator of renal disease, becoming elevated by glomerular injury, tubular damage, or poor blood flow to the kidneys (prerenal failure). Serum urea (and BUN) levels are influenced by diet and are low in necrotic liver disease.

In which of the following conditions is PSA least likely to be increased? A. Precancerous lesions of the prostate B. Postprostate biopsy C. Benign prostatic hypertrophy D. Post-digital rectal examination

Correct Answer: D 18. D PSA is a serine protease responsible for liquefaction of the seminal fluid. PSA has been used successfully to monitor for recurrence and follow the response of patients to androgen-suppression therapy. Currently, it is one of the few FDA-approved tumor markers for cancer screening. Although digital rectal examination raises the prostatic acid phosphatase level, it does not increase the concentration of PSA in the plasma. In addition to prostate cancer, PSA may be increased in acute or chronic prostate inflammation, benign prostate hypertrophy, and after transurethral prostate resection or prostate biopsy. As a result, the specificity of PSA is approximately 60% and the predictive value of a positive result approximately 30%.

In addition to measuring blood glucose, Hgb A1c, and microalbumin, which test should be done on diabetic persons once per year? A. Urine glucose B. Urine ketones C. Plasma fructosamines D. Estimated glomerular filtration rate

Correct Answer: D 19. D While urinary glucose can identify persons who may have diabetes, it is not sensitive enough to manage glucose control on a daily basis, and has been replaced by whole-blood glucose monitoring or continuous glucose monitoring. While the urinary ketone test is a useful screening test for diabetic and other forms of ketosis, the plasma β hydroxybutyrate test should be used to identify and monitor ketosis in diabetic persons. Fructosamine is a useful adjunct to Hgb A1c to identify poor control of blood glucose in the past 2-4 weeks, but has not been recommended for routine use in all diabetic patients.

Which of the following will shift the O2 dissociation curve to the left? A. Anemia B. Hyperthermia C. Hypercapnia D. Alkalosis

Correct Answer: D 21. D A left shift in the oxyhemoglobin dissociation curve signifies an increase in the affinity of Hgb for O2. This occurs in alkalosis, hypothermia, and in those hemoglobinopathies such as Hgb Chesapeake that increase the binding of O2 to heme. A right shift in the oxyhemoglobin dissociation curve lowers the affinity of Hgb for O2. This occurs in anemia due to increased 2,3-diphosphoglycerate (2,3-DPG), with increased body temperature, increased hydrogen ion concentration, hypercapnia (increased PCO2), and in some hemoglobinopathies, such as Hgb Kansas.

SITUATION: A physician calls to request a CK on a sample already sent to the laboratory for coagulation studies. The sample is 2-hour-old citrated blood and has been stored at 4°C. The plasma shows very slight hemolysis. What is the best course of action and the reason for it? A. Perform the CK assay on the sample because no interferent is present B. Reject the sample because it is slightly hemolyzed C. Reject the sample because it has been stored too long D. Reject the sample because the citrate will interfere

Correct Answer: D 22. D CK activity is lost with excessive storage, the most labile isoenzyme being CK-1. However, CK in serum is stable at room temperature for about 4 hours and up to 1 week at 4°C provided that an optimized method is used. Slight hemolysis does not interfere because CK is absent from RBCs. More significant hemolysis may cause positive interference by contributing ATP, glucose-6-PO4, and adenylate kinase to the serum. Calcium chelators remove magnesium as well as calcium and should not be used.

Which would be consistent with partially compensated respiratory acidosis? A. pH increased; PCO2 increased; Bicarbonate increased B. pH increased; PCO2 decreased; Bicarbonate decreased C. pH decreased; PCO2 decreased; Bicarbonate decreased D. pH decreased; PCO2 increased; Bicarbonate increased

Correct Answer: D 23. D Acidosis = low pH; respiratory = disturbance of PCO2; a low pH is caused by increased PCO2. In partially compensated respiratory acidosis, the metabolic component of the buffer system, bicarbonate, is retained. This helps to compensate for retention of PCO2 by titrating hydrogen ions. The compensatory component always moves in the same direction as the cause of the acid-base disturbance.

For which drug group are both peak and trough measurements usually required? A. Antiarrhythmics B. Analgesics C. Tricyclic antidepressants D. Aminoglycoside antibiotics

Correct Answer: D 23. D Aminoglycoside antibiotics cause damage to the eighth cranial nerve at toxic levels, resulting in hearing loss. When given at subtherapeutic doses, they fail to resolve infection. Most drugs falling in the other classes have a narrow peak-trough difference but are highly toxic when blood levels exceed the therapeutic range. Usually, these can be safely monitored by measuring trough levels.

In addition to polarography, what other electrochemical method can be used to measure glucose in plasma? A. Conductivity B. Potentiometry C. Anodic stripping voltammetry D. Amperometry

Correct Answer: D 23. D In some critical care analyzers, amperometric measurement of glucose is used. The glucose oxidase is impregnated into the membrane covering the electrode. It reacts with glucose in the sample, forming H2O2. This diffuses across the membrane to the anode of the electrode, where it is oxidized to O2. The electrons produced are used to reduce oxygen at the cathode, completing the current path. At the anode (usually platinum), 2H2O2 → 4e- + 2O2 + 4H+. At the cathode (usually silver), O2 + 4H+ + 4e- → 2H2O. The net equation is 2H2O2 → O2 + 2H2O.

SITUATION: Results of an iron profile are: -serum Fe = 40 μg/dL -ferritin = 40μg/L (reference range 15-200) -TIBC = 400 μg/dL -transferrin = 300 mg/dL These results indicate: A. Error in calculation of TIBC B. Serum iron falls before ferritin in iron deficiency C. A defect in iron transport and not Fe deficiency D. Excess release of ferritin caused by injury

Correct Answer: D 24. D Serum ferritin levels fall before iron or TIBC in iron deficiency, and a low level of serum ferritin is diagnostic. However, low tissue levels of ferritin may be masked by increased release into the blood in liver disease, infection, and acute inflammation. Although this patient's serum ferritin is within reference limits, serum iron is low and percent saturation is only 10%. Note that the TIBC and transferrin results are both elevated and agree. TIBC can be estimated by multiplying the serum transferrin by 1.4. These results point to iron deficiency.

Which substance is used to generate the light signal in electrochemiluminescence? A. Acridinium B. Luminol C. Dioxetane phosphate D. Ruthenium

Correct Answer: D 25. D All of these substances are chemiluminescent. Dioxetane phosphate is excited by alkaline phosphatase. Acridinium and luminol are excited by hydrogen peroxide. In electrochemiluminesence, ruthenium is used to label antibody or antigen. Antigen-antibody complexes containing the ruthenium label are bound to paramagnetic particles via a strepavidin-biotin reaction. The paramagnetic particles are attracted to an electrode surface. The flowcell is washed with a solution containing tripropylamine (TPA) to remove unbound ruthenium label. At the electrode surface, the TPA is oxidized and the electrons excite the ruthenium, causing production of 620-nm light.

Which serum protein should be measured in a patient suspected of having Wilson's disease? A. Hemopexin B. Alpha-1 antitrypsin C. Haptoglobin D. Ceruloplasmin

Correct Answer: D 25. D α-1 antitrypsin, haptoglobin, and ceruloplasmin are acute phase proteins and will be increased in inflammatory diseases. Ceruloplasmin is an α-2 globulin that binds the majority of the serum copper. Levels are low in almost all patients with Wilson's disease, an autosomal recessive disorder caused by accumulation of copper in liver, brain, kidney, and other tissues. Low ceruloplasmin may occur in patients with nephrosis, malnutrition, and hepatobiliary disease. Therefore, the diagnosis of Wilson's disease is made by demonstrating decreased plasma ceruloplasmin, increased urinary copper, and the presence of Kayser-Fleischer rings (brown deposits at the edge of the cornea).

Which of the following disorders is associated with lactate acidosis? A. Diarrhea B. Renal tubular acidosis C. Hypoaldosteronism D. Alcoholism

Correct Answer: D 26. D Lactate acidosis often results from hypoxia, which causes a deficit of nicotinamide adenine dinucleotide, the oxidized form (NAD+). This promotes the reduction of pyruvate to lactate, regenerating NAD+ needed for glycolysis. In alcoholic acidosis, oxidation of ethanol to acetaldehyde consumes the NAD+. In diabetes, lactate acidosis can result from depletion of Krebs cycle intermediates. Diarrhea and renal tubular acidosis result in metabolic acidosis via bicarbonate loss. Hypoaldosteronism causes metabolic acidosis via hydrogen and potassium ion retention.

Which test is the most specific for establishing a diagnosis of Cushing's disease (pituitary Cushing's)? A. Low-dose dexamethasone suppression B. High-dose dexamethasone suppression C. Twenty-four-hour urinary free cortisol D. Petrosal sinus sampling following corticotropin-releasing hormone stimulation

Correct Answer: D 29. D Although dexamethasone suppression tests have a high sensitivity, some patients without Cushing's syndrome have indeterminate results (e.g., values between 5 and 10 μg/dL) or abnormal results owing to medications or other conditions. When corticotropin-releasing hormone is given intravenously, patients with Cushing's disease have an exaggerated ACTH response. Samples are drawn from the sinuses draining the pituitary gland and from the peripheral blood. In patients with pituitary tumors, the ACTH will be several times higher in the sinus samples than in the peripheral blood samples.

In absorption spectrophotometry: A. Absorbance is directly proportional to transmittance B. Percent transmittance is directly proportional to concentration C. Percent transmittance is directly proportional to the light path length D. Absorbance is directly proportional to concentration

Correct Answer: D 3. D Beer's law states that A = a × b × c, where a is the absorptivity coefficient (a constant), b is the path length, and c is concentration. Absorbance is directly proportional to both b and c. Doubling the path length results in incident light contacting twice the number of molecules in solution. This causes absorbance to double, the same effect as doubling the concentration of molecules.

Select the hormone which when elevated is associated with galactorrhea, pituitary adenoma, and amenorrhea. A. E2 B. Progesterone C. Follicle-stimulating hormone (FSH) D. Prolactin

Correct Answer: D 3. D Serum prolactin may be increased from hypothalamic dysfunction or pituitary adenoma. When levels are greater than five times the URL, a pituitary tumor is suspected. Prolactin is measured by enzyme immunoassay (EIA).

Which of the following is characteristic of type 1 diabetes mellitus? A. Requires an oral glucose tolerance test for diagnosis B. Is the most common form of diabetes mellitus C. Usually occurs after age 40 D. Requires insulin replacement to prevent ketosis

Correct Answer: D 3. D Type 1, or juvenile, diabetes is also called insulin- dependent diabetes because patients must be given insulin to prevent ketosis. Type 1 accounts for only about 10%-20% of cases of diabetes mellitus, and is usually diagnosed by a fasting plasma glucose. Two consecutive results ≥126 mg/dL is diagnostic. Approximately 95% of patients produce autoantibodies against the beta cells of the pancreatic islets. Other autoantibodies may be produced against insulin, glutamate decarboxylase, and tyrosine phosphorylase IA2. There is genetic association between type 1 diabetes and human leukocyte antigens (HLA) DR3 and DR4.

SITUATION: A urine sample is received in the laboratory with the appropriate custody control form, and a request for drug of abuse screening. Which test result would be cause for rejecting the sample? A. Temperature after collection 95°F B. pH 5.0 C. Specific gravity 1.005 D. Creatinine 5 mg/dL

Correct Answer: D 30. D Approximately 5 per 1,000 urine samples received for DAU testing have been adulterated by either dilution, substitution, or addition of substances such as glutaraldehyde that interfere with testing. The majority of these situations can be detected by determining temperature (90°F-100°F) pH (4.5-8.0), specific gravity (1.003-1.019), and creatinine (≥20 mg/dL). All of the values listed are within the limits of an acceptable sample with the exception of creatinine. Dry reagent strips are available that test for pH, specific gravity, creatinine, nitrite, peroxide, pyridinium, and glutaraldehyde.

What effect does hematocrit have on POCT tests for whole-blood glucose monitoring? A. Low hematocrit decreases glucose readings on all devices B. High hematocrit raises glucose readings on all devices C. The effect is variable and dependent on the enzyme/coenzyme system D. Low hematocrit raises readings and high hematocrit lowers readings unless corrected

Correct Answer: D 31. D Hematocrit affects POCT glucose measurements. High hematocrit lowers the glucose because RBC glucose concentration is lower than plasma concentration. Other factors include binding of oxygen to hemoglobin and the slower diffusion of glucose onto the solid phase—both of which occur when the hematocrit is high. Bias due to an abnormal hematocrit can be avoided by simultaneously measuring the conductivity of the sample. The hematocrit is calculated and used to mathematically correct the glucose measurement.

All of the following are requirements for a QC material except: A. Long-term stability B. The matrix is similar to the specimens being tested C. The concentration of analytes reflects the clinical range D. Analyte concentration must be independent of the method of assay

Correct Answer: D 31. D Quality control materials are stable, made of the same components as the specimen, cover the dynamic linear range of the assay, and can be used for multiple analytes. The target mean for QC samples is determined from replicate assays by the user's method, not the "true" concentration of the analyte. Out-of-control results are linked to analytic performance rather than to the inherent accuracy of the method.

Which substance has the longest detection time? A. Amphetamines B. Cocaine C. Benzodiazepines D. Marijuana

Correct Answer: D 31. D Some drugs have a long half-life, and can be detected for longer periods after use, but the detection window also depends on other variables such as dosage, frequency of use, and method sensitivity. Marijuana is stored in fatty tissue and is metabolized slowly. In persons who use marijuana several times per week, cannabinoids can be detected several weeks after last use. For chronic daily users, this extends to months after discontinuation. Other drugs with detection windows of a week or more include long-acting barbiturates, LSD, anabolic steroids, and phencyclidine (PCP).

All of the following are required when measuring magnesium by atomic absorption spectrophotometry except: A. A hollow cathode lamp with a magnesium cathode B. A chopper to prevent optical interference from magnesium emission C. A monochromator to isolate the magnesium emission line at 285 nm D. A 285-nm reference beam to correct for background absorption

Correct Answer: D 32. D Atomic absorption requires a lamp with a cathode made from the metal to be assayed. The lamp emits the line spectrum of the metal, providing the wavelength that the atoms can absorb. The chopper pulses the source light, allowing it to be discriminated from light emitted by excited atoms. A monochromator eliminates light emitted by the ideal gas in the lamp. Deuterium (wide bandpass light) or Zeeman correction (splitting the incident light into side bands by a magnetic field) may be used to correct for background absorption.

Which assay using 24-hour urine is considered the best single screening test for pheochromocytoma? A. Total urinary catecholamines B. VMA C. Homovanillic acid (HVA) D. Metanephrines

Correct Answer: D 32. D Catecholamines are metabolized to metanephrines and VMA. Urinary catecholamines are increased by exercise and dietary ingestion. Measurement of 24-hour urinary metanephrine is about 95% sensitive for pheochromocytoma, and is the best single test. Specificity and sensitivity for detecting pheochromocytoma approach 100% when both VMA and metanephrines are measured.

Which of the following is classified as a mucopolysaccharide storage disease? A. Pompe's disease B. von Gierke disease C. Hers' disease D. Hurler's syndrome

Correct Answer: D 32. D Hurler's syndrome is an autosomal recessive disease resulting from a deficiency of iduronidase. Glycosaminoglycans (mucopolysaccharides) accumulate in the lysosomes. Multiple organ failure and mental retardation occur, resulting in early mortality. Excess dermatan and heparin sulfate are excreted in urine. Other mucopolysaccharidoses (MPS storage diseases) are Hunter's, Scheie's, Sanfilippo's, and Morquio's syndromes.

When measuring calcium by atomic absorption spectrophotometry, which is required? A. An organic extraction reagent to deconjugate calcium from protein B. An internal standard C. A magnesium chelator D. Lanthanum oxide to chelate phosphates

Correct Answer: D 33. D An acidic diluent such as hydrochloric acid (HCl) will displace calcium bound to albumin. However, calcium forms a thermostable bond with phosphate that causes chemical interference in atomic absorption. Lanthanum displaces calcium, forming lanthanum phosphate, and eliminates interference from phosphates. Unlike in some colorimetric methods for calcium (e.g., o-cresolphthalein complexone), magnesium does not interfere because it does not absorb the 422.7 nm emission line from the calcium-hollow cathode lamp.

After installing a new analyzer and reviewing the results of patients for 1 month, the lead technologist notices a greater frequency of patients with abnormally high triglyceride results. Analysis of all chemistry profiles run the next day indicated that triglyceride results are abnormal whenever the test is run immediately after any sample that is measured for lipase. These observations point to which type of error? A. Specificity of the triglyceride reagents B. Precision in pipetting of lipemic samples C. Bias caused by sequence of analysis D. Reagent carryover

Correct Answer: D 33. D Carryover errors are usually attributed to interference caused by a sample with a very high concentration of analyte preceding a normal sample. However, reagent carryover may also occur on automated systems that use common reagent delivery lines or reusable cuvettes. In the case of lipase methods, triglycerides used in the reagent may coat the reagent lines or cuvettes interfering with the triglyceride measurements that directly follow.

A patient has a plasma myoglobin of 10 μg/L at admission. Three hours later, the myoglobin is 14 μg/L and the troponin I is 0.02 μg/L (reference range 0-0.03 μg/L). These results are consistent with which condition? A. Skeletal muscle injury B. Acute myocardial infarction C. Unstable angina D. No evidence of myocardial or skeletal muscle injury

Correct Answer: D 33. D This person displays very low plasma myoglobin (reference range for females is approximately 17-75 μg/L). The TnI result is also within normal limits. These results are consistent with baseline levels and no evidence of cardiac or skeletal muscle injury. TnI cutpoints for diagnosis of AMI are dependent upon the method, and may be higher than the upper limit of normal. Troponin results above the upper reference limit but below the cutpoint for AMI indicate myocardial injury and increased risk for AMI.

Which of the following abnormal laboratory results is found in von Gierke's disease? A. Hyperglycemia B. Increased glucose response to epinephrine administration C. Metabolic alkalosis D. Hyperlipidemia

Correct Answer: D 34. D Von Gierke's disease (type 1 glycogen storage disease) results from a deficiency of glucose-6-phosphatase. This blocks the hydrolysis of glucose-6-PO4 to glucose and Pi, preventing degradation of glycogen to glucose. The disease is associated with increased triglyceride levels because fats are mobilized for energy and lactate acidosis caused by increased glycolysis. A presumptive diagnosis is made when intravenous galactose administration fails to increase serum glucose, and can be confirmed by demonstrating glucose-6-phosphatase deficiency or decreased glucose production in response to epinephrine.

Which of the following tests is least essential to the operation of an emergency department at a general hospital? A. Carboxyhemoglobin B. Osmolality C. Salicylate D. Lead

Correct Answer: D 36. D The vast majority of acute toxicology situations seen in the emergency department (ED) involve poisoning with alcohol, acetaminophen, salicylate, abuse substances, or carbon monoxide. Emergency departments should offer a minimum of these tests. In the absence of specific tests for abuse substances or a comprehensive drug screen, the serum osmolality measured by freezing point depression is a sensitive surrogate test for drug and alcohol overdose. In the ED environment, a difference between measured and calculated osmolality greater than 10 mOsm/Kg almost always indicates drug or alcohol poisoning. Toxicity from lead poisoning and most other trace metals is usually a chronic condition that does not often require immediate access to laboratory testing.

Which test becomes abnormal in the earliest stage of the acute coronary syndrome? A. Myosin light chain 1 B. CK-MB isoforms C. Myoglobin D. High-sensitivity C-reactive protein

Correct Answer: D 37. D The acute coronary syndrome (ACS) refers to the evolution of coronary artery events that lead up to AMI. Coronary artery disease (CAD) begins with formation of a plaque comprised of lipid from dead endothelium that proliferates into the artery lumen. The plaque becomes disrupted and the vessel wall inflamed in the asymptomatic stage of CAD. If platelet activation occurs and results in thrombosis, blood flow becomes significantly reduced, resulting in angina. This signals the transition to more advanced disease in which ischemia to heart muscle occurs and eventually to AMI. Myoglobin and CK-MB isoforms are not increased until the end stage of ACS. High-sensitivity C-reactive protein (hs-CRP) is an ultrasensitive CRP assay that accurately measures CRP below 1 mg/L. CRP is an acute-phase protein increased in inflammation. Levels of CRP between 3.2-10 mg/L signal low-grade inflammation, which occurs in the asymptomatic phase of ACS. Such inflammation occurs when coronary artery plaques become disrupted, and therefore, persons with CAD who have a mildly increased CRP are at high risk of disease progression. Glycogen phosphorylase-BB and albumin cobalt binding are increased by ischemia. Troponins are increased when there is unstable angina and ischemic injury, and indicate an increased risk for AMI.

Which of the following statements applies to both measurement of VMA and metanephrines in urine? A. Both can be oxidized to vanillin and measured at 360 nm without interference from dietary compounds B. Both can be measured immunochemically after hydrolysis and derivatization C. Both require acid hydrolysis prior to measurement D. Both can be measured by specific HPLC and MS assays

Correct Answer: D 37. D VMA and metanephrines can both be measured as vanillin after oxidation with periodate. However, these methods are affected by dietary sources of vanillin; coffee, chocolate, bananas, and vanilla must be excluded from the diet. Metanephrines, VMA, and HVA are most often measured by HPLC-EDC.

Which of the following conditions will cause an increased anion gap? A. Diarrhea B. Hypoaldosteronism C. Hyperkalemia D. Renal failure

Correct Answer: D 38. D An increased anion gap occurs when there is production or retention of anions other than bicarbonate or chloride (measured anions). For example, in renal failure, retention of phosphates and sulfates (as sodium salts) increases the anion gap. Other common causes of metabolic acidosis with an increased anion gap are diabetic ketoacidosis and lactate acidosis. The anion gap may also be increased in the absence of an acid-base disorder. Common causes include hypocalcemia, drug overdose, and laboratory error when measuring electrolytes.

Which test is the most sensitive in detecting early monoclonal gammopathies? A. High-resolution serum protein electrophoresis B. Urinary electrophoresis for monoclonal light chains C. Capillary electrophoresis of serum and urine D. Serum-free light chain immunoassay

Correct Answer: D 38. D Immunonephelometric free light chain assays can detect monoclonal protein production before the mass is sufficient to cause a monoclonal spike on protein electrophoresis or capillary electrophoresis, but will be positive only in cases where monoclonal light chain production occurs. Therefore, measurement of free light chains is recommended along with protein electrophoresis when testing for myeloma. Free light chains are normally present in serum because L chains are made at a faster rate than H chains. However, in cases where free L chains are the result of monoclonal plasma cell proliferation, the kappa:lambda ratio will be abnormal in addition to one of the L chain types being elevated.

Which test is the most useful way to evaluate the response to treatment for multiple myeloma? A. Measure of total immunoglobulin B. Measurement of 24-hour urinary light chain concentration (Bence-Jones protein) C. Capillary electrophoresis of M-protein recurrence D. Measurement of serum-free light chains

Correct Answer: D 39. D Unlike electrophoresis methods, serum free light chain assays are quantitative and an increase in free light chain production with an abnormal kappa:lambda ratio occurs earliest in recurrence of myeloma. Light chains have a shorter plasma half life than intact Ig and therefore, the reduction in free light chain concentration is an earlier indicator of treatment effect than measurement of intact Ig. It is not subject to the variation in 24-hour urinary light chain measurement caused by sample collection error and abnormal renal function. A 50% or more reduction in serum free light chain concentration is considered a partial response to treatment. A full response is indicated by reduction to within normal limits and a return of the kappa:lambda ratio to normal. An abnormal free light chain ratio has a 3.5-fold higher risk of progression to myeloma in persons with monoclonal gammopathy of undetermined significance (MGUS).

Which of the following cardiac markers derived from neutrophils predicts an increased risk for myocardial infarction? A. Phospholipase A2 (PLA2) B. Glycogen phosphorylase BB (GPBB) C. Soluble CD40 ligand (sCD40l) D. Myeloperoxidase (MPO)

Correct Answer: D 40. D All of the answer choices are markers for acute coronary syndrome and increased risk of AMI. MPO is released from neutrophils and is thought to destabilize the arterial plaque by oxidizing both LDL and HDL and reducing nitric oxide levels in the coronary arteries. Levels in the upper third quartile predict an increased risk of a coronary event even when troponin is normal. GPBB is released from myocytes early in an ischemic episode and becomes abnormal about 2 hours after an AMI. sCD40l is released from activated platelets and indicates an unstable plaque. PLA2 is produced by the arterial wall. It removes a fatty acid from phospholipids and increases the amount of oxidized LDL, leading to foam cell formation. Like hs-CRP, it is a marker for an inflamed plaque.

Which of the following is more commonly associated with a nonmalignant form of monoclonal gammopathy (MGUS)? A. Bone marrow plasma cells comprise 20% of nucleated cells B. Monoclonal protein (M-protein) concentration is 3.5 g/dL C. M-protein is IgG D. Age greater than 60 at the time of monoclonal protein discovery

Correct Answer: D 40. D MGUS is the most common cause of monoclonal gammopathy. About 3% of the U.S. population at age 50 and 5% at age 70 have MGUS. The absence of bone lesions and organ damage, plasma cells below 10% of nucleated bone marrow cells, and M-protein below 3.0 g/dL are characteristic of MGUS as opposed to myeloma or other malignant gammopathy. About 50% of persons with MGUS have IgH gene translocations or chromosome 13 deletion associated with multiple myeloma. The risk of transformation of MGUS to malignant disease is about 1% per year.

Which condition is most often associated with a high serum iron level? A. Nephrosis B. Chronic infection or inflammation C. Polycythemia vera D. Noniron deficiency anemias

Correct Answer: D 41. D Anemia associated with chronic infection causes a low serum iron, but unlike iron deficiency, causes a low (or normal) TIBC and does not cause low ferritin. Noniron deficiency anemias such as pernicious anemia and sideroblastic anemia produce high serum iron and low TIBC. Nephrosis causes iron loss by the kidneys. Polycythemia is associated with increased iron within the RBCs and depletion of iron stores.

SITUATION: A blood sample in a red-stoppered tube is delivered to the laboratory for electrolytes, calcium, and phosphorus. The tube is approximately half full and is accompanied by a purple-stoppered tube for a complete blood count that is approximately three-quarters full. The chemistry results are as follows: -Na: 135 mmol/L -K: 11.2 mmol/L -Cl: 103 mmol/L -HCO3: 14 mmol/L -Ca: 2.6 mg/dL -InP: 3.8 mg/dL What is the most likely explanation of these serum calcium results? A. Severe hemolysis during sample collection B. Laboratory error in the calcium measurement C. The wrong order of draw was used for vacuum tube collection D. Some anticoagulated blood was added to the red-stoppered tube

Correct Answer: D 41. D The potassium and the calcium results are above and below physiological limit values, respectively. Although hemolysis could explain the high potassium, hemolysis does not cause a significant change in serum calcium. The wrong order of draw could result in the falsely low calcium value but would not be sufficient to cause a result that is incompatible with life (and does not explain a grossly elevated potassium). The results and the condition of the tubes indicate that blood from a full tube collected in K3 EDTA was added to the clot tube, chelating the calcium and increasing the potassium.

Which condition can result in acidosis? A. Cystic fibrosis B. Vomiting C. Hyperaldosteronism D. Excessive O2 therapy

Correct Answer: D 41. D When O2 saturation of venous blood is greatly elevated, Hgb cannot release O2. Oxyhemoglobin cannot bind CO2 or hydrogen ions and acidosis results. Pure O2 may cause neurological damage, leading to convulsion and blindness, especially in infants. It can induce respiratory failure by causing pulmonary hemorrhage, edema, and hyalinization. The other three conditions cause alkalosis. Vomiting and cystic fibrosis cause loss of chloride, resulting in hypovolemia and intestinal bicarbonate absorption. Hyperaldosteronism causes hypokalemia; this results in increased renal H+ excretion and a shift of H+ into cells in exchange for K+

Which explanation is the best interpretation of the following BUN bias plot? See Harr pg 233 A. The new method consistently overestimates the BUN by a constant concentration B. The new method is greater than the reference method but not by a statistically significant margin C. The new method is lower than the reference method by 5 mg/dL D. The new method is lower than the reference and the magnitude is concentration dependent

Correct Answer: D 42. D A bias plot compares the bias (candidate method minus reference method) to the result of the reference method. Ideally, points should be scattered equally on both sides of the zero line. When the majority of points is below the zero line, the candidate method is negatively biased (lower than the reference). In this case, the difference between the methods increases in proportion to the BUN concentration. This type of plot occurs when the slope of the linear regression line is low.

The results shown in the table above are obtained from three consecutive serum samples using an automated random access analyzer that samples directly from a bar-coded tube. Calibration and QC performed at the start of the shift are within the acceptable range, and no error codes are reported by the analyzer for any tests on the three samples. Upon results verification, what is the most appropriate course of action? A. Report the results and proceed with other tests since no analytical problems are noted B. Repeat the controls before continuing with further testing, but report the results C. Check sample identification prior to reporting D. Do not report BUN results for these

Correct Answer: D 44. D BUN is elevated 5- to 10-fold for three consecutive patients in the absence of any other laboratory evidence of renal disease. The glucose results show conclusively that the samples are not from the same patient. Therefore, the BUN results must be caused by a systematic error, and should not be reported. Further testing for BUN should cease until the analytical components of the BUN assay are completely evaluated and the cause of these results identified and corrected. This is demonstrated by successful recalibration and performance of controls within acceptable limits. Following this, the BUN assay should be repeated on the three samples along with all other specimens with a spurious BUN result that have occurred since the start of the shift.

Which statement regarding the diagnosis of iron deficiency is correct? A. Serum iron levels are always higher at night than during the day B. Serum iron levels begin to fall before the body stores become depleted C. A normal level of serum ferritin rules out iron deficiency D. A low serum ferritin is diagnostic of iron deficiency

Correct Answer: D 44. D Serum iron levels are falsely elevated by hemolysis and subject to diurnal variation. Levels are highest in the morning and lowest at night, but this pattern is reversed in persons who work at night. A low ferritin is specific for iron deficiency. However, only about 1% of ferritin is in the vascular system. Any disease that increases ferritin release may mask iron deficiency.

Which of the following tests is consistently abnormal in osteoporosis? A. High urinary calcium B. High serum Pi C. Low serum calcium D. High urine or serum N-telopeptide of type 1 collagen

Correct Answer: D 45. D Commonly used markers for other bone diseases such as serum or urinary calcium, inorganic phosphorus, total alkaline phosphatase (ALP), and vitamin D are neither sensitive nor specific for osteoporosis. Calcium and phosphorus are usually within normal limits. Although estrogen deficiency reduces formation of 1,25 hydroxyvitamin D (1,25 hydroxycholecalciferol), promoting postmenopausal osteoporosis, the 1,25 hydroxyvitamin D is low in only 30%-35% of cases, and low levels may be caused by other bone disorders. Serum markers for osteoporosis include both N-telopeptide of type 1 collagen (NTx) and C-telopeptide of type 1 collagen (CTx). These can be used to follow treatment with resorption antagonists (bisphosphonates) because they decrease significantly when therapy is successful.

A quantitative sandwich enzyme immunoassay for intact serum hCG was performed on week 4 and the result was 40,000 mIU/mL (reference range 10,000-80,000 mIU/mL). The physician suspected a molar pregnancy and requested that the laboratory repeat the test checking for the hook effect. Which process would identify this problem? A. Obtain a new plasma specimen and heat inactivate before testing B. Obtain a urine specimen and perform the assay C. Perform a qualitative pregnancy test D. Perform a serial dilution of the sample and repeat the test

Correct Answer: D 47. D The hook effect is the result of excessive antigen concentration and results in a dose response (calibration) curve that reverses direction at very high antigen concentrations. It occurs in two-site double antibody sandwich assays when both the capture antibody and the enzyme-conjugated antibody are incubated with the antigen at the same time. The excess antigen saturates both antibodies preventing formation of a double antibody sandwich. The hook effect can cause results to be sufficiently low to cause misdiagnosis. It can be detected by diluting the sample (antigen) in which case the assay result will be greater than in the undiluted sample. An alternative solution is to perform the test using a competitive binding assay or a sandwich assay in which the enzyme-labeled antibody is not added until after separation of free and bound antigen.

What role does vitamin D measurement play in the management of osteoporosis? A. Vitamin D deficiency must be demonstrated to establish the diagnosis B. Vitamin D is consistently elevated in osteoporosis C. A normal vitamin D level rules out osteoporosis D. Vitamin D deficiency is a risk factor for developing osteoporosis

Correct Answer: D 48. D Vitamin D assay is not used to diagnose osteoporosis. Vitamin D deficiency is a cause of secondary osteoporosis, and together with low PTH, calcium, and estrogen are important risk factors. If one or more of these is abnormal, then bone resorption or remodeling may be abnormal, predisposing one to osteoporosis. Deficiency of vitamin D also causes rickets (called osteomalacia in adults), a condition in which bones become soft owing to reduced deposition of hydroxyapatite.

Which of the following results falls within the diagnostic criteria for diabetes mellitus? A. Fasting plasma glucose of 120 mg/dL B. Two-hour postprandial plasma glucose of 160 mg/dL C. Two-hour plasma glucose of 180 mg/dL following a 75 g oral glucose challenge D. Random plasma glucose of 250 mg/dL and presence of symptoms

Correct Answer: D 5. D The American Diabetes Association recommends the following criteria for diagnosing diabetes mellitus: fasting glucose ≥ 126 mg/dL, casual (random) glucose ≥ 200 mg/dL in the presence of symptoms (polyuria, increased thirst, weight loss), glucose ≥ 200 mg/dL at 2 hours after an oral dose of 75 g of glucose, and hemoglobin A1c ≥ 6.5%. A diagnosis of diabetes mellitus is indicated if any one or combination of these four criteria is met on more than a single testing event. The fasting plasma glucose test requires at least 8 hours with no food or drink except water. The 2-hour postloading test should be conducted according to the oral glucose tolerance guidelines currently recommended by the World Health Organization.

In which case might a very low plasma TSH result not correlate with thyroid status? A. Euthyroid sick syndrome B. Congenital hypothyroidism C. When TBG is elevated D. After high-dose corticosteroid treatment

Correct Answer: D 50. D In persons with severe chronic diseases or who have hCG-secreting tumors, TSH production may be suppressed. Some drugs, especially high doses of corticosteroids, will suppress TSH production. Low TSH levels not matching thyroid status can also be seen in patients who have recently been treated for hyperthyroidism because there is a delay in the pituitary response. High-sensitivity TSH assays that can measure as little as 0.01 mIU/L and free T4 and T3 can help differentiate these conditions from clinical hyperthyroidism. If the TSH is below .03 mIU/L and the free hormone levels are increased, this points to hyperthyroidism. Lab values in euthyroid sick syndrome may mimic mild hypothyroidism. In euthyroid sick syndrome, thyroid function will be normal, but TSH may be slightly increased owing to lower levels of free T3. In euthyroid sick syndrome, the rT3 will be increased.

Serial TnI assays are ordered on a patient at admission, 3 hours, and 6 hours afterwards. The samples were collected in heparinized plasma separator tubes. Following are the results (reference range 0-0.03 μg/L) -Admission = 0.03 μg/L -3 hours = 0.07 μg/L 6 hours = 0.02 μg/L These results indicate: A. A positive test for acute myocardial infarction B. Unstable angina C. Cardiac injury of severity less than myocardial infarction D. Random error with the 3-hour sample

Correct Answer: D 50. D Troponin assays produce very little fluorescence or chemiluminescence when plasma levels are within the reference range and near the minimum detection limit of the assay. Fibrin, tube additives, and heterophile antibodies have been known to cause spurious elevations, and this result should be treated as a random error because the result before and after are both normal.

When comparing the laboratory's monthly mean to its peer group to determine if bias is present, what statistic is most appropriate? A. F test B. Linear regression analysis C. Correlation coefficient D. Standard deviation index

Correct Answer: D 51. D The standard deviation index (SDI) compares the lab's mean to the peer group's mean in terms of standard deviations instead of concentration. This normalizes the value so that it is independent of mean, and allows performance comparisons for any analyte. The SDI equals the lab's mean minus the peer group's mean divided by the peer group's standard deviation. It has a similar probability distribution to a t test and a value greater than 2.0 is considered significant.

Which of the following conditions is associated with a low serum magnesium? A. Addison's disease B. Hemolytic anemia C. Hyperparathyroidism D. Pancreatitis

Correct Answer: D 54. D Low magnesium can be caused by gastrointestinal loss, as occurs in diarrhea and pancreatitis (loss of Mg and Ca as soaps). Hyperparathyroidism causes increased release of both calcium and magnesium from bone. Addison's disease (adrenocorticosteroid deficiency) may be associated with increased magnesium accompanying hyperkalemia. Hemolytic anemia causes increased release of magnesium as well as potassium from damaged red blood cells (RBCs).

Which of the following statements regarding ALP is true? A. In normal adults, the primary tissue source is fast-twitch skeletal muscle B. Geriatric patients have a lower serum ALP than other adults C. Serum ALP levels are lower in children than in adults D. Pregnant women have a higher level of serum ALP than other adults

Correct Answer: D 56. D ALP is higher in children than in adults due to bone growth. Children and geriatric patients have higher serum ALP due to increased bone isoenzyme. Serum ALP levels are often two- or threefold higher than the URL in the third term of pregnancy. In nonpregnant normal adults, serum ALP is derived from liver and bone. Liver, bone, placental, renal, and intestinal isoenzymes of ALP can be separated by electrophoresis, and many other ALP isoenzymes have been identified by isofocusing.

Which route of administration is associated with 100% bioavailability? A. Sublingual B. Intramuscular C. Oral D. Intravenous

Correct Answer: D 6. D When a drug is administered intravenously, all the drug enters the bloodstream, and therefore, the bioavailable fraction is 1.0. All other routes of administration require absorption through cells, and this process reduces the bioavailable fraction. The bioavailable fraction for a drug given orally can be calculated by dividing the peak blood concentration after oral administration by the peak drug concentration after IV administration. A value of 0.7 or higher is desired for drugs given orally.

Which electrolyte is least likely to be elevated in renal failure? A. Potassium B. Magnesium C. Inorganic phosphorus D. Sodium

Correct Answer: D 60. D Reduced glomerular filtration coupled with decreased tubular secretion causes accumulation of potassium, magnesium, and inorganic phosphorus. Poor tubular reabsorption of sodium offsets reduced glomerular filtration. Unfiltered sodium draws both chloride and water, causing osmotic equilibration between filtrate, serum, and the tissues. In renal disease, serum sodium is often normal, although total body sodium is increased owing to fluid and salt retention.

Select the chemical that is used in most HPLC procedures to decrease solvent polarity. A. Hexane B. Nonane C. Chloroform D. Acetonitrile

Correct Answer: D 61. D All of the compounds mentioned have nonpolar properties. Because most HPLC is reverse phase (a polar solvent is used), hexane and nonane are too nonpolar. Acetonitrile is more polar and less toxic than chloroform and along with methanol is a common polarity modifier for HPLC.

Which condition is least likely to be associated with increased serum ALP? A. Osteomalacia B. Biliary obstruction C. Hyperparathyroidism and hyperthyroidism D. Osteoporosis

Correct Answer: D 62. D ALP is elevated in osteomalacia (rickets), bone cancer, and bone disease secondary to hyperthyroidism and hyperparathyroidism, but total ALP it is high in less than 30% of osteoporosis patients. Pancreatic disease associated with biliary obstruction, such as cancer at the head of the pancreas, is associated with elevated ALP.

Which statement regarding the measurement of bilirubin by the Jendrassik-Grof method is correct? A. The same diluent is used for both total and direct assays to minimize differences in reactivity B. Positive interference by Hgb is prevented by the addition of HCl after the diazo reaction C. The color of the azobilirubin product is intensified by the addition of ascorbic acid D. Fehling's reagent is added after the diazo reaction to reduce optical interference by hemoglobin

Correct Answer: D 67. D The Jendrassik-Grof method uses HCl as the diluent for the measurement of direct bilirubin because unconjugated bilirubin is poorly soluble at low pH. Total bilirubin is measured using an acetate buffer with caffeine added to increase the solubility of the unconjugated bilirubin. After addition of diazotized sulfanilic acid and incubatiion, the diazo group is reduced by ascorbic acid, and Fehling's reagent is added to alkalinize the diluent. At an alkaline pH the product changes from pink to blue, shifting the absorbance maximum to 600 nm where Hgb does not contribute significantly to absorbance.

Which type of monochromator produces the purest monochromatic light in the UV range? A. A diffraction grating and a fixed exit slit B. A sharp cutoff filter and a variable exit slit C. Interference filters and a variable exit slit D. A prism and a variable exit slit

Correct Answer: D 7. D Diffraction gratings and prisms both produce a continuous range of wavelengths. A diffraction grating produces a uniform separation of wavelengths. A prism produces much better separation of high-frequency light because refraction is greater for higher-energy wavelengths. Instruments using a prism and a variable exit slit can produce UV light of a very narrow bandpass. The adjustable slit is required in order to allow sufficient light to reach the detector to set 100%T.

The most commonly employed method of assay for plasma or serum lipase is based on: A. Hydrolysis of olive oil B. Rate turbidimetry C. Immunoassay D. Peroxidase coupling

Correct Answer: D 77. D Although all of the methods cited are available, the most commonly used method for lipase assay is based upon the hydrolysis of a synthetic diglyceride substrate yielding 2-monoglyceride. This is hydrolyzed and forms glycerol, which is phosphorylated and forms glycerol-3-phosphate. This is oxidized by glycerophosphate oxidase, yielding hydrogen peroxide. 1,2 diglyceride + H2O Lipase--> 2-monoglyceride + fatty acid 2- monoglyceride + H2O Monoglyceride esterase --> glycerol + fatty acid glycerol + ATP Glycerol kinase--> glycerol-3-phosphate + ADP glycerol-3-PO4 + O2 Glycerophosphate oxidase--> dihydroxyacetone phosphate + H2O2 H2O2 + 4-aminoantipyrene + TOOS Peroxidase--> quinoneimine dye + H2O

Which of the following conditions is most commonly associated with an elevated level of total protein? A. Glomerular disease B. Starvation C. Liver failure D. Malignancy

Correct Answer: D 8. D Malignant disease is usually associated with increased immunoglobulin and acute-phase protein production. However, nutrients required for protein synthesis are consumed, causing reduced hepatic albumin production. Glomerular damage causes albumin and other low molecular weight proteins to be lost through the kidneys. Liver failure and starvation result in decreased protein synthesis.

The increase in the level of serum enzymes used to detect cholestatic liver disease is caused mainly by: A. Enzyme release from dead cells B. Leakage from cells with altered membrane permeability C. Decreased perfusion of the tissue D. Increased production and secretion by cells

Correct Answer: D 8. D The amount of enzyme in the serum can be increased by necrosis, altered permeability, secretion, or synthesis. It is also dependent upon tissue perfusion, enzyme half-life, molecular size, and location of the enzyme within the cell. Most enzymes are liberated by necrosis, but a few, such as ALP and γ-glutamyltransferase, are produced and secreted at a greater rate in obstructive liver disease.

When calibrating a semiautomatic pipet that has a fixed delivery of 10.0 μL using a gravimetric method, what should be the average weight of deionized water transferred? A. 10.0 μg B. 100.0 μg C. 1.0 mg D. 10.0 mg

Correct Answer: D 80. D Gravimetric and spectrophotometric analysis are the two methods used to verify pipet volume accuracy and precision. Since spectrophotometric analysis involves dilution, gravimetric analysis is associated with greater certainty. At 20°C, the density of pure water is 0.99821 g/mL. Therefore, each microliter weighs almost exactly 1.0 mg.

Which statement regarding gestational diabetes mellitus (GDM) is correct? A. Is diagnosed using the same oral glucose tolerance criteria as in nonpregnancy B. Converts to diabetes mellitus after pregnancy in 60%-75% of cases C. Presents no increased health risk to the fetus D. Is defined as glucose intolerance originating during pregnancy

Correct Answer: D 9. D Control of GDM reduces perinatal complications such as respiratory distress syndrome, high birth weight, and neonatal jaundice. Women at risk are usually screened between 24 and 28 weeks' gestation. The screening test can be performed nonfasting and consists of an oral 50-g glucose challenge followed by serum or plasma glucose measurement at 1 hour. A result ≥ 140 mg/dL is followed by a 2-hour or 3-hour oral glucose tolerance test to confirm gestational diabetes. For the 3-hour test, a 100-g dose of glucose is used and at least two of the following cutoffs must be exceeded: fasting, ≥ 95 mg/dL or higher; 1 hour, ≥ 180 mg/dL or higher; 2 hour ≥ 155 mg/dL or higher; 3 hour, ≥ 140 mg/dL or higher. The same cutpoints are used for the 2-hour test except that a 75-g dose is used. GDM converts to diabetes mellitus within 10 years in 30%-40% of cases. ADA recommends testing persons with GDM for diabetes 6-12 weeks after delivery.

Which of the following enzymes is considered most tissue specific? A. Creatine kinase (CK) B. Amylase C. Alkaline phosphatase (ALP) D. Alcohol dehydrogenase (ADH)

Correct Answer: D 9. D No enzyme is truly tissue specific and diagnostic accuracy depends upon recognizing changes in plasma levels that characterize different diseases. This includes the mass or activity of enzyme released, its rise, peak, and return to normal, the isoenzyme(s) released, and the concomitant changes of other enzymes. Alanine aminotransferase and alcohol dehydrogenase are primarily increased in necrotic liver disease.

What condition is characterized by an elevation of total bilirubin primarily due to an increase in the conjugated bilirubin fraction? A. Hemolyticjaundice B. Neonatal jaundice C. Crigler-Najjar syndrome D. Obstructive jaundice

D. "Obstructive jaundice" is a term applied to conditions in which the common bile duct is obstructed because of gallstone formation, spasm, or neoplasm. Such an obstruction blocks the flow of bile from the gallbladder into the small intestine. This impedance of bile flow will result in a backflow of bile from the gallbladder into the sinusoids of the liver and ultimately into the peripheral circulation. Because the liver is not initially involved and the disorder is of posthepatic origin, the increased levels of bilirubin in the blood are caused by the backflow of conjugated bilirubin. If the disorder is allowed to progress, the continued backflow of bile will cause parenchymal cell destruction. Such cellular necrosis will result in a depression of the conjugating ability of the liver, and an elevation of unconjugated bilirubin levels in the blood will ensue.

Which of the following tests would most likely be included in a routine lipid profile? A. Total cholesterol, triglyceride, fatty acid, chylomicron B. Total cholesterol, triglyceride, HDL cholesterol, phospholipid C. Triglyceride, HDL cholesterol, LDL cholesterol, chylomicron D. Total cholesterol, triglyceride, HDL cholesterol, LDL cholesterol

D. A "routine" lipid profile would most likely consist of the measurement of total cholesterol, triglyceride, HDL cholesterol, and LDL cholesterol. These measurements are most easily adapted to today's multichannel chemistry analyzers. Both total cholesterol and triglyceride use enzymatic techniques to drive the reaction to completion. HDL cholesterol and LDL cholesterol are commonly requested tests to help determine patient risk for coronary heart disease. The HDL is separated from other lipoproteins using a precipitation technique, immunotechniques, and/or polymers and detergents. The nonprecipitation techniques are preferred because they can give better precision, be adapted to an automated chemistry analyzer, and be run without personnel intervention. LDL cholesterol may be calculated using the Friedewald equation, or it may be assayed directly using selective precipitation methods or direct homogeneous techniques.

Most atomic absorption spectrophotometers incorporate a beam chopper and a tuned amplifier. The purpose of these components is to avoid errors that would be caused by A. Variations in flame temperature B. Deterioration of the hollow-cathode lamp C. Stray light from the hollow-cathode lamp D. Measurement of light emitted by the analyte

D. A beam chopper is a device for interrupting a beam of light so that a pulsed beam is produced. In an atomic absorption spectrophotometer, if the light entering the flame from the hollowcathode lamp is pulsed, then the light leaving the flame will consist of unabsorbed pulsed light and impulsed light from the flame and from a small amount of emission by excited atoms of the analyte. The detector has an amplifier that is tuned to recognize and amplify only the pulsed signal. Thus errors caused by light from the flame and light emitted by the analyte are avoided. However, the beam chopper and tuned amplifier do not compensate for errors introduced by variations in flame temperature or deterioration of the hollow-cathode lamp. AAS may be used to measure such analytes as lead, zinc, copper, aluminum, magnesium, calcium, and lithium.

What are the principles of operation for a chloride analyzer that generates silver ions as part of its reaction mechanism? A. Potentiometry and amperometry B. Amperometry and polarography C. Coulometry and potentiometry D. Amperometry and coulometry

D. A chloride coulometer employs a coulometric system based on Faraday's law, which states that in an electrochemical system, the number of equivalent weights of a reactant oxidized or reduced is directly proportional to the quantity of electricity used in the reaction. The quantity of electricity is measured in coulombs. The coulomb is the unit of electrical quantity; 1 coulomb of electricity flowing per minute constitutes a current of 1 ampere. Thus, if the current is constant, the number of equivalent weights of reactant oxidized or reduced depends only on the duration of the current. In the chloride coulometer, the electrochemical reaction is the generation of Ag+ ions by the passage of a direct current across a pair of silver electrodes immersed in a conducting solution containing the sample to be assayed for chloride. As the Ag+ ions are generated, they are immediately removed from solution by combining with chloride to form insoluble silver chloride. When all the chloride is precipitated, further generation of Ag+ ions causes an increase in conductivity of the solution. Thus the instrument provides an electrometric titration, in which the titrant is Ag+ ions and the endpoint of the titration is indicated by the increase in conductivity of the solution. Amperometry is used to measure the increase in conductivity. The amperometric circuit includes a second pair of silver electrodes that are immersed in the solution. They are provided with a small, steady, and constant voltage. The appearance of free Ag+ ions in the solution generates a sharp increase in conductivity, which, in turn, causes a sudden rise in the current between the electrodes in the amperometric circuit. This increase in current activates a relay that stops the further generation of Ag+ ions and also stops an automatic timer placed in the circuit to measure the total duration of current in the coulometric circuit. Although this system is no longer used for routine analysis of serum, it is still employed for sweat chloride analysis.

Which of the following conditions is not associated with hyponatremia? A. Addison disease B. Diarrhea C. Diuretic therapy D. Gushing syndrome

D. A decreased serum sodium concentration, or hyponatremia, is associated with a variety of disorders, including (1) Addison disease, which involves the inadequate secretion of aldosterone, resulting in decreased reabsorption of sodium by the renal tubules; (2) diarrhea, which involves the impaired absorption from the gastrointestinal tract of dietary sodium and of sodium from the pancreatic juice, causing an excessive quantity of sodium to be excreted in the feces; (3) diuretic therapy, which causes a loss of water with concurrent loss of electrolytes, including sodium; and (4) renal tubular disease, which involves either the insufficient reabsorption of sodium in the tubules or a defect in the Na+-H+ tubular exchange mechanism. A diagnosis of Gushing syndrome is incorrect because the disorder is associated with hypernatremia.

The substance to be measured reacts with a specific macromolecule of limited binding capacity. Which of the following assays does not employ this principle? A. Chemiluminescence immunoassay B. Enzyme-multiplied immunoassay technique C. Fluorescent polarization immunoassay D. High-performance liquid chromatography

D. A number of immunoassay methods have been developed for the quantification of hormones, drugs, tumor markers, and other analytes that are present in small concentrations in the blood. The overall principle involved is the same. That is, the substance to be measured reacts with a specific macromolecule of limited binding capacity; frequently, this binder is an antibody. All these assays are similarly dependent on the closeness with which the unknown species and the standard react with the binder. These assays differ only in the specific reagents used. The ELISA system depends on enzyme-labeled antigen. Competitive protein binding (CPB) is a general term for any system that uses serum protein or tissue receptors for binding agents. Other methods based on antigen-antibody reactions, include such assays as fluorescent polarization immunoassay (FPI), enzyme-multiplied immunoassay technique, and chemiluminescence assays. Although hormones may be quantified using high-performance liquid chromatography, its principle is based on differential partitioning of compounds and not on antigen-antibody reactions as for the immunoassays.

To calibrate the pH electrode in a pH/ blood gas analyzer, it is necessary that A. The barometric pressure be known and used for adjustments B. Calibrating gases of known high and low concentrations be used C. The calibration be performed at room temperature D. Two buffer solutions of known pH be used

D. A pH/blood gas analyzer contains a pHsensitive glass electrode, a PCO^ electrode, and a PO2 electrode. The glass electrode is calibrated by comparison with two primary standard buffers of known pH. Because pH readings are temperature sensitive, the calibration must be carried out at a constant temperature of 37°C. pH readings are not appreciably sensitive to changes in barometric pressure. Note that if the PCO2 and PC>2 electrodes were also to be calibrated, then it would be essential to know the barometric pressure, because that affects the PCC>2 and PO2 calibrating gases.

Which of the following is false about a photomultiplier tube? A. Converts radiant energy (light) to electrical energy (current) B. Amplifies the current significantly C. Has a very rapid response time D. Is composed of an iron plate and a layer of selenium

D. A photomultiplier tube (PMT) has two functions: (1) It is a transducer that converts light to electricity; and (2) it amplifies the signal within the tube. Amplification can be as great as one million times. The emission of electrons by a light-sensitive surface—that is, the conversion of light energy to electrical energy—is virtually instantaneous. Hence, PMTs have a very rapid response time. An iron plate and a layer of selenium are partial descriptions of the

Which of the following disorders is not a form of hepatic jaundice? A. Cirrhosis B. Crigler-Najjar syndrome C. Hepatitis D. Neoplasm of common bile duct

D. Abnormal conditions characterized by jaundice may be classified according to their type of liver involvement. The three types of jaundice are prehepatic, hepatic, and posthepatic. Hepatic jaundice may be subdivided into two groups on the basis of the type of excessive bilirubin: conjugated bilirubin or unconjugated bilirubin. Gilbert syndrome and Dubin-Johnson syndrome are disorders in which the process of bilirubin transport is malfunctioning. Both Crigler-Najjar syndrome and neonatal jaundice, a physiological disorder, are due to a deficiency in the enzyme-conjugating system. Disorders such as viral hepatitis, toxic hepatitis, and cirrhosis cause damage and destruction of liver cells so that the ability of the liver to remove unconjugated bilirubin from the blood and to conjugate it with glucuronic acid becomes impaired. As these disorders progress, the level of unconjugated bilirubin in the blood rises. There is also an increase, although not as great as that of unconjugated bilirubin, in blood levels of conjugated bilirubin. The cause is a leakage of conjugated bilirubin from damaged parenchymal cells into the sinusoids. Neoplasm of the common bile duct is a form of posthepatic jaundice.

Anticoagulated whole blood is the preferred specimen in determining exposure to what compound? A. Methanol B. Mercury C. Acetaminophen D. Carbon monoxide

D. After absorption, mercury rapidly accumulates in many organs and in the central nervous system, with only minor quantities found in the blood. The excretion of mercury by the kidney generally forms the basis for measurement of exposure. The preferred specimen in screening for exposure to methanol or acetaminophen is serum. Whole blood is required for determining carbon monoxide exposure, because practically all the inhaled carbon monoxide is found in erythrocytes bound to hemoglobin. Following release of carbon monoxide from hemoglobin, the CO gas can be measured using gas chromatography. The percent carboxyhemoglobin saturation of whole blood can be determined by differential spectrophotometry or by using an automated, wavelength system such as a CO-oximeter.

In addition to performing hemoglobin electrophoresis, a solubility test may be performed to detect the presence of what hemoglobin? A. A, B. C C. F D. S

D. Although hemoglobin electrophoresis is the recommended method for hemoglobin identification, solubility testing may be warranted for large-scale screening for hemoglobin S. Solubility testing is possible because the solubility properties of most hemoglobins differ enough from those of hemoglobin S. In this method, sodium hydrosulfite acts as a reducing agent to deoxygenate hemoglobin. In the presence of hemoglobin S, the concentrated phosphate buffer test solution will become turbid because deoxygenated hemoglobin S is insoluble in the buffer solution. Hemoglobins Aj, C, D, and F, when present, will remain soluble in the phosphate buffer solution and show no visible signs of turbidity. Therefore, the detection of turbidity is associated with the presence of hemoglobin S.

The abnormal metabolism of several of the amino acids has been linked with disorders classified as inborn errors of metabolism. What technique is used to differentiate among several different amino acids? A. Electrophoresis B. Microbiological analysis C. Enzyme immunoassay D. Chromatography

D. Although microbiological analysis and chemical analysis may be employed to detect and quantify a specific amino acid, chromatographic analysis is preferred as a screening technique for amino acid abnormalities or when differentiation among several amino acids is necessary. Thin-layer chromatography, either one- or two-dimensional, is being used in conjunction with a mixture of ninhydrin-collidine for color development. To quantify amino acids high-performance liquid chromatography, ionexchange chromatography, and tandem mass spectrometry are used.

Given the following information for a rate reaction, calculate the activity of a serum specimen for alanine aminotransferase in international units per liter (IU/L). Time Absorbance 1 min 1.104 Specimen volume = 20 \jL 2 min 1.025 Reagent volume = 3.0 ml_ 3 min 0.950 Molar absorptivity for NADH at 340 nm = 6.22 x 103 L/mol-cm 4 min 0.873 Light path = 1 cm A. 186 B. 198 C. 1857 D. 1869

D. An international unit (IU) is defined as the enzyme activity that catalyzes the conversion of of substrate in 1 minute under standardwhen a rate method is employed, the following equation is used: total assay AA/min X volume (mL) X I06|xmol/tnol Absorptivity X light path X specimen coefficient (cm) volume (mL) 0.077 X 3.02 mL X 106 (xmol/mol 6.22 X 103 L/mol • cm X 1 cm X 0.02 mL = 1869 IU/L It is important to remember that the total assay volume includes the volume of reagent, diluent, and sample used in the particular assay and that the total assay volume and specimen volume should be expressed in the same units.

Because of similar electrophoretic mobilities, several hemoglobins cannot be differentiated on cellulose acetate medium. Electrophoresis of hemoglobins at pH 6.2 on agar gel may be useful in differentiating which hemoglobins? A. AI from A2 B. A! from D C. A, from E D. CfromA2

D. At pH 6.2 on agar gel, hemoglobins exhibit different electrophoretic mobilities in comparison with hemoglobins electrophoresed at pH 8.6 on cellulose acetate. The order of migration of hemoglobins on cellulose acetate, proceeding from the most anodal hemoglobin to the most cathodal hemoglobin, is respectively A[ and F, followed by G, D, and S, which migrate with the same mobility, followed by the group A2, C, O, and E, which migrate the most slowly with the same mobility. This migration pattern is in contrast to agar gel electrophoresis at pH 6.2 in which the order of migration, from the most anodal hemoglobin to the most cathodal hemoglobin, is, respectively, C and S, followed by hemoglobins A,, A2, D, E, and G, which migrate as a group with the same mobility, followed by F. The different migration patterns seen with these two media systems are useful in differentiating hemoglobins that migrate with the same electrophoretic mobility. In the case of hemoglobins A2 and C, which migrate with the same mobility on cellulose acetate, it is not possible to discern which hemoglobin is present in a particular blood specimen. By electrophoresing this specimen on agar gel at pH 6.2, hemoglobin A2 may be differentiated from hemoglobin C because hemoglobin A2 exhibits mobility similar to that of hemoglobin A|, whereas hemoglobin C migrates alone closest to the anode.

In assaying an analyte with a single-beam atomic absoiption spectrophotometer, what is the instrument actually measuring? A. Intensity of light emitted by the analyte on its return to the ground state B. Intensity of light that the analyte absorbs from the hollow-cathode lamp C. Intensity of light that the analyte absorbs from the flame D. Intensity of the beam from the hollowcathode lamp after it has passed through the analyte-containing flame

D. Atomic absorption spectrophotometry (AAS) is based on the principle that atoms in a basic ground state are capable of absorbing energy in the form of light at a specific wavelength. In a single-beam AAS, the amount of light that the analyte absorbs from the hollow-cathode lamp is what we wish to know. However, what is actually measured is the intensity of the beam after it has passed through the flame. This measurement is made with and without sample in the flame. In this way, the instrument calculates the amount of light absorbed because of the presence of the analyte in the flame. Because most samples usually have the analyte in the form of a compound or an ion, the analyte must first be converted to nonionized atoms. This is achieved by heating in a flame. About 99% of the atoms of analyte in the flame are in the ground state and, therefore, are capable of absorbing energy at the appropriate wavelength. Hence, light absorbed is essentially proportional to the concentration of the analyte. The light source in AAS is a hollowcathode lamp in which the cathode contains the element that is to be measured.

Express 30 mg/dL of urea nitrogen as urea. A. 14 mg/dL B. 20 mg/dL C. 50 mg/dL D. 64 mg/dL

D. Because the substances classified as nonprotein- nitrogen (NPN) compounds were quantified by assaying for their nitrogen content, it became customary to express urea as urea nitrogen. When urea was expressed as urea nitrogen, a comparison could be made between the concentration of urea and the concentration of other NPN compounds. When it is necessary to convert urea nitrogen values to urea, the concentration may be calculated easily by multiplying the urea nitrogen value by 2.14. This factor is derived from the molecular mass of urea (60 daltons) and the molecular weight of its two nitrogen atoms (28): 60 / 28 = 2.14

Which of the following factors will not adversely affect the accurate quantification of bilirubin in serum? A. Lipemia B. Hemolysis C. Exposure to light D. Specimen refrigeration

D. Bilirubin will deteriorate when exposed to either white or UV light. This deterioration is also temperature sensitive. Thus, specimens for bilirubin analysis should be stored in the dark at refrigerator temperature until the assay can be performed. Lipemia should be avoided, due to its interference with spectrophotometric analyses. Because hemoglobin reacts with diazo reagent, use of hemolyzed specimens should be avoided. Hemolysis will cause bilirubin results to be falsely low.

Which term describes a congenital disorder that is characterized by a split in the albumin band when serum is subjected to electrophoresis? A. Analbuminemia B. Anodic albuminemia C. Prealbuminemia D. Bisalbuminemia

D. Bisalbuminemia is a congenital disorder that does not exhibit any clinical manifestations. The only sign of this disorder is the splitting of albumin into two distinct bands when serum is subjected to electrophoresis. The extra albumin band may occur either anodically or cathodically to the normal albumin band depending on its speed of migration. The intensity of the two bands when quantified by densitometry may show that the two forms are of equal concentration. In a less common variation the abnormal albumin band may represent only 10-15% of the total albumin concentration.

To produce reliable results, when should blood specimens for lipid studies be drawn? A. Immediately after eating B. Anytime during the day C. In the fasting state, approximately 2 to 4 hours after eating D. In the fasting state, approximately 9 to 12 hours after eating

D. Blood specimens for lipid studies should be drawn in the fasting state at least 9 to 12 hours after eating. Although fat ingestion only slightly affects cholesterol levels, the triglyceride results are greatly affected. Triglycerides peak at about 4 to 6 hours after a meal, and these exogenous lipids should be cleared from the plasma before analysis. The presence of chylomicrons, as a result of an inadequate fasting period, must be avoided because of their interference in spectrophotometric analyses.

A patient's total cholesterol/HDL cholesterol ratio is 10.0. What level of risk for coronary heart disease does this result indicate? A. No risk B. Half average risk C. Average risk D. Twice average risk

D. Both total cholesterol and HDL cholesterol are independent measurable indicators of risk of coronary heart disease (CHD). By relating total and HDL cholesterol in a mathematical way, physicians can obtain valuable additional information in predicting risk for CHD. Risk of CHD can be quantified by the ratio of total cholesterol to HDL cholesterol along the following lines: Ratio Risk CHD 3.43 half average 4.97 average 9.55 two times average 24.39 three times average Thus this patient shows approximately twice the average risk for CHD. Risk ratios for CHD can easily be calculated by instrument and/or laboratory computers given the total and HDL cholesterol values. Reports indicating level of risk based on these results can be programmed by the laboratory and/or manufacturer.

Although serum elevations are not generally seen in early stages, which of the following tumor markers are elevated in more advanced stages of breast cancer? A. CEA and AFP B. AFP and C A 125 C. PSA and CA 15-3 D. CA 15-3 and CA 549

D. CA 15-3 and CA 549 are oncofetal antigens that are glycoprotein in nature. CA 15-3 is found on mammary epithelium. Increased serum levels of CA 15-3 are found in breast, pancreatic, lung, colorectal, and liver cancers. CA 549 is found in the cell membrane and luminal surface of breast tissue. Increased serum levels of CA 549 are found in breast, lung, prostate, and colon cancers. Although both CA 15-3 and CA 549 are elevated in more advanced stages of breast cancer, neither is helpful in detecting early stages of breast cancer.

Which of the following is an example of a phenothiazine drug? A. Cyclosporine B. Theophylline C. Phenytoin D. Chlorpromazine

D. Chlorpromazine (Thorazine ) and thioridazine are examples of phenothiazines and are used in the treatment of psychoses. Although the drugs themselves have a relatively short halflife, metabolites may be found in the urine for many weeks after cessation of therapy. Screening for phenothiazines is often done by specific chromatographic techniques or by the less specific ferric perchloricnitric (FPN) colorimetric reagent. Quantification is done by HPLC and fluorescent polarization immunoassay (FPIA).

What compound normally found in urine may be used to assess the completeness of a 24-hour urine collection? A. Urea B. Uric acid C. Creatine D. Creatinine

D. Creatine is predominantly found in muscle cells, where the quantity of creatine is proportional to muscle mass. As muscle metabolism proceeds, creatine is freed from its high-energy phosphate form, and the creatine, thus liberated, forms the anhydride creatinine. The quantity of creatinine formed daily is a relatively constant amount because it is related to muscle mass. Therefore, it has been customary to quantify the creatinine present in a 24-hour urine specimen as an index of the completeness of the collection.

Which of the following is not characteristic of cystic fibrosis? A. Decreased bicarbonate concentration in duodenal fluid B. Decreased lipase activity in duodenal fluid C. Decreased amylase activity in duodenal fluid D. Increased trypsin in feces

D. Cystic fibrosis is inherited as an autosomal recessive trait. It is a systemic disease that affects the exocrine glands, causing gastrointestinal malabsorption, pancreatic insufficiency, and pulmonary disease. Cystic fibrosis is characterized by increased concentrations of chloride and sodium in sweat. With pancreatic insufficiency, the amount of lipase, amylase, trypsin, and bicarbonate secreted into the duodenum is decreased. Because the three enzymes contribute to digestion of fats, starches, and proteins, respectively, children with this disorder suffer from malabsorption.

Which of the following is a homogeneous immunoassay where separation of the bound from the free labeled species is not required? A. Radioimmunoassay B. Enzyme-linked immunosorbent assay C. Immunoradiometric assay D. Enzyme-multiplied immunoassay technique

D. Enzyme-multiplied immunoassay technique (EMIT) is an example of a homogeneous immunoassay technique. A homogeneous assay is one in which separation of the bound and free fraction is unnecessary. The antigen is labeled with an enzyme and competes with the unknown antigen for binding sites on the antibody. The enzyme-labeled antigen that remains in the free fraction is enzymatically active. Therefore, the free labeled antigen can be determined by its action on a substrate in the presence of boundlabeled fraction. This type of assay is used commonly on automated instruments. The other techniques mentioned in the question, RIA, ELISA, and IRMA, are termed heterogeneous immunoassays because they require the physical separation of the bound from the free fraction before actual measurement.

Which of the following disorders is characterized by an inability to transport bilirubin from the sinusoidal membrane into the hepatocyte? A. Carcinoma of the common bile duct B. Crigler-Najjar syndrome C. Dubin-Johnson syndrome D. Gilbert syndrome

D. Gilbert syndrome is a preconjugation transport disturbance. In this disorder the hepatic uptake of bilirubin is defective because the transportation of bilirubin from the sinusoidal membrane to the microsomal region is impaired. Gilbert syndrome is inherited as an autosomal dominant trait characterized by increased levels of unconjugated bilirubin.

Which test may be performed to assess the average plasma glucose level that an individual maintained during a previous 2- to 3-month period? A. Plasma glucose B. Two-hour postprandial glucose C. Oral glucose tolerance D. Glycated hemoglobin

D. Glycated hemoglobin is a collective term encompassing the three glycated hemoglobin fractions—hemoglobin A)a, hemoglobin Alb, and hemoglobin Alc. Hb Alc is the fraction of Hb AI that is present in the greatest concentration. Some commercially available column chromatography methods measure the three fractions collectively. Glycated hemoglobin refers to the specific red cell hemoglobin A types to which a glucose molecule becomes irreversibly attached. The greater the glucose concentration in the plasma, the greater the number of hemoglobin molecules that will become glycated. Because red blood cells have an average life span of 120 days and the glycation is irreversible, measurement of glycated hemoglobin reflects the average plasma glucose level of an individual during the previous 2- to 3-month period. This test is used as a monitor of diabetic control.

What compound chelates iron and is the immediate precursor of heme formation? A. Porphobilinogen B. Protopoiphyrinogen IX C. Uroporphyrinogen III D. Protoporphyrin IX

D. Heme is derived from a series of biochemical reactions that begin with the formation of porphobilinogen from succinyl coenzyme A and glycine. Because porphobilinogen is a monopyrrole, four molecules of porphobilinogen condense and cyclize to form the porphyrinogen precursors of protoporphyrin IX. Protoporphyrin IX chelates iron to form heme and is, therefore, the immediate precursor of heme formation.

Acetaminophen is particularly toxic to what organ? A. Heart B. Kidney C. Spleen D. Liver

D. Hepatotoxicity is common in acetaminophen overdose. It is particularly important to be able to determine the acetaminophen serum level rapidly so that the elimination half-life of the drug can be estimated. Hepatic necrosis is more common when the half-life exceeds 4 hours and is very likely when it exceeds 12 hours. The concentration of acetaminophen can be measured by HPLC, colorimetric, EMIT, and fluorescence polarization methods.

A mother brings her daughter, a 22-yearold medical technology student, to her physician. The patient is hyperventilating and has glossy eyes. The mother explains that her daughter is scheduled to take her final course exam the next morning. The patient had been running around frantically all day in a worried state and then started to breathe heavily. Blood gases are drawn in the office with the following results: pH = 7.58, PCO2 = 55 mm Hg, HCOj = 18 mmol/L .What do these data indicate? A. Metabolic alkalosis, partially compensated B. Respiratory acidosis, uncompensated C. A dual problem of acidosis D. An error in one of the blood gas measurements

D. Here the pH and case information indicate alkalosis, but both the metabolic (decreased HCO^) and respiratory (increased PCO2) components indicate acidosis. Most likely there is a problem/error in one or more of the measurements.

Fluorometers are designed so that the path of the exciting light is at a right angle to the path of the emitted light. What is the purpose of this design? A. Prevent loss of emitted light B. Prevent loss of the excitation light C. Focus emitted and excitation light upon the detector D. Prevent excitation light from reaching the detector

D. In a fluorometer, light from the excitation lamp travels in a straight line, whereas the fluorescent light is radiated in all directions. If the detector for the emitted fluorescent light is placed at a right angle to the path of the excitation light, the excitation light will not fall on the detector. In addition, baffles can be placed around the cuvet to avoid reflection of the exciting light from the surface of the cuvet to the detector. The right-angle configuration does not prevent loss of the exciting or the emitted light.

A serum ferritin level may not be a useful indicator of iron-deficiency anemia in patients with what type of disorder? A. Chronic infection B. Malignancy C. Viral hepatitis D. All the above

D. In cases of iron-deficiency anemia uncomplicated by other diseases, serum ferritin levels correlate well with the evidence of iron deficiency obtained by marrow examination for stainable iron. This indicates that ferritin is released into the serum in direct proportion to the amount stored in tissues. In iron deficiency, serum ferritin levels fall early in the disease process. However, in certain disorders there is a disproportionate increase in serum ferritin in relation to iron stores. Examples include chronic infections, chronic inflammation, malignancies, and liver disease. For individuals who have these chronic disorders or iron deficiency, it is common for their serum ferritin levels to appear normal.

Which of the following is not associated with potassium? A. Has no renal threshold B. Increased serum level in acidosis C. Hemolysis causes false increase in serum levels D. Major anion of intracellular fluid

D. In contrast to sodium, which is the principal plasma cation, potassium is the principal cellular cation. After absorption in the intestinal tract, potassium is partially filtered from the plasma by the kidneys. It is then almost completely reabsorbed from the glomerular filtrate by the proximal tubules and subsequently reexcreted by the distal tubules. Unlike sodium, potassium exhibits no renal threshold, being excreted into the urine even in K+-depleted states. In acidotic states, as in renal tubular acidosis in which the exchange of Na+ for H+ is impaired, the resulting retention of potassium causes an elevation in serum K+ levels. Hemolysis must be avoided in blood specimens that are to be used for K+ analysis because erythrocytes contain a potassium concentration 23 times greater than serum K+ levels. If the red blood cells are hemolyzed, a significant increase in serum K+ will result.

When electrophoresis is performed, holes appear in the staining pattern, giving the stained protein band a doughnut-like appearance. What is the probable cause of this problem? A. Protein denatured and will not stain properly B. Ionic strength of the buffer was too high C. Protein reached its isoelectric point and precipitated out D. Protein concentration was too high

D. In electrophoresis, each band in the stained protein pattern should be uniformly colored; that is, no holes should appear within an individual band. Such a doughnut-like appearance occurs when the protein is present in too high a concentration, thus exceeding the complexing ability of the stain. To overcome this problem, dilute elevated specimens before rerunning the electrophoresis.

A healthy, active 10-year-old boy with no prior history of illness comes to the lab after school for a routine chemistry screen in order to meet requirements for summer camp. After centrifugation, the serum looks cloudy. The specimen had the following results: blood glucose = 135 mg/dL, total cholesterol =195 mg/dL, triglyceride =185 mg/dL. What would be the most probable explanation for these findings? The boy A. Is at risk for coronary artery disease B. Has type 1 diabetes mellitus that is undiagnosed C. Has an inherited genetic disease causing a lipid imbalance D. Was most likely not fasting when the specimen was drawn

D. In evaluating lipid profile results, it is important to start with the integrity of the sample. From the case history, it is doubtful that a 10-year-old healthy, active boy would be suffering from a lipid or glucose disorder manifesting these kinds of results. Furthermore, the boy came in for testing after school. It is improbable that a 10-yearold boy would be able to maintain a 9- to 12-hour fast during the school day. In this case, the boy should have been thoroughly interviewed by the laboratory staff before the blood test to determine if he was truly fasting. Specimen integrity is the first thing that must be ensured before running any glucose or lipid tests.

Which of the following does not apply to gas-liquid chromatography? A. Separation depends on volatility of the sample. B. Separation depends on the sample's solubility in the liquid layer of the stationary phase. C. Stationary phase is a liquid layer adsorbed on the column packing. D. Mobile phase is a liquid pumped through the column.

D. In gas-liquid chromatography (GLC), the stationary phase is a liquid adsorbed on particles packed in a column. The mobile phase is a gas that passes through the column. Because the sample is carried in the mobile phase, it must be volatile at the temperature of the column so that it can be carried by the gas. In addition, separation is dependent on the solubility of the solute in the liquid layer of the stationary phase.

Which of the following is not the reason that preventive maintenance schedules are required? A. Keep instrument components clean B. Replace worn parts C. Extend the life of the equipment D. Keep personnel busy when the laboratory work is slow

D. In order to prevent excessive downtime and costly repairs, a preventive maintenance schedule should be devised, implemented, and recorded for all laboratory equipment. Preventive maintenance procedures include the cleaning of instrument components, the replacing of worn parts, and the adjusting of certain parts or parameters. Following a preventive maintenance schedule will help to extend the life of the equipment. It is important that all laboratory personnel recognize the need for routine maintenance and follow prescribed maintenance schedules.

What is the immediate precursor of bilirubin formation? A. Mesobilirubinogen B. Verdohemoglobin C. Urobilinogen D. Biliverdin

D. In the catabolic process of hemoglobin degradation, the alpha-carbon methene bridge of the tetrapyrrole ring structure of heme opens oxidatively to form verdohemoglobin. Verdohemoglobin is a complex composed of biliverdin, iron, and the protein globin. This complex then undergoes degradation in which iron is removed and returned to the body iron stores, the globin portion is returned to the amino acid pool, and the biliverdin undergoes reduction to form bilirubin. It is biliverdin, therefore, that is the immediate precursor of bilirubin formation. Mesobilirubinogen and urobilinogen represent intestinal breakdown products of bilirubin catabolism.

Which of the following statements may be associated with the activity of insulin? A. Increases blood glucose levels B. Decreases glucose uptake by muscle and fat cells C. Stimulates release of hepatic glucose into the blood D. Stimulates glycogenesis in the liver

D. Insulin may be described as an anabolic, polypeptide hormone. Insulin stimulates glucose uptake by muscle cells (which increases protein synthesis), by fat cells (which increases triglyceride synthesis), and by liver cells (which increases lipid synthesis and glycogenesis). If cellular uptake of glucose is stimulated, the glucose concentration in the circulation decreases.

Which of the following statements can be associated with the enzymatic assay of ammonia? A. Increase in absorbance monitored at 340 nm B. Nicotinamide-adenine dinucleotide (NAD+) required as a cofactor C. Ammonium ion isolated from specimen before the enzymatic step D. Reaction catalyzed by glutamate dehydrogenase

D. Ion-exchange, ion-selective electrode, and enzymatic methods have been employed for the analysis of ammonia in plasma specimens. Because the enzymatic method is a direct assay, prior separation of ammonium ions is not required. The enzymatic reaction catalyzed by glutamate dehydrogenase follows: 2-Oxoglutarate + NHj + NADPH <==> Glutamate + NADP+ + H2O The rate of oxidation of NADPH to NADP+ is followed as a decreasing change in absorbance at 340 nm.

For which of the following laboratory instalments should preventive maintenance procedures be performed and recorded? A. Analytical balance B. Centrifuge C. Chemistry analyzer D. All the above

D. It is imperative that preventive maintenance procedures be performed and the results recorded for all laboratory instrumentation. This includes maintenance of analytical balances, refrigerators, freezers, centrifuges, ovens, water baths, heating blocks, thermometers, pipetters, dilutors, automated analyzers, and all other laboratory equipment used for analyzing specimens. Preventive maintenance is performed at scheduled times such as per shift, daily, weekly, monthly, or yearly.

Which of the following statements is true about partially compensated respiratory alkalosis? A. PCC>2 is higher than normal. B. HCO^ is higher than normal. C. More CC>2 is eliminated through the lungs by hyperventilation. D. Renal reabsorption of HCC>3 is decreased.

D. Laboratory results from arterial blood gas studies in partially compensated respiratory alkalosis are as follows: pH slightly increased, PCO2 decreased, HCO^ decreased, and total CO2 decreased. Respiratory alkalosis is a disturbance in acid-base balance that is caused by hyperventilation associated with such conditions as fever, hysteria, and hypoxia. Respiratory alkalosis is characterized by a primary deficiency in physically dissolved CO2 (decreased PCO2). This decrease in the level of PCO2 is due to hyperventilation, causing the accelerated loss of CO2 by the lungs. This loss of CO2 alters the normal 20:1 ratio of cHCO^/PCO2, causing an increase in the blood pH level. In respiratory alkalosis, because the initial defect is in the lungs, the kidneys respond as the major compensatory system. Ammonia production in the kidneys is decreased, Na4"-!-^ exchange is decreased with the retention of IT1", and bicarbonate reabsorption is decreased. By decreasing the bicarbonate reabsorption into the bloodstream, the kidneys attempt to reestablish the 20:1 ratio and normal blood pH. In a partially compensated state, as the blood bicarbonate level decreases, the blood pH begins to return toward normal but continues to be slightly alkaline. In a fully compensated state the blood pH is normal.

Which chromatography system is commonly used in conjunction with mass spectrometry? A. High-performance liquid B. Ion-exchange C. Partition D. Gas-liquid

D. Mass spectrometry is used in the clinical laboratory in conjunction with gas or liquid chromatography (GC-MS). In gas chromatography a compound is identified by its retention time. If two compounds have very similar retention times, the compound may be misidentified. Gas chromatography complements mass spectrometry in that the eluted peak is subjected to mass spectrometric analysis for molecular weight determination. Use of the two systems in tandem allows for more accurate identification of compounds.

A 4-year-old female presents with a palpable abdominal mass, pallor, and petechiae. Based on family history, clinical findings, and the patient's physical examination, neuroblastoma is suspected. Which of the following does not support such a diagnosis? A. Increased blood dopamine levels B. Increased blood epinephrine levels C. Increased urinary homovanillic acid D. Decreased urinary vanillylmandelic acid

D. Neuroblastoma is a solid malignant tumor found in the medulla of the adrenal gland, or it may arise from the extra-adrenal sympathetic chain. More commonly the disease occurs in children under the age of 5 years. Metastasis may occur to the liver, bone, bone marrow, or brain. Neuroblastoma is characterized by tumor production of epinephrine, norepinephrine, and dopamine, so all three hormones will be increased in the blood. The end product of dopamine metabolism is homovanillic acid (HVA). The end product of the catecholamines, epinephrine and norepinephrine, is vanillylmandelic acid (VMA). Both HVA and VMA will be excreted in excess in the urine.

A physician orders several laboratory tests on a 55-year-old male patient who is complaining of pain, stiffness, fatigue, and headaches. Based on the following serum test results, what is the most likely diagnosis? Alkaline phosphatase—significantly increased Gamma-glutamyltransferase—normal A. Biliary obstruction B. Cirrhosis C. Hepatitis D. Osteitis deformans

D. Osteitis deformans, also known as Paget disease, is a chronic disorder of bone. This disorder is characterized by a significant increase in the serum alkaline phosphatase level. Gammaglutamyltransferase will be normal in bone disease, because this enzyme is not found in bone tissue. However, in hepatobiliary disease both enzymes would characteristically be elevated.

Which of the following does not have an effect on plasma calcium levels? A. Parathyroid hormone B. Vitamin D C. Calcitonin D. Aldosterone

D. PTH, calcitonin, vitamin D, plasma proteins, and plasma phosphates are factors that influence plasma calcium levels. PTH is a hormone important in maintaining plasma calcium levels. It mobilizes calcium from bones. It increases the synthesis of one of the vitamin D derivatives, thereby causing an increase in bone resorption and intestinal absoiption of calcium. When normal calcium levels are restored, PTH secretion is cut off (negative- feedback mechanism). Calcitonin (thyrocalcitonin) is a hormone secreted by the thyroid gland in response to elevated levels of plasma calcium. It acts by inhibiting bone resorption of calcium, thereby preventing significant variations in plasma calcium concentrations. Hydroxylation of vitamin D gives a derivative that will increase the intestinal absorption of calcium and phosphates.

In the collection of plasma specimens for lactate determinations, which of the following anticoagulants would be more appropriate? A. Sodium heparin B. Sodium citrate C. EDTA D. Oxalate plus fluoride

D. Plasma lactate concentrations are increased in cases of lactic acidosis. The accumulation of lactate in the blood results from any mechanism that produces oxygen deprivation of tissues and, thereby, anaerobic metabolism. Lactate concentrations in whole blood are extremely unstable because of the rapid production and release of lactate by erythrocytes as a result of glycolysis. One method of stabilizing blood lactate levels in specimen collection is to add an enzyme inhibitor such as fluoride or iodoacetate to the collection tubes. Heparin, ethylenediaminetetra-acetic acid (EDTA), and oxalate will act as anticoagulants but will not prevent glycolysis in the blood sample.

Which of the following is a qualitative screening test for porphobilinogen that may be performed to aid in the diagnosis of the porphyrias? A. Caraway test B. Gutmantest C. Jendrassik-Grof test D. Watson-Schwartz test

D. Porphobilinogen is a precursor compound in the biosynthesis of heme. In acute intermittent porphyria, excess amounts of porphobilinogen are excreted in the urine. The Watson-Schwartz test employs ;>dimethylaminobenzaldehyde reagent (also known as Ehrlich's aldehyde reagent) to form a red condensation product with porphobilinogen.

Which of the following hormones does not promote an increase in blood glucose levels? A. Growth hormone B. Cortisol C. Glucagon D. Insulin

D. Regulation of the blood glucose concentration depends on a number of hormones. These include insulin, glucagon, cortisol, epinephrine, growth hormone, adrenocorticotropic hormone, and thyroxine. Of these hormones, insulin is the only one that decreases the blood glucose level. Glucagonis produced in the pancreas by the alpha cells. Glucagon promotes an increase in the blood glucose concentration by its stimulatory effect on glycogenolysis in the liver. Cortisol is produced by the adrenal cortex. It stimulates gluconeogenesis, thus increasing the blood level of glucose. Epinephrine is produced by the adrenal medulla. It promotes glycogenolysis, thus increasing blood glucose. Growth hormone and adrenocorticotropic hormone are produced by the anterior pituitary gland. Both hormones are antagonistic to insulin and hence increase blood glucose. Thyroxine is produced by the thyroid gland. It not only stimulates glycogenolysis but also increases the intestinal absorption rate of glucose.

Of the following blood glucose levels, which would you expect to result in glucose in the urine? A. 60mg/dL B. 120mg/dL C. 150mg/dL D. 225mg/dL

D. Renal threshold is defined as the plasma level that must be exceeded in order for the substance to appear in the urine. The renal threshold for glucose is 180 mg/dL. This means that the blood glucose level must exceed 180 mg/dL in order for glucose to be excreted in the urine.

Which of the following analytes would not commonly be measured when monitoring complications of diabetes mellitus? A. Serum urea nitrogen B. Urinary albumin C. Serum creatinine D. Serum bilirubin

D. Research has demonstrated that there is a correlation between blood glucose levels in diabetes mellitus and the development of longterm complications. These complications may include such disorders as retinopathy, neuropathy, atherosclerosis, and renal failure. Thus, quantifying such blood analytes as urea, creatinine, and lipids as well as urinary albumin can aid in monitoring diabetic individuals.

The surfactant/albumin ratio by fluorescence polarization is performed to assess what physiological state? A. Hyperlipidemia B. Coronary artery disease C. Hemolytic disease of the newborn D. Fetal lung maturity

D. Respiratory distress syndrome (RDS), also referred to as hyaline membrane disease, is commonly seen in preterm infants. A deficiency of pulmonary surfactant causes the infant's alveoli to collapse during expiration, resulting in improper oxygenation of capillary blood in the alveoli. Currently, the surfactant/albumin ratio by fluorescence polarization is performed using amniotic fluid to assess fetal lung maturity. The amniotic fluid is mixed with a fluorescent dye. When the dye binds to albumin there is a high polarization, and when the dye binds to surfactant there is a low polarization. Thus the surfactant/ albumin ratio is determined. The units are expressed as milligrams of surfactant per gram of albumin, with fetal lung maturity being sufficient with values greater than 50 mg/g. Older methodologies have employed the determinations of phosphatidylglycerol, foam stability, and lecithin/sphingomyelin (L/S) ratio. The L/S ratio is based on the physiological levels of lecithin and sphingomyelin. Lecithin is a surfactant that prepares lungs to expand and take in air. Sphingomyelin is incorporated into the myelin sheath of the central nervous system of the fetus. The amounts of lecithin and sphingomyelin produced during the first 34 weeks of gestation are approximately equal; however, after the 34th week, the amount of lecithin synthesized greatly exceeds that of sphingomyelin. At birth, an L/S ratio of 2:1 or greater would indicate sufficient lung maturity.

A 10-year-old female presents with varicella. The child has been experiencing fever, nausea, vomiting, lethargy, and disorientation. A diagnosis of Reye syndrome is determined. Which of the following laboratory results is not consistent with the diagnosis? A. Elevated serum AST B. Elevated serum ALT C. Elevated plasma ammonia D. Elevated serum bilirubin

D. Reye syndrome is associated with viral infections, exogenous toxins, and salicylate use. The disorder generally manifests itself in children from 2 to 13 years of age. The laboratory findings that support a diagnosis of Reye syndrome include increased levels of serum aspartate and alanine transaminases (greater than 3 times the reference range), increased plasma ammonia level (can exceed 100 (jig/dL), and prolonged prothrombin time (3 sec or more than the control). In Reye syndrome the serum bilirubin level is generally within the reference range.

A sample of blood is collected for glucose in a sodium fluoride tube before the patient has had breakfast. The physician calls 2 hours later and requests that determination of blood urea nitrogen (BUN) be performed on the same sample rather than obtaining another specimen. The automated analyzer in your laboratory utilizes the urease method to quantify BUN. What should you tell the physician? A. Will gladly do the test if sufficient specimen remains B. Could do the test using a micromethod C. Can do the BUN determination on the automated analyzer D. Cannot perform the procedure

D. Sodium fluoride is a weak anticoagulant that acts as a preservative for glucose. It functions as a glucose preservative by inhibiting glycolysis. However, it is not suitable for use with many enzyme procedures. In the determination of BUN, where urease activity is utilized, the high concentration of fluoride in the plasma acts as an enzyme inhibitor, preventing the necessary chemical reaction.

Which of the following is most associated with the membrane structure of nerve tissue? A. Cholesterol B. Triglyceride C. Phospholipids D. Sphingolipids

D. Sphingolipids, most notably sphingomyelin, are the major lipids of the cell membranes of the central nervous system (i.e., the myelin sheath). Like phospholipids, Sphingolipids are amphipathic and contain a polar, hydrophilic head and a nonpolar, hydrophobic tail, making them excellent membrane formers. Although sometimes considered a subgroup of phospholipids, sphingomyelin is derived from the amino alcohol sphingosine instead of glycerol.

Which of the following is the most potent androgen? A. Androstenedione B. Dehydroepiandrosterone C. Androsterone D. Testosterone

D. Testosterone is the most potent of the body's androgens. One of the major functions of the testes is to produce testosterone. It is metabolized to the 17-ketosteroids, etiocholanolone and androsterone, but testosterone is not itself a 17-ketosteroid. The 17-ketosteroids, dehydroepiandrosterone (DHEA), androsterone, and androstenedione, all have androgenic properties but are much weaker than testosterone.

Your lab routinely uses a precipitation method to separate HDL cholesterol. You receive a slightly lipemic specimen for HDL cholesterol. The total cholesterol and triglyceride for the specimen were 450 and 520 mg/dL, respectively. After adding the precipitating reagents and centrifuging, you notice that the supernatant still looks slightly cloudy. What is your next course of action in analyzing this specimen? A. Perform the HDL cholesterol test; there is nothing wrong with this specimen. B. Take off the supernatant and recentrifuge. C. Take off the supernatant and add another portion of the precipitating reagent to it and recentrifuge. D. Send specimen to a lab that offers other techniques to separate more effectively the HDL cholesterol.

D. The Abell-Kendall assay is commonly used to separate HDL cholesterol from other lipoproteins. In this precipitation technique a heparin sulfate-manganese chloride mixture is used to precipitate the LDL and VLDL cholesterol fractions. This technique works well as long as there is no significant amount of chylomicrons or lipemia in the specimen and/or the triglyceride is under 400 mg/dL. Incomplete sedimentation is seen as cloudiness or turbidity in the supernatant after centrifugation. It indicates the presence of other lipoproteins and leads to over estimation of HDL cholesterol. The lipemic specimens may be cleared and the HDL cholesterol separated more effectively by using ultrafiltration, extraction, latex immobilized antibodies, and/or ultracentrifugation. These techniques are usually not available in a routine laboratory.

When screening urine for toxic concentrations of certain substances, which of the following will not be identified by the Reinsch test? A. Bismuth B. Arsenic C. Mercury D. Cyanide

D. The Reinsch test is applied to urine and is based on the ability of copper to reduce most metal ions to their metallic states in the presence of acid. Cyanide is not a metal and, therefore, will not be reduced. Increased urinary levels of arsenic, bismuth, antimony, and mercury will coat the copper with dull black, shiny black, blue-black, and silver-gray deposits, respectively. The test is intended as a rapid screening method only, and results should be confirmed by more sensitive and specific methods.

What percentage decrease in plasma or urinary estriol, in comparison with the previous day's level, is considered significant during pregnancy? A. 5 B. 10 C. 25 D. 40

D. The concentration of estriol in maternal plasma or in a 24-hour sample of maternal urine is often used as an indicator of fetal distress or placental failure. A single value of either serum or urine estriol has relatively little value unless it can be related accurately to the gestational week. When sequential estriol determinations are made during pregnancy, a pattern of stable or steadily falling values may indicate a problem pregnancy. For serum or urine estriols, any individual value that is 30-50% less than the previous value or the average of the previous 3 days' values is significant.

The creatinine clearance test is routinely used to assess the glomerular filtration rate. Given the following information for an average-size adult, calculate a creatinine clearance. Urine creatinine—120 mg/dL Plasma creatinine—1.2 mg/dL Urine volume for 24 hours—1520 mL A. 11 mL/min B. 63 mL/min C. 95 mL/min D. 106 mL/min

D. The creatinine clearance test is used to assess the glomerular filtration rate. An accurately timed 24-hour urine specimen and a blood sample, drawn in the middle of the 24-hour urine collection, are required. The creatinine concentrations of the urine specimen and the plasma are determined, and these values, along with the urine volume, are used to determine the creatinine clearance. The body surface area will not be used in the calculation because the clearance is being done on an average-size adult. The following general mathematical formula is used to calculate creatinine clearance: — X V = Creatinine clearance (mL/min) where U = urine creatinine concentration in milligrams per deciliter, P = plasma creatinine concentration in milligrams per deciliter, and V = volume of urine per minute, with volume expressed in milliliters and 24 hours expressed as 1440 minutes. Applying this formula to the problem presented in the question: 120 mg/dL 1520 mL/24 hr 1.2 mg/dL 1440 min/24 hr = 106 mL/min It should be noted that both the size of the kidney and the body surface area of an individual influence the creatinine clearance rate. Because normal values for creatinine clearance are based on the average adult body surface area, it is necessary that the clearance rate be adjusted when the body surface area of the individual being tested differs significantly from the average adult area. This type of adjustment is especially critical if the individual is an infant, a young child, or an adolescent. The body surface area may be calculated from an individual's height and weight, or it may be determined from a nomogram. The average body surface area is accepted as being 1.73 m2. The mathematical formula used to calculate a creatinine clearance when the body surface area of the individual is required follows: U 1.73 — X V X = Creatinine clearance P A (mL/min/standard surface area) where 1.73 = standard adult surface area in square meters and A = body surface area of the individual in square meters.

What may be the cause of neonatal physiological jaundice of the hepatic type? A. Hemolytic episode caused by an ABO incompatibility B. Stricture of the common bile duct C. Hemolytic episode caused by an Rh incompatibility D. Deficiency in the bilirubin conjugation enzyme system

D. The enzyme uridine diphosphate glucuronyltransferase catalyzes the conjugation of bilirubin with glucuronic acid. In newborns, especially premature infants, this liver enzyme system is not fully developed or functional. Because of this deficiency in the enzyme system, the concentration of unconjugated bilirubin rises in the blood, because only the conjugated form may be excreted through the bile and urine. The increased levels of unconjugated bilirubin will cause the infant to appear jaundiced. Generally, this condition persists for only a short period because the enzyme system usually becomes functional within several days after birth. Neonatal physiological jaundice resulting from an enzyme deficiency is hepatic in origin. Hemolytic jaundice resulting from either Rh or ABO incompatibility is a prehepatic type of jaundice, whereas a stricture of the common bile duct is classified as posthepatic jaundice.

During pregnancy, estriol is synthesized in the placenta from formed in the . A. Estradiol, mother B. Estradiol, fetus C. 16a-Hydroxy-DHEA-S,mother D. 16a-Hydroxy-DHEA-S, fetus

D. The formation of estriol during pregnancy involves mainly the fetoplacental unit. Dehydroepiandrosterone sulfate (DHEA-S) and its 16a-hydroxy-DHEA-S derivative are formed mainly by the fetal adrenal glands and to a lesser degree by the liver. The fetus possesses 16ahydroxylase activity, which is needed to convert dehydroepiandrosterone sulfate (DHEA-S) to 16a-hydroxy-DHEA-S. The 16a-hydroxy- DHEA-S compound is metabolized by the placenta to estriol. The placenta lacks certain enzymes needed for the conversion of simple precursors such as acetate, cholesterol, and progesterone to estrogens. Thus, the placenta must rely on immediate precursors produced in the fetus. In the case of estriol, the placenta utilizes the 16a-hydroxy-DHEA-S precursor made in the adrenal glands of the fetus. The latter compound crosses into the placenta, which takes over with the necessary enzymes to complete the synthesis of estriol. This estriol produced in the placenta is rapidly reflected in the maternal plasma and far exceeds maternal synthesis of estriol. Thus measurement of estriol in the maternal blood or urine is a sensitive indicator of the integrity of the fetoplacental unit. A defect in either the fetus or the placenta will be reflected by a decrease in estriol production.

Which of the following describes the basis for the freezing point osmometer? A. The freezing point depression is directly proportional to the amount of solvent present. B. The freezing point depression varies as the logarithm of the concentration of solute. C. The freezing point is raised by an amount that is inversely proportional to the concentration of dissolved particles in the solution. D. The freezing point is lowered by an amount that is directly proportional to the concentration of dissolved particles in the solution.

D. The freezing point of an aqueous solution is lowered 1.86°C for every osmole of dissolved particles per kilogram of water. These particles may be ions (e.g., Na+ and Cl~), or undissociated molecules such as glucose. The freezing point osmometer is an instrument designed to measure the freezing point of solutions. It uses a thermistor that is capable of measuring very small changes in temperature.

Which of the following disorders is not characterized by an elevated serum myoglobin? A. Renal failure B. Vigorous exercise C. Acute myocardial infarction D. Hepatitis

D. The heme protein myoglobin can bind oxygen reversibly and is found in cardiac and striated muscles. In cases of acute myocardial infarction, myoglobin increases within 1-3 hours of the infarct. Myoglobin is not cardiac specific, and increased serum levels also occur in vigorous exercise, intramuscular injections, rhabdomyolysis, and muscular dystrophy. Because myoglobin is a relatively small protein and able to be excreted by the kidneys, elevated serum levels occur in renal failure.

Bence Jones proteinuria is a condition characterized by the urinary excretion of what type of light chain? A. Kappa light chains B. Lambda light chains C. Both kappa and lambda light chains D. Either kappa or lambda light chains

D. The immunoglobulins are composed of both heavy and light chains. In Bence Jones proteinuria, there is an overproduction of one type of light chain by a single clone of plasma cells. Therefore, the plasma cells produce either an excessive amount of kappa light chains or an excessive amount of lambda light chains. The light-chain type produced is in such abundance that the renal threshold is exceeded, resulting in the excretion of free light chains of the kappa or lambda type in the urine. The type of light chain excreted in the urine may be identified by performing immunoelectrophoresis on a concentrated urine specimen. In addition, immunoturbidimetric and immunonephelometric methods may also be used.

There are five immunoglobulin classes: IgG, IgA, IgM, IgD, and IgE. With which globulin fraction do these immunoglobulins migrate electrophoretically? A. Alphapglobulins B. Alpha2-globulins C. Betapglobulins D. Gamma-globulins

D. The immunoglobulins, IgG, IgA, IgM, IgD, and IgE, migrate electrophoretically with the gamma-globulin fraction. The normal serum levels of the IgD and IgE classes are so low that these two immunoglobulins do not normally contribute to the intensity of the stained gammaglobulin electrophoretic fraction. The primary component of the gamma fraction consists of IgG, with IgA and IgM contributing to the intensity of the stained fraction to a lesser degree. In disease states the concentration relationship between the immunoglobulins may be significantly altered from the normal.

When measuring K+ with an ion-selective electrode by means of a liquid ionexchange membrane, what antibiotic will be incorporated into the membrane? A. Monactin B. Nonactin C. Streptomycin D. Valinomycin

D. The ion-exchange electrode is a type of potentiometric, ion-selective electrode that consists of a liquid ion-exchange membrane that is made of an inert solvent and an ion-selective neutral earner material. A collodion membrane may be used to separate the membrane solution from the sample solution being analyzed. Because of its ability to bind K+, the antibiotic valinomycin is used as the neutral carrier for the K+-selective membrane. The antibiotics nonactin and monactin are used in combination as the neutral carrier for the NH^-selective membrane. A special formulation is used to make a selective glass membrane for the measurement of sodium.

The kinetic methods for quantifying serum triglyceride employ enzymatic hydrolysis. The hydrolysis of triglyceride may be accomplished by what enzyme? A. Amylase B. Leucine aminopeptidase C. Lactate dehydrogenase D. Lipase

D. The kinetic methods used for quantifying serum triglycerides use a reaction system of coupling enzymes. It is first necessary to hydrolyze the triglycerides to free fatty acids and glycerol. This hydrolysis step is catalyzed by the enzyme lipase. The glycerol is then free to react in the enzyme-coupled reaction system that includes glycerokinase, pyruvate kinase, and lactate dehydrogenase or in the enzyme-coupled system that includes glycerokinase, glycerophosphate oxidase, and peroxidase.

A 42-year-old woman is admitted to the hospital with complaints of abdominal pain and inability to eat, which have gotten worse during the past several weeks. Although the pain had been uncomfortable, what alarmed her was noticing a slight yellow color in her eyes. Blood was drawn and the test results follow: total bilirubin 3.9 mg/dL, direct bilirubin 2.7 mg/dL, AST slightly elevated (3 times the upper limit of the reference range), ALT slightly elevated (3 times the upper limit of the reference range), alkaline phosphatase markedly elevated (6 times the upper limit of the reference range), and urine urobilinogen decreased. What diagnosis do these test results support? A. Viral hepatitis B. Cirrhosis C. Exposure to toxic chemicals D. Biliary obstruction

D. The laboratory test results suggest that the woman has posthepatic biliary obstruction. The diagnosis is supported by the greater increase in alkaline phosphatase in contrast to the lesser increases in ALT and AST, the greater increase in the direct (conjugated) bilirubin level, and the decrease in urine urobilinogen. In biliary obstruction, increased synthesis of alkaline phosphatase is induced, with more produced in hepatocytes adjacent to biliary canaliculi. When an obstruction occurs in the biliary system, which may be caused by such disorders as gallstones in the common bile duct or a tumor in the region of the ampulla of Vater, the conjugated bilirubin made in the liver is unable to pass into the intestines. This will increase the bilirubin level in the blood, as well as result in less production of urobilinogen in the intestines. Thus, less urobilinogen will be transported in the enterohepatic circulation and less urobilinogen will be excreted in the urine. The other choices, viral hepatitis, exposure to toxic chemicals, and cirrhosis, are hepatic disorders that affect the hepatocytes directly and thus liver function. In such cases, there would be hepatocyte injury or tissue necrosis resulting in greater elevation of ALT and AST as compared to alkaline phosphatase, and the unconjugated bilirubin would be the fraction with the more significant increase because hepatocyte function is compromised. Urobilinogen would also be increased in the urine because of the inability of the liver to process it.

In spectrophotometric analysis, what is the purpose of the reagent blank? A. Correct for interfering chromogens B. Correct for lipemia C. Correct for protein D. Correct for color contribution of the reagents

D. The reagent blank contains the same reagents as those used for assaying the specimen. By adjusting the spectrophotometer to IOQ%T (or 0 absorbance) with the reagent blank, the instrument automatically subtracts the color contributed by the reagents from each succeeding reading of specimens, controls, and standards. This technique is used both in manual procedures and automated instruments. Because the reagent blank does not contain sample, there is no correction for interfering chromogens or lipemia.

Of the total serum calcium, free ionized calcium normally represents approximately what percent? A. 10 B. 40 C. 50 D. 90

D. The renal tubular reabsorption of phosphate is controlled by the action of parathyroid hormone (PTH) on the kidney. Increased PTH secretion from any cause will lead to a decreased tubular reabsorption of phosphate (increased urine phosphate and decreased serum phosphate). The test is useful in distinguishing serum hypercalcemia that is a result of excess PTH production by the parathyroid glands from hypercalcemia due to other causes (e.g., bone disease).

The bicarbonate ion concentration may be calculated from the total CO2 and PCO2 blood levels by using which of the following formulas? A. 0.03 x (PCO2 - total CO2) B. (total CO2 + 0.03) x PCO2 C. 0.03 x (total CO2 - PO2) D. total CO2 - (0.03 x PCO2)

D. The solubility coefficient of CO? gas (dissolved CO2) in normal blood plasma at 37°C is 0.03 mmol/L/mm Hg. The concentration of dissolved CO2 found in plasma is calculated by multiplying the PCO2 blood level by the solubility coefficient (0.03). The predominant components of total CO2 are bicarbonate (95%) and carbonic acid (5%). The bicarbonate ion concentration in millimoles per liter can be calculated by subtracting the product of (0.03 mmol/L/mm Hg x PCO2 mm Hg), which represents carbonic acid, from the total CO2 concentration (millimoles per liter).

Proteins, carbohydrates, and lipids are the three major biochemical compounds of human metabolism. What is the element that distinguishes proteins from carbohydrate and lipid compounds? A. Carbon B. Hydrogen C. Oxygen D. Nitrogen

D. The three major biochemical compounds that exert primary roles in human intermediary metabolism are proteins, carbohydrates, and lipids. The presence of nitrogen in all protein compounds distinguishes proteins from carbohydrates and lipids. Protein compounds contain approximately 16% nitrogen. Although there are only 20 common a-amino acids that are found in all proteins and a total of 40 known amino acids, a protein compound may contain from fifty to thousands of amino acids. The uniqueness of any protein is dictated by the number, type, and sequencing of the a-amino acids that compose it. The a-amino acids are linked to each other through peptide bonds. A peptide bond is formed through the linkage of the amino group of one amino acid to the carboxyl group of another amino acid.

Enzymes that catalyze the transfer of groups between compounds are classified as belonging to which enzyme class? A. Hydrolases B. Eyases C. Oxidoreductases D. Transferases

D. There are six major classes of enzymes. The International Commission of Enzymes of the International Union of Biochemistry has categorized all enzymes into one of these classes: oxidoreductases, transferases, hydrolases, lyases, isomerases, and ligases. Transferases are enzymes that catalyze the transfer of groups, such as amino and phosphate groups, between compounds. Transferases frequently need coenzymes, such as pyridoxal-5'-phosphate (P-5-P), for the amino transfer reactions. Aspartate and alanine aminotransferases, creatine kinase, and gammaglutamyltransferase are typical examples.

Which of the following is false about cardiac troponin I (cTnl) as it relates to AMI? A. Increase above reference interval seen in 3 to 6 hours B. Measure initially and serially in 3- to 6-hour intervals C. Remains elevated 5 to 10 days D. Expressed in regenerating and diseased skeletal muscle and cardiac muscle disorders

D. Troponin is a group of three proteins that function in muscle contraction by binding to the thin filaments of cardiac and skeletal striated muscle. The three proteins are known as troponin T (TnT), troponin I (Tnl), and troponin C (TnC). With AMI, the cardiac-specific isoforms of troponin are released into the blood; the two of clinical interest are cTnl and cTnT. Cardiac troponin I (cTnl) will show an increase that exceeds the reference interval in approximately 3-6 hours following an AMI. Quantification should be done serially starting with an initial measurement at presentation followed by testing at 3-6 hours, 6-9 hours, and 12-24 hours. cTnl will remain elevated for 5-10 days. Unlike cTnT, which is expressed in small quantities in regenerating and diseased skeletal muscle, cTnl is not, which makes it specific for cardiac muscle.

When mixed with phosphotungstic acid, what compound causes the reduction of the former to a tungsten blue complex? A. Urea B. Ammonia C. Creatinine D. Uric acid

D. Uric acid may be quantified by reacting it with phosphotungstic acid reagent in alkalinesolution. In this reaction, uric acid is oxidized to allantoin and the phosphotungstic acid is reduced, forming a tungsten blue complex. The intensity of the tungsten blue complex is proportional to the concentration of uric acid in the specimen.

When performing spectrophotometer quality assurance checks, what is the holmium oxide glass filter used to assess? A. Linearity B. Stray light C. Absorbance accuracy D. Wavelength accuracy

D. Wavelength calibration of a spectrophotometer is performed to verify that the radiant energy emitted from the monochromator through the exit slit is the same as the wavelength selector indicates. The glass filters holmium oxide, used in the UV and visible ranges, and didymium, used in the visible and near IR regions, are employed to check wavelength accuracy. Solutions of stable chromogens such as nickel sulfate may be used. Source lamps may be replaced with mercury-vapor or deuterium lamps. These lamps have strong emission lines and provide the most accurate method of wavelength calibration.

When measuring CK-MB, which of the following would provide the most sensitive method? A. Electrophoretic B. Colorimetric C. Kinetic D. Mass immunoassay

D. When measuring CK-MB, the mass immunoassay is more sensitive because it is quantifying the amount of enzyme present. This is in contrast to a kinetic method, which measures enzyme activity by means of the enzyme catalyzing a reaction and the product of that reaction being measured. Electrophoretic methods also measure enzyme activity based on colored product or fluorescent product formation.

Which of the following is not a type of support media used for serum protein electrophoresis? A. Agarose gel B. Cellulose acetate C. Acrylamide D. Celite

D. When serum is applied to a support medium placed in a buffer solution of alkaline pH and subjected to an electrical field, the serum proteins will be separated into fractions for identification and quantification. Support media that may be used for electrophoretic separations include agarose gel, starch gel, cellulose acetate, and acrylamide. The pore size of the agarose gel and cellulose acetate is large enough that the protein molecules are able to move freely through the media with a resolution of between five to seven fractions. Because the pore size of starch gel and acrylamide is somewhat smaller, the resolution of approximately 20 fractions is possible with this type of medium. Agarose gel and cellulose acetate are the more commonly used media in the routine clinical laboratory. Celite provides the inert supporting phase in gas-liquid chromatography.

Given the following information, calculate the plasma osmolality in milliosmoles per kilogram: sodium—142 mmol/L; glucose—130 mg/dL; urea nitrogen— 18mg/dL. A. 290 B. 291 C. 295 D. 298

D. When the osmolality has been both measured in the laboratory and calculated, the osmolal gap may then be determined by subtracting the calculated osmolality from the measured. Plasma osmolality may be calculated when the plasma sodium, glucose, and urea nitrogen values are known. The equation for calculating osmolality expresses Na+, glucose, and urea nitrogen in mmol/L (SI units). To convert glucose and urea nitrogen from mg/dL to mmol/L, the conversion factors 0.056 and 0.36 are used, respectively. For sodium, the factor 2 is used to count the cation (sodium) once and its corresponding anion once. Because glucose and urea nitrogen are undissociated molecules, they are each counted once. Use the following equation: Calculated osmolality (mOsm/kg) = 2.0 Na+(mmol/L) + Glucose (mmol/L) + Urea nitrogen (mmol/L) = 2.0 (142 mmol/L) + (0.056 X 130 mg/dL) + (0.36 X 18 mg/dL) 284 + 7.3 + 6.5 = 298 mOsm/kg

What does hydrolysis of sucrose yield? A. Glucose only B. Galactose and glucose C. Maltose and glucose D. Fructose and glucose

D. When two monosaccharides condense with loss of a molecule of water, a disaccharide is formed. Disaccharides, therefore, can be hydrolyzed into two monosaccharides. The most important disaccharides are maltose, lactose, and sucrose. On hydrolysis, sucrose will yield one molecule of glucose and one molecule of fructose. Maltose can be hydrolyzed into two molecules of glucose. Lactose can be hydrolyzed into glucose and galactose.

Which of the following is an acute-phase reactant protein able to inhibit enzymatic proteolysis and having the highest concentration of any of the plasma proteolytic inhibitors? A. C-reactive protein B. Haptoglobin C. ct2-Macroglobulin D. cq-Antitrypsin

D. ap Antitrypsin is an acute-phase reactant protein whose concentration increases in response to inflammation, a i-Antitrypsin inhibits the selfdestruction of one's own tissue by forming inactive complexes with proteolytic enzymes. In this way the enzymes are inhibited, and tissue destruction through self-digestion is avoided. a\- Antitrypsin has been found to have the highest concentration in serum of any of the plasma proteolytic inhibitors. It is an effective inhibitor of the enzymes chymotrypsin, plasmin, thrombin, collagenase, and elastase. The primary effect of aj-antitrypsin may be seen in the respiratory tract and the closed spaces of the body where physiological pH values are maintained, a j-Antitrypsin is least effective in the stomach and intestines.

In which of the following disorders would the maternal serum level of aj-fetoprotein not be elevated? A. Neural tube defect B. Spinabifida C. Fetal distress D. Down syndrome

D. ctpFetoprotein, synthesized by the fetus, peaks at 13 weeks and declines at 34 weeks of gestation. When concern exists for the wellbeing of the fetus, maternal serum AFP is measured between 15 and 20 weeks of gestation. An increased AFP level in maternal serum is associated with such disorders as neural tube defects, spina bifida, and fetal distress. A decreased AFP level in maternal serum is characteristic of Down syndrome.

Which of the following is not associated with human chorionic gonadotropin? A. (3 subunit confers immunogenic specificity B. Used to confirm pregnancy C. Used as a tumor marker D. Found in hepatoma

D. hCG is a dimer consisting of alpha and beta polypeptide chains, with the (3 subunit conferring immunogenic specificity. Although hCG is more commonly associated with testing to confirm pregnancy, it is also associated with certain forms of cancer. (3-hCG is used as a tumor marker for hydatidiform mole, gestational choriocarcinoma, and placental-site trophoblastic tumor. hCG's utility also extends to monitoring the success of therapy in testicular and ovarian germ cell tumors. In addition, increased levels of hCG have been identified in hematopoietic malignancy, melanoma, gastrointestinal tract neoplasms, sarcoma, and lung, breast, and renal cancers.

Total CK (creatine kinase)

- ↑ in muscle, cardiac or brain damage - Higher reference ranges in males due to greater muscle mass and physical activity

Direct spectrophotometry

bilirubin testing for newborns only. -- due to lack of interfering compounds

Gamma-glutamyl transferase (GGT)

blood test to detect increased enzymes that can indicate cirrhosis, hepatitis, acute pancreatitis, acute cholecystitis, or nephrosis, and to test for Helicobacter pylori antibodies often increased after alcohol intake

glucagon, ACTH, GH, cortisol, Human placental lactogen, epinephrine, T3& T4

list the hormones that cause an increase in serum glucose

serum iron, TIBC, Nephelometry, ferritin

what are they ways to test Iron levels in the body?

BNP, pro-BNP

what test are used to access congestive heart failure

liver

where are proteins synthesized and catabolized?

Which of the following tests would be particularly useful in determining isopropanolexposure? A. Serum osmolality and urine acetone B. Urine osmolality and serum osmolality C. Urine acetone and urine osmolality D. Serum sodium and serum acetone

A. A significant fraction of absorbed isopropanol is metabolized to acetone and rapidly excreted in the urine. Because of isopropanol's relatively low molecular weight, exposure to this compound will in most cases significantly increase the patient's serum osmolality. Of course, other alcohols will have a similar effect. Urine osmolality exhibits a wide variability throughout the day and, therefore, would be of little use in determining isopropanol exposure. Serum sodium would be only secondarily affected by isopropanol exposure.

Which is the most predominant buffer system in the body? A. Bicarbonate/carbonic acid B. Acetate/acetic acid C. Phosphate/phosphorous acid D. Hemoglobin

A. Because of its high concentration in blood, the bicarbonate/carbonic acid pair is the most important buffer system in the blood. This buffer system is also effective in the lungs and in the kidneys in helping to regulate body pH. The other buffers that also function to help maintain body pH are the phosphate, protein, and hemoglobin buffer systems. The acetate buffer system is not used by the body to regulate pH.

Which of the following does not accurately describe direct bilirubin? A. Insoluble in water B. Conjugated in the liver C. Conjugated with glucuronic acid D. Excreted in the urine of jaundiced patients

A. Direct bilirubin was so named because of its ability in the van den Bergh method to react directly with diazotized sulfanilic acid without the addition of alcohol. Such a direct reaction is possible because direct bilirubin is conjugated in the liver with glucuronic acid, thereby making it a polar, water-soluble compound. Because conjugated bilirubin is both water soluble and not protein bound, it may be filtered through the glomerulus and excreted in the urine of jaundiced patients. Indirect bilirubin is a protein-bound unconjugated compound that is soluble in alcohol but not in water, and because of these properties, it is unable to be excreted in the urine.

Which globin chains compose hemoglobin Aj? A. Two alpha chains and two beta chains B. Two alpha chains and two delta chains C. Two alpha chains and two gamma chains D. Two beta chains and two delta chains

A. Hemoglobin is a tetramer composed of four globin chains, four heme groups, and four iron atoms. In adult hemoglobin, or hemoglobin AI, there are two alpha chains and two beta chains. Hemoglobin A2, which comprises less than 4% of the normal adult hemoglobin, is composed of two alpha chains and two delta chains. Hemoglobin F, or fetal hemoglobin, is composed of two alpha chains and two gamma chains.

A method calls for extracting an acidic drug from urine with an anion exchange column. The pKa of the drug is 6.5. Extraction is enhanced by adjusting the sample pH to: A. 8.5 B. 6.5 C. 5.5 D. 4.5

Correct Answer: A 7. A Extraction of a negatively charged drug onto an anion exchange (positively charged) column is optimal when more than 99% of the drug is in the form of anion. The extraction pH should be 2 pH units above the pKa of an acidic drug. When pH = pKa the drug will be 50% ionized, and when pH is greater than pKa the majority of drug is anionic.

Plate 7 shows the electrophoresis of hemoglobin (Hgb) samples performed on agarose gel, pH 8.8. The control sample is located in lanes 2 and 10 and contains Hgb A, S, and C. Which sample(s) are from neonates? A. Samples 1 and 5 B. Sample 3 C. Sample 7 D. Samples 8 and 9

Correct Answer: A 7. A Neonates and infants up to 6 months old have Hgb F levels between 8% and 40%. The Hgb F level falls to below 2% in children over 2 years old. Hgb F is more acidic than Hgb S, and less acidic than Hgb A. Therefore, at an alkaline pH, Hgb F has a greater net negative charge than Hgb S but a lesser net negative charge than Hgb A, and migrates between Hgb A and Hgb S.

What is the term that describes the sum of carbonic acid and bicarbonate in plasma? A. Total CO2 B. Standard bicarbonate C. Buffer base D. Base excess

A. The concentration of total CO2 (ctCO2) or carbon dioxide content is a measure of the concentration of bicarbonate, carbonate, carbamino compounds, carbonic acid, and dissolved carbon dioxide gas (PCO2) in the plasma. Bicarbonate makes up approximately 95% of the total CO2 content, but most laboratories are not equipped to directly measure bicarbonate. Therefore, total CO2 is generally quantified. The bicarbonate concentration may be estimated by subtracting the H2CO3 concentration (measured in terms of PCO2 and converted to H2CO3) from the total CO7 concentration.

Which of the following carbohydrates is a polysaccharide? A. Starch B. Sucrose C. Lactose D. Glucose

A. There are three major classifications of carbohydrates: monosaccharides, disaccharides, and polysaccharides. Starch is classified as a polysaccharide because its structure is composed of many molecules of glucose (a monosaccharide) condensed together. Monosaccharides (e.g., glucose) are carbohydrates with the general molecular formula Cn(H20)n, that cannot be broken down to simpler substances by acid hydrolysis. Disaccharides (e.g., sucrose, lactose) are condensation products of two molecules of monosaccharides with loss of one molecule of water.

Which of the following reagents is used in a colorimetric method to quantify the concentration of serum calcium? A. Cresolphthalein complexone B. Lanthanum C. Malachite green D. Amino-naphthol-sulfonic acid

A. Total serum calcium concentration is often determined by the spectrophotometric quantification of the color complex formed with cresolphthalein complexone. Magnesium will also form a color complex and, therefore, is removed by reacting the serum with 8-hydroxyquinoline. Calcium concentration is determined with the use of a variety of other reagents and most reliably by means of atomic absorption spectrophotometry.

A person has a fasting triglyceride level of 240 mg/dL. The physician wishes to know the patient's non-HDL cholesterol level. What cholesterol fractions should be measured? A. Total cholesterol and HDL cholesterol B. Total cholesterol and LDL cholesterol C. HDL cholesterol and LDL cholesterol D. Total cholesterol and chylomicrons

Correct Answer: A 70. A When the HDL cholesterol is subtracted from the total cholesterol, the result is called the non-HDL cholesterol. This result, the sum of LDL cholesterol and VLDL cholesterol, represents the fraction with atherogenic remnant lipoproteins as well as LDL cholesterol. People who have a fasting triglyceride ≥ 200 mg/dL are at increased risk for coronary artery disease owning to atherogenic VLDL remnants, and the treatment goal is to have a non-HDL cholesterol no more than 30 mg/dL greater than the LDL cholesterol.

In acute pancreatitis, a significant increase in which serum enzyme would be expected diagnostically? A. Creatine kinase B. Amylase C. Alkaline phosphatase D. Aspartate aminotransferase

B. Amylase and lipase are the two most important enzymes in evaluating pancreatic function. The values of amylase and lipase activity are significantly elevated in acute pancreatitis and obstruction of the pancreatic duct. In most cases of acute pancreatitis, the lipase activity stays elevated longer than amylase activity.

Which of the following conditions is associated with a high level of S-type amylase? A. Mumps B. Intestinal obstruction C. Alcoholic liver disease D. Peptic ulcers

Correct Answer: A 71. A Both salivary and pancreatic amylases designated S-type and P-type, respectively, are present in normal serum. High amylase occurs in mumps, ectopic pregnancy, biliary obstruction, peptic ulcers, alcoholism, malignancies, and other nonpancreatic diseases. Isoenzymes can be separated by electrophoresis (S-type is faster than P-type), but more commonly immunoinhibition of S-type amylase is used to rule out mumps, malignancy, and ectopic pregnancy, which give rise to high S-type amylase.

Which glucose method can employ a polarographic oxygen electrode? A. Hexokinase B. Glucose oxidase C. Glucose dehydrogenase D. o-Toluidine

B. Glucose in the presence of oxygen is oxidized to gluconic acid and hydrogen peroxide. This reaction is catalyzed by glucose oxidase. By using a polarographic oxygen electrode, the rate of oxygen consumption is measured and related to the concentration of glucose in the sample.

The measurement of oxygen in blood by means of a PO2 electrode involves which of the following? A. Wheatstone bridge arrangement of resistive elements sensitive to oxygen concentration B. Direct relationship between amount of oxygen in the sample and amount of current flowing in the measuring system C. Change in current resulting from an increase of free silver ions in solution D. Glass electrode sensitive to H+ ions

B. In a blood gas analyzer, the electrode for measuring the partial pressure of oxygen (POz) in the blood is an electrochemical cell consisting of a platinum cathode and a Ag/AgCl anode connected to an external voltage source. The cathode and anode are immersed in buffer. A polypropylene membrane selectively permeable to gases separates the buffer from the blood sample. When there is no oxygen diffusing into the buffer, there is practically no current flowing between the cathode and the anode because they are polarized. When oxygen diffuses into the buffer from a sample, it is reduced at the cathode. The electrons necessary for this reduction are produced at the anode. Hence a current flows; the current is directly proportional to the PO2 in the sample. The current flowing in the system is measured. Plotting the voltage change versus current change gives a polarogram. The voltage at which the sharp rise in current occurs is characteristic of the electrochemical reaction involved; that is, characteristic of the analyte. The amount of increase in current (i.e., the wave height) is proportional to the concentration of analyte. In anodic stripping voltammetry, a negative potential is applied to one of the electrodes. Trace metal ions in the solution are thereby reduced and plated onto the anodic electrode. This is a preconcentrating step. The plated electrode is then used as the anode in a polarographic cell. The metal is thereby stripped off the anode. The current flow during the stripping provides a polarogram that both identifies and quantifies the trace metals. The method is particularly appropriate for assaying heavy metals such as lead in blood.

In the urea method, the enzymatic action of urease is inhibited when blood for analysis is drawn in a tube containing what anticoagulant? A. Sodium heparin B. Sodium fluoride C. Sodium oxalate D. Ethylenediaminetetra-acetic acid

B. In addition to the fact that sodium fluoride is a weak anticoagulant, it also functions as an antiglycolytic agent and is used as a preservative for glucose in blood specimens. With the urease reagent systems for the quantification of urea, the use of sodium fluoride must be avoided because of its inhibitory effect on this system. Additionally, contamination from the use of ammonium oxalate and ammonium heparin must be avoided, because urease catalyzes the production of ammonium carbonate from urea. In several methods, the ammonium ion formed reacts proportionally to the amount of urea originally present in the sample. Anticoagulants containing ammonium would contribute falsely to the urea result.

Ion-exchange chromatography separates solutes in a sample based on the A. Solubility of the solutes B. Sign and magnitude of the ionic charge C. Adsorption ability of the solutes D. Molecular size

B. Ion-exchange chromatography uses synthetic ion-exchange resins. They may be cationor anion-exchange resins. They can be used in either a column or a thin layer. Separation of mixtures of substances by ion-exchange chromatography depends primarily on the sign and the ionic charge density of the substances being separated.

In an electrolytic cell, which of the following is the half-cell where reduction takes place? A. Anode B. Cathode C. Combination electrode D. Electrode response

B. Oxidation involves the loss of electrons, and reduction the gain of electrons. In an electrolytic cell composed of two different half-cells—for example, zinc in zinc sulfate and copper in copper sulfate—electrons will flow from the anode to the cathode. Thus reduction takes place at the cathode, whereas oxidation occurs at the anode. "Combination electrode" refers to the combining of indicator and reference electrodes into a single unit. "Electrode response" refers to the ability of an ion-selective electrode to respond to a change in concentration of the ion being measured by exhibiting a change in potential.

Which of the following has an effect on plasma calcium levels? A. Sodium B. Inorganic phosphate C. Potassium D. Iron

B. Plasma phosphates influence plasma calcium levels. Case studies show that there is a reciprocal relationship between calcium and phosphorus. A decrease in plasma calcium will be accompanied by an increase in plasma inorganic phosphate.

When using EMIT, the enzyme is coupled to A. Antibody B. Antigen C. Substrate D. Coenzyme

B. The components needed in EMIT include the free unlabeled drug (unlabeled antigen) in the serum specimen, antibody specific to the drug being quantified, enzyme-labeled drug (labeled antigen), and substrate and coenzyme specific for the enzyme. In this method, the enzyme is coupled to the drug, producing an enzyme-labeled drug also referred to as an enzyme-labeled antigen. This enzyme-labeled complex competes with free unlabeled drug in the serum sample for the binding sites on the antibody. EMIT therapeutic drug monitoring assays are available for a variety of drugs that are included in the categories of antimicrobial, antiepileptic, antiasthmatic, cardioactive, and antineoplastic drugs. The EMIT system is not limited only to drug assays but is also available for hormone testing.

Which of the following tissues does not secrete steroid hormones? A. Ovaries B. Pituitary gland C. Testes D. Adrenal cortex

B. The pituitary gland produces protein hormones such as adrenocorticotropic hormone, thyroid-stimulating hormone, follicle-stimulating hormone, growth hormone, and prolactin. Steroid hormones include €21 corticosteroids and progesterone, C^ androgens, and Cjg estrogens. The mineralo- and gluco-corticosteroids are secreted only by the adrenal glands, but the other steroids listed are secreted by the ovaries, testes, adrenal glands, and placenta to a varying extent, depending on the individual's sex.

For assessing carcinoma of the prostate, quantification of PSA has virtually replaced the measurement of which of the following enzymes? A. Alkaline phosphatase B. Acid phosphatase C. Alanine aminotransferase D. Trypsin

B. The quantification of serum prostate-specific antigen has replaced measurement of serum acid phosphatase for assessing carcinoma of the prostate. PSA measurement in conjunction with the digital rectal examination is recommended for prostate cancer screening. In addition, PSA can be used to stage and monitor therapy of prostatic cancer.

Free drug levels can generally be determined by analyzing what body fluid? A. Whole blood B. Ultrafiltrate of plasma C. Urine D. Protein-free filtrate of plasma

B. The term "free drug" refers to the fraction of drug in the plasma not bound to protein. For the determination of free drug concentrations, urine would not be the proper specimen because the rate of drug excretion depends mainly on conjugation or metabolism and not on protein binding. In preparation of a protein-free nitrate of plasma, the drugs bound to protein would also enter the filtrate because they are dissociated when the protein is denatured. Saliva is a form of plasma ultrafiltrate and with some restrictions as to sampling and type of drug analyzed can be used for free-drug monitoring. Methods for equilibrium dialysis and for preparation of ultrafiltrates of plasma are now available and can provide excellent samples for free-drug analyses of some compounds.

The term "lipid" encompasses a wide variety of compounds characterized as being insoluble in water but soluble in nonpolar solvents. Which of the following vitamins is not classified as fat soluble? A. Vitamin A B. Vitamin C C. Vitamin D D. Vitamin E

B. The term "lipid" encompasses a large group of compounds, including the sterols, fatty acids, triglycerides, phosphatides, bile pigments, waxes, and fat-soluble vitamins. Vitamins A, D, E, and K are classified as fat-soluble vitamins. Thiamine (Bj), riboflavin (B2), pyridoxine (B6), cyanocobalamin (B12), niacin, pantothenic acid, lipoic acid, folic acid, inositol, and ascorbic acid (C) are classified as water-soluble vitamins and as such are not lipid compounds.

When it is not possible to perform a creatinine assay on a fresh urine specimen, to what pH level should the urine be adjusted? A. 3.0 B. 5.0 C. 7.0 D. 9.0

C. Creatinine assays are preferably performed on fresh urine specimens. If an acid urine specimen is kept for a time, any creatine in the urine will be converted to creatinine. In alkaline urine, an equilibrium situation will occur between the creatine and creatinine present in the specimen. To avoid either of these situations, it is recommended that the urine be adjusted to pH 7.0 and that the specimen be frozen. It is thought that at a neutral pH, the integrity of the urine specimen will be maintained because it will require days or even weeks for equilibrium to occur between the two compounds.

Which of the following is an autoantibody that binds to TSH receptor sites on thyroid cell membranes, preventing thyroidstimulating hormone from binding? A. Antithyroglobulin antibodies B. Thyroid antimicrosomal antibodies C. Thyrotropin-receptor antibodies D. Antithyroid peroxidase antibodies

C. Currently, the suggested term for autoantibodies that bind to TSH receptor sites is thyrotropin-receptor antibodies (TRAb). The thyrotropin-receptor antibodies (TRAb) are thyroid- stimulating immunoglobulins (TSI) that are IgG autoantibodies and are able to bind to the thyroid-stimulating hormone (TSH) receptor sites on thyroid cell membranes, thus preventing TSH from binding. These autoantibodies interact with the receptors similarly to TSH, thus stimulating the thyroid to secrete thyroid hormones. Because these autoantibodies do not respond to the negative feedback system as does TSH, hyperthyroidism is the end result. The majority of patients with Graves hyperthyroid disease exhibit high liters of TRAb.

When do the highest levels of gonadotropins occur? A. During the follicular phase of the menstrual cycle B. During the luteal phase of the menstrual cycle C. At the midpoint of the menstrual cycle D. Several days prior to ovulation

C. During the menstrual cycle, folliclestimulating hormone (FSH) levels decrease in the later part of the follicular stage. Luteinizing hormone (LH) gradually increases during the follicular stage. At midcycle, both FSH and LH levels spike. Following this spike, in the luteal stage or second half of the menstrual cycle, FSH and LH levels gradually decrease. In postmenopausal women the ovaries stop secreting estrogens. In response the gonadotropins, FSH and LH, rise to their highest levels. The reason is the feedback system between estrogen secretion by the gonads and the secretion of releasing factors by the hypothalamus; a decreased estrogen level causes increased secretion of FSHreleasing factor and LH-releasing factor.

Which of the following characteristics will a protein have at its isoelectric point? A. Net negative charge B. Net positive charge C. Net zero charge D. Mobility

C. Proteins are dipolar or zwitterion compounds because they contain amino acids that exhibit both negative and positive charges. The isoelectric point (pi) of a protein refers to the pH at which the number of positive charges on the protein molecule equals the number of negative charges, causing the protein to have a net charge of zero. Because the protein exhibits electrical neutrality at its isoelectric point, it is unable to migrate in an electrical field.

When pituitary adenoma is the cause of decreased estrogen production, an increase of which hormone is most frequently responsible? A. Prolactin B. FSH C. LH D. Thyroid-stimulating hormone (TSH)

Correct Answer: A 8. A Prolactinoma can result in anovulation because high levels of prolactin suppress release of LHRH (gonadotropin-releasing hormone), causing suppression of growth hormone (GH), FSH, and estrogen. Prolactinoma is the most commonly occurring pituitary tumor accounting for 40%-60%. Adenomas producing FSH have a frequency of about 20%, while those pituitary tumors secreting LH and TSH are rare.

What is the normal renal threshold of sodium (measured in millimoles per liter)? A. 80-85 B. 90-110 C. 110-130 D. 135-148

C. Sodium is the principal cation found in the plasma. The normal serum sodium level is 136-145 mmol/L, whereas in urine the sodium concentration ranges between 40 and 220 mmol/ day, being dependent on dietary intake. Because sodium is a threshold substance, it is normally excreted in the urine when the serum sodium concentration exceeds 110-130 mmol/L. When serum levels fall below 110 mmol/L, all the sodium in the glomerular filtrate is virtually reabsorbed in the proximal and distal tubules. This reabsorption process is influenced by the hormone aldosterone.

To achieve the best levels of sensitivity and specificity, to what type of detector system could a gas chromatograph be coupled? A. UV spectrophotometer B. Bichromatic spectrophotometer C. Mass spectrometer D. Fluorescence detector

C. The column and carrier gas flow rate used in gas-liquid chromatography are important aspects of the separation and resolving power of the system. When the column eluent is introduced into a mass spectrometer, additional information pertaining to elemental composition, position of functional groups, and molecular weight may be determined for the puipose of identifying compounds (e.g., drugs in biological samples). Mass spectrometers consist of a vacuum system, ion source, mass filter, and detector.

Which of the following is not quantified in the triple test for Down syndrome? A. cq-Fetoprotein B. Unconjugated estriol C. Progesterone D. Human chorionic gonadotropin

C. The triple test for Down syndrome includes quantification of ctpfetoprotein (AFP), unconjugated estriol (uE3), and human chorionic gonadotropin (hCG) in the maternal serum. These measurements should be done between 16 and 18 weeks gestation, and they are useful in detecting neural tube defects and Down syndrome. In Down syndrome, the AFP and uE3 levels are low, whereas the hCG level is elevated. These test results are related to gestational age and are expressed as a multiple of the median (MoM), meaning the maternal serum result is divided by the median result of the corresponding gestational population.

Respiratory acidosis is described as a(n): A. increase in CO2 content and PCO2 with a decreased pH B. decrease in CO2 content with an increased pH C. increase in CO2 content with an increased pH D. decrease in CO2 content and PCO2 with a decreased pH

Correct Answer: A

The most specific method for the assay of glucose utilizes: A. hexokinase B. glucose oxidase C. glucose-6-phosphatase D. glucose dehydrogenase

Correct Answer: A

Zinc protoporphyrin or free erythrocyte protoporphyrin measurements are useful to assess blood concentrations: A. lead B. mercury C. arsenic D. berryllium

Correct Answer: A

A trend in QC results is most likely caused by: A. Deterioration of the reagent B. Miscalibration of the instrument C. Improper dilution of standards D. Electronic noise

Correct Answer: A 23. A A trend occurs when six or more consecutive quality control results either increase or decrease in the same direction; however, this is not cause for rejection until a multirule is broken. Trends are systematic errors (affecting accuracy) linked to an unstable reagent, calibrator, or instrument condition. For example, loss of volatile acid from a reagent causes a steady pH increase, preventing separation of analyte from protein. This results in lower QC results each day.

Which of the following assays has the poorest precision? Analyte; Mean (mmol/L); Standard Deviation A. Ca; 2.5; 0.3 B. K; 4.0; 0.4 C. Na; 140; 4.0 D. Cl; 100; 2.5

Correct Answer: A 26. A Although calcium has the lowest s, it represents the assay with poorest precision. Relative precision between different analytes or different levels of the same analyte must be evaluated by the coefficient of variation (CV) because standard deviation is dependent upon the mean. CV = s × 100/Mean. This normalizes standard deviation to a mean of 100. The CV for calcium in the example is 12.0%.

Orders for uric acid are legitimate stat requests because: A. Levels above 10 mg/dL cause urinary tract calculi B. Uric acid is hepatotoxic C. High levels induce aplastic anemia D. High levels cause joint pain

Correct Answer: A 34. A Uric acid calculi form quickly when the serum uric acid level reaches 10 mg/dL. They are translucent compact stones that often lodge in the ureters, causing postrenal failure.

What component of a freezing point osmometer measures the sample temperature? A. Thermistor B. Thermocouple C. Capacitor D. Electrode

Correct Answer: A 50. A A thermistor is a temperature-sensitive resistor. The resistance to current flow increases as temperature falls. The temperature at which a solution freezes can be determined by measuring the resistance of the thermistor. Resistance is directly proportional to the osmolality of the sample.

In which liver disease is the DeRitis ratio (ALT:AST) usually greater than 1.0? A. Acute hepatitis B. Chronic hepatitis C. Hepatic cirrhosis D. Hepatic carcinoma

Correct Answer: A 50. A ALT prevails over AST in hepatitis; however, AST is greater than ALT in carcinoma, alcoholic liver disease, and cirrhosis of the liver.

Which measurement principle is employed in a vapor pressure osmometer? A. Seebeck B. Peltier C. Hayden D. Darlington

Correct Answer: A 52. A The Seebeck effect refers to the increase in voltage across the two junctions of a thermocouple caused by a difference in the temperature at the junctions. Increasing osmolality lowers the dew point of a sample. When sample is cooled to its dew point, the voltage change across the thermocouple is directly proportional to osmolality.

Which statement regarding bone-specific ALP is true? A. The bone isoenzyme can be measured immunochemically B. Bone ALP is increased in bone resorption C. Bone ALP is used for the diagnosis of osteoporosis D. There are two distinct bone isoenzymes

Correct Answer: A 59. A Bone ALP assays (Ostase and Alkphase-B) use monoclonal antibodies to measure the bone isoenzyme in mass units. The assays may be used to monitor bone remodeling by osteoblasts in osteoporosis, and thus, are useful for following treatment. Bone specific ALP is not sufficiently sensitive to diagnose osteoporosis, and antibodies may cross-react with other ALP isoenzymes, depending on their source.

Which of the following statements is true? A. Apoenzyme + prosthetic group = holoenzyme B. A coenzyme is an inorganic molecule required for activity C. Cofactors are as tightly bound to the enzyme as prosthetic groups D. All enzymes have optimal activity at pH 7.00

Correct Answer: A 6. A A coenzyme is an organic molecule required for full enzyme activity. A prosthetic group is a coenzyme that is tightly bound to the apoenzyme and is required for activity. Cofactors are inorganic atoms or molecules needed for full catalytic activity. Pyridoxyl- 5 ́-phosphate is a prosthetic group for ALT and AST. Consequently, patients with low levels of pyridoxal- 5 ́-phosphate (P-5 ́-P) (vitamin B6 deficiency) may have reduced transaminase activity in vitro. Enzymes can have diverse pH (and temperature) optimas.

Hyperalbuminemia is caused by: A. Dehydration syndromes B. Liver disease C. Burns D. Gastroenteropathy

Correct Answer: A 6. A A high serum albumin level is caused only by dehydration or administration of albumin. Liver disease, burns, gastroenteropathy, nephrosis, starvation, and malignancy cause hypoalbuminemia.

Which of the following reagents is used in the direct HDL cholesterol method? A. Sulfated cyclodextrin B. Magnesium sulfate and dextran sulfate C. Anti-apoA-I D. Manganese heparin

Correct Answer: A 65. A The direct HDL cholesterol method most commonly employed uses cholesterol esterase and oxidase enzymes conjugated to polyethylene glycol. In the presence of sulfated cyclodextrin, the enzymes do not react with non-HDL cholesterol molecules. Anti-apoA-I binds to HDL and is not used in HDL assays.

A condition in which erythrocyte protoporphyrin is increased is: A. acute intermittent porphyria B. iron deficiency anemia C. porphyria cutanea tarda D. acute porphyric attack

Correct Answer: B

Lipoprotein (a), or Lp(a), is significant when elevated in serum because it: A. Is an independent risk factor for atherosclerosis B. Blocks the clearance of VLDLs C. Displaces apo-AI from HDLs D. Is linked closely to a gene for obesity

Correct Answer: A 67. A Lp(a) is a complex of apo-B100 and protein (a) formed by a disulfide bridge. The complex is structurally similar to plasminogen and is thought to promote coronary heart disease by interfering with the normal fibrinolytic process. Lp(a) is measured by immunoassay; however, the measurement will vary depending on the type of antibodies used and their epitope specificity.

What component is used in a GC-MS but not used in an LC-MS? A. Electron source B. Mass filter C. Detector D. Vacuum

Correct Answer: A 67. A The mass spectrometer requires a sample that is suspended in a gas phase, and therefore, the sample from a GC can be directly injected into the mass spectrometer. While chemical ionization of the sample is possible, most GC-MS instruments utilize electron ionization. Electrons are produced by applying 70 electron volts to a filament of tungsten or rhenium under vacuum. The electrons collide with the neutral molecules coming from the GC, splitting them into fragments. The array of fragments is a unique identifier of each molecule.

A fasting serum sample form an asymptomatic 43-year-old woman is examined visually and chemically with the following results: -initial appearance of serum: milky -appearance of serum after overnight refrigeration: cream layer over turbid serum -triglyceride level: 2,000 mg/dL (22.6 mmol/L) -cholesterol level: 550 mg/dL (14.25 mmol/L) This sample contains predominantly: A. chylomicrons, alone B. chylomicrons and very low-density lipoproteins (VLDL) C. very low-density lipoproteins (VLDL) and low-density lipoproteins D. high-density lipoproteins (HDL)

Correct Answer: B

A patient has the following thyroid profile: -total T4: decreased -free T4: decreased -thyroid peroxidase antibody: positive -TSH: decreased This patient most probably has: A. hyperthyroidism B. hypothyroidism C. a normal thyroid D. Graves disease

Correct Answer: B

A salicylate level is performed to detect toxicity caused by ingestion of excess: A. acetaminphen B. aspirin C. ibuprofen D. pseudoephedrine

Correct Answer: B

A serum sample demonstrates an elevated result when tested with the Jaffe reaction. This indicates; A. prolonged hypothermia B. renal functional impairment C. pregnancy D. arrhythmia

Correct Answer: B

A serum sample was assayed bilirubin at 10 am, and the result was 12 mg/dL (205.6 umol/L). The same sample was retested at 3 pm. The result now is 8 mg/dL (136.8 umol/L). The most likely explanation for this discrepancy is; A. the reagent has deteriorated B. the sample was exposed to light C. a calculation error in the first assay D. the sample was not refrigerated

Correct Answer: B

A 9-month-old boy from Israel has gradually lost the ability to sit up, and develops seizures. he has an increased amount of a phospholipid called GM2-ganglioside in his neurons, and he lacks the enzyme hexosaminidase A in his leukocytes. These findings suggest: A. Neimann-Pick disease B. Tay-Sachs disease C. phenylketonuria D. Hurler syndrome

Correct Answer: B

A breakdown product of hemoglobin is: A. lipoprotein B. bilirubin C. hematoxylin D. Bence Jones protein

Correct Answer: B

Calcium concentration in the serum is regulated by: A. insulin B. parathyroid hormone C. thyroxine D. vitamin C

Correct Answer: B

The presence of which of the following isoenzymes indicates acute myocardial damage? A. CKMM B. CKMB C. CKBB D. none

Correct Answer: B

Which of the following enzymes catalyzes the conversion of starch to glucose and maltose? A. malate dehydrogenase (MD) B. amylase (AMS) C. creatine kinase (CK) D. isocritic dehydrogenase (ICD)

Correct Answer: B

Amino transferase enzymes catalyze the: A. exchange of amino groups and sulfhydryl groups between alpha-amino and sulfur-containing acids B. exchange of amino and keto groups between alpha-amino and alpha-keto acids C. hydrolysis of amino acids and keto acids D. reversible transfer of hydrogen form amino acids to coenzyme

Correct Answer: B

Electrophoretic movement of proteins toward the anode will decrease by increasing the: A. Buffer pH B. Ionic strength of the buffer C. Current D. Voltage

Correct Answer: B 12. B Electrophoresis is the migration of charged molecules in an electric field. Increasing the strength of the field by increasing voltage (or current) increases migration. However, increasing ionic strength decreases the migration of proteins. Counterions (cations) in the buffer move with the proteins, reducing their electromagnetic attraction for the anode.

As part of a hyperlipidemia screening program, the following resutls were obtianed on a 25-year-old woman 6 hours after eating: -triglycerides: 260 mg/dL (2.86 mmol/L) -cholesterol: 120 mg/dL (3.12 mmol/L) Which of the following is the best interpretation of these results? A. both results are normal, and not affected by the recent meal B. cholesterol is normal, but triglycerides are elevated, which may be attributed to the recent meal C. both results are elevated, indicating a metabolic problem in addition to the nonfasting state D. both results are below normal despite the recent meal, indicating a metabolic problem

Correct Answer: B

Aspartate aminotransferase (AST) and alanine aminotransferase (ALT) are both elevated in which of the following diseases? A. muscular dystrophy B. viral hepatitis C. pulmonary emboli D. infectious monoculeosis

Correct Answer: B

Assays for free T4 measure hormone not bound to thyroxine-binding prealbumin, thyroxine-binding globulin and: A. thyrotopin-releasing hormone B. albuminr C. free T3 D. thyroid-stimulating hormone

Correct Answer: B

A chromatogram for blood alcohol (GC) gives broad trailing peaks and increased retention times for ethanol and internal standard. This is most likely caused by: A. A contaminated injection syringe B. Water contamination of the column packing C. Carrier gas flow rate that is too fast D. Oven temperature that is too high

Correct Answer: B 17. B Increased oven temperature or gas flow rate will shorten retention times and decrease peak widths. Syringe contamination may cause the appearance of ghost peaks. Water in a PEG column such as Carbowax used for measuring volatiles causes longer retention times and loss of resolution.

Below are the results of a protein electrophoresis: See BOC pg 94 Pic 2 These results are consistent with a(n): A. normal serum protein pattern B. normal CSF protein pattern C. abnormal serum protein pattern D. abnormal CSF protein pattern

Correct Answer: B

Cerebrospinal fluid for glucose assay should be: A. refrigerated B. analyzed immediately C. heated to 56C D. stored at room temperature after centrifugation

Correct Answer: B

Clinical assays for tumor markers are most important for: A. screening for he presence of cancer B. monitoring the curse of a known cancer C. confirming the absence of disease D. identifying patients at risk for cancer

Correct Answer: B

Decreased serum iron associate with increased TIBC is compatible with which of the following disease states? A. anemia of chronic infection B. iron deficiency anemia C. chronic liver disease D. nephrosis

Correct Answer: B

Detection of carriers of hereditary coproporphyria should include analysis of: A. 24 hour urine for porphobilinogen B. fresh morning urine for delta-aminolevulinic acid C. erythrocyte protoporphyrin D. 24 hour urine for porphyrin

Correct Answer: B

During a normal pregnancy, quantitative human chorionic gonadotropin (HCG) levels peak how many weeks after the las menstrual period? A. 2-4 B. 8-10 C. 14-16 D. 18-20

Correct Answer: B

Estrogen and progesterone receptor assays are useful in identifying patients who are likely to benefit form endocrine therapy to treat which of the following? A. ovarian cancer B. breast cancer C. endometriosis D. amenorrhea

Correct Answer: B

Free therapeutic drug levels are usually higher when serum protein concentrations are below normal. In which of the following conditions would this most likely occur? A. acute inflammation B. nephrotic syndrome C. pregnancy D. mulitiple myeloma

Correct Answer: B

Given the following results; -alkaline phosphatase: slight increase -aspartate aminotransferase: slight increase -alanine aminotransferase: slight increase -gamma-glutamyl transferase: slight increase This is most consistent with: A. acute hepatitis B. chronic hepatitis C. obstructive jaundice D. liver hemangioma

Correct Answer: B

How is primary hypocortisolism (Addison disease) differentiated from secondary hypocortisolism (of pituitary origin)? A. adrenal corticotropic hormone (ACTH) is decreased in pirmary and elevated in secondary B. adrenal corticotropic hormone (ACTH) is elevated is primary and decreased in secondary C. low aldosterone and hypoglycemia present with secondary hypocortisolism D. normal cortisol levels and blood pressure with primary hypocortisolism

Correct Answer: B

If a fasting glucose was 90 mg/dL, which of the following 2-hour postprandial glucose results would most closely represent normal glucose metabolism? A. 25 mg/dL (1.4 mmol/L) B. 50 mg/dL (2.3 mmol/L) C. 180 mg/dL (5.5 mmol/L) D. 260 mg/dL (14.3 mmol/L)

Correct Answer: B

If the total bilirubin is 3.1 mg/dL ( 53.0 umol/L) and the conjugated bilirubin is 2.0 mg/dL (34.2 umol/L), the unconjugated bilirubin is: A. 0.5 mg/dL( 8.6 umol/L) B. 1.1 mg/dL ( 18.8 umol/L) C. 2.2 mg/dL (37.6 umol/L) D. 5.1 mg/dL (87.2 umol/L)

Correct Answer: B

Refer to the following illustration: See BOC pg 106 Pic 1 The class of phospholipid surfactants represented by the dotted line on the amniotic fluid analysis shown above is though to originate in what fetal organ system? A. cardiovascular B. pulmonary C. hepatic D. placental

Correct Answer: B

Urinary HVA is most often assayed to detect: A. Pheochromocytoma B. Neuroblastoma C. Adrenal medullary carcinoma D. Psychiatric disorders such as manic depression

Correct Answer: B 38. B HVA is the major metabolite of dopa, and urinary HVA is elevated in more than 75% of neuroblastoma patients. Neuroblastomas also usually produce VMA from norepinephrine. Thus, HVA and VMA are assayed together and this increases the sensitivity of detection to around 90%.

Sodium determination by indirect ion selective electrode is falsely decreased by A. elevated chloride levels B. elevated lipid levels C. decreased protein levels D. decreased albumin levels

Correct Answer: B

Specimen for blood gas determination should be drawn into a syringe containing: A. no preservative B. heparin C. EDTA D. oxalate

Correct Answer: B

TSH is produced by the: A. hypothalamus B. pituitary gland C. adrenal cortex D. thyroid

Correct Answer: B

Technical problems encountered during the collection of an amniotic fluid specimen caused doubt as to whether the specimen was amniotic in origin. Which 1 of the following procedures would best establish that the fluid is amniotic in origin? A. measurement of absorbance at 450 nm B. creatinine measurement C. lecithin/sphingomyelin ration D. human amniotic placental lactogen (HPL)

Correct Answer: B

The creatinine clearance (mL/min) is equal to: A. urinary creatinine (mg/L)/[volume of urine (mL/min) x plasma creatinine (mg/L)] B. [urinary creatinine (mg/L) x volume (mL/min)] / plasma creatinine (mg/L) C. urinary creatinine (mg/L) / [volume of urine (mL / hour) x plasma creatinine (mg/L) D. [urinary creatinine (mg/L) x volume (mL/hour)] / plasma creatinine (mg/L)

Correct Answer: B

A blood creatinine value of 5.0 mg/dL (442.0 umol/L) is most likely to be found with which of the following blood values? A. osmolality: 292 mOsm/kg B. uric acid: 8 mg/dL (475.8 umol/L) C. urea nitrogen: 80 mg/dL (28.56 mmol/L) D. ammonia: 80 ug/dL (44 umol/L)

Correct Answer: C

An elevated serum iron with normal iron biding capacity is most likely associate with: A. iron deficiency anemia B. renal damage C. pernicious anemia D. septicemia

Correct Answer: C

In the Jendrassik-Grof reaction for total bilirubin, bilirubin reacts with diazotized sulfanilic acid to form: A. diazo bilirubin B. biliverdin C. azobilirubin D. bilirubin glucuronide

Correct Answer: C

The illustration below represents a Lineweaver-Burk plot of 1/v vs 1/[S] is an enzyme reactions and the following assumptions should be made: See BOC pg. 126 pic 2 -The enzyme concentration was the same for reactions A and B -The enzyme concentration was in excess for reactions A and B -Reaction A occurred under ideal conditions Which of the following statements about reaction B is true? A. it illustrates noncompetitive inhibition B. it illustrates competitive inhibition C. it illustrates neither competitive nor noncompetitive inhibition D. is could be the result of heavy metal conamination

Correct Answer: B

The biuret reaction for the analysis of serum protein depends on the number of: A. free amino groups B. free carboxyl groups C. peptide bonds D. tyrosine residues

Correct Answer: C

Turbidity in serum suggests elevation of: A. cholesterol B. total protein C. chylomicrons D. albumin

Correct Answer: C

Which of the following chemical determinations may be of help in establishing the presence of seminal fluid? A. lactic dehydrogenase (LD) B. isocitrate dehydrogenase (ICD) C. acid phophatase D. alkaline phosphatase

Correct Answer: C

The major action of angiotensin II is: A. increased pituitary secretion of vasopressin B. increased vasoconstriction C. increased parathromone secretion by the parathyroid D. decreased adrenal secretion fo aldosterone

Correct Answer: B

The most heat labile fraction of alkaline phosphatase is obtained form A. liver B. bone C. intestine D. placenta

Correct Answer: B

The recommended initial thyroid function test for either a healthy, asymptomatic patient or a patient with symptoms which may be related to a thyroid disorder is: A. free thyroxine (free T4) B. thyroid-stimulating hormone (TSH) C. total thyroxine (T4) D. triiodothyronine (T3)

Correct Answer: B

The substance that is measured to estimate the serum concentration of triglycerides by most methods is: A. phopholipids B. glycerol C. fatty acids D. pre-beta lipoprotein

Correct Answer: B

The troponin complex consists of: A. troponin T, calcium an d tropomyosin B. troponin C, troponin I and troponin T C. troponin I, actin, and tropomyosin D. troponin C, myoglobin, and actin

Correct Answer: B

Upon development of a thin-layer chromatogram for drug analysis all drug spots (including the standards) had migrate with the solvent front. The most probable cause for this would be: A. environmental temperature too warm B. incorrect aqueous to nonaqueous solvent mixture C. too much sample applied D. chromatogram dried to quickly

Correct Answer: B

What specimen preparation is commonly used to perform the alkaline phosphatase isoenzyme determination? A. serum is divided into 2 aliquots, one is frozen and the other is refrigerated B. serum is divided into 2 aliquots, one is heated at 56C and the other is unheated C. no preparation is necessary since the assay uses EDTA plasma D. protein-free filtrate is prepared first

Correct Answer: B

Which of the following elevates carboxyhemoglobin? A. nitrite poisoning B. exposure to carbon monoxide C. sulfa drug toxicity D. sickle cell anemia

Correct Answer: B

Which of the following enzyme substrates is the most specific for prostatic acid phosphatase for quantitative endpoint reactions? A. p-nitrophenylphosphate B. thymolphthalein monphosphate C. beta-naphthol-phosphate D. beta-glycerophosphate

Correct Answer: B

Which of the following enzymes of heme biosynthesis is inhibitied by lead? A. aminolevulinate synthase B. porphobilinogen synthase C. uroporphyrinogen synthase D. bilirubin synthetase

Correct Answer: B

Which of the following is an example of a peptide bond? See BOC pg 94 Pic 1 A. A B. B C. C D. D

Correct Answer: B

Which of the following lipid results would be expected to be falsely elevated on a serum specimen form a nonfasting patient? A. cholesterol B. triglyceride C. HDL D. LDL

Correct Answer: B

Which of the following methods for quantitation of high-density lipoprotein is most suited for clinical laboratory use? A. Gomori procedure B. homogeneous C. column chromatography D. agarose gel electrophoresis

Correct Answer: B

Which instrument requires a primary and secondary monochromator? A. Spectrophotometer B. Atomic absorption spectrophotometer C. Fluorometer D. Nephelometer

Correct Answer: C 22. C A fluorometer uses a primary monochromator to isolate the wavelength for excitation, and a secondary monochromator to isolate the wavelength emitted by the fluorochrome.

Which of the following statements regarding total CK is true? A. Levels are unaffected by strenuous exercise B. Levels are unaffected by repeated intramuscular injections C. Highest levels are seen in Duchenne's muscular dystrophy D. The enzyme is highly specific for heart injury

Correct Answer: C 23. C Total CK is neither sensitive nor specific for AMI. An infarct can occur without causing an elevated total CK. Exercise and intramuscular injections cause a significant increase in total CK. Crush injuries and muscular dystrophy can increase the total CK up to 50 times the URL.

Which of the following represents the Henderson-Hasselbalch equation as applied to blood pH? - A. pH = 6.1 + log HCO3-/PCO2 B. pH = 6.1 + log HCO3 /(0.03 × PCO2) C. pH = 6.1 + log dCO2/HCO3- D. pH = 6.1 + log (0.03 × PCO2)/HCO3-

Correct Answer: B 1. B The Henderson-Hasselbalch equation describes the pH of a buffer comprised of a weak acid and its salt. pH = pKa + log salt/acid, where pKa is the negative logarithm of the dissociation constant of the acid. In this case, the salt is sodium bicarbonate and the acid is the dissolved CO2, which is equal to 0.03 (mmol/L per mm Hg) x PCO2. The pKa includes both the hydration and dissociation constant for dissolved CO2 in blood, 6.1 and is termed pK ́.

Referring to the Levy-Jennings chart, what is the first day in the month when the run should be rejected and patient results should be repeated? A. Day 6 B. Day 7 C. Day 8 D. Day 9

Correct Answer: C 33. C Although the trend is apparent across QC levels by day 7, the patient results would not be rejected until day 8 when the 41s rule is broken. An advantage to plotting control data is that trends can be identified before results are out of control and patient data must be rejected. In this case, corrective steps should have been implemented by day 7 to avoid the delay and expense associated with having to repeat the analysis of patient samples.

Thyroid hormones are derived from the amino acid: A. Phenylalanine B. Methionine C. Tyrosine D. Histidine

Correct Answer: C 39. C Thyroid hormones are derived from the enzymatic modification of tyrosine residues on thyroglobulin. Tyrosine is halogenated enzymatically with iodine, forming monoiodotyrosine (MIT) and diiodotyrosine (DIT). Enzymatic coupling of these residues form T3 (3,5,3 ́-triiodothyronine) and T4 (3,5,3 ́,5 ́-tetraiodothyronine). These are hydrolyzed from thyroglobulin, forming active hormones.

What is the pH of a 0.05 M solution of acetic acid? Ka = 1.75 × 10-5, pKa = 4.76 A. 1.7 B. 3.0 C. 4.3 D. 4.6

Correct Answer: B 11. B Weak acids are not completely ionized, and pH must be calculated from the dissociation constant of the acid (in this case 1.75 × 10-5). Ka = [H+] × [Ac-] / [HAc] 1.75 x 10-5 = [H+] × [Ac-] / 5.0 x 10-2 Since [H+] = [Ac-] X2 = (1.75 × 10-5 ) × (5.0 × 10-2 ) = 8.75 × 10-7 x = √8.75 × 10-7 = [H+] = 9.35 × 10-4 M pH = -Log 9.35 × 10-4 M = 3.0 Alternatively, pH = 1/2 (pKa - Log HA) pH = 1/2 (4.76 - Log 5.0 × 10-2) = 1/2 (4.76 + 1.30) = 3.0

Which set of results is most likely in a female with hypogonadotropic ovarian failure? A. Increased LH, FSH, and estrogen B. Decreased LH, FSH, and estrogen C. Decreased prolactin and estrogen D. Increased LH and FSH, and decreased estrogen

Correct Answer: B 12. B Hypogonadotropic ovarian failure is the result of pituitary dysfunction. It may be caused by low levels of both LH and FSH, or it may be caused by high levels of prolactin as in prolactinoma, since prolactin will inhibit LHRH and result in low LH and FSH.

Which isoenzyme of ALP migrates farthest toward the anode when electrophoresed at pH 8.6? A. Placental B. Bone C. Liver D. Intestinal

Correct Answer: C 58. C Liver ALP isoenzymes migrate farthest toward the anode, but fast and slow variants occur. The slow liver ALP band is difficult to distinguish from placental and bone ALP. The order from cathode to anode is: - Renal→Intestinal→Bone→Placental→Liver + Improved separation of bone and liver isoenzymes can be achieved by incubating the serum with neuraminidase prior to electrophoresis. The enzyme reduces the sialic content of the bone isoenzyme, causing it to migrate at a slower rate.

Which of the following diseases is caused by a deficiency of sphingomyelinase? A. Gaucher disease B. Fabry disease C. Niemann-Pick disease D. Tay-Sachs disease

Correct Answer: C 58. C The diseases mentioned result from inborn errors of lipid metabolism (lipidoses) caused by deficiency of an enzyme needed for lipid degradation. Specific lipids accumulate in the lysosomes. Niemann-Pick disease results from a deficiency of sphingomyelinase; Gaucher disease from β-glucocerebrosidase; Fabry disease (sex linked) from α-galactosidase A; and Tay-Sachs from N acetylglucosaminidase A.

Which of the following enzymes is usually depressed in liver disease? A. Elastase-1 B. GLD C. Pseudocholinesterase D. Aldolase

Correct Answer: C 78. C Pseudocholinesterase is found mainly in the liver and functions to hydrolyze acetylcholine. It is depressed by organophosphate insecticides and drugs that function as cholinesterase inhibitors and the serum assay is used to presumptively identify cases of insecticide poisoning. Levels of pseudocholinesterase are decreased in patients with liver disease as a result of depressed synthesis. In cirrhosis and hepatoma, there is a 50%-70% reduction in serum level and a 30%-50% reduction in hepatitis. Elastase-1 is a pancreatic digestive enzyme that breaks down connective tissue protein. Its level in feces is reduced in persons with pancreatic insufficiency. GLD is increased in necrotic jaundice, and aldolase in necrotic jaundice and muscle disease.

A 3-year-old child was evaluated for abdominal pain and anorexia by a physician. A CBC revealed a hemoglobin of 9.8 g/dL (98 g/L) and basophilic stippling of the RBCs. The doctor should order further tests to check for poisoning form: A. arsenic B. iron C. mercury D. lead

Correct Answer: D

A person suspected of having metabolic alkalosis would have which of the following laboratory findings? A. CO2 content and PO2 elevated, pH decreased B. CO2 content decreased and pH elevated C. CO2 content, PCO2, and pH decreased D. CO2 content and pH elevated

Correct Answer: D

Carcinoembryonic antigen (CEA) is most likely to be produced in a malignancy involving the: A. brain B. testes C. bone D. colon

Correct Answer: D

In the liver, bilirubin is converted to: A. urobilinogen B. urobilin C. bilirubin-albumin complex D. bilirubin diglucuronide

Correct Answer: D

Pellarga is associated with deficiency of which of the following vitamins? A. A B. B1 C. thiamine D. niacin

Correct Answer: D

The first step in analyzing a 24-hour urine specimen for quantitative urine protein is: A. subculture the urine for bacteria B. add the appropriate preservative C. screen for albumin using a dipstick D. measure the total volume

Correct Answer: D

Which of the following is the formula for calculating absorbance give the percent transmittance (%T) of a solution? A. 1- log (%T) B. log (% T)-2 C. 2 x log (%T) D. 2-log (%T)

Correct Answer: D

The normal difference between alveolar and arterial PO2 (PAO2-PaO2 difference) is: A. 3 mm Hg B. 10 mm Hg C. 40 mm Hg D. 50 mm Hg

Correct Answer: B 12. B The PAO2-PaO2 difference results from the low ratio of ventilation to perfusion in the base of the lungs. The hemoglobin in the blood coming from the base of the lung has a lower O2 saturation. This blood will take up O2 from the plasma of blood leaving well-ventilated areas of the lung, thus lowering the mixed arterial PO2.

Which could account for drug toxicity following a normally prescribed dose? A. Decreased renal clearance caused by kidney disease B. Discontinuance or administration of another drug C. Altered serum protein binding caused by disease D. All of these options

Correct Answer: D 17. D Therapeutic drug monitoring is necessary for drugs that have a narrow therapeutic index. Individual differences alter pharmacokinetics, causing lack of correlation between dose and drug blood level. These include age, diet, ingestion with or without food, genetic factors, exercise, smoking, pregnancy, metabolism of other drugs, protein binding, and disease states.

Which product is measured in the coupling step of the urease-UV method for BUN? A. CO2 B. Dinitrophenylhydrazine C. Diphenylcarbazone D. NAD+

Correct Answer: D 22. D In the urease-UV method, urease is used to hydrolyze urea, forming CO2 and ammonia. Glutamate dehydrogenase catalyzes the oxidation of NADH, forming glutamate from 2-oxoglutarate and ammonia. The glutamate dehydrogenase reaction is used for measuring both BUN and ammonia.

SITUATION: Results of an iron profile are: -Serum Fe = 40 μg/dL -TIBC = 400 μg/dL -ferritin = 50 μg/L All of the following tests are useful in establishing a diagnosis of Fe deficiency except: A. Protein electrophoresis B. Erythrocyte zinc protoporphyrin C. Serum transferrin D. Hgb electrophoresis

Correct Answer: D 25. D Electrophoresis may show an elevated β-globulin (transferrin) characteristic of iron deficiency, or inflammation that would help explain a normal ferritin. Zinc protoporphyrin is elevated in iron deficiency and in lead poisoning. Hemoglobinopathies and thalassemias are not associated with iron deficiency.

Which aminoaciduria results in the overflow of branched chain amino acids? A. Hartnup's disease B. Alkaptonuria C. Homocystinuria D. Maple syrup urine disease

Correct Answer: D 25. D Valine, leucine, and isoleucine accumulate due to branched-chain decarboxylase deficiency in maple syrup urine disease. These are transaminated to ketoacids that are excreted, giving urine a maple sugar odor. Alkaptonuria is caused by homogentisic acid oxidase deficiency, causing homogentisic aciduria. Homocystinuria is a no-threshold-type aminoaciduria that usually results from cystathionine synthase deficiency.

Which statement regarding nephelometry is true? A. Nephelometry is less sensitive than absorption spectrophotometry B. Nephelometry follows Beer's law C. The optical design is identical to a turbidimeter except that a HeNe laser light source is used D. The detector response is directly proportional to concentration

Correct Answer: D 27. D In nephelometry, the detector output is proportional to concentration (as opposed to turbidimetry where the detector is behind the cuvette). The detector(s) is (are) usually placed at an angle between 25° and 90° to the incident light, depending upon the application. Nephelometers, like fluorometers, are calibrated to read zero with the light path blocked, and sensitivity can be increased up to 1,000 times by amplification of the detector output or increasing the photomultiplier tube dynode voltage.

SITUATION: A sample for ammonia assay is taken from an IV line that had been capped and injected with lithium heparin (called a heparin lock). The sample is drawn in a syringe containing lithium heparin, and immediately capped and iced. The plasma is separated and analyzed within 20 minutes of collection, and the result is 50 μg/dL higher than one measured 4 hours before. What is the most likely explanation of these results? A. Significantly greater physiological variation is seen with patients having systemic, hepatic, and gastrointestinal diseases B. The syringe was contaminated with ammonia C. One of the two samples was collected from the wrong patient D. Stasis of blood in the line caused increased ammonia

Correct Answer: D 31. D Falsely elevated blood ammonia levels are commonly caused by improper specimen collection. Venous stasis and prolonged storage cause peripheral deamination of amino acids, causing a falsely high ammonia level. Plasma is the sample of choice since ammonia levels increase with storage. Lithium heparin and EDTA are acceptable anticoagulants; the anticoagulant used should be tested to make sure it is free of ammonia. A vacuum tube can be used if filled completely. Serum may be used provided the tube is iced immediately, and the serum is separated as soon as the sample clots. The patient should be fasting and must not have smoked for 8 hours because tobacco smoke can double the plasma ammonia level.

Which of the following trace elements is considered an essential micronutrient? A. Thallium B. Aluminum C. Mercury D. Selenium

Correct Answer: D 37. D Trace elements can be divided into two categories, those that have no known biological purpose and those that do. The former include thallium, mercury, lead, cadmium, and aluminum. All others can be considered essential, including arsenic that has been shown necessary for normal methionine metabolism. Most trace elements are of medical importance because excessive levels lead to toxicity. However, a deficiency of trace elements such as selenium, zinc, and copper are commonly caused by total parenteral nutrition and are medically important.

Which statement about enzymes is true? A. An enzyme alters the Gibb's free energy of the reaction B. Enzymes cause a reaction with a positive free energy to occur spontaneously C. An enzyme's natural substrate has the highest Km D. A competitive inhibitor will alter the apparent Km of the reaction

Correct Answer: D 4. D Enzymes alter the energy of activation by forming a metastable intermediate, the enzyme substrate complex. Enzymes do not alter the free energy or direction of a reaction. Competitive inhibitors bind to the active site where the enzyme binds substrate and are overcome by increasing the substrate concentration.

Which of the following conditions is associated with an increase in ionized calcium (Cai) in the blood? A. Alkalosis B. Hypoparathyroidism C. Hyperalbuminemia D. Malignancy

Correct Answer: D 42. D Increased Cai occurs in hyperparathyroidism, malignancy, and acidosis. Cai is elevated in primary hyperparathyroidism due to resorption of calcium from bone. Many nonparathyroid malignancies create products called parathyroid hormone-related proteins that stimulate the parathyroid receptors of cells. Acidosis alters the equilibrium between bound and free calcium, favoring ionization. Hyperalbuminemia increases the total calcium by increasing the protein- bound fraction, but does not affect the Cai.

Which of the following conditions will increase total T4 by increasing TBG? A. Acute illness B. Anabolic steroid use C. Nephrotic syndrome D. Pregnancy or estrogens

Correct Answer: D 42. D Pregnancy and estrogens are the most common cause of increased TBG. Other causes include hepatitis, morphine, and clofibrate therapy. Acute illness, anabolic steroids, and nephrotic syndrome decrease the level of TBG. Normal pregnancy causes an elevated serum total T4. Suitable assays are available that estimate free T4 and T3 and these should be used instead of total hormone assays.

A new test for prostate cancer is found to have a sensitivity of 80.0% and a specificity of 84.0%. If the prevalence of prostate cancer is 4.0% in men over 42 years old, what is the predictive value of a positive test result (PV+) in this group? A. 96.0% B. 86.0% C. 32.4% D. 17.2%

Correct Answer: D 48. D The predictive value of a positive test (PV+) is defined as the percentage of persons with a positive test result who will have the disease or condition. It is dependent upon the sensitivity of the test and the prevalence of the disease in the population tested. PV+ is calculated by multiplying the true positives by 100, then dividing by the sum of true positives and false positives. % PV+ = TP × 100 (TP + FP) where TP equals (sensitivity × prevalence) and FP equals (1 - specificity) × (1 - prevalence) = 0.80 × 0.04 × 100 / (0.80 × 0.04) + [(1 - 0.84) × (1 - 0.04)] = 0.032 × 100 / 0.032 + (0.96 × 0.16) = 17.2%

Which of the following conditions is most likely to cause a falsely high creatinine clearance result? A. The patient uses the midstream void procedure when collecting his or her urine B. The patient adds tap water to the urine container because he or she forgets to save one of the urine samples C. The patient does not empty his or her bladder at the conclusion of the test D. The patient empties his or her bladder at the start of the test and adds the urine to the collection

Correct Answer: D 5. D Urine in the bladder should be eliminated and not saved at the start of the test because it represents urine formed prior to the test period. The other conditions (choices A-C ) will result in falsely low urine creatinine or volume and, therefore, falsely lower clearance results. Error is introduced by incomplete emptying of the bladder when short times are used to measure clearance. A 24-hour timed urine is the specimen of choice. When filtrate flow falls below 2 mL/min, error is introduced because tubular secretion of creatinine occurs. The patient must be kept well hydrated during the test to prevent this.

Which of the following statements is correct? A. Both HDL and LDL are homogenous B. There are several subfractions of LDL but not HDL C. There are several subfractions of HDL but not LDL D. There are several subfractions of both HDL and LDL

Correct Answer: D 52. D There are 7 subfractions of LDL and 10 subfractions of HDL. These are grouped into subclasses defined by their molecular sizes. In general, the small, dense LDL subclasses contain more oxidized LDL and are more atherogenic than the larger LDL molecules. The larger HDL subfractions comprising the HDL-3 subclass are associated with a lower risk of coronary artery disease.

Which of the following conditions is most likely to produce an elevated plasma potassium? A. Hypoparathyroidism B. Cushing's syndrome C. Diarrhea D. Digitalis overdose

Correct Answer: D 58. D Digitalis toxicity causes potassium to leave the cells and enter the extracellular fluid, resulting in hyperkalemia. Renal failure, hemolytic anemia and Addison's disease are other frequent causes of hyperkalemia. Hypoparathyroidism indirectly causes hypokalemia by inducing alkalosis via increased renal retention of phosphate and bicarbonate. Cushing's syndrome (adrenal cortical hyperfunction) results in low potassium and elevated sodium. Diarrhea causes loss of sodium and potassium.

A serum ALP level greater than twice the elevation of GGT suggests: A. Misidentification of the specimen B. Focal intrahepatic obstruction C. Acute alcoholic hepatitis D. Bone disease or malignancy

Correct Answer: D 65. D In obstructive jaundice, GGT is elevated more than ALP. A disproportionate increase in ALP points to a nonhepatic source of ALP, often bone disease. GGT is the most sensitive marker of acute alcoholic hepatitis, rising about fivefold higher than ALP or transaminases.

SITUATION: A plasma sample from a person in a coma as a result of an automobile accident gave the following results: -Total CK 480 IU/L -Myoglobin 800 μg/L -CK-MB 8 μg/L -Troponin I 0.02 μg/L What is the best interpretation of these results? A. The person had a heart attack that caused the accident B. The accident caused traumatic injury, but no heart attack occurred C. A heart attack occurred in addition to a stroke D. It is not possible to tell whether a heart attack occurred because of the extensive trauma

Correct Answer: B 14. B The automobile accident caused both brain damage (coma) and muscle damage (myoglobin). The sandwich assay for MB uses antibodies to both the M and B subunits of CK-MB and therefore, is not subject to interference from CK-BB that could have resulted from brain injury. The CK relative index is 1.6, which is lower than would be expected if the CK-MB were derived from heart damage. Since the TnI is within normal limits, the slight increase in CK-MB is due to the gross release of CK from skeletal muscle.

Which of the following methods would yield reliable quantification of ethanol in the presence of isopropanol? A. Reaction with permanganate and chromotropic acid B. Conway diffusion followed by dichromate reaction C. Alcohol dehydrogenase reaction D. Gas-liquid chromatography

D. Gas-liquid chromatography (GLC) is one of the few methods that can quantify ethanol reliably in the presence of isopropanol (2-propanol) or other alcohols. Examples of the analytical problems associated with quantifying alcohols are as follows: Isopropanol significantly cross-reacts (6%) in the widely used alcohol dehydrogenase (ADH) method for ethanol; other alcohols will cross-react with dichromate methods for ethanol; and other alcohols will cross-react with the permanganate- chromotropic acid method, which is sometimes used for the identification of methanol. Because GLC is not generally available in stat laboratories, for patients with suspected exposure to alcohols other than ethanol, a variety of other laboratory and clinical findings are often used.

Which statement best describes the level of GH in patients with pituitary adenoma associated with acromegaly? A. The fasting GH level is always elevated at least twofold B. Some patients will require a glucose suppression test to establish a diagnosis C. A normal fasting GH level rules out acromegaly D. Patients produce a lower concentration of insulin-like growth factor I (IGF-1) than expected from their GH level

Correct Answer: B 15. B Approximately 90% of patients with acromegaly will have an elevated fasting GH level, but 10% will not. In addition, a single measurement is not sufficient to establish a diagnosis of acromegaly because various metabolic and nutritional factors can cause an elevated serum GH in the absence of pituitary disease. The glucose suppression test is used to diagnose acromegaly. An oral dose of 100 g of glucose will suppress the serum GH level at 1 hour (postadministration) to below 1 μg/L in normal patients, but not in patients with acromegaly. Patients with acromegaly also have high levels of IGF-1, also called somatomedin C, which is overproduced by the liver in response to excess release of GH.

SITUATION: A patient has the following electrolyte results: -Na = 130 mmol/L -Cl = 105 mmol/L -K = 4.8 mmol/L -TCO2 = 26 mmol/L Assuming acceptable QC, select the best course of action. A. Report these results B. Check the albumin, total protein, Ca, P, and Mg results; if normal, repeat the sodium test C. Request a new sample D. Recalibrate and repeat the potassium test

Correct Answer: B 15. B The anion gap of this sample is < 4 mmol/L. This may result from laboratory error, retention of an unmeasured cation (e.g., calcium), or low level of unmeasured anion such as phosphorus or albumin. The sodium is inappropriately low for the chloride and bicarbonate and should be repeated if no biochemical cause is apparent.

According to American Diabetes Association criteria, which result is consistent with a diagnosis of impaired fasting glucose? A. 99 mg/dL B. 117 mg/dL C. 126 mg/dL D. 135 mg/dL

Correct Answer: B 17. B Impaired fasting glucose is defined as a plasma glucose ≥100 but <126 mg/dL. A fasting glucose of 126 or higher on two consecutive occasions indicates diabetes. A fasting glucose of 99 mg/dL is considered normal.

SITUATION: An amylase result is 550 U/L. A 1:4 dilution of the specimen in NaCl gives 180 U/L (before mathematical correction for dilution). The dilution is repeated with the same results. The technologist should: A. Report the amylase as 550 U/L B. Report the amylase as 720 U/L C. Report the amylase as 900 U/L D. Dilute the sample 1:10 in distilled water and repeat

Correct Answer: B 18. B A 1:4 dilution refers to 1 part serum and 3 parts diluent; the result is multiplied by 4 to determine the serum concentration. Serum may contain wheat germ gluten or other natural amylase inhibitors that, when diluted, result in increased enzyme activity. Serum for amylase should always be diluted with normal saline because chloride ions are needed for amylase activity.

Which of the following may be a sampling source of error for an automated instrument? A. Short sample B. Air bubble in bottom of sample cup C. Fibrin clot in sample probe D. All the above

D. With automated instruments, the quality of the specimen and its handling are critical to producing accurate test results. Sampling errors can occur that cause falsely low results to be generated. These errors include short sampling, air pocket in the bottom of the sample cup, and fibrin clots in the sample probe.

Which of the following conditions is associated with hypophosphatemia? A. Rickets B. Multiple myeloma C. Renal failure D. Hypervitaminosis D

Correct Answer: A 44. A Rickets can result from dietary phosphate deficiency, vitamin D deficiency, or an inherited disorder of either vitamin D or phosphorus metabolism. Vitamin D-dependent rickets (VDDR) can be reversed by megadoses of vitamin D. Type 1 is caused by a deficiency in renal cells of 1-α-hydroxylase, an enzyme that converts 25 hydroxyvitamin D to the active form, 1,25 hydroxyvitamin D. Type 2 is caused by a deficiency in the vitamin D receptor of bone tissue. Vitamin D-resistant rickets (VDRR) is caused by a deficiency in the renal reabsorption of phosphate. Consequently, affected persons (usually men because it is most commonly X-linked) have a normal serum calcium and a low Pi

competitive inhibitors

bind to the active site of an enzyme, competing with the substrate

CK, AST, Aldolase

in muscular disorders such as muscular dystrophy there are increase in what markers?

AST, ALT, LD, ALP, 5'NT, GGT

what are the liver markers?

[(Na)+ (K)]-[(Cl)+(HCO3)]

which anion gap formula has a reference range of 10-20mM/L

(Na)-[(Cl)+ (HCO3)]

which anion gap formula has a reference range of 7-16mM/L?

Which of the following hormones initiates its response by binding to cytoplasmic receptors? A. Estradiol B. Epinephrine C. Growth hormone D. Follicle-stimulating hormone

A. At the cellular level, the site of action of the peptide and catecholamine hormones is different from that of the steroid and thyroid hormones. The peptide and catecholamine hormones bring about their effects by combining with receptors on or in the cell membranes of the target cells. In some cases, this binding to the membrane results in activation of adenylate cyclase, which sets in motion the so-called second-messenger mechanism of hormone action. On the other hand, steroid and thyroid hormones act predominantly by diffusing through the target cell membranes and combining with cytoplasmic or nucleic receptors to form a complex that then brings about the hormone's action.

Which of the following techniques is more commonly used to measure vitamins? A. High-performance liquid chromatography B. Spectrophotometry C. Nephelometry D. Microbiological

A. HPLC is a commonly used technique for the measurement of vitamins. Measurement by HPLC tends to be rapid, sensitive, and specific. Other techniques employed include spectrophotometric, fluorometric, and microbiological assays.

Which of the following is not associated with carcinoembryonic antigen? A. Increased levels seen with malignancies of the lungs B. Quantified by using capillary electrophoresis C. Used to monitor treatment of colon cancer D. Glycoprotein in nature

B. Carcinoembryonic antigen (CEA), a glycoprotein, is found in increased amounts in serum when malignant tumors of the colon, lung, pancreas, stomach, and breast are present. Care must be exercised in treating CEA as a diagnostic test, because elevated values are also seen in smokers, hepatitis patients, and patients with several other nonmalignant disorders. Clinically, CEA is more valuable in prognosis and treatment monitoring. Enzyme immunoassay and other types of immunoassays are available for the quantification of CEA.

Which of the following is not associated with growth hormone? A. Somatotropin B. Secreted by posterior pituitary C. Hypersecretion results in acromegaly D. Affects lipid, carbohydrate, and protein metabolism

B. Growth hormone (somatotropin) is a polypeptide secreted by the anterior pituitary. It is essential to the growth process of cartilage, bone, and a variety of soft tissues. It also plays an important role in lipid, carbohydrate, and protein metabolism of adults. During the growth phase of humans, hyposecretion of somatotropin results in dwarfism, whereas hypersecretion, conversely, causes pituitary gigantism. After the growth phase, hypersecretion of somatotropin causes acromegaly. Diagnosis of hypersecretion or hyposecretion of growth hormone usually requires the use of suppression or provocative tests of growth hormone release. Growth hormone levels may be quantified using immunoassay methods, including chemiluminescence immunoassay.

What is the anticoagulant of choice for blood gas analysis? A. EDTA B. Heparin C. Sodium fluoride D. Citrate

B. Heparin is the best anticoagulant to use in drawing blood for blood gas analyses because it does not affect the value of the blood pH. This is also critical to PO2 measurements because alterations in blood pH will cause concomitant changes in PO2 values. Several heparin salts are available for use as anticoagulants. Sodium heparinate, 1000 U/mL, is commonly used. Ammonium heparinate may be substituted for the sodium salt when it is necessary to perform additional testing, such as electrolyte analysis, on the blood gas sample.

Heroin is synthesized from what drug? A. Diazepam B. Morphine C. Ecgonine D. Chlorpromazine

B. Heroin (diacetylmorphine), an abused drug, is a derivative of morphine. The morphine used in its synthesis is generally obtained from opium. Although heroin itself is not pharmacologically active, it does have a rapid onset of action. It is converted quickly to 6-acetylmorphine and then hydrolyzed to morphine, both of which are pharmacologically active. So heroin abuse can be detected by measuring its metabolite morphine in the blood or urine.

The laboratory receives a request that assays for urinary aminolevulinic acid, porphobilinogen, uroporphyrin, and coproporphyrin are to be performed on a patient. Which of the following will not contribute to the integrity of the sample when these assays are performed on the same urine specimen? A. Refrigeration B. Addition of hydrochloric acid C. 24-hour urine collection D. Use of a brown bottle

B. When measurement of aminolevulinic acid, porphobilinogen, uroporphyrin, or coproporphyrin is requested, a 24-hour urine specimen should be collected. The urine should be refrigerated during collection and stored in a brown bottle to protect light-sensitive compounds. Because porphobilinogen is more stable under alkaline conditions and aminolevulinic acid is more stable under acid conditions, sodium bicarbonate should be added as a compromise to maintain the pH near 7.

What dye may be used for staining protein bands following electrophoresis? A. Fat red 7B B. Sudan black B C. Ponceau S D. Oil red O

C. Amido black 10B, Coomassie brilliant blue, and Ponceau S are dyes that are used to stain serum proteins after electrophoresis. Once the serum protein bands are stained, they may be quantified by scanning the support media at the appropriate wavelength with a densitometer. Oil red O and fat red 7B are dyes that are used to stain lipoproteins following electrophoresis.

Exogenous triglycerides are transported in the plasma in what form? A. Phospholipids B. Cholesteryl esters C. Chylomicrons D. Free fatty acids

C. Chylomicrons are protein-lipid complexes composed primarily of triglycerides and containing only small amounts of cholesterol, phospholipids, and protein. After food ingestion, the chylomicron complexes are formed in the epithelial cells of the intestines. From the epithelial cells, the chylomicrons are released into the lymphatic system, which transports chylomicrons to the blood. The chylomicrons may then carry the triglycerides to adipose tissue for storage, to organs for catabolism, or to the liver for incorporation of the triglycerides into very-low-density lipoproteins (VLDLs). Chylomicrons are normally cleared from plasma within 6 hours after a meal.

The bandpass of a spectrophotometer is 10 nm. If an instalment is set at 540 nm, the wavelengths that are permitted to impinge on the sample will be within what wavelength range? A. 530-540 nm B. 530-550 nm C. 535-545 nm D. 540-550 nm

C. The bandpass or bandwidth is the range of wavelengths that are passed by a monochromator. In the example given, the bandpass will permit a 10-nm range of wavelengths to pass through the monochromator and impinge on the sample solution in the cuvet. Thus, 540 ± 5 nm (10-nm bandpass) will be equivalent to a wavelength range of 535-545 nm.

What does an increase in the serum enzyme levels indicate? A. Decreased enzyme catabolism B. Accelerated enzyme production C. Tissue damage and necrosis D. Increased glomerular filtration rate

C. The majority of serum enzymes that are of interest clinically are of intracellular origin. These enzymes function intracellularly, with only small amounts found in serum as a result of normal cellular turnover. Increased serum levels are due to tissue damage and necrosis, where the cells disintegrate and leak their contents into the blood. Thus, elevated serum levels of intracellular enzymes are used diagnostically to assess tissue damage.

Which of the following techniques is based on electro-osmotic flow? A. Capillary electrophoresis B. Zone electrophoresis C. Iontophoresis D. Isoelectric focusing

A. Capillary electrophoresis is based on electroosmotic flow (EOF). When an electric field is applied, the flow of liquid is in the direction of the cathode. Thus, EOF regulates the speed at which solutes move through the capillary. Cations migrate the fastest, because EOF and electrophoretic attraction are in the direction of the cathode.

Which of the following is a spectrophotometric method for quantifying serum chloride? A. Ferric perchlorate B. Ammonium molybdate C. Bathophenanthroline D. Cresolphthalein complexone

A. Chloride can be quantified by the spectrophotometric ferric perchlorate method. The reagent reacts with chloride to form a colored complex. Other methods employed are the spectrophotometric mercuric thiocyanate method, the coulometric-amperometric titration method, and the ion-selective electrode method, which is employed by many automated analyzers.

Which of the following stimulates the production of singlet oxygen at the surface of the sensitizer particle in a luminescent oxygen channeling immunoassay? A. Radiant energy B. Heat energy C. Enzymatic reaction D. Fluorescent irradiation

A. In a luminescent oxygen channeling immunoassay the antigen links to two antibodycoated particles. The first is an antibody-coated sensitizer particle containing a photosensitive dye (singlet oxygen source), and the second is an antibody-coated particle (singlet oxygen receptor) containing a precursor chemiluminescent compound and a fluorophore. Radiant energy is used to irradiate the immunocomplex, which stimulates the production of singlet oxygen at the surface of the sensitizer particle. The singlet oxygen diffuses to the second particle being held in close proximity.

Which of the following is the major mineralocorticoid? A. Aldosterone B. Cortisol C. Corticosterone D. Testosterone

A. The corticosteroids, produced by the adrenal cortex, may be classified as glucocorticoids or mineralocorticoids. Cortisol is the primary glucocorticoid, and aldosterone is the primary mineralocorticoid. Aldosterone functions as a regulator of salt and water metabolism. Aldosterone promotes water retention and sodium resorption with potassium loss in the distal convoluted tubules of the kidney.

Which of the following drugs is used as a bronchodilator? A. Theophylline B. Phenytoin C. Amikacin D. Clozapine

A. Theophylline is a bronchodilator that is used to treat asthma. The therapeutic range is 10-20 |xg/mL, and use must be monitored to avoid toxicity. Use of theophylline has been replaced where possible with (3-adrenergic agonists, which are available in the inhaled form.

Identification of the urinary metabolite benzoylecgonine would be useful in determining exposure to which of the following drugs? A. Codeine B. Cocaine C. Amphetamine D. Propoxyphene

B. Cocaine is an abused drug and not available for therapeutic use. After absorption, cocaine in the blood is rapidly converted into ecgonine and benzoylecgonine. Because of the kidney's concentrating effect, examination of the urine for the metabolites is a sensitive method of determining exposure to cocaine.

In what way is the "normal" population reference interval for total cholesterol in America different from that of other clinical chemistry parameters (i.e., protein, sodium, BUN, creatinine, etc.)? A. Established units for total cholesterol are mg/dL; no other chemistry test has these units. B. Reference interval is artificially set to reflect good health even though Americans as a group have "normally" higher total cholesterol levels. C. Total cholesterol reference interval must be interpreted in line with triglyceride, phospholipid, and sphingolipid values. D. Total cholesterol reference interval is based on a manual procedure, whereas all other chemistry parameters are based on automated procedures.

B. Historically, total cholesterol levels of Americans have been below 300 mg/dL. Other countries, however, have relatively lower population cholesterol levels. The prevalent diet of these countries, however, may be vegetarian or fish, as opposed to meat, oriented. Higher total cholesterol resulting from a meat diet has been established. Clinical studies have also shown an increased risk of CAD in individuals with total cholesterol greater than 200 mg/dL. Thus, the upper reference interval of acceptable total cholesterol was artificially lowered to 200 mg/dL to reflect the lower risk of CAD associated with it.

Which of the following reagents can be used to measure protein in cerebrospinal fluid? A. Biuret B. Coomassie brilliant blue C. Ponceau S D. Bromcresol green

B. The concentration of total protein in cerebrospinal fluid (CSF) is 15^5 mg/dL. Such a low level of protein requires a method with sufficient sensitivity such as Coomassie brilliant blue. Turbidimetric methods can also be used to quantify protein in CSF. Neither biuret nor Ponceau S has the sensitivity needed, and bromcresol green measures only albumin and does not react with the globulins.

Which of the following is not characteristic of Dubin-Johnson syndrome? A. Impaired excretion of bilirubin into the bile B. Hepatic uptake of bilirubin is normal C. Inability to conjugate bilirubin D. Increased level of bilirubin in urine

C. In Dubin-Johnson syndrome, the transport of conjugated (direct) bilirubin from the microsomal region to the bile canaliculi is impaired. In this rare familial disorder, plasma conjugated bilirubin levels are increased because of defective excretion of bilirubin in the bile. Because conjugated bilirubin is water soluble, increased amounts of bilirubin are found in the urine.

Detection of which of the following substances is most useful to monitor the course of a patient with testicular cancer? A. alpha-fetoprotein B. carcinoembryonic antigen C. prolactin D. testosterone

Correct Answer: A

Factors that contribute to a PCO2 electrode requiring 60-120 seconds to reach equilibrium include the: A. difussion characteristics of the membrane B. actual blood PO2 C. type of calibrating standard (i.e. liquid or humidified gas) D. potential of the polarizing mercury cell

Correct Answer: A

In amniotic fluid, the procedure used to detect hemolytic disease of the newborn is: A. measurement of absorbance at 450 nm B. creatinine C. lecithin/sphingomyelin ratio D. estriol

Correct Answer: A

In regard to drug distribution patterns, which of the following statements is false? A. Drug metabolism is slower in newborns than adults. B. Drug metabolism is more rapid for 6-year-old children than for adults. C. Renal clearance of drugs is faster in newborns than adults. D. Drug metabolism often changes during pubescence.

C. Persons involved in therapeutic drug monitoring should consider not only the properties of the various drugs but also the populations to which they are administered. The neonate is particularly susceptible to drug toxicity because of renal and hepatic immaturity, which leads to an increased drug half-life in comparison with that seen in adults. The neonatal pattern of drug elimination is reversed rapidly several weeks after birth, and children generally metabolize drugs more rapidly than adults. With the onset of puberty, the rate of drug metabolism generally slows and approaches the adult rate of drug use.

Secondary hyperparathyroidism is often the result of A. Vitamin C deficiency B. Liver disease C. Renal disease D. Thyroid disease

C. Secondary hyperparathyroidism is a disorder that represents the response of a normally functioning parathyroid gland to chronic hypocalcemia. In most patients, the hypocalcemia is the result of renal disease or vitamin D deficiency. Vitamin D deficiency decreases intestinal calcium absoiption, resulting in hypocalcemia. The hypocalcemia resulting from renal disease is more complex. It can result either from the increased serum phosphate level caused by decreased glomerular filtration or from the decreased synthesis of 1,25-dihydroxy vitamin D3 in kidney disease.

The secretion of which of the following is controlled by growth hormone? A. Growth hormone-releasing hormone B. Corticotropin-releasing hormone C. Somatomedin D. Somatostatin

C. Somatomedins, insulin-like growth factors I and II, is the designation given to a family of small peptides whose formation in the liver is under the control of growth hormone. The somatomedins exhibit similar activity as insulin and are active in stimulating many aspects of cell growth, particularly that of cartilage. Blood levels of somatomedin have been determined by radioimmunoassay methods, and acromegalic adults have been shown to have significantly elevated levels in comparison with normal adults.

Measuring which of the following compounds is useful in the diagnosis of steatorrhea? A. Vitamin B12 B. Vitamin C C. Carotenoids D. Folic acid

C. The definitive test for the diagnosis of steatorrhea (fat malabsorption) is the fecal fat determination that usually is done with a 72-hour collection. Carotenoids are a group of fat-soluble compounds that are precursors of vitamin A (retinol). The carotenoids are not synthesized in humans, and their absorption depends on intestinal fat absoiption. Therefore, the serum carotene level is sometimes used as a simple screening test for steatorrhea. In addition to steatorrhea, other conditions, such as poor diet, liver disease, and high fever, can result in below-normal carotene levels. Folic acid and vitamins C and B12 are water soluble and would not be useful for determining fat absoiption.

Which of the following is useful in the detection and management of carcinoma of the prostate? A. total prostate-specific antigen B. prostatic acid phosphatase C. human chorionic gonadotropin D. alpha-fetaprotein

Correct Answer: A

In what condition would an increased level of serum albumin be expected? A. Malnutrition B. Acute inflammation C. Dehydration D. Renal disease

C. There are no physiological diseases that cause increased production of albumin by the liver. Elevated serum albumin is only associated with dehydration. It is a relative increase that will return to normal when fluids are administered to alleviate the dehydration. Disorders such as malnutrition, acute inflammation, and renal disease are characterized by decreased serum albumin levels.

A cardiac glycoside that is used in the treatment of congenital heart failure and arrhythmias by increasing the force and velocity of myocardial contraction is: A. digoxin B. acetaminophen C. lithium D. phenytoin

Correct Answer: A

Aspartate amino transferase (AST) is characteristically elevated in disease of the: A. liver B. kidney C. intestine D. pancreas

Correct Answer: A

In a pH meter reference electrodes may include: A. silver-silver chloride B. quihydrone C. hydroxide D. hydrogen

Correct Answer: A

The following results were obtained: -urine creatinine: 90 mg/dL (7956 umol/L) -serum creatinine: 0.90 mg/dL (79.6 umol/L) -patient's total body surface: 1.73 m^2 (average =1.7 m^2) -total urine volume in 24 hours: 1500 mL Given the above date, the patient's creatinine clearance, in mL/min is: A. 104 B. 124 C. 144 D. 150

Correct Answer: A

When myocardial infarction occurs, the first enzyme to become elevated is: A. CK B. LD C. AST D. ALT

Correct Answer: A

Which of the following tumor markers is classified as a tumor suppressor gene? A.BRCA-1 B. Carcinoembryonic antigen (CEA) C. Human chorionic gonadotropin (hCG) D. Nuclear matrix protein

Correct Answer: A 1. A Tumor markers may be enzymes, hormones, receptors, oncofetal (glycoprotein) antigens, or oncogenes. BRCA-1 is located on the long arm of chromosome 17 and carries an 85% lifetime risk of breast or ovarian cancer when present. Its product functions in DNA repair and slows cell proliferation.

Correction of pH for a patient with a body temperature of 38°C would require: A. Subtraction of 0.015 B. Subtraction of 0.01% C. Addition of 0.020 D. Subtraction of 0.020

Correct Answer: A 15. A The pH decreases by 0.015 for each degree Celsius above the 37°C. Because the blood gas analyzer measures pH at 37°C, the in vivo pH would be 0.015 pH units below the measured pH.

SITUATION: A patient's biochemistry results are: -ALT = 55 IU/L -glucose = 87 mg/dL -Na = 142 mmol/L -Ca = 8.4 mg/dL -AST = 165 IU/L -LD = 340 IU/L -K = 6.8 mmol/L -Pi = 7.2 mg/dL Select the best course of action. A. Report results along with an estimate of the degree of hemolysis B. Repeat LD but report all other results C. Request a new sample D. Dilute the serum 1:2 and repeat AST and LD

Correct Answer: A 19. A Results indicate a moderately hemolyzed sample. Because sodium, calcium, and glucose are not significantly affected, results should be reported along with an estimate of visible hemolysis. The physician may reorder affected tests of interest.

Which is normally the most abundant corticosteroid hormone secreted by the adrenal cortex? A. Cortisol B. Dehydroepiandrosterone C. Aldosterone D. Corticosterone

Correct Answer: A 20. A Cortisol is the most abundant adrenal hormone, and abnormal levels have pronounced effects on carbohydrate and lipid metabolism. Cortisol is a 21-carbon steroid with a dihydroxyacetone group at C17 and hydroxyl group at C11 that account for its glucocorticoid potency. Plasma and urinary cortisol measurements are used to diagnose most types of adrenocortical dysfunction. Dehydroepiandrosterone (DHEA), an adrenal androgen, is the next most abundant adrenal hormone. Aldosterone is the principal mineral corticoid made by the adrenals, and corticosterone is the immediate precursor to aldosterone. Both regulate salt balance.

Which of the following is the primary mechanism of compensation for metabolic acidosis? A. Hyperventilation B. Release of epinephrine C. Aldosterone release D. Bicarbonate excretion

Correct Answer: A 27. A In metabolic acidosis, the respiratory center is stimulated by chemoreceptors in the carotid sinus, causing hyperventilation. This results in increased release of CO2. Respiratory compensation begins almost immediately unless blocked by pulmonary disease or respiratory therapy. Hyperventilation can bring the PCO2 down to approximately 10-15 mm Hg.

Which of the following plots is best for detecting all types of QC errors? A. Levy-Jennings B. Tonks-Youden C. Cusum D. Linear regression

Correct Answer: A 28. A The Levy-Jennings plot is a graph of all QC results with concentration plotted on the y axis and run number on the x axis. The mean is at the center of the y axis, and concentrations corresponding to -2 and +2s are highlighted. Results are evaluated for multirule violations across both levels and runs. Corrective action for shifts and trends can be taken before QC rules are broken.

Which of the following conditions is classified as normochloremic acidosis? A. Diabetic ketoacidosis B. Chronic pulmonary obstruction C. Uremic acidosis D. Diarrhea

Correct Answer: A 31. A Bicarbonate deficit will lead to hyperchloremia unless the bicarbonate is replaced by an unmeasured anion. In diabetic ketoacidosis, acetoacetate and other ketoacids replace bicarbonate. The chloride remains normal or low and there is an increased anion gap.

Which PCO2 value would be seen in maximally compensated metabolic acidosis? A. 15 mm Hg B. 30 mm Hg C. 40 mm Hg D. 60 mm Hg

Correct Answer: A 32. A In metabolic acidosis, hyperventilation increases the ratio of bicarbonate to dissolved CO2. The extent of compensation is limited by the rate of both gas diffusion and diaphragm contraction. The lower limit is between 10 and 15 mm Hg PCO2, which is the maximum compensatory effect.

Which metabolite is most often increased in carcinoid tumors of the intestine? A. 5-Hydroxyindolacetic acid (5-HIAA) B. 3-Methoxy-4-hydroxyphenylglycol (MHPG) C. 3-Methoxydopamine D. HVA

Correct Answer: A 33. A 5-HIAA is a product of serotonin catabolism. Excess levels are found in urine of patients with carcinoid tumors composed of argentaffin cells. Carcinoid tumors are usually found in the intestine or lung, and produce serotonin and 5-hydroxytryptophan, a serotonin precursor. Serotonin is deaminated by monamine oxidase, forming 5-HIAA, and the 5-HIAA is excreted in the urine. Some carcinoid tumors produce mainly 5-hydroxytryptophan because they lack an enzyme needed to convert it to serotonin. The 5-hydroxytryptophan is converted by the kidneys to serotonin resulting in high urinary serotonin. Both 5-HIAA and serotonin are usually measured by HPLC with EDC.

Referring to the Levy-Jennings chart, what analytical error is present during the second half of the month? A. Shift B. Trend C. Random error D. Kurtosis

Correct Answer: A 34. A A shift is characterized by six consecutive points lying on the same side of the mean. This occurs from day 15 to day 20. Shifts are caused by a change in the assay conditions that affect the accuracy of all results, such as a change in the concentration of the calibrator; change in reagent; a new lot of reagent that differs in composition; or improper temperature setting, wavelength, or sample volume. The term kurtosis refers to the degree of flatness or sharpness in the peak of a set of values having a Gaussian distribution.

Which of the following factors is most likely to cause a falsely low result when using the BCG dye-binding assay for albumin? A. The presence of penicillin B. An incubation time of 120 seconds C. The presence of bilirubin D. Lipemia

Correct Answer: A 10. A BCG and BCP are not significantly affected by bilirubin or hemolysis, although negative interference caused by free Hgb has been reported with some BCG methods. Lipemic samples may cause positive interference, which can be eliminated by serum blanking. Incubation times as long as 2 minutes result in positive interference from globulins, which react with the dye. Penicillin and some other anionic drugs bind to albumin at the same site as the dye, causing falsely low results.

For drugs with first-order elimination, which statement about drug clearance is true? A. Clearance = elimination rate ÷ serum level B. It is most often performed by the liver C. It is directly related to half-life D. Clearance rate is independent of dose

Correct Answer: A 10. A First-order elimination represents a linear relationship between the amount of drug eliminated per hour and the blood level of drug. For drugs following linear kinetics, clearance equals the elimination rate divided by the drug concentration in blood. When clearance (in milligrams per hour) and f are known, the dose per hour needed to give a desired average drug level at steady state can be calculated. Clearance is inversely related to the drug's half-life and is accomplished mainly by the kidneys.

Which of the following laboratory results is consistent with primary hypoparathyroidism? A. Low calcium; high inorganic phosphorus Pi B. Low calcium; low Pi C. High calcium; high Pi D. High calcium; low Pi

Correct Answer: A 43. A Parathyroid hormone deficiency causes reduced resorption of calcium from bone, increased renal excretion of calcium, and decreased renal excretion of phosphorus. It is distinguished from other causes of hypocalcemia by Cai, which is reduced only by primary hypoparathyroidism and alkalosis.

Select the lipoprotein fraction that carries most of the endogenous triglycerides. A. VLDL B. LDL C. HDL D. Chylomicrons

Correct Answer: A 44. A The VLDL is formed in the liver largely from chylomicron remnants and hepatic-derived triglycerides. Therefore, the VLDL transports the majority of endogenous triglycerides, while the triglycerides of chylomicrons are derived entirely from dietary absorption.

Which of the following statements is true regarding reverse T3 (rT3)? A. Formed in the blood by degradation of T4 B. Physiologically active, but less than T3 C. Decreased in euthyroid sick syndrome D. Interferes with the measurement of serum T3

Correct Answer: A 46. A Reverse T3 is formed from the deiodination of T4 in the blood. It is an inactive isomer of T3, (3,3 ́,5 ́-triiodothyronine). Reverse T3 is increased in acute and chronic illness and is used to identify patients with euthyroid sick syndrome.

Which condition gives rise to the highest serum level of transaminases? A. Acute hepatitis B. Alcoholic cirrhosis C. Obstructive biliary disease D. Diffuse intrahepatic cholestasis

Correct Answer: A 49. A The transaminases usually reach 20-50 times the URL in acute viral and toxic hepatitis. Both transaminases are moderately increased (5-10 × URL) in infectious mononucleosis, toxic hepatitis, diffuse intrahepatic obstruction, lymphoma, and cancer of the liver, and slightly increased (2-5 × URL) in cirrhosis and extrahepatic obstruction.

In gas chromatography, the elution order of volatiles is usually based upon the: A. Boiling point B. Molecular size C. Carbon content D. Polarity

Correct Answer: A 60. A The order of elution is dependent upon the velocity of the analyte. Usually, the lower the boiling point of the compound, the greater its velocity or solubility in carrier gas.

Which of the following conditions is associated with hypernatremia? A. Diabetes insipidus B. Hypoaldosteronism C. Burns D. Diarrhea

Correct Answer: A 62. A Diabetes insipidus results from failure to produce ADH. Because the collecting tubules are impermeable to water in the absence of ADH, severe hypovolemia and dehydration result. Hypovolemia stimulates aldosterone release, causing sodium reabsorption, which worsens the hypernatremia. Burns, hypoaldosteronism, diarrhea, and diuretic therapy are common causes of hyponatremia.

Which electrolyte level best correlates with plasma osmolality? A. Sodium B. Chloride C. Bicarbonate D. Calcium

Correct Answer: A 69. A Sodium and chloride are the major extracellular ions. Chloride passively follows sodium, making sodium the principal determinant of plasma osmolality.

Blood PCO2 may be measured by: A. direct colorimetric measurement of dissolved CO2 B. a self-contained potentiometric electrode C. measurement of CO2-saturated hemoglobin D. measurement of CO2 consumed at the cathode

Correct Answer: B

During pregnancy, the form of estrogen measured in urine is mostly: A. estrodiol B. estriol C. estrone D. pregnanediol

Correct Answer: B

Kernicterus is an abnormal accumulation of bilirubin in: A. heart tissue B. brain tissue C. liver tissue D. kidney tissue

Correct Answer: B

Most automated blood gas analyzers directly measure: A. pH, HCO3 and % O2 saturation B. pH, PCO2 and PO2 C. HCO3, PCO2 and PO2 D. pH, PO2 and % O2 saturation

Correct Answer: B

Refer to the following illustration: See BOC pg. 86 The serum protein electrophorsis patter shown below was obtained on cellulose acetate at pH 8.6 Identify the serum protein fraction on the left of the illustration. A. gamma globulin B. albumin C. alpha-1 globulin D. alpha-2 globulin

Correct Answer: B

Scurvy is associated with deficiency of which fo the following vitamins? A. A B. C C. niacin D. thiamine

Correct Answer: B

Urea concentration is calculated from the BUN by multiplying by a factor of: A. 0.5 B. 2.14 C. 6.45 D. 14

Correct Answer: B 16. B BUN is multiplied by 2.14 to give the urea concentration in mg/dL. BUN (mg/dL) = urea × (% N in urea ÷ 100) Urea = BUN × 1/(% N in urea ÷100) Urea = BUN × (1/0.467) = 2.14

In peroxidase-coupled glucose methods, which reagent complexes with the chromogen? A. Nitroprusside B. Phenol C. Tartrate D. Hydroxide

Correct Answer: B 29. B The coupling step in the Trinder glucose oxidase method uses peroxidase to catalyze the oxidation of a dye by H2O2. Dyes such as 4-aminophenozone or 4-aminoantipyrine are coupled to phenol to form a quinoneimine dye that is red and is measured at about 500 nm.

In uncompensated metabolic acidosis, which of the following will be normal? A. Plasma bicarbonate B. PCO2 C. p50 D. Total CO2

Correct Answer: B 30. B The normal compensatory mechanism for metabolic acidosis is respiratory hyperventilation. In uncompensated cases, the PCO2 is not reduced, indicating a concomitant problem in respiratory control.

Examine the Levy-Jennings chart at the bottom of the previous page and identify the QC problem that occurred during the first half of the month. See Harr pg 229 A. Shift B. Trend C. Random error D. Kurtosis

Correct Answer: B 32. B A trend is characterized by six consecutive decreasing or increasing control results. The value for both controls becomes progressively higher from day 4 to day 9. Trends are caused by changes to the test system that increase over time, such as deterioration of reagents or calibrators, progressive changes in temperature, evaporation, light exposure, and bacterial contamination. A trend is a type of SE because all results are affected. Conversely, RE affects some results (but not others) in an unpredictable manner. Control rules affected by RE are 13s and R4s

Which statement about iron methods is true? A. Interference from Hgb can be corrected by a serum blank B. Colorimetric methods measure binding of Fe2+ to a ligand such as ferrozine C. Atomic absorption is the method of choice for measurement of serum iron D. Serum iron can be measured by potentiometry

Correct Answer: B 45. B Atomic absorption is not the method of choice for serum iron because matrix error and variation of iron recovered by extraction cause bias and poor precision. Most methods use HCl to deconjugate Fe3+ from transferrin followed by reduction to Fe2+. This reacts with a neutral ligand such as ferrozine, tripyridyltriazine (TPTZ), or bathophenanthroline to give a blue complex. Anodic stripping voltammetry can also be used to measure serum iron. Hemolysis must be avoided because RBCs contain a much higher concentration of iron than does plasma.

In which condition is the measurement of acid phosphatase clinically useful? A. Measuring the prostatic isoenzyme to screen for prostate cancer B. Measuring the enzyme in a vaginal swab extract C. The diagnosis of hemolytic anemia D. As a marker for bone regeneration

Correct Answer: B 66. B The PSA test is clinically more sensitive than prostatic acid phosphatase in detecting prostatic cancer. The clinical use of prostatic acid phosphatase is confined to the investigation of sexual assault. Acid phosphatase activity > 50 IU/L establishes the presence of seminal fluid in the vaginal sample. Tartrate-resistant acid phosphatase is used as a cytochemical marker for hairy-cell leukemia, and may be measured in serum to identify diseases with increased osteoclast activity, particularly malignancies involving bone.

What is the pH of a solution of HNO3, if the hydrogen ion concentration is 2.5 × 10-2 M? A. 1.0 B. 1.6 C. 2.5 D. 2.8

Correct Answer: B 8. B For a strong acid, the pH is equal to the negative logarithm of the hydrogen ion concentration. pH = -Log H+ pH = -Log 0.025 pH = 1.6

A 10-year old child was admitted to pediatrics with an initial diagnosis of skeletal muscle disease. The best confirmatory tests would be: A. creatine kinase and isocitrate dehydogenase B. gamma-glutamyl transferase and alkaline phosphatase C. aldolase and creatine kinase D. lactate dehydrogenase and malate dehydrogenase

Correct Answer: C

A carbonate salt used to control manic-depressive disorder is: A. digozin B. acetaminophen C. lithium D. phenytoin

Correct Answer: C

A scanning of a CK isoenzyme fractionation revealed 2 peaks: a slow cathodic peak (CKMM) and an intermediate peak (CKMB). A possible interpretation for this pattern is: A. barin tumor B. muscular dystrophy C. myocardial infaction D. viral hepatitis

Correct Answer: C

The glycated hemoglobin value represents the integrated values of glucose concentration during the preceding: A. 1-3 weeks B. 4-5 weeks C. 6-8 weeks D. 16-20 weeks

Correct Answer: C

The principle excretory form of nitrogen is: A. amino acids B. creatinine C. urea D. uric acid

Correct Answer: C

Which condition is a common cause of stray light? A. Unstable source lamp voltage B. Improper wavelength calibration C. Dispersion from second-order spectra D. Misaligned source lamp

Correct Answer: C 10. C Stray light is caused by the presence of any light other than the wavelength of measurement reaching the detector. It is most often caused by second-order spectra, deteriorated optics, light dispersed by a darkened lamp envelope, and extraneous room light.

A patient's BUN is 60 mg/dL and serum creatinine is 3.0 mg/dL. These results suggest: A. Laboratory error measuring BUN B. Renal failure C. Prerenal failure D. Patient was not fasting

Correct Answer: C 18. C BUN is affected by renal blood flow as well as by glomerular and tubular function. When blood flow to the kidneys is diminished by circulatory insufficiency (prerenal failure), glomerular filtration decreases and tubular reabsorption increases due to slower filtrate flow. Because urea is reabsorbed, BUN levels rise higher than creatinine. This causes the BUN:creatinine ratio to be greater than 10:1 in prerenal failure.

Serum protein and immunofixation electrophoresis are ordered on a patient. The former is performed, but there is no evidence of a monoclonal protein. Select the best course of action. A. Perform quantitative Ig G, A, M B. Perform the IFE on the serum C. Report the result; request a urine sample for protein electrophoresis D. Perform IFE on the serum and request a urine sample for IFE

Correct Answer: C 26. C An area of restricted mobility should be identified on serum protein electrophoresis before IFE is performed. About one out of four patients with multiple myeloma have monoclonal free λ or κ chains in urine only, and therefore, urine electrophoresis should be included in initial testing.

Select the correct order of Hgb migration on agarose or cellulose acetate at pH 8.6. A. - C→F→S→A + B. - S→C→A→F + C. - C→S→F→A + D. - S→F→A→C +

Correct Answer: C 28. C Hgb A2 is the slowest of the normal Hgbs, and Hgb A is the fastest. Hgb F migrates just behind Hgb A. Hgb S migrates midway between Hgb A2 and Hgb A. Hgbs C, CHarlem (Georgetown), O, and E migrate with Hgb A2. Hgbs G and DPunjab and Hgb OArab migrate with Hgb S.

Which of the following statements regarding enzymatic reactions is true? A. The enzyme shifts the equilibrium of the reaction to the right B. The enzyme alters the equilibrium constant of the reaction C. The enzyme increases the rate of the reaction D. The enzyme alters the energy difference between reactants and products

Correct Answer: C 3. C An enzyme will accelerate the rate of a reaction, reducing the time required to reach equilibrium. The concentration of reactants and products at equilibrium will be the same with or without the enzyme.

Which of the following statements regarding the catecholamines is true? A. They are derived from tryptophan B. They are produced by the zona glomerulosa of the adrenal cortex C. Plasma levels show both diurnal and pulsed variation D. They are excreted in urine primarily as free catecholamines

Correct Answer: C 31. C Catecholamines—epinephrine, norepinephrine, and dopamine—are produced from the amino acid tyrosine by the chromaffin cells of the adrenal medulla. Plasma and urinary catecholamines are measured in order to diagnose pheochromocytoma. Symptoms include hypertension, headache, sweating, and other endocrine involvement. Plasma catecholamines are oxidized rapidly to metanephrines and VMA; only about 2% is excreted as free catecholamines. The zona glomerulosa is the outermost portion of the adrenal cortex, where aldosterone is mainly produced.

A patient has the following arterial blood gas results: -pH = 7.56 -PO2 = 100 mm Hg -PCO2 = 25 mm Hg -HCO3 = 22 mmol/L These results are most likely the result of which condition? A. Improper specimen collection B. Prolonged storage C. Hyperventilation D. Hypokalemia

Correct Answer: C 33. C The pH is alkaline (reference range 7.35-7.45) and this can be caused by either low PCO2 or increased bicarbonate. This patient has a normal bicarbonate (reference range 22-26 mmol/L) and a low PCO2 (reference range 35-45 mm Hg). Low PCO2 is always caused by hyperventilation, and therefore, this is a case of uncompensated respiratory alkalosis. The acute stages of respiratory disorders are often uncompensated. Prolonged storage would cause the pH and PO2 to fall, and the PCO2 to rise. Hypokalemia causes alkalosis, but usually is associated with the retention of CO2 as compensation.

The protein composition of HDL is what percentage by weight? A. Less than 2% B. 25% C. 50% D. 90%

Correct Answer: C 45. C About 50% of the weight of HDL is protein, largely apo A-I and apo A-II. The HDL is about 30% phospholipid and 20% cholesterol by weight. The HDL binds and esterifies free cholesterol from cells and transports it to the liver, where it can be eliminated in the bile.

If the steady-state drug level is too high, the best course of action is to: A. Decrease the dose B. Decrease the dose interval C. Decrease the dose and decrease the dose interval D. Change the route of administration

Correct Answer: A 14. A Decreasing both dose and dosing interval will have offsetting effects on peak and trough blood levels. The appropriate dose can be calculated if the clearance or Vd and f are known. For example, the initial dose is calculated by multiplying the desired peak blood drug concentration by the Vd.

What is the primary means of solute separation in HPLC using a C18 column? A. Anion exchange B. Size exclusion C. Partitioning D. Cation exchange

Correct Answer: C 57. C Stationary phases (column packings) used in HPLC separate solutes by multiple means, but in reverse-phase HPLC the relative solubility between the mobile phase and stationary phase is most important and depends upon solvent polarity, pH, and ionic strength.

Which of the following buffers is used in the IFCC recommended method for ALP? A. Glycine B. Phosphate C. 2-Amino-2-methyl-1-propanol D. Citrate

Correct Answer: C 64. C The Szasz modification of the Bowers-McComb method measures the hydrolysis of p-nitrophenyl phosphate, and continuously monitors the formation of p-nitrophenol at 405 nm. AMP buffer chelates phosphorus, preventing product inhibition; Zn+2 and Mg+2 are added to the substrate to activate ALP. HEDTA is used to chelate the excess Zn+2, which is inhibitory at high concentrations.

The determination of the oxygen saturation of hemoglobin is best accomplished by: A. Polychromatic absorbance measurements of a whole-blood hemolysate B. Near infrared transcutaneous absorbance measurement C. Treatment of whole blood with alkaline dithionite prior to measuring absorbance D. Calculation using PO2 and total hemoglobin by direct spectrophotometry

Correct Answer: A 14. A Measurement of oxyhemoglobin, deoxyhemoglobin (reduced hemoglobin), carboxyhemoglobin, methemoglobin, and sulfhemoglobin can be accomplished by direct spectrophotometry at multiple wavelengths and analysis of the absorptivity coefficients of each pigment at various wavelengths. The O2 saturation is determined by dividing the fraction of oxyhemoglobin by the sum of all pigments. This eliminates much of the error that occurs in the other methods when the quantity of an abnormal hemoglobin pigment is increased.

Which of the following 2-hour glucose challenge results would be classified as impaired glucose tolerance (IGT)? Two-hour serum glucose: A. 130 mg/dL B. 135 mg/dL C. 150 mg/dL D. 204 mg/dL

Correct Answer: C 8. C With the exception of pregnant females, impaired glucose tolerance is defined by the ADA as a serum or plasma glucose at 2 hours following a 75-g oral glucose load of ≥140 mg/dL and < 200 mg/dL. Persons who have a fasting plasma glucose of ≥100 but < 126 mg/dL are classified as having impaired fasting glucose (IFG). Both IGT and IFG are risk factors for developing diabetes later in life. Such persons are classified as having prediabetes and should be tested annually.

Beriberi is associated with deficiency of vitamin: A. A B. C C. niacin D. thiamine

Correct Answer: D

The urinary excretion produce measured as an indicator of epinephrine production is: A. dopamine B. dihydroxyphenylalanine (DOPA) C. homovanillic acid D. vanillylmandelic acid (VMA)

Correct Answer: D

Unless blood gas measurements are made immediately after sampling, in vitro glycolysis of the blood causes a: A. rise in pH and PCO2 B. fall in pH and a rise in PO2 C. rise in pH and a fall in PO2 D. fall in pH and a rise in PCO2

Correct Answer: D

Which of the following conditions will interfere with the measurement of LD? A. Slight hemolysis during sample collection B. Storage at 4°C for 3 days C. Storage at room temperature for 16 hours D. Use of plasma collected in heparin

Correct Answer: A 17. A RBCs are rich in LD-1 and LD-2, and even slight hemolysis will falsely elevate results. Hemolytic, megaloblastic, and pernicious anemias are associated with LD levels of 10-50 times the URL. LD is stable for 2 days at room temperature or 1 week at 4°C; however, freezing causes deterioration of LD-5. The activity of LD is inhibited by EDTA, which binds divalent cations; serum or heparinized plasma should be used.

The syndrome of inappropriate antidiuretic hormone secretion (SIADH) causes: A. Low serum vasopressin B. Hypernatremia C. Urine osmolality to be lower than plasma D. Low serum electrolytes

Correct Answer: D 2. D SIADH results in excessive secretion of vasopressin (ADH) from the posterior pituitary, causing fluid retention and low plasma osmolality, sodium, potassium, and other electrolytes by hemodilution. It is suspected when urine osmolality is higher than plasma, but urine sodium concentration is normal or increased. Patients with sodium depletion have a urine osmolality higher than plasma, but low urine sodium.

Select the enzyme that is most specific for β-D-glucose. A. Hexokinase B. G-6-PD C. Phosphohexisomerase D. Glucose oxidase

Correct Answer: D 24. D Glucose oxidase is the most specific enzyme reacting with only β-D-glucose. However, the peroxidase coupling reaction used in the glucose oxidase method is subject to positive and negative interference. Therefore, hexokinase is used in the reference method.

In the Oliver-Rosalki method for CK, adenosine monophosphate (AMP) is added to the substrate in order to: A. Inhibit adenylate kinase B. Block the oxidation of glutathione C. Increase the amount of ADP that is available D. Block the action of diadenosine pentaphosphate

Correct Answer: A 19. A Positive interference in the Oliver-Rosalki method can occur when adenylate kinase is present in the serum from hemolysis or damaged tissue. Adenylate kinase hydrolyzes ADP, forming AMP and ATP (2 ADP AK --> AMP + ATP). This reaction is inhibited by adding AMP and diadenosine pentaphosphate (Ap5A) to the substrate.

Which of the following statements describes a nonkinetic enzyme assay? A. Initial absorbance is measured followed by a second reading after 5 minutes B. Absorbance is measured at 10-second intervals for 100 seconds C. Absorbance is monitored continuously for 1 minute using a chart recorder D. Reflectance is measured from a xenon source lamp pulsing at 60 Hz

Correct Answer: A 2. A A kinetic assay uses several evenly spaced absorbance measurements to calculate the change in absorbance per unit time. A constant change in absorbance per unit of time occurs only when the rate of the reaction is zero order (independent of substrate concentration). Enzyme activity is proportional to rate only under zero-order conditions.

Which of two instruments can be assumed to have the narrower bandpass? Assume that wavelength is accurately calibrated. A. The instrument giving the highest absorbance for a solution of 0.1 mmol/L NADH at 340 nm B. The instrument giving the lowest %T for a solution of nickel sulfate at 700 nm C. The instrument giving the highest %T reading for 1.0% v/v HCl at 350 nm D. The instrument giving the most linear plot of absorbance versus concentration

Correct Answer: A 2. A Bandpass is defined by the range of wavelengths passed through the sample at the specified wavelength setting. It can be measured using any solution having a narrow absorbance peak (e.g., NADH at 340 nm). The instrument producing the purest monochromatic light will have the highest absorbance reading.

The term pharmacokinetics refers to the: A. Relationship between drug dose and the drug blood level B. Concentration of drug at its sites of action C. Relationship between blood concentration and therapeutic response D. The relationship between blood and tissue drug levels

Correct Answer: A 2. A Pharmacokinetics is the mathematical expression of the relationship between drug dose and drug blood level. When the appropriate formula is applied to quantitative measures of drug dose, absorption, distribution, and elimination, the blood concentration can be accurately determined.

Select the most appropriate single screening test for thyroid disease. A. Free thyroxine index B. Total T3 assay C. Total T4 D. TSH assay

Correct Answer: D 43. D TSH is produced by the anterior pituitary in response to low levels of free T4 or T3. A normal TSH rules out thyroid disease. TSH is low in primary hyperthyroidism and high in primary hypothyroidism.

Which isoenzyme of ALP is most heat stable? A. Bone B. Liver C. Intestinal D. Placental

Correct Answer: D 57. D Placental ALP and tumor-associated isoenzymes such as the Regan isoenzyme associated with lung cancer are the only isoenzymes that retain activity when serum is heated to 65° C for 10 minutes. Heat inactivation is used primarily to distinguish liver ALP from bone ALP. If less than 20% activity remains after heating serum to 56°C for 10 minutes, then bone ALP is most likely present.

Which monochromator specification is required in order to measure the true absorbance of a compound having a natural absorption bandwidth of 30 nm? A. 50-nm bandpass B. 25-nm bandpass C. 15-nm bandpass D. 5-nm bandpass

Correct Answer: D 8. D Bandpass refers to the range of wavelengths passing through the sample. The narrower the bandpass, the greater the photometric resolution. Bandpass can be made smaller by reducing the width of the exit slit. Accurate absorbance measurements require a bandpass less than one-fifth the natural bandpass of the chromophore.

Which of the following compounds is not a precursor of the estrogens? A. Progesterone B. Testosterone C. Cholesterol D. Aldosterone

D. Acetate, cholesterol, progesterone, and the male sex hormones testosterone and androstenedione all serve as precursors for the synthesis of estrogens. The major pathway for conversion of testosterone to estradiol is in the ovaries. The major pathway for conversion of androstenedione to estrone is outside the ovaries.

Which statement concerning gammaglutamyltransferase is false? A. Present in almost all cells of the body B. Elevated in liver and some pancreatic diseases C. Elevated in chronic alcoholism D. Elevated in bone disease

D. Gamma-glutamyltransferase (GGT) catalyzes the transfer of gamma-glutamyl groups from peptides to an appropriate acceptor. GGT is found in almost all cells. The highest amount of GGT is found in the kidney, and slightly less is found in the liver and pancreas. Diagnostically, the assay of GGT is widely used to investigate hepatic disease. Increased values are seen in a variety of liver disorders and in conditions that are characterized by secondary liver involvement, including acute pancreatitis, pancreatic carcinoma, infectious mononucleosis, alcoholism, and cardiac insufficiency. Normal GGT levels are seen in bone disorders, in growing children, and during pregnancy.

Which of the following conditions is usually associated with an acute inflammatory pattern? A. Myocardial infarction (MI) B. Malignancy C. Rheumatoid arthritis D. Hepatitis

Correct Answer: A 22. A MI produces a pattern of acute inflammation usually associated with tissue injury. This pattern results from production of acute phase proteins including α1-antitrypsin, α1-antichymotrypsin, and haptoglobin. It is also seen in early infection, pregnancy, and early nephritis. Malignancy, rheumatoid arthritis, and hepatitis are associated with a chronic inflammatory pattern. This differs from the acute pattern by the addition of a polyclonal gammopathy.

Which enzyme deficiency is responsible for phenylketonuria (PKU)? A. Phenylalanine hydroxylase B. Tyrosine transaminase C. p-Hydroxyphenylpyruvic acid oxidase D. Homogentisic acid oxidase

Correct Answer: A 23. A PKU is an overflow aminoaciduria resulting from the accumulation of phenylalanine. It is caused by a deficiency of phenylalanine hydroxylase, which converts phenylalanine to tyrosine. Excess phenylalanine accumulates in blood. This is transaminated, forming phenylpyruvic acid, which is excreted in the urine.

Select the coupling enzyme used in the hexokinase method for glucose. A. Glucose-6-phosphate dehydrogenase B. Peroxidase C. Glucose dehydrogenase D. Glucose-6-phosphatase

Correct Answer: A 25. A The hexokinase reference method uses a protein-free filtrate prepared with barium hydroxide (BaOH) and zinc sulfate (ZnSO4). Hexokinase catalyzes the phosphorylation of glucose in the filtrate using ATP as the phosphate donor. Glucose-6-phosphate (glucose-6-PO4) is oxidized to 6-phosphogluconate and NAD+ is reduced to NADH using G-6-PD. The increase in absorbance at 340 nm is proportional to glucose concentration. Although hexokinase will phosphorylate some other hexoses including mannose, fructose, and glucosamine, the coupling reaction is entirely specific for glucose-6-PO4 eliminating interference from other sugars.

Which of the following is false about PSA? A. Serum quantified using immunoassays B. Single-chain glycoprotein C. Used as a tumor marker D. Not elevated in benign prostatic hyperplasia

D. PSA is a single-chain glycoprotein whose function aids in the liquefaction of seminal coagulum. PSA is found specifically in the prostate gland, and elevated levels are associated with prostate cancer and benign prostatic hyperplasia (BPH). Thus, combining the quantification of PSA with the performance of the digital rectal examination is more beneficial for prostate cancer detection. Immunoassays using enzyme, fluorescent, and chemiluminescent labels are available to quantify PSA.

In a nonmyocardial as opposed to a myocardial cause of an increased serum or plasma CK-MB, which would be expected? A. An increase in CK-MB that is persistent B. An increase in the percent CK-MB as well as concentration C. The presence of increased TnI D. A more modest increase in total CK than CK-MB

Correct Answer: A 26. A Plasma CK-MB becomes abnormal 4 hours postinfarction, peaks in 16-20 hours, and usually returns to normal within 48 hours. In some noncardiac causes of elevated plasma CK-MB such as muscular dystrophy, there is a persistent elevation of both total CK and CK-MB. TnI and TnT are cardiac-specific markers. They become elevated slightly before CK-MB when a CK-MB URL of 4 μg/L is used, remain elevated for 7-10 days following an AMI, and are not increased in muscular dystrophy, malignant hyperthermia, or crush injuries that are associated with an increase in the concentration of CK-MB. Absolute CK-MB increases are evaluated cautiously, when CK-MB is less than 2.5% of total enzyme because noncardiac sources may be responsible.

What analyte is measured using the Jaffe reaction? A. Urea B. Uric acid C. Ammonia D. Creatinine

D. The Jaffe reaction, which was described in 1886, is still used for creatinine analysis. The Jaffe reaction employs the use of an alkaline picrate solution that reacts with creatinine to form a bright orange-red complex. A drawback to this procedure is its lack of specificity for creatinine, because noncreatinine chromogens, glucose, and proteins are also able to react with alkaline picrate.

Which compounds originally condense to form aminolevulinic acid? A. Oxoglutarate and aspartate B. Isocitrate and coenzyme II C. Oxalacetate and malate D. Succinyl coenzyme A and glycine

D. The biochemical synthesis of the porphyrins consists of a series of reactions. Succinyl coenzyme A and glycine are the two compounds that originally condense to form aminolevulinic acid (ALA). Through a second condensation reaction, two molecules of ALA condense and cyclize to form porphobilinogen. Porphobilinogen is a monopyrrole structure and the precursor of porphyrin synthesis.

Which immunoglobulin class is able to cross the placenta from the mother to the fetus? A. IgA B. IgD C. IgE D. IgG

D. The only immunoglobulin class that is able to cross the placenta from the mother's circulation to the fetus is IgG. Therefore, at birth, there is very little immunoglobulin present in the infant except for the maternal IgG. After birth, as the infant comes in contact with antigens, the levels of IgG, IgA, and IgM slowly increase.

Ingestion of which of the following drugs may cause hypoglycemia? A. Ethanol B. Propranolol C. Salicylate D. All the above

D. There are greater than 100 causes of hypoglycemia. Among the causes is the ingestion of certain drugs. Use of ethanol, propranolol, and salicylate has been linked to the occurrence of hypoglycemia.

Given the following data, calculate the coefficient of variation for glucose. -Analyte: Glucose -Mean: 76 mg/dL -Standard Deviation: 2.3 A. 3.0% B. 4.6% C. 7.6% D. 33.0%

Correct Answer: A 27. A The coefficient of variation is calculated by dividing the standard deviation by the mean and multiplying by 100. % CV = s /x *100 × = 2.3 / 7.6 × 100 = 3.0% The CV is the most appropriate statistic to use when comparing the precision of samples that have different means. For example, when comparing the precision of the level 1 control to the level 2 control, the coefficient of variation normalizes the variance to be independent of the mean. The control with the lower CV is the one for which the analysis is more precise.

5'NT

biliary obstruction

Total bilirubin

conjugated + unconjugated= ________ - reacts slowly with Diazo reagent

Hyponatremia

low sodium in the blood

Triglycerides

primary form of lipid storage

hypertension

pro-BNP and BNP are released by ventricle walls in response to _______ and volume overload

Myoglobin

produced by the muscles including the heart increased in muscle and renal damage

urea, ammonia

protein breakdown in the body produces _______ and _______

Pre-hepatic jaundice

results from excessive breakdown of red blood cells liver function= N conjugated= N in urine= none unconjugated/urobili= increased

Turbidimetric method

used in urine and CSF total protein. --based on light scattering from a bacterial suspension; no way to differentiate between dead and living cells; although this is the fastest method to count cells

2Na+ (glucose/18)+ (BUN/2.8)

what is the formula to calculate osmolality

In serum protein electrophoresis, when a buffer solution of pH 8.6 is used, which of the following characterizes the proteins? A. Exhibit net negative charge B. Exhibit net positive charge C. Exhibit charge neutrality D. Migrate toward the cathode

A. Buffer solutions of pH 8.6 are commonly used for seaim protein electrophoresis. At this alkaline pH, the serum proteins have a net negative charge. Therefore, the negatively charged serum proteins migrate toward the anode. This is true for all the proteins except the gamma-globulins, which tend to show the phenomenon of endosmosis.

Which of the following statements about carbonic anhydrase (CA) is true? A. Catalyzes conversion of CC>2 and F^O to HHCO3 in red blood cells B. Causes shift to the left in oxygen dissociation curve C. Catalyzes formation of H2CO3 from CO2 and H2O in the tissues D. Inactive in renal tubular cells

A. Carbonic anhydrase (CA) is an enzyme found in red blood cells that catalyzes the reversible hydration of CO2 to bicarbonate and a proton: CA H2O + CO2 < * HHCO3 The proton, in turn, is buffered by the histidine portion of the hemoglobin molecule that activates the release of oxygen. It is at this point that oxyhemoglobin is converted to deoxyhemoglobin. In the alveoli of the lungs, CA catalyzes the conversion of H2CO3 to CO2 and H2O. The CO2 is then exhaled. Carbonic anhydrase is an intracellular enzyme of erythrocytes and renal tubular cells, and it is not found normally in any significant concentration in the plasma. It is not associated with the oxygen dissociation curve.

What collective term encompasses the reduction products stercobilinogen, urobilinogen, and mesobilirubinogen? A. Urobilinogen B. Mesobilirubinogen C. Urobilin D. Bilirubin

A. Conjugated bilirubin and a small amount of unconjugated bilirubin will pass from the bile into the small intestine. In the small intestine, enzyme systems of anaerobic bacteria are able to reduce bilirubin to the reduction products mesobilirubinogen, stercobilinogen, and urobilinogen. These three reduction products of bilirubin catabolism are collectively referred to as urobilinogen.

When measuring enzyme activity, if the instrument is operating 5°C lower than the temperature prescribed for the method, how will the results be affected? A. Lower than expected B. Higher than expected C. Varied, showing no particular pattern D. All will be clinically abnormal.

A. Factors that affect enzyme assays include temperature, pH, substrate concentration, and time of incubation. For each clinically important enzyme, the optimum temperature and pH for its specific reaction are known. When lower than optimum temperature or pH is employed, the measured enzyme activity will be lower than the expected activity value. As temperature increases, the rate of the reaction increases. Generally, a twofold increase in reaction rates will be observed with a 10°C rise in temperature. However, once the optimum temperature is exceeded, the reaction rate falls off as enzyme denaturation occurs at temperatures ranging from 40 to 70°C.

Which of the following statements is false about anodic stripping voltammetry (ASV)? A. Based on potentiometry B. Occurs in an electrochemical cell C. Involves preconcentration of the analyte by electroplating D. Used to measure lead

A. In polarography, an electrochemical cell is used. A gradually increasing voltage is applied between the two electrodes of the cell that are in contact with a solution containing the analyte.

Which parameter is used in mass spectrometry to identify a compound? A. Ion mass-to-charge ratio B. Molecular size C. Absorption spectrum D. Retention time

A. Mass spectrometry identifies a compound based on the principle of charged particles moving through a magnetic or electric field, with ions being separated from other charged particles according to their mass-to-charge ratios. The mass spectrum produced is unique for a particular compound. It also identifies the positioning of functional groups of the compound. Mass spectrometry is useful in the clinical laboratory for drug identification.

Which of the following is not a reagent required in an enzymatic serum glucose method? A. NAD+ B. Glucose oxidase C. Peroxidase D. Reduced chromogen

A. The glucose oxidase method for quantifying glucose employs two coupled enzymatic reactions. In the first reaction, which is catalyzed by glucose oxidase, glucose in the presence of oxygen is oxidized to gluconic acid and hydrogen peroxide. In the second reaction, peroxidase catalyzes a reaction between hydrogen peroxide and the reduced form of a chromogenic oxygen acceptor, such as o-dianisidine, forming an oxidized colored product that is read spectrophotometrically.

Which immunoglobulin class, characterized by its possession of a secretory component, is found in saliva, tears, and body secretions? A. IgA B. IgD C. IgG D. IgM

A. The immunoglobulin class IgA is found in both plasma and body secretions, with the two types being differentiated by their sedimentation coefficients. Plasma IgA has an average sedimentation coefficient of 7S, and secretory IgA has a sedimentation coefficient of US. Secretory IgA is present in saliva, tears, and secretions of nasal, gastrointestinal, and tracheolbronchial origin. Secretory IgA is dimeric in structure and possesses a glycoprotein secretory component attached to its heavy chains and a J polypeptide. The principal immunoglobulin found in secretions is IgA, with only trace amounts of IgG being present. The presence of IgM, IgD, or IgE in secretions has not been detected.

Which of the following is the Hollander insulin test used to confirm? A. Hyperglycemia B. Vagotomy C. Pancreatectomy D. Insulinoma

B. In cases of peptic ulcer, treatment may include surgery that severs the vagus nerve. This severing is known as vagotomy, which, if complete, prevents the secretion of gastrin and HC1 by the stomach. The Hollander insulin test is performed to assess the completeness of the vagotomy. If the vagotomy is complete, the hypoglycemia caused by the administration of insulin will not exert its normal stimulatory effect on gastric HC1 and pepsinogen secretion.

THC (A9-tetrahydrocannabinol) is the principal active component of what drug? A. Benzodiazepine B. Marijuana C. Morphine D. Codeine

B. THC (A9-tetrahydrocannabinol) is the principal active component of marijuana. Homogeneous enzyme immunoassay methods test for the presence of THC metabolites, especially 11- nor-A9-THC-9-carboxylic acid, which is the primary urinary metabolite. Metabolites appear in urine within hours of smoking and continue to be detectable for 3 to 5 days following exposure.

The physician is concerned that a pregnant patient may be at risk for delivering prematurely. What would be the best biochemical marker to measure to assess the situation? A. Inhibin A B. ctpFetoprotein C. Fetal fibronectin D. Human chorionic gonadotropin

C. Fibronectin is an adhesive glycoprotein that functions with collagen to support cell adhesion. It is a normal constituent in the placenta and amniotic fluid. As labor begins, a change occurs in cell adhesion that affects the placenta and uterine wall. The level of fetal fibronectin increases in the secretions of the cervix and vagina. When this occurs prematurely, the increase in fetal fibronectin is used to predict risk of premature birth. Inhibin A, ctrfetoprotein, human chorionic gonadotropin, and unconjugated estriol are used together in the quadruple test to assess risk for such disorders as Down syndrome.

Which of the following enzyme activities can be determined by using a dilute olive oil emulsion substrate, whose hydrolyzed product is monitored as a decrease in turbidity or light scatter? A. Alkaline phosphatase B. Amylase C. Lipase D. Trypsin

C. Lipase activity can be determined using a dilute olive oil emulsion as the substrate. The fatty micellar complexes absorb light as well as scatter light. Lipase catalyzes the hydrolysis of these triglyceride complexes, forming fatty acid and glycerol products. With the degradation of the micellar complexes, clearing of the reagent mixture occurs, causing changes in turbidity and light scatter. The rate at which the turbidity decreases can be monitored spectrophotometrically at 400 nm, or the decrease in light scatter can be measured using a nephelometer. The rate of these changes can be equated to the lipase activity present in the serum specimen.

Serum protein electrophoresis is routinely performed on the serum obtained from a clotted blood specimen. If a plasma specimen is substituted for serum, how will the electrophoresis be affected? A. Electrophoresis cannot be performed because the anticoagulant will retard the mobilities of the protein fractions. B. Electrophoresis cannot be performed because the anticoagulant will cause migration of the protein fractions in the direction of the cathode. C. Electrophoresis will show an extra fraction in the beta-gamma region. D. Electrophoresis will show an extra fraction in the prealbumin area.

C. Protein electrophoresis is performed on a serum specimen. If plasma is substituted for serum, the electrophoresis will show an extra fraction in the beta-gamma region, because fibrinogen is a beta2-globulin. This extra fraction represents the protein fibrinogen that is present in a plasma specimen. Fibrinogen contributes approximately 0.2-0.4 g/dL to the total protein concentration.

Which of the following characterizes respiratory acidosis? A. Excess of bicarbonate B. Deficit of bicarbonate C. Excess of dissolved carbon dioxide (PCO2) D. Deficit of dissolved carbon dioxide (PCO2)

C. The normal ratio of bicarbonate ions to dissolved carbon dioxide is 20:1 and pH = 6.1 + log 20/1. An excess of dissolved CO2 (e.g., increase in PCC^) will increase the denominator in the equation or decrease the ratio of bicarbonate ions to dissolved CO?. The pH will decrease; that is, the plasma becomes more acidic. The amount of dissolved CO2 (PCC^) in the blood is related to respiration. Hence, this condition is termed "respiratory acidosis."

Which of the following is not associated with insulin? A. Synthesized from proinsulin B. Synthesized by (3-cells in the pancreas C. C-peptide is active form D. Two-chain polypeptide

C. The protein hormone insulin is synthesized in the pancreas by the (3-cells of the islets of Langerhans. Insulin, a two-chain polypeptide, consists of 51 amino acids. A single-chain preproinsulin is cleaved to proinsulin, which is the immediate precursor of insulin. Proinsulin is hydrolyzed to form insulin, a two-chain polypeptide, and inactive C-peptide. Insulin promotes the entry of glucose into tissue cells.

IN the immunoinhibition phase of CKMB procedure: A. M subunit is inactivated B. B subunit is inactivated C. MB is inactivated D. BB is inactivated

Correct Answer: A

Severe diarrhea causes: A. metabolic acidosis B. metabolic alkalosis C. respiratory acidosis D. respiratory alkalosis

Correct Answer: A

What is the best method to diagnose lactase deficiency? A. H2 breath test B. plasma aldolase level C. LDH level D. D-xylose test

Correct Answer: A

Which of the following stains is used for lipoprotein electrophoresis? A. Oil Red O B. Coomassie Brilliant Blue C. Amido Black D. Ponceau S

Correct Answer: A 20. A Oil Red O and Sudan Black B stain neutral fats and are used to stain lipoproteins as well as fat in urine or stool. The other stains are used for proteins. Coomassie Brilliant Blue is more sensitive than Ponceau S or Amido Black, and all three stains have slightly greater affinity for albumin than globulins. In addition, silver nitrate may be used to stain CSF proteins because it has greater sensitivity than the other stains.

Foradrugthatfollowsfirst-orderpharmacokinetics, adjustment of dosage to achieve the desired blood level can be made using which formula? A. New dose = current dose × desired concentration concentration at B. New dose = current dose × concentration at steady state desired C. New dose = concentration at steady state × half-life desired concentration D. New dose = concentration at steady state × desired current dose concentration

Correct Answer: A 22. A Most drugs follow first-order pharmacokinetics, meaning the clearance of drug is linearly related to the drug dose. The dose of such drugs can be adjusted by multiplying the ratio of the current dose to blood concentration by the desired drug concentration, provided the blood concentration is measured at steady state.

Quantitative determination of Hgb A2 and Hgb F are best performed by: A. High-performance liquid chromatography B. Alkali denaturation C. Electrophoresis D. Direct bichromatic spectrophotometry

Correct Answer: A 27. A Hgb A2 and Hgb F are often quantitated to diagnose persons with thalassemia. The method of choice is HPLC using cation exchange chromatography. Hemoglobins are eluted from the column in order of increasing positive charge using a sodium phosphate buffer to produce a gradient of increasing ionic strength. Hemoglobin F elutes from the column earlier than Hgb A2 because it is less positively charged.

The following conditions are all causes of alkalosis. Which condition is associated with respiratory (rather than metabolic) alkalosis? A. Anxiety B. Hypovolemia C. Hyperaldosteronism D. Hypoparathyroidism

Correct Answer: A 28. A Respiratory alkalosis is caused by hyperventilation, which leads to decreased PCO2. Anxiety and drugs such as epinephrine that stimulate the respiratory center are common causes of respiratory alkalosis. Excess aldosterone increases net acid excretion by the kidneys. Low parathyroid hormone causes increased bicarbonate reabsorption, resulting in alkalosis. Hypovolemia increases the relative concentration of bicarbonate. This is common and is called dehydrational alkalosis, chloride responsive alkalosis, or alkalosis of sodium deficit.

Which Hgb separates from Hgb S on citrate (acid) agar, but not agarose or cellulose acetate? A. Hgb DPunjab B. Hgb E C. Hgb CHarlem (Georgetown) D. Hgb OArab

Correct Answer: A 32. A Hgbs OArab, E, and CHarlem migrate to the same position as Hgbs A2 and C on agarose × or cellulose acetate) at pH 8.6. Hgb DPunjab migrates to the same position as Hgb S on agarose, but moves with Hgb A on citrate agar. Agarose is a purified form of agar; it lacks the sulfated pectins required to separate Hgbs DPunjab and G from Hgb S, and Hgbs E, CHarlem, and OArab from Hgb C. Hgb CHarlem is a sickling Hgb and it migrates to the same position as Hgb S on citrate (acid) agar.

The reference potential of a silver-silver chloride electrode is determined by the: A. Concentration of the potassium chloride filling solution B. Surface area of the electrode C. Activity of total anion in the paste covering the electrode D. The concentration of silver in the paste covering the electrode

Correct Answer: A 36. A The activity of any solid or ion in a saturated solution is unity. For a silver electrode covered with silver chloride paste, the Nernst equation is E = E° - RT/nF × 2.3 log10 [Ag° × Cl- ]/[AgCl]. Because silver and silver chloride have an activity of 1.0, and all components except chloride are constants, the potential of the reference electrode is determined by the chloride concentration of the filling solution. E = Eo - RT/nF × 2.3 log10[Cl- ] = E° - 59.2 mV × log[Cl- ] (at room temperature)

Which wavelength would be absorbed strongly by a red-colored solution? A. 450 nm B. 585 nm C. 600 nm D. 650 nm

Correct Answer: A 4. A A solution transmits light corresponding in wavelength to its color, and usually absorbs light of wavelengths complementary to its color. A red solution transmits light of 600-650 nm and strongly absorbs 400-500 nm light.

What are the likely laboratory findings in a person suspected of having Wilson's disease? A. Blood copper and ceruloplasmin low, urinary copper excretion high B. Blood and urine copper concentration high, ceruloplasmin low C. Blood and urine copper concentration high, ceruloplasmin high D. Blood and urine copper concentration low, ceruloplasmin low

Correct Answer: A 40. A Wilson's disease is an autosomal recessive disease in which copper transport is abnormal. The gene causing the disease codes for an ATPase (called Wilson's protein or ATP7B) that is needed to excrete copper into bile and incorporate copper into ceruloplasmin. There are over 200 reported mutations of this gene. The absence of Wilson's protein results in failure to load ceruloplasmin with copper, dramatically reducing its half-life in blood. Therefore, blood levels of ceruloplasmin are low, and blood levels of copper are usually low because there is little ceruloplasmin to bind it. Copper deposits in tissues, particularly the liver and brain, causing necrosis, and excess is excreted in the urine.

A patient has an elevated serum T3 and free T4 and undetectable TSH. What is the most likely cause of these results? A. Primary hyperthyroidism B. Secondary hyperthyroidism C. Euthyroid with increased thyroxine-binding proteins D. Euthyroid sick syndrome

Correct Answer: A 47. A An undetectable TSH with increased T3 is caused by primary hyperthyroidism (suppression via high free thyroid hormone). In secondary hyperthyroidism, the TSH will be elevated in addition to at least the T3. Patients with an increased thyroxine-binding protein level will have an increase in total T3 but not free T4 or TSH. Patients with euthyroid sick syndrome usually have a low total T3 due to deficient conversion of T4 to T3, normal free T4, and a normal or slightly elevated TSH.

Which set of results is most likely in an adult male with primary testicular failure? A. Increased LH, FSH, and decreased testosterone B. Decreased LH, FSH, and testosterone C. Decreased testosterone, androstenedione, and FSH D. Increased androstenedione, decreased testosterone, and normal FSH

Correct Answer: A 9. A Primary testicular failure produces a picture that is hypergonadotropic. The LH and FSH are increased because the pituitary gland is normal and responds to decreased free testosterone. Androstenedione is an adrenal androgen and is unaffected. In testicular failure secondary to pituitary deficiency (hypogonadotropic testicular failure), the LH, FSH, and testosterone are low.

125I has a physical half-life of 60.0 days. A sample today had activity of 10,000 CPM/mL. How many days form today will the count be 1250 CPM/mL? A. 60 B. 180 C. 240 D. 1250

Correct Answer: B

90% of the copper present in the blood is bound to: A. transferrin B. ceruloplasmin C. albumin D. cryoglobulin

Correct Answer: B

A 2-year-old child with a decreased serum T4 is described as being somewhat dwarfed, stocky, overweight, and having coarse features. Of the following, the most informative additional laboratory test would be the serum: A. thyroxine binding globulin (TBG) B. thyroid-stimulating hormone (TSH) C. triiodothyronine (T3) D. cholesterol

Correct Answer: B

At blood pH 7.40, what is the ratio fo bicarbonate to carbonic acid? A. 15:1 B. 20:1 C. 25:1 D. 30:1

Correct Answer: B

Refer to the following graph: See BOC pg 109 Pic 1 The HCG levels shown in the above graph most probably represent: A. hydatidiform mole following miscarriage at 4 months B. normal pregnancy C. development of hydatidiform mole D miscarriage at 2 months with retained placenta

Correct Answer: B

Serum from a patient with metastatic carcinoma of the prostate was separated form the clot and stored at room temperature. The following results were obtained: Patient value; Reference range -Ca++: 10.8 mg/dL (2.7 mmol/L); 8.8-10.3 mg/dL (2.2-2.6 mmol/L) -LD: 420 U/L; 50-150 U/L -acid phosphatase: 0.1 U/L; 0-5.5 U/L The technician should repeat the: A. LD using dilute serum B. acid phosphatase with freshly drawn serum C. LD with fresh serum D. test using plasma

Correct Answer: B

Serum levels that define hypoglycemia pre-term or low birth with infants are: A. the same as adults B. lower than adults C. the same as a normal full-term infant D. higher than a normal full-term infant

Correct Answer: B

The electrophoretic pattern of plasma sample as compared to a serum sample shows a: A. broad prealbumin peak B. sharp fibrinogen peak C. diffuse pattern because of the presence of anticoagulants D. decreased globulin fraction

Correct Answer: B

What battery of tests if most useful in evaluating an anion gap of 22 mEq/L (22 mmol/L)? A. Ca++, Mg++, PO-4, and pH B. BUN, creatinine, salicylate and methanol C. AST, ALT, LD and amylase D. glucose, CK, myoglobin and cryoglobulin

Correct Answer: B

When performing a manual protein analysis on a xanthocrhomic spinal fluid, the technician should: A. perform the test as usual B. make a patient blank C. centrifuge the specimen D. dilute the specimen with deionized water

Correct Answer: B

Which of the following steroids is an adrenal cortical hormone? A. angiotensinogen B. aldosterone C. epinephrine D. growth hormone

Correct Answer: B

Which one of the following values obtained during a glucose tolerance test are diagnostic of diabetes mellitus? A. 2-hour specimen =150 mg/dL (8.3 mmol/L) B. fasting plasma glucose = 126 mg/dL (6.9 mmol/L) C. fasting plasma glucose = 110 mg/dL (6.1 mmol/L) D. 2-hour specimen = 180 mg/dL (9.9 mmol/L)

Correct Answer: B

Which of the following enzymes is activated by calcium ions? A. CK B. Amylase C. ALP D. LD

Correct Answer: B 10. B Most enzymes require metals as activators or cofactors. CK and ALP require Mg+2 for full activity, and amylase requires Ca+2. Metals required for activity should be components of the substrate used for enzyme analysis. The substrate must also contain anions required (e.g., Cl- for amylase) and should not contain inhibiting cations or anions (e.g., Zn+2 and Mn+2 for CK).

What is the pH of a buffer containing 40.0 mmol/L NaHC2O4 and 4.0 mmol/L H2C2O4? (pKa = 1.25) A. 1.35 B. 2.25 C. 5.75 D. 6.12

Correct Answer: B 12. B The Henderson-Hasselbalch equation can be used to determine the pH of a buffer containing a weak acid and a salt of the acid. pH = pKa + log salt/Acid = 1.25 + log 40.0 mmol/L / 4.0 mmol/L = 1.25 + log 10 = 2.25

Which of the following is a potential source of error in the hexokinase method? A. Galactosemia B. Hemolysis C. Sample collected in fluoride D. Ascorbic acid

Correct Answer: B 27. B The hexokinase method can be performed on serum or plasma using heparin, EDTA, citrate, or oxalate. RBCs contain glucose-6-PO4 and intracellular enzymes that generate NADH, causing positive interference. Therefore, hemolyzed samples require a serum blank correction (subtraction of the reaction rate with hexokinase omitted from the reagent).

Uric acid is derived from the: A. Oxidation of proteins B. Catabolism of purines C. Oxidation of pyrimidines D. Reduction of catecholamines

Correct Answer: B 32. B Uric acid is the principal product of purine (adenosine and guanosine) metabolism. Oxidation of proteins yields urea along with CO2, H2O, and inorganic acids. Catecholamines are oxidized, forming vanillylmandelic acid (VMA) and homovanillic acid (HVA).

A single-point calibration is performed between each blood gas sample in order to: A. Correct the electrode slope B. Correct electrode and instrument drift C. Compensate for temperature variance D. Prevent contamination by the previous sample

Correct Answer: B 35. B Calibration using a single standard corrects the instrument for error at the labeled value of the calibrator but does not correct for analytic errors away from the set point. A two-point calibration adjusts the slope response of the electrode, eliminating proportional error caused by poor electrode performance.

What is the first day in the second half of the month that patient results would be rejected? A. Day 16 B. Day 17 C. Day 18 D. Day 19

Correct Answer: B 35. B The 41s rule is broken across QC levels on day 17. This means that four consecutive controls are greater than ±1s from the mean. QC rules that are sensitive to SE are applied across both runs and levels to increase the probability of error detection. These are 22s, 41s, and 10×.

Which statement accurately describes serum transaminase levels in AMI? A. ALT is increased 5- to 10-fold after an AMI B. AST peaks 24-48 hours after an AMI and returns to normal within 4-6 days C. AST levels are usually 20-50 times the upper limit of normal after an AMI D. Isoenzymes of AST are of greater diagnostic utility than the total enzyme level

Correct Answer: B 48. B ALT may be slightly elevated after an AMI. AST levels can be up to 5-10 times the URL after AMI, but elevations of this range are also seen in patients with muscular dystrophy, crush injury, pulmonary embolism, infectious mononucleosis, and cancer of the liver.

What is the percentage of serum calcium that is ionized (Cai)? A. 30% B. 45% C. 60% D. 80%

Correct Answer: B 52. B Calcium exists in serum in three forms: protein bound, ionized, and complexed (as undissociated salts). Only Cai is physiologically active. Protein bound and Cai each account for approximately 45% of total calcium, and the remaining 10% is complexed.

Which of the following statements about the phosphatases is true? A. They hydrolyze adenosine triphosphate and related compounds B. They are divided into two classes based upon pH needed for activity C. They exhibit a high specificity for substrate D. They are activated by Pi

Correct Answer: B 55. B Phosphatases are classified as either alkaline or acid depending upon the pH needed for optimum activity. The phosphatases hydrolyze a wide range of monophosphoric acid esters. ALP is inhibited by phosphorus (product inhibition). The International Federation of Clinical Chemistry (IFCC) recommended method employs 2-amino-2-methyl-1-propanol, a buffer that binds Pi

The urea nitrogen concentration of a serum sample was measured to be 15 mg/dL. urea; NH2CONH2 atomic weight: N=14 C=12 O=16 H=1 The urea concentration of the same sample, in mg/dL, is: A. 15 B. 24 C. 32 D. 40

Correct Answer: C

Total iron-binding capacity measures the serum iron transporting capacity of: A. hemoglobin B. ceruloplasmin C. transferrin D. ferritin

Correct Answer: C

In which case would eGFR derived from the plasma creatinine likely give a more accurate measure of GFR than measurement of plasma cystatin C? A. Diabetic patient B. Chronic renal failure C. Post-renal transplant D. Chronic hepatitis

Correct Answer: C 12. C Cystatin C is eliminated almost exclusively by the kidneys and plasma levels are not dependent on age, sex, or nutritional status. However, plasma levels are affected by some drugs, including those used to prevent renal transplant rejection. Increased plasma levels have been reported in chronic inflammatory diseases and cancer. Formulas are available to calculate eGFR from plasma cystatin C, but unlike for creatinine, the formulas must be matched to the method of assay. The eGFR derived from cystatin C can detect a fall in GFR sooner and may be more sensitive for diabetic and other populations at risk for chronic kidney disease. As a screening test for eGFR, it has about the same predictive value as eGFR derived from creatinine.

What is the blood pH when the partial pressure of carbon dioxide (PCO ) is 60 mm Hg and the bicarbonate concentration is 18 mmol/L? A. 6.89 B. 7.00 C. 7.10 D. 7.30

Correct Answer: C 3. C Solve using the Henderson-Hasselbalch equation. pH = pK ́ + log HCO3-/(0.03 × PCO2), where pK ́, the negative logarithm of the combined hydration and dissociation constants for dissolved CO2 and carbonic acid, is 6.1 and 0.03 is the solubility coefficient for CO2 gas. pH = 6.1 + log 18/(0.03 × 60) = 6.1 + log 18/1.8 pH = 6.1 + log 10. Because log 10 = 1, pH = 7.10

Which of the following statements regarding iron metabolism is correct? A. Iron absorption is decreased by alcohol ingestion B. Normally, 40%-50% of ingested iron is absorbed C. The daily requirement is higher for pregnant and menstruating women D. Absorption increases with the amount of iron in the body stores

Correct Answer: C 37. C For adult men and nonmenstruating women, approximately 1-2 mg/day of iron is needed to replace the small amount lost mainly by exfoliation of cells. Because 5%-10% of dietary iron is absorbed normally, the daily dietary requirement in this group is 10-20 mg/day. Menstruating women have an additional requirement of 1 mg/day and pregnant women 2 mg/day. Absorption efficiency will increase in iron deficiency and decrease in iron overload. Iron absorption is enhanced by low gastric pH and is increased by alcohol ingestion.

The study of pharmacogenomics involves which type of testing? A. Family studies to determine the inheritance of drug resistance B. Testing drugs with cell cultures to determine the minimum toxic dosage C. Testing for single nucleotide polymorphisms known to affect drug metabolism D. Comparison of dose-response curves between family members

Correct Answer: C 4. C Pharmacogenomics refers to the study of genes that affect the performance of a drug in an individual. One method is to test for single nucleotide polymorphisms (SNPs) using DNA microarrays in genes such as those that code for the cytochrome P450 enzymes involved in the metabolism of many drugs. Genetic variations of one such enzyme may account for individual pharmacokinetic differences and can be used to predict the efficacy of the drug.

Which condition is associated with the lowest percent saturation of transferrin? A. Hemochromatosis B. Anemia of chronic infection C. Iron deficiency anemia D. Noniron deficiency anemia

Correct Answer: C 40. C Percent saturation = Serum Fe × 100/TIBC. Normally, transferrin is one-third saturated with iron. In iron deficiency states, the serum iron falls but transferrin rises. This causes the numerator and denominator to move in opposite directions, resulting in very low percent saturation (about 10%). The opposite occurs in hemochromatosis and sideroblastic anemia, resulting in an increased percent saturation.

Select the order of mobility of lipoproteins electrophoresed on cellulose acetate or agarose at pH 8.6. A. - Chylomicrons→pre-β →β→α+ B. - β→pre-β→α→chylomicrons + C. - Chylomicrons →β→pre-β→α + D. - α→β→pre-β→chylomicrons +

Correct Answer: C 42. C Although pre-β lipoprotein is lower in density than β lipoprotein, it migrates faster on agarose or cellulose acetate owing to its more negative apoprotein composition. When lipoproteins are separated on polyacrylamide gel, pre-β moves slower than β lipoprotein. Molecular sieving causes migration to correlate with lipoprotein density when PAGE is used.

A new method for BUN is evaluated by comparing the results of 40 paired patient samples to the urease-UV method. Normal and high controls were run on each shift for 5 days, five times per day. The results are as follows: -Linear Regression:ŷ = -0.3 + 0.90x -Low Control: -x = 14.2 mg/dL; s = 1.24 -High Control: -x = 48.6 mg/dL; s = 1.12 What is the total analytical error estimate for a sample having a concentration of 50 mg/dL? A. -2.2 mg/dL B. -2.8 mg/dL C. -7.5 mg/dL D. -10.0 mg/dL

Correct Answer: C 45. C Linear regression analysis gives an estimate of SE, which is equal to (ŷ - xc) where xc is the expected concentration, and ŷ is the value predicted by the linear regression equation. SE = [-0.3 + (0.9 × 50 mg/dL)] - 50.0 mg/dL = 44.7-50.0 = -5.3 mg/dL The standard deviation of the new method for the high control is used to estimate the RE because the mean of this control is nearest to the expected concentration of 50 mg/dL. RE is estimated by ±1.96 × s. RE = 1.96 × 1.12 = ± 2.2 mg/dL Total analytical error (TE) is equal to the sum of SE and RE. TE = SE + RE = -5.3 mg/dL + (-2.2 mg/dL) = -7.5 mg/dL

Given the serum protein electrophoresis pattern shown, which transaminase results would you expect? See Harr pg 325 A. Within normal limits for both B. Marked elevation of both (20-50-fold normal) C. Mild elevations of both (2-5-fold normal) D. Marked elevation of AST but normal ALT

Correct Answer: C 49. C The protein electrophoresis and densitometric scan show a significantly reduced albumin and polyclonal gammopathy. The densitometric scan shows beta-gamma bridging that supports a diagnosis of hepatic cirrhosis. In this condition one would expect two- to fivefold increases of both transaminases with an ALT:AST ratio below 1.

What is the normal ratio of bicarbonate to dissolved carbon dioxide (HCO3-:dCO2) in arterial blood? A. 1:10 B. 10:1 C. 20:1 D. 30:1

Correct Answer: C 5. C When the ratio of HCO3-:dCO2 is 20:1, the log of salt/acid becomes 1.3. Substituting this in the Henderson-Hasselbalch equation and solving for pH gives pH = 6.1 + log 20; pH = 6.1 + 1.3 = 7.4. Acidosis results when this ratio is decreased, and alkalosis when it is increased.

The serum level of which of the following laboratory tests is decreased in both VDDR and VDRR? A. Vitamin D B. Calcium C. Pi D. Parathyroid hormone

Correct Answer: C 50. C Persons with VDDR and VDRR have a low Pi . However, persons with VDDR have decreased serum calcium, as well. Parathyroid hormone (PTH) is increased in persons with VDDR because calcium is the primary stimulus for PTH release, but not in persons with VDRR. Vitamin D levels vary depending upon the type of rickets and the vitamin D metabolite that is measured. 1,25(OH)D, the active form of vitamin D, is low in type 1 but high in type 2 VDDR. It may be either normal or low in VDRR.

Which of the following statements regarding transaminases is true? A. ALT is often increased in muscular disease, pancreatitis, and lymphoma B. ALT is increased in infectious mononucleosis, but AST is usually normal C. ALT is far more specific for liver diseases than is AST D. Substrate depletion seldom occurs in assays of serum from hepatitis cases

Correct Answer: C 52. C ALT is far more specific for liver disease than AST. High ALT may result from nonhepatic causes such as AMI, muscle injury or disease, and severe hemolysis, but nonhepatic sources can be ruled out by a high direct bilirubin. Elevated ALT (e.g., >65 IU/L) is used along with immunologic tests for hepatitis to disqualify blood donors. AST is increased in muscle disease, MI, pancreatitis, and lymphoma. Both transaminases are moderately increased in infectious mononucleosis.

What is the lipid testing protocol for adults recommended by the National Cholesterol Education Program (NCEP) to evaluate risk for atherosclerosis beginning at age 20? A. Total cholesterol, fasting or nonfasting every year B. Total cholesterol, fasting, every 2 years C. Lipid profile, fasting, every 5 years D. LDL cholesterol, fasting, every 2 years

Correct Answer: C 53. C Because LDL cholesterol, HDL cholesterol, VLDL cholesterol, and triglycerides are all risk factors for coronary artery disease, NCEP recommends a fasting lipid profile to include triglycerides, total cholesterol, HDL cholesterol, and LDL cholesterol be performed every 5 years beginning at age 20. However, because LDL cholesterol is the target of treatment, therapeutic goals are based on the LDL cholesterol. New guidelines recommend an LDL cholesterol goal below 70 mg/dL for the highest-risk persons.

Treatment recommendations for patients with coronary heart disease are based upon measurement of which analyte? A. HDL cholesterol B. Apo-B100 C. LDL cholesterol D. Total cholesterol

Correct Answer: C 55. C NECP has identified LDL cholesterol as the target of therapy for reducing the risk of heart attack because lowering LDL cholesterol has proven to be an effective intervention. The greater the risk of coronary heart disease, the lower the cutpoint for intervention. For persons at high risk (a 10-year risk of heart attack > 20%) the cutpoint is ≥ 100 mg/dL for initiation of statin therapy. For highest-risk persons (those that have acute coronary syndrome, and multiple or uncontrolled risk factors) the treatment goal is LDL cholesterol below 70 mg/dL.

Which of the following values is the threshold critical value (alert or action level) for low plasma potassium? A. 1.5 mmol/L B. 2.0 mmol/L C. 2.5 mmol/L D. 3.5 mmol/L

Correct Answer: C 59. C The reference range for potassium is 3.6-5.4 mmol/L. However, values below 2.5 mmol/L require immediate intervention because below that level there is a grave risk of cardiac arrhythmia, which can lead to cardiac arrest. The upper alert level for potassium is usually 6.5 mmol/L, except for neonatal and hemolyzed samples. Above this level, there is danger of cardiac failure.

What is the PCO2 if the dCO2 is 1.8 mmol/L? A. 24 mm Hg B. 35 mm Hg C. 60 mm Hg D. 72 mm Hg

Correct Answer: C 6. C Dissolved CO2 is calculated from the measured PCO2 × 0.0306, the solubility coefficient for CO2 gas in blood at 37°C. dCO2 = PCO2 × 0.03 Therefore, PCO2 = dCO2 /0.03 PCO2 = 1.8 mmol/L ÷ 0.03 mmol/ L per mm Hg = 60 mm Hg

Select the most appropriate adult reference range for fasting blood glucose. A. 40-105 mg/dL (2.22-5.82 mmol/L) B. 60-140 mg/dL (3.33-7.77 mmol/L) C. 65-99 mg/dL (3.61-5.50 mmol/L) D. 75-150 mg/dL (4.16-8.32 mmol/L)

Correct Answer: C 6. C Reference ranges vary slightly depending upon method and specimen type. Enzymatic methods specific for glucose have an upper limit of normal no greater than 99 mg/dL. This is the cutoff value for impaired fasting plasma glucose (prediabetes) recommended by the American Diabetes Association. Although 65 mg/dL is considered the 2.5 percentile, a fasting level below 50 mg/dL is often seen without associated clinical hypoglycemia, and neonates have a lower limit of approximately 40 mg/dL owing to maternal insulin.

Which of the following enzymes is common to all enzymatic methods for triglyceride measurement? A. Glycerol phosphate oxidase B. Glycerol phosphate dehydrogenase C. Glycerol kinase D. Pyruvate kinase

Correct Answer: C 60. C All enzymatic triglyceride methods require lipase to hydrolyze triglycerides, and glycerol kinase to phosphorylate glycerol, forming glycerol-3-phosphate. The most common method couples glycerol kinase with glycerol phosphate oxidase and peroxidase. 1. Triglyceride + H2O Lipase --> glycerol + fatty acids 2. Glycerol + ATP GK --> glycerol-3-phosphate + ADP 3. Glycerol-3-phosphate + O2 GPO --> dihydroxyacetone phosphate + H2O2 H2O2 + phenol + 4-aminophenazone Px --> quinoneimine dye + H2O GK = glycerol kinase; GPO = glycerol phosphate oxidase; Px = peroxidase

Select the reagent needed in the coupling enzyme reaction used to generate a colored product in the cholesterol oxidase method for cholesterol. A. Cholestahexaene B. H2O2 C. 4-Aminoantipyrine D. Cholest-4-ene-3-one

Correct Answer: C 61. C In the cholesterol oxidase method, cholesterol ester hydrolase converts cholesterol esters to free cholesterol by hydrolyzing the fatty acid from the C3-OH group. Cholesterol oxidase catalyzes the oxidation of free cholesterol at the C3-OH group forming cholest-4-ene-3-one and hydrogen peroxide. The peroxide is used in a peroxidase reaction to oxidize a dye (e.g., 4-aminoantipyrine), which couples to phenol, forming a red quinoneimine complex.

What method is used to introduce the sample into a mass spectrometer for analysis of a trace element? A. Electrospray ionization B. Laser desorption C. Inductively charged plasma (ICP) ionization D. Direct injection

Correct Answer: C 74. C Mass spectrometers can be used to measure trace metals, but the atoms need to be vaporized and ionized like molecules before they enter the mass filter. This is done by introducing the sample into a very hot plasma (6,000-10,000°K) called a torch. The torch is made by circulating argon through inner and outer quartz tubes. The tubes are wrapped with a coil of wire that receives a radio frequency. This creates current flow through the wire and a magnetic field at the torch end. Argon atoms are excited by the current and magnetic field and ionize. When the argon is ignited by a spark, it forms the plasma. The sample is mixed with argon at the other end to create an aerosol. When it reaches the torch, the solvent is evaporated and the energy from the torch and collisions with argon ions cause ejection of outer- shell electrons, forming cations of the element. ICP-MS is used to measure any trace element that readily forms cations.

The reference method for serum lipase is based upon: A. Assay of triglycerides following incubation of serum with olive oil B. Rate turbidimetry C. Titration of fatty acids with dilute NaOH following controlled incubation of serum with olive oil D. Immunochemical assay

Correct Answer: C 76. C The reference method of Cherry and Crandall is based upon the titration of fatty acids formed by the hydrolysis of an emulsion of olive oil after incubation for 24 hours at 37°C. Because most of the activity occurs within the first 3 hours, the incubation time may be shortened to as little as 1 hour without loss of clinical utility.

Which enzyme is most likely to be elevated in the plasma of a person suffering from a muscle wasting disorder? A. 5 ́-Nucleotidase B. Pseudocholinesterase C. Aldolase D. Glutamate dehydrogenase

Correct Answer: C 79. C 5'-Nucleotidase is increased primarily in obstructive liver disease and liver cancer. When elevated along with ALP, it identifies the liver as the source of ALP. Glutamate dehydrogenase is increased in necrotic liver diseases along with transaminases, but because of its distribution it is elevated to a greater extent in toxic hepatitis and therefore is useful as a marker for halothane (anesthesia) toxicity. Aldolase is found in all tissues and is increased in many conditions including myocardial infarction, viral hepatitis, and myelocytic leukemia. However, like CK, the greatest increase is seen in skeletal muscle-wasting disease such as muscular dystrophies.

A 1-year-old girl with a hyperlipoproteinemia and lipase deficiency has the following lipid profile -cholesterol: 300 mg/dL (7.77 mmol/L) -LDL: increased -HDL: decreased -triglycerides: 200 mg/dL (2.26 mmol/L) -chylomircrons: present A serum specimen from this patient that was refrigerated overnight would most likely be: A. clear B. cloudy C. creamy layer over cloudy serum D. creamy layer over clear serum

Correct Answer: D

A 21-year-old man with nauseas, vomitting, and jaundice has the following laboratory findings: Test; patient; Reference range -total serum bilirubin; 8.5 mg/dL (145.4 umol/L); 0-1.0 mg/dL (0.0- 17.1 umol/L) -conjugated serum bilirubin; 6.1 mg/dL (104.3 umol/L); 0-0.5 mg/dL (0.0-8.6 umol/L) -urine urobilinogen; increased -fecal urobilinogen: decreased -urine bilirubin: positive -AST: 200 U/L; 0-50 U/L -alkaline phosphatase: 170 U/L; 0-150 U/L These can best be explained as representing: A. unconjugated hyperbilirubinemia, probably due to hemolysis B. unconjugated hyperbilirubinemia, probably due to toxic liver damage C. conjugate hyperbilirubinemia, probably due to biliary tract disease D. conjugated hyperbilirubinemia, probably due to hepatocellular obstruction

Correct Answer: D

A common cause of respiratory alkalosis is: A. vomiting B. starvation C. asthma D. hyperventilaiton

Correct Answer: D

A patient has the following results: Patient values: Reference values -serum iron: 250 ug/dL 944.9 umol/L); 60-150 ug/dL (10.7 - 26.0 umol/L) -TIBC: 350 ug/dL (62.7 umol/L); 300-350 ug/dL) (53.7 - 62.7 umol/L) The best conclusion is that has: A. normal iron status B. iron deficiency anemia C. chronic disease D. iron hemochromatosis

Correct Answer: D

A patient is admitted with biliary cirrhosis. If a serum protein electrophoresis is performed, which of the following globulin fractions will be most elevated? A. alpha-1 B. alpha-2 C. beta D. gamma

Correct Answer: D

A patient with glomerulonephritis is most likely to present with the following serum results: A. creatinine decreased B. calcium increased C. phosphorous decreased D. BUN increased

Correct Answer: D

A physician suspects his patient has pancreatitis. Which test(s) would be most indicative of this disease? A. creatinine B. LD isoenzymes C. beta-hydroxybutyrate D. amylase

Correct Answer: D

Aldosterone is released by the adrenal cortex upon stimulation by: A. renin B. angiotensinogen C. angiotensin I D. angiotensin II

Correct Answer: D

IN the Jaffe reaction, creatinine reacts with A. alkaline sulfasalazine solution to produce an orange-yellow complex B. potassium iodide to form a reddish-purple complex C. sodium nitroferricyanide to yield a reddish-brown color D. alkaline picrate solution to yield an orange-red complex

Correct Answer: D

In a specimen collected for plasma glucose analysis, sodium fluoride: A. serves as a coenzyme of hexokinase B. prevents reactivity of non-glucose reducing substances C. precipitates proteins D. inhibits glycolysis

Correct Answer: D

In using ion-exchange chromatographic methods, falsely increased levels of Hgb A2, might be demonstrated in the presence of: A. iron deficiency anemia B. pernicious anemia C. thalassemias D. Hgb S

Correct Answer: D

In which of the following conditions does decreased activity of glucuronyl transferase result in increased unconjugated bilirubin and kernicterus in neonates? A. Gilbert disease B. Rotor syndrome C. Dubin-Johnson syndrome D. Crigler-Jajjar syndrome

Correct Answer: D

Increased serum albumin concentrations are seen in which of the following conditions? A. nephrotic syndrome B. acute hepatitis C. chronic inflammation D. dehydration

Correct Answer: D

Isoenzyme assays are performed to improve: A. precision B. accuracy C. sensitivity D. specificity

Correct Answer: D

Lithium therapy is widely used in the treatment of: A. hypertension B. hyperactivity C. aggression D. manic-depressive (bipolar) disorder

Correct Answer: D

Magnesium carbonate is added in an iron binding capacity determination in order to: A. allow color to develop B. precipitate protein C. bind with hemoglobin iron D. remove excess unbound iron

Correct Answer: D

Malid dehydrogenas is added to the aspartate aminotransaminase (AST) reaction to catalyze the conversion of: A. alpha-ketoglutarate to aspartate B. alpha-ketoglutarate to malate C. aspartate to oxalacetate D. oxalacetate to malate

Correct Answer: D

Refer to the following illustration: See BOC pg 102 Which of the following is the most likely interpretation of the LD isoenzyme scan illustrated above? A. myocardial infarction B. megaloblastic anemia C. acute pancreatitis D. viral hepatitis

Correct Answer: D

THe principle of the tablet test for bilirubin in urine or feces is: A. the reaction between bile and 2,4-dichloronitrobenzene to a yellow color B. the liberation of oxygen by bile to oxidize orthotolidine to a blue -purple color C. chemical coupling of bile with a diazonium slat to form a brown color D. chemical coupling of bilirubin with a diazonium salt to form a purple color

Correct Answer: D

The bicarbonate and carbonic acid ration is calculate form an equation by: A. Siggaard-Andersen B. Gibbs-Donnan C. Natelson D. Henderson-Hasselbalch

Correct Answer: D

The first step in the quantitation of serum iron is: A. direct reaction with appropriate chromogen B. iron saturation of transferrin C. free iron precipitation D. separation of iron form transferring

Correct Answer: D

The following results were obtained in a creatinine clearance evaluation: -urine concentration: 84 mg/dL -urine volume: 1, 440 mL/24 hr -serum concentration: 1.4 mg/dL -body surface area: 1.60 m2 (average =1.73 m2) The creatinine clearance in mL/min is: A. 6 B. 22 C. 60 D. 65

Correct Answer: D

The function of the major lipid components of the very low-density lipoproteins (VLDL) is to transport: A. cholesterol form peripheral cells to the liver B. cholesterol and phospholipids to peripheral cells C. exogenous triglycerides D. endogenous triglycerides

Correct Answer: D

The hemoglobin that is resistant to alkali (KOH) denaturation is: A. A B. A2 C. C D. F

Correct Answer: D

The international Federation for Clinical Chemistry (IFCC) recommends the use of methods such as the Bessey-Lowry Brock method for determining alkaline phosphatase activity. The substrate used in this type of method is: A. monophophate B. phenylphosphate C. disodium phenylphosphate D. para-nitrophenylphosphate

Correct Answer: D

The metabolite 11-nor-tetrahydrocannabinol-9-COOH can be detected by immunoassay 3-5 days after a single use of: A. methoamphetamine B. cocaine C. benzodiazepine D. marijuana

Correct Answer: D

Urobilinogen is formed in the: A. kidney B. spleen C. liver D. intestine

Correct Answer: D

Which of the following calcium procedures utilizes lanthanum chloride to eliminate interfering substances? A. o-cresolphthalein complexone B. precipitation with chloranilic acid C. chelation with EDTA D. atomic absorption spectrophotometry

Correct Answer: D

Which of the following clinical disorders is associated with the greatest elevation of lactate dehydrogenase isoenzyme 1? A. pneumonia B. glomerulonephritis C. pancreatitis D. pernicious anemia

Correct Answer: D

Which of the following serum proteins migrate with the beta-globulins on cellulose acetate at pH 8.6? A. ceruloplasmin B. hemoglobin C. haptoglobin D. C3 component of complement

Correct Answer: D

Which of the following statements most correctly describes the utility of the clinical laboratory assays for tumor markers? A. tumor markers are useful to screen asymptomatic patients for tumor B. tumor markers are highly specific C. tumor markers indicate the likelihood of an individual developing a tumor D. tumor markers are useful in tracking the efficacy of treatment

Correct Answer: D

Which one of the following sets of results is consistent with primary hypothyroidism, (eg, Hashimoto thyroiditis): Result; TSH; T4 ( free thyroxine); Antimicrosomal antibody -result A; decreased; decreased; positive -result B; increased; increase; positive -result C; normal; decreased; negative -result D; increased; decreased; positive A. result A B. result B C. result C D. result D

Correct Answer: D

Which statement about steady-state drug levels is true? A. The absorbed drug must be greater than the amount excreted B. Steady state can be measured after two elimination half-lives C. Constant intravenous infusion will give the same minima and maxima as an oral dose D. Oral dosing intervals give peaks and troughs in the dose-response curve

Correct Answer: D 11. D When drugs are infused intravenously, both the distribution and elimination rates are constant. This eliminates the peaks and troughs seen in the dose-response curve. Peak and trough levels are characteristics of intermittent dosing regimens. The steady state is reached when drug in the next dose is sufficient only to replace the drug eliminated since the last dose. Steady state can be measured after five drug half-lives because blood levels will have reached 97% of steady state.

SITUATION: The following lab results are reported. Which result is most likely to be erroneous? Arterial blood gases: -pH = 7.42 -pCO2 = 38.0 mm Hg -pO2 = 90 mm Hg bicarbonate = 24 mmol/L. Plasma electrolytes: -Na = 135 mmol/L -K = 4.6 mmol/L -Cl = 98 mmol/L -TCO2 = 33 mmol/L A. pH B. Na C. K D. TCO2

Correct Answer: D 12. D The pH, pCO2, and bicarbonate are normal, and therefore, agree. The electrolytes are normal also, but the TCO2 is increased significantly. The reference range for venous TCO2 is 22-28 mmol/L. Although TCO2 is the sum of bicarbonate and dissolved CO2, the venous TCO2 is determined almost entirely by the bicarbonate, since dCO2 is lost as CO2 gas when the venous blood is exposed to air during processing. A TCO2 value of 32 mmol/L would be expected in a person with metabolic alkalosis.

If the peak level is appropriate but the trough level too low at steady state, then the dose interval should: A. Be lengthened without changing the dose per day B. Be lengthened and dose rate decreased C. Not be changed, but dose per day increased D. Be shortened, but dose per day not changed

Correct Answer: D 13. D Increasing the dose rate may result in peak drug levels in the toxic range. Decreasing the dosing interval will raise the trough level so that it is maintained in the therapeutic range. The trough level is affected by the drug clearance rate. If clearance increases, then trough level decreases.

What is the maximum recommended storage time and temperature for an arterial blood gas sample drawn in a plastic syringe? Storage Time; Temperature A. 10 min; 2°C-8°C B. 20 min; 2°C-8°C C. 30 min; 2°C-8°C D. 30 min; 22°C

Correct Answer: D 17. D Arterial blood gas samples collected in plastic syringes should be stored at room temperature because cooling the sample allows oxygen to enter the syringe. Storage time should be no more than 30 minutes because longer storage results in a significant drop in pH and PO2 and increased PCO2.

How many milliliters of glacial acetic acid are needed to prepare 2.0 L of 10.0% v/v acetic acid? A. 10.0 mL B. 20.0 mL C. 100.0 mL D. 200.0 mL

Correct Answer: D 2. D The expression percent v/v refers to the volume of one liquid in mL present in 100.0 mL of solution. To calculate, multiply the percentage (as mL) by the volume required (mL), then divide by 100 (mL). (10.0 mL × 2000.0 mL) ÷ 100.0 mL = 200.0 mL To prepare 2.0 L of a 10.0% v/v solution of acetic acid, add approximately 1.0 L of deionized H2O to a 2.0-L volumetric flask. Add 200.0 mL of glacial acetic acid and mix. Then, add sufficient deionized H2O to bring the meniscus to the 2.0-L line and mix again.

Two consecutive controls are both beyond -2s from the mean. How frequently would this occur on the basis of chance alone? A. 1:100 B. 5:100 C. 1:400 D. 1:1,600

Correct Answer: D 21. D QC results follow a Gaussian or normal distribution. Ninety-five percent of the results fall within ±2s of the mean; therefore, 2.5 out of 100 (1:40) are above +2s and 2.5 out of 100 are below -2s. The probability of two consecutive controls being beyond -2s is the product of their individual probabilities. 1/40 × 1/40 = 1/1,600 trials by chance.

A technologist is asked to use the serum from a clot tube left over from a chemistry profile run at 8 a.m. for a stat ionized calcium (Cai) at 11 a.m. The technologist should: A. Perform the assay on the 8 a.m. sample B. Perform the test only if the serum container was tightly capped C. Perform the assay on the 8 a.m. sample only if it was refrigerated D. Request a new sample

Correct Answer: D 28. D Cai is pH dependent. Heparinized blood is preferred because it can be assayed immediately. Serum may be used, but the specimen must remain tightly capped while clotting and centrifuging, and analyzed as soon as possible.

Which of the following conditions is associated with both metabolic and respiratory alkalosis? A. Hyperchloremia B. Hypernatremia C. Hyperphosphatemia D. Hypokalemia

Correct Answer: D 29. D Hypokalemia is both a cause and result of alkalosis. In alkalosis, hydrogen ions may move from the cells into the extracellular fluid and potassium into the cells. In hypokalemia caused by overproduction of aldosterone, hydrogen ions are secreted by the renal tubules. This increase in net acid excretion results in metabolic alkalosis.

A lipemic sample gives a sodium of 130 mmol/L on an analyzer that uses a 1:50 dilution of serum or plasma before introducing it to the ion selective electrodes. The same sample gives a sodium of 142 mmol/L using a direct (undiluted) ion selective electrode. Assuming acceptable quality control, which of the following is the most appropriate course of action? A. Report a sodium result of 136 mmol/L B. Ultracentrifuge the sample and repeat by ISE C. Dilute the sample 1:4 and repeat by ISE D. Report the undiluted ion selective electrode result

Correct Answer: D 3. D Lipemic samples give lower results for sodium (pseudohyponatremia) when diluted prior to measurement because the H2O phase is mostly diluent and a significant component of the sample volume is displaced by lipid. Direct ISEs measure sodium in the plasma water, more accurately reflecting patient status.

Given the following QC chart, identify the day in which a violation of the R4s QC rule occurs. See Harr pg 231 A. Day 3 B. Day 8 C. Day 10 D. Day 15

Correct Answer: D 36. D An R4s error is defined as the algebraic difference between two controls within the same run. In this Levy-Jennings plot, on day 15, Level 1 is above the +2s limit (approximately +2.5s) and Level 2 is below the -2s limit (approximately -2.5s). These controls are approximately 5s apart (+2.5s minus -2.5s = +5s).

Two methods for total cholesterol are compared by running 40 paired patient samples in duplicate on each instrument. The following results are obtained: Instrument; Mean;Standard Deviation -Method x (reference method); 235 mg/dL; 3.8 method) -Method y (candidate method); 246 mg/dL; 3.4 Assuming the samples are collected and stored in the same way and the analysis done by a technologist who is familiar with both methods, what is the bias of method y? A. 0.4 B. 7.2 C. 10.6 D. 11.0

Correct Answer: D 40. D The bias is defined as the difference between the means of the two methods and is calculated using the formula: bias = y - ×. The bias is an estimate of SE. The student's t test is used to determine if bias is statistically significant. The t statistic is the ratio of bias to the standard error of the mean difference. The greater the bias, the higher the t score.

SITUATION: A patient previously diagnosed with primary hypothyroidism and started on thyroxine replacement therapy is seen for follow-up testing after 2 weeks. The serum-free T4 is normal but the TSH is still elevated. What is the most likely explanation for these results? A. Laboratory error in measurement of free T4 B. Laboratory error in measurement of TSH C. In vitro drug interference with the free T4 assay D. Results are consistent with a euthyroid patient in the early phase of therapy

Correct Answer: D 42. D Results of thyroid tests (especially in hospitalized patients) may sometimes appear discrepant because medications and nonthyroid illnesses can affect test results. The pituitary is slow to respond to thyroxine replacement, and 6-8 weeks are usually required before TSH levels fall back to normal. In the early stage of therapy, the patient should be monitored by the free T4 result. This patient's free T4 is normal, indicating that replacement therapy is adequate. The high TSH sometimes seen in treated patients is called pituitary lag.

In addition to the number of true negatives (TN), which of the following measurements is needed to calculate specificity? A. True positives B. Prevalence C. False negatives D. False positives

Correct Answer: D 46. D The clinical specificity of a laboratory test is defined as the true negatives divided by the sum of true negatives and false positives (FP). % Specificity = TN × 100 / TN + FP Specificity is defined as the percentage of disease-free people who have a negative test result. The probability of false positives is calculated from the specificity as: 1-(% specificity)

Which of the following statements regarding the metabolism of bilirubin is true? A. It is formed by hydrolysis of the α methene bridge of urobilinogen B. It is reduced to biliverdin prior to excretion C. It is a by-product of porphyrin production D. It is produced from the destruction of RBCs

Correct Answer: D 47. D Synthesis of porphyrins results in production of heme and metabolism of porphyrins other than protoporphyrin IX yields uroporphyrins and coproporphyrins, not bilirubin. Reticuloendothelial cells in the spleen digest Hgb and release the iron from heme. The tetrapyrrole ring is opened at the α methene bridge by heme oxygenase, forming biliverdin. Bilirubin is formed by reduction of biliverdin at the γ methene bridge. It is complexed to albumin and transported to the liver.

Which of the following is associated with Tangier disease? A. Apoprotein C-II deficiency B. Homozygous apo-B100 deficiency C. Apoprotein C-II activated lipase D. Apoprotein A-I deficiency

Correct Answer: D 51. D Deficiency of apo A-I is seen in Tangier disease, a familial hypocholesterolemia. Heterozygotes have about half of the normal level of HDL (familial hypoalphalipoproteinemia) and homozygotes have almost no detectable HDL. Tangier disease is caused by a mutation of the ATP-binding cassette gene. The deficient gene prevents apo A-I from binding lipids, and it is rapidly catabolized. Abetalipoproteinemia results from defective hepatic transport of apo-B100, and is also inherited as an autosomal recessive condition. LDL is absent, and the condition is associated with hemolytic anemia and central nervous system damage.

Select the most sensitive marker for alcoholic liver disease. A. GLD B. ALT C. AST D. γ-Glutamyltransferase (GGT)

Correct Answer: D 52. C ALT is far more specific for liver disease than AST. High ALT may result from nonhepatic causes such as AMI, muscle injury or disease, and severe hemolysis, but nonhepatic sources can be ruled out by a high direct bilirubin. Elevated ALT (e.g., >65 IU/L) is used along with immunologic tests for hepatitis to disqualify blood donors. AST is increased in muscle disease, MI, pancreatitis, and lymphoma. Both transaminases are moderately increased in infectious mononucleosis.

Which of the following conditions is associated with hypokalemia? A. Addison's disease B. Hemolytic anemia C. Digoxin intoxication D. Alkalosis

Correct Answer: D 57. D Addison's disease (adrenocortical insufficiency) results in low levels of adrenal corticosteroid hormones, including aldosterone and cortisol. Because these hormones promote reabsorption of sodium and secretion of potassium by the collecting tubules, patients with Addison's disease display hyperkalemia and hyponatremia. Hemolytic anemia and digoxin intoxication cause release of intracellular potassium. Alkalosis causes potassium to move from the extracellular fluid into the cells as hydrogen ions move from the cells into the extracellular fluid to compensate for alkalosis.

Which of the following methods for HDL cholesterol is the reference method? A. Manganese-heparin B. Magnesium-phosphotungstate C. Magnesium-dextran D. Ultracentrifugation

Correct Answer: D 63. D Ultracentrifugation of plasma in a potassium bromide solution with a density of 1.063 is used to separate HDL from LDL and VLDL. The HDL fraction is transferred from the bottom of the tube and assayed for cholesterol content by the Abell-Kendall method. The remaining three methods rely upon selective precipitation of lipoproteins containing apoprotein B using a polyanionic solution. All of these methods are subject to interference by very high triglycerides and vary somewhat in specificity depending on the efficiency of precipitation.

Which of the following amylase substrates is recommended by the IFCC? A. Starch B. Maltodextrose C. Maltotetrose D. Blocked maltohepatoside

Correct Answer: D 68. D Amylase is commonly measured using synthetic substrates. In the IFCC-recommended method, p-nitrophenyl maltohepatiside is used. One end of the polymer is covalently linked to p-nitrophenol and the other is linked to 4,6 ethylidine to prevent its hydrolysis by α-glucosidase. Amylase hydrolyzes the substrate from both ends producing fragments of 2, 3, and 4 glucose subunits. α-Glucosidase hydrolyzes the subunits containing p-nitrophenyl groups, forming glucose and p-nitrophenol. The increase absorbance at 405 nm is proportional to amylase activity.

`Laboratory tests are performed on a postmenopausal, 57-year-old female as part of an annual physical examination. The patient's casual plasma glucose is 220 mg/dL, and the glycated hemoglobin (Hb AIC) is 11%. Based on this information, how should the patient be classified? A. Normal glucose tolerance B. Impaired glucose tolerance C. Gestational diabetes mellitus D. Type 2 diabetes mellitus

D. A casual plasma glucose should be less than 200 mg/dL. The reference range for glycated hemoglobin (Hb Alc) is 4-6%. Because the individual is a postmenopausal, 57-year-old female, with abnormal test results being found as part of an annual physical examination, the most likely diagnosis is type 2 diabetes mellitus. The ADA recommends that in the absence of unequivocal hyperglycemia, the glucose result should be confirmed by repeating the casual glucose or performing a fasting plasma glucose on a subsequent day. The ADA does not recommend Hb Alc as a screening test for diabetes mellitus.

Which hemoglobin may be differentiated from other hemoglobins on the basis of its resistance to denature in alkaline solution? A. A, B. A2 C. C D. F

D. Although hemoglobin differentiation is best achieved by use of electrophoresis, hemoglobin F may be differentiated from the majority of human hemoglobins because of its alkali resistance. Hemoglobin F is able to resist denaturation and remain soluble when added to an alkaline solution. In contrast to hemoglobin F, most hemoglobins will denature in alkaline solution and precipitate on the addition of ammonium sulfate. After 1 year of age, the normal concentration of hemoglobin F is less than 1% of the total hemoglobin. However, hemoglobin F may be present in elevated concentrations in disorders that include thalassemia, sickle cell disease, and aplastic anemia.

Which disorder is not associated with an elevated protein level in cerebrospinal fluid? A. Bacterial meningitis B. Multiple sclerosis C. Cerebral infarction D. Hyperthyroidism

D. CSF, an ultrafiltrate of blood plasma, is made in the choroid plexus of the ventricles of the brain. Protein quantification is among the tests generally ordered on CSF; other tests include glucose, culture and sensitivity, and differential cell count. The reference range for CSF protein is 15-45 mg/dL. CSF protein may be quantified using turbidimetric (e.g., sulfosalicylic acid and benzethonium chloride) or dyebinding methods (e.g., Coomassie brilliant blue). Elevated levels of CSF protein are found in such disorders as bacterial, viral, and fungal meningitis; multiple sclerosis; neoplasm; disk herniation; and cerebral infarction. Low levels of CSF protein are found in hyperthyroidism and in CSF leakage from the central nervous system.

What is the major active metabolite of the anticonvulsant drug primidone? A. Phenytoin B. Acetazolamide C. NAPA D. Phenobarbital

D. Following absoiption, primidone is metabolized primarily to phenobarbital and secondarily to phenylethylmalonamide (PEMA). Both metabolites have anticonvulsant activity, and both have a longer half-life than primidone. Generally, only serum phenobarbital and primidone concentrations are monitored. Determination of phenobarbital is particularly important when another anticonvulsant phenytoin is also administered because the metabolic rate of primidone conversion to phenobarbital may be increased, with a resulting accumulation of phenobarbital in the blood.

In gout, what analyte deposits in joints and other body tissues? A. Calcium B. Creatinine C. Urea D. Uric acid

D. Gout is a pathological condition that may be caused by a malfunction of purine metabolism or a depression in the renal excretion of uric acid. Two of the major characteristics of gout are hyperuricemia and a deposition of uric acid as monosodium urate crystals in joints, periarticular cartilage, bone, bursae, and subcutaneous tissue. Such a deposition of urate crystals causes inflammation of the affected area and precipitates an arthritic attack.

Which type of photodetector employs a linear arrangement that allows it to respond to a specific wavelength resulting in complete UV/visible spectrum analysis? A. Photomultiplier tube B. Phototube C. Barrier layer cell D. Photodiode array

D. Photodiode array detectors are designed with 256 to 2048 photodiodes that are arranged in a linear fashion. This arrangement allows each photodiode to respond to a specific wavelength that results in a continuous UV/visible spectrum. Resolution is generally 1-2 nm.

A serum creatinine was found to be 6.0 mg/dL. Which of the following urea nitrogen serum results would support the same pathological condition? A. 6 mg/dL B. 20 mg/dL C. 35 mg/dL D. 70 mg/dL

D. Serum urea nitrogen and creatinine levels are frequently requested together so that their ratio can be evaluated. The normal ratio of serum urea nitrogen to creatinine ranges between 10:1 and 20:1. Abnormal values obtained when kidney function tests are performed may be the result of a prerenal, renal, or postrenal malfunction. The ratio of urea nitrogen to creatinine is sometimes used as an index in the assessment of kidney function and as a means of differentiating the source of the malfunction.

In the plasma, an excess in the concentration of bicarbonate without a change in PCO2 from normal will result in what physiological state? A. Respiratory acidosis B. Respiratory alkalosis C. Metabolic acidosis D. Metabolic alkalosis

D. The acid-base equilibrium of the blood is expressed by the Henderson-Hasselbalch equation: pH = pK' + log cHCOj (PCO2 X 0.03) In this buffer pair, pK' =6.1. Normally, the ratio of the concentration of bicarbonate ions cHCO^ to the concentration of carbonic acid expressed as (PCO2 x 0.03) in the plasma is 20:1. The bicarbonate component of the equation is considered to be the "metabolic" component, controlled by the kidneys. The carbonic acid component is considered the "respiratory" component, controlled by the lungs. An excess of bicarbonate without a change in PCO2 will increase the ratio of bicarbonate to carbonic acid. Therefore, the pH will increase; that is, the plasma becomes more alkaline.

Which of the following best represents the reference (normal) range for arterial pH? A. 7.35-7.45 B. 7.42-7.52 C. 7.38-7.68 D. 6.85-7.56

Correct Answer: A 4. A The reference range for arterial blood pH is 7.35-7.45 and is only 0.03 pH units lower for venous blood owing to the buffering effects of hemoglobin (Hgb) known as the chloride isohydric shift. Most laboratories consider less than 7.20 and greater than 7.60 the critical values for pH.

Upon which principle is the biuret method based? A. The reaction of phenolic groups with CuIISO4 B. Coordinate bonds between Cu+2 and carbonyl and amine groups of peptide bonds. C. The protein error of indicator effect producing color when dyes bind protein D. The reaction of phosphomolybdic acid with protein

Correct Answer: B 2. B Biuret is a compound with two carbonyl groups and three amino groups and forms coordinate bonds with Cu+2 in the same manner as does protein. Therefore, proteins and peptides are both measured in the biuret reaction. The biuret reagent consists of an alkaline solution of copper II sulfate. Tartrate salts are added to keep the copper in solution and prevent turbidity. Potassium iodide prevents autoreduction of Cu+2.

When establishing QC limits, which of the following practices is inappropriate? A. Using last month's QC data to determine current target limits B. Exclusion of any QC results greater than ±2s from the mean C. Using control results from all shifts on which the assay is performed D. Using limits determined by reference laboratories using the same method

Correct Answer: B 25. B Data between ±2 and ±3s must be included in calculations of the next month's acceptable range. Elimination of these values would continuously reduce the distribution of QC results, making "out-of-control" situations a frequent occurrence. Generally, QC results greater than 3s are not used to calculate next month's mean.

Which statement regarding thyroid hormones is true? A. Circulating levels of T3 and T4 are about equal B. T3 is about 10-fold more active than T4 C. The rate of formation of monoiodotyrosine and diiodotyrosine is about equal D. Most of the T3 present in plasma is from its direct release from thyroid storage sites

Correct Answer: B 40. B The rate of DIT synthesis is twice that of MIT and the rate of coupling favors formation of T4. Levels of T4 are about 50 times those of T3, but T3 is approximately 10 times more active physiologically. Eighty percent of circulating T3 is derived from enzymatic conversion of T4 by T4 5 ́-deiodinase.

Select the coupling enzyme used in the kinetic AST reaction of Henry. A. LD B. Malate dehydrogenase C. Glutamate dehydrogenase D. G-6-PD

Correct Answer: B 45. B The method of Henry for AST uses malate dehydrogenase (MD) to reduce oxaloacetate to malate. The electrons come from NADH forming NAD+. Aspartate + α-ketoglutarate AST --> Oxaloacetate + Glutamate Oxaloacetate + NADH + H+ MD --> Malate + NAD+

Which of the following mechanisms accounts for the elevated plasma level of β lipoproteins seen in familial hypercholesterolemia (formerly type II hyperlipoproteinemia)? A. Hyperinsulinemia B. ApoB-100 receptor defect C. ApoC-II activated lipase deficiency D. ApoE3 deficiency

Correct Answer: B 48. B The production of excess insulin leads to hypertriglyceridemia and is one mechanism responsible for familial endogenous hypertriglyceridemia. ApoC-II is an activator of lipoprotein lipase, and a homozygous deficiency results in high plasma chylomicrons and VLDL. ApoE3 deficiency is synonymous with inheritance of two apo-E2 alleles that lead to β dyslipoproteinemia. Familial hypercholesterolemia is inherited as an autosomal dominant trait. The classical form results from one of many mutations affecting the LDL receptor that cause it to have a lower affinity for LDL. A related hypercholesterolemia common in people of European ancestry results from a mutation of the apo-B100 gene that causes LDL to have a lower affinity for the LDL receptor. Together, they make familial hypercholesterolemia the most common inherited hyperlipoproteinemia with a frequency over 1:500.

The following plot represents a study of a screening test for malignant prostate cancer using plasma PSA (ng/mL). The outcome measured was positive cytology results obtained by biopsy. What concentration gives the highest sensitivity with the least number of unnecessary biopsies? See Harr pg 237 A. 2.6 B. 3.6 C. 3.8 D. 5.2

Correct Answer: B 55. B A receiver operating characteristic (ROC) curve is used to identify the test result, giving the highest sensitivity with the least number of false-positive results. Sensitivity (true positives) is plotted against false positives. The number in the uppermost left corner represents the highest detection with the lowest number of false positives. In this case, a result of 3.6 ng/mL detects 72% of malignancies with 1 in 10 (10%) false positives.

Which electrolyte measurement is least affected by hemolysis? A. Potassium B. Calcium C. Pi D. Magnesium

Correct Answer: B 56. B Potassium, phosphorus, and magnesium are the major intracellular ions, and even slight hemolysis will cause falsely elevated results. Serum samples with visible hemolysis (20 mg/dL free Hgb) should be redrawn.

Which statement about colorimetric bilirubin methods is true? A. Direct bilirubin must react with diazo reagent under alkaline conditions B. Most methods are based upon reaction with diazotized sulfanilic acid C. Ascorbic acid can be used to eliminate interference caused by Hgb D. The color of the azobilirubin product is independent of pH

Correct Answer: B 66. B Unconjugated bilirubin is poorly soluble in acid, and therefore, direct bilirubin is assayed using diazotized sulfanilic acid diluted in weak HCl. The direct diazo reaction should be measured after no longer than 3 minutes to prevent reaction of unconjugated bilirubin, or the diazo group can be reduced using ascorbate or hydroxylamine preventing any further reaction.

Which of the following tumor markers is used to monitor persons with breast cancer for recurrence of disease? A. Cathepsin-D B. CA-15-3 C. Retinoblastoma gene D. Estrogen receptor (ER)

Correct Answer: B 7. B CA-15-3 shares the same antigenic determinant as CA-27.29. Both are present on MUC1, a mucinous protein on the cell membrane of various tissues. The markers are used to monitor treatment and recurrence of breast cancer. However, abnormal plasma levels are seen in many nonmalignant conditions, and the test is not used for diagnostic purposes. CA-125 is a glycoprotein antigen shed by approximately 75% of ovarian cancers. It is an FDA-approved tumor marker for monitoring recurrence of ovarian cancer and evaluating the effectiveness of chemotherapy. Cathepsin-D and ER assays are performed to determine the prognosis of persons with breast cancer. Overexpression of cathepsin-D is associated with a higher relapse rate. Breast tissue that is negative for ER is poorly responsive to hormone suppression (tamoxifen) therapy. The retinoblastoma gene (RB) is a tumor-suppressor gene found to be missing in persons with retinoblastoma. Various mutations of the gene have been reported in breast, lung, bladder, and other cancers.

A drug has a half-life of 6 hours. If a dose is given every 6 hours, a stead-state drug level would usually be achieved in: A. 3-5 hours B. 10-12 hours C. 24-42 hours D. 48-50 hours

Correct Answer: C

Acidosis and alkalosis are best defined as fluctuations in blood pH and CO2 content due to changes in: A. Bohr effect B. O2 content C. bicarbonate buffer D. carbonic anhydrase

Correct Answer: C

If the pKa is 6.1, the CO2 content is 25 mM/L, the salt equals the total CO2 content minus the carbonic acid; the carbonic acid equals 0.03 x PCO2 and PCO2- 40 mmHg, it may be concluded that: A. pH=6.1 + log [(40-0.03)/(0.03)] B. pH=6.1 + log [(25-0.03)/(0.03)] C. pH=6.1 + log [(25-1.2)/(1.2)] D. pH= 6.1 + log [(1.2)/(1.2-25)]

Correct Answer: C

Premature atherosclerosis can occur when which of the following becomes elevated? A. chylomicrons B. prostaglandins C. low-density lipoproteins D. high-density lipoproteins

Correct Answer: C

Quantitation of Na+ and K+ by ion-selective electrode is the standard method because: A. dilution is required for flame photometry B. there is no lipoprotein interference C. of advances in electrochemistry D. of the absence of an internal standard

Correct Answer: C

Refer to the folloowing illustration: See BOC pg. 125 Pic 1 This illustration represents the change in absorbance at 340 nm over a period of 8 minutes in an assay for lactate dehydrogenase. True statement about this figure include: A. the reaction is follows zero order kinetics between 5 and 8 minutes B. the reaction is proceeding form lactate to pyruvate C. nonlinearity after 6 minutes is due to substrate exhaustion D. the change in absorbance is due to reduction of NAD to NADH

Correct Answer: C

Select the test which evaluates renal tubular function: A. IVP B. creatinine clearance C. osmolarity D. microscopic urinalysis

Correct Answer: C

Serum concentrations of vitamin B12 are elevated in: A. pernicious anemia in relapse B. patients on chronic hemodialysis C. chronic granulocytic leukemia D. Hodgkin disease

Correct Answer: C

An international unit (IU) of enzyme activity is the quantity of enzyme that: A. Converts 1 μmol of substrate to product per liter B. Forms 1 mg of product per deciliter C. Converts 1 μmol of substrate to product per minute D. Forms 1 μmol of product per liter

Correct Answer: C 1. C The IU is a rate expressed in micromoles per minute. Activity is reported as IUs per liter (IU/L) or mIU/mL. The SI unit for enzyme activity is the katal (1 katal converts 1 mol of substrate to product in 1 second).

SITUATION: A peak blood level for orally administered theophylline (therapeutic range 8-20 mg/L) measured at 8 a.m. is 5.0 mg/L. The preceding trough level was 4.6 mg/L. What is the most likely explanation of these results? A. Laboratory error made on peak measurement B. Specimen for peak level was collected from wrong patient C. Blood for peak level was drawn too soon D. Elimination rate has reached maximum

Correct Answer: C 10. C Sample collection time is critical for accurate therapeutic drug monitoring. Blood for trough levels must be collected immediately before the next dose. Blood collection time for peak levels must not occur prior to complete absorption and distribution of drug. This usually requires 1-2 hours for orally administered drugs. The therapeutic range for theophylline is 8-20 mg/L. These results are most consistent with a peak sample having been drawn prior to complete absorption of the drug.

Which tumor marker is used to determine the usefulness of trastuzumab (Herceptin) therapy for breast cancer? A. PR B. CEA C. HER-2/neu D. Myc

Correct Answer: C 10. C Trastuzumab is an antibody to the HER-2/neu gene product, a tyrosine kinase receptor protein. HER- 2/neu is an oncogene that is overexpressed in some breast cancers. Overexpression is associated with a more aggressive clinical course but responds to treatment with trastuzumab, which blocks the attachment of growth factor to the receptor. The progesterone receptor, like the ER, is used to identify persons with breast cancer who are more likely to respond to estrogen-suppression therapy. Myc is a group of oncogenes that are activated in various cancers, including lung, breast, colon, stomach, leukemia, and lymphoma. HER-2/neu is measured in plasma by immunoassay. ER, PR, and myc are measured in tissue and not plasma using immunohistological stains or FISH.

Which type of filter is best for measuring stray light? A. Wratten B. Didymium C. Sharp cutoff D. Neutral density

Correct Answer: C 12. C Sharp cutoff filters transmit almost all incident light until the cutoff wavelength is reached. At that point, they cease to transmit light. Because they give an "all or none effect," only stray light reaches the detector when the selected wavelength is beyond the cutoff.

In which condition is the LD most likely to be within normal limits? A. Hepatic carcinoma B. Pulmonary infarction C. Acute appendicitis D. Crush injury

Correct Answer: C 14. C LD is increased slightly to moderately in most causes of liver disease. Smallest elevations are seen in obstructive jaundice and highest in hepatic carcinoma and toxic hepatitis, where levels can reach 10-fold the upper reference limit. LD is also increased in crush injury and muscular dystrophies due to skeletal muscle damage, and in pulmonary infarction owing to embolism formation. Amylase is increased in a majority of persons with acute appendicitis, but LD is not.

Which of the following assays is recommended as a screening test for colorectal cancer in persons over 50 years old? A. CEA B. AFP C. Occult blood D. Fecal trypsin

Correct Answer: C 15. C Bleeding in the gastrointestinal tract occurs during the early stages of colorectal cancer when treatment can be most effective. Although occult blood can be caused by many other GI problems, it is not associated with benign polyps and has a sensitivity of over 80% for detection of colorectal cancer. CEA is elevated in less than 60% of such cases. AFP is elevated in only about 5% of colon cancers. Fecal trypsin is not a marker for colorectal cancer, but α1-antitrypsin is present in the stool in a majority of malignant colon tumors owing to intestinal protein loss.

Which component is required in a spectrophotometer in order to produce a spectral absorbance curve? A. Multiple monochromators B. A reference optical beam C. Photodiode array D. Laser light source

Correct Answer: C 15. C There are two ways to perform spectral scanning for compound identification. One is to use a stepping motor that continuously turns the monochromator so that the wavelength aligned with the exit slit changes at a constant rate. A more practical method is to use a diode array detector. This consists of a chip embedded with as many as several hundred photodiodes. Each photodiode is aligned with a narrow part of the spectrum produced by a diffraction grating, and produces current proportional to the intensity of the band of light striking it (usually 1-2 nm in range). The diode signals are processed by a computer to create a spectral absorbance or transmittance curve.

A stat plasma lithium determined using an ion-selective electrode is measured at 14.0 mmol/L. Select the most appropriate course of action. A. Immediately report this result B. Check sample for hemolysis C. Call for a new specimen D. Rerun the lithium calibrators

Correct Answer: C 16. C Lithium in excess of 2.0 mmol/L is toxic (in some laboratories 1.5 mmol/L is the upper therapeutic limit). A level of 14 mmol/L would not occur unless the sample were contaminated with lithium. This would most likely result from collection in a green-stoppered tube containing the lithium salt of heparin.

Which statement regarding reflectometry is true? A. The relation between reflectance density and concentration is linear B. Single-point calibration can be used to determine concentration C. 100% reflectance is set with an opaque film called a white reference D. The diode array is the photodetector of choice

Correct Answer: C 20. C Reflectometry does not follow Beer's law, but the relationship between concentration and reflectance can be described by a logistic formula or algorithm that can be solved for concentration. For example, K/S = (1 - R) 2/2R, where K = Kubelka-Munk absorptivity constant, S = scattering coefficient, R = reflectance density. K/S is proportional to concentration. The white reference is analogous to the 100%T setting in spectrophotometry and serves as a reference signal. Dr = log R0/R1, where Dr is the reflectance density, R0 is the white reference signal, and R1 is the photodetector signal for the test sample.

Which of the following serum protein electrophoresis results suggests an acute inflammatory process? Albumin; α1; α2; β; γ A. Decreased; Increased; Decreased; Normal; Normal B. Normal; Increased; Normal; Increased; Increased C. Decreased; Increased; Increased; Normal; Normal D. Increased; Increased; Increased; Increased; Increased

Correct Answer: C 21. C Acute inflammation is characterized by increased production of acute phase proteins. These include α1-antitrypsin, α1-acid glycoprotein, α1-antichymotrypsin, and haptoglobin. Albumin is slightly decreased. γ- and β-fractions are normal.

SITUATION: Hgb electrophoresis is performed and all of the Hgbs have greater anodal mobility than usual. A fast Hgb (Hgb H) is at the edge of the gel and bands are blurred. The voltage is set correctly, but the current reading on the ammeter is too low. Select the course of action that would correct this problem. A. Reduce the voltage B. Dilute the buffer and adjust the pH C. Prepare fresh buffer and repeat the test D. Reduce the running time

Correct Answer: C 27. C Increased mobility, decreased resolution, and low current result from low ionic strength. Reducing voltage will slow migration but will not improve resolution. Diluting the buffer will reduce the current, resulting in poorer resolution.

Blood ammonia levels are usually measured in order to evaluate: A. Renal failure B. Acid-base status C. Hepatic coma D. Gastrointestinal malabsorption

Correct Answer: C 28. C Hepatic coma is caused by accumulation of ammonia in the brain as a result of liver failure. The ammonia increases central nervous system pH and is coupled to glutamate, a central nervous system neurotransmitter, forming glutamine. Blood and cerebrospinal fluid ammonia levels are used to distinguish encephalopathy caused by cirrhosis or other liver disease from nonhepatic causes and to monitor patients with hepatic coma.

SITUATION: An EDTA sample for TnI assay gives a result of 0.04 ng/mL (reference range 0-0.03 ng/mL). The test is repeated 3 hours later on a new specimen and the result is 0.06 ng/mL. A third sample collected 6 hours later gives a result of 0.07 ng/mL. The EKG showed no evidence of ST segment elevation (STEMI). What is the most likely explanation? A. A false-positive result occurred due to matrix interference B. Heparin should have been used instead of EDTA, which causes false positives C. The patient has suffered cardiac injury D. The patient has had an ischemic episode without cardiac injury

Correct Answer: C 35. C EDTA is the additive of choice for troponin assays because it avoids microclots that can lead to false positive results when serum or heparinized plasma is used. Spurious false positives caused by matrix effects usually revert to normal when the test is repeated on a new sample. An AMI will cause the TnI to increase in subsequent tests. Results between 0.04-0.10 ng/mL are the result of cardiac injury, and indicate either AMI or an increased short-term risk of AMI.

The ion-selective membrane used to measure potassium is made of: A. High-borosilicate glass membrane B. Polyvinyl chloride dioctylphenyl phosphonate ion exchanger C. Valinomycin gel D. Calomel

Correct Answer: C 38. C Valinomycin is an antibiotic with a highly selective reversible-binding affinity for potassium ions. Sodium electrodes are usually composed of a glass membrane with a high content of aluminum silicate. Calcium and lithium ion-selective electrodes are made from organic liquid ion exchangers called neutral carrier ionophores. Calomel is made of mercury covered with a paste of mercurous chloride (Hg°/Hg2Cl2) and is used as a reference electrode for pH.

Which reagent is used in the Jendrassik-Grof method to solubilize unconjugated bilirubin? A. 50% methanol B. N-butanol C. Caffeine D. Acetic acid

Correct Answer: C 65. C A polarity modifier is required to make unconjugated bilirubin soluble in diazo reagent. The Malloy-Evelyn method uses 50% methanol to reduce the polarity of the diazo reagent. Caffeine is used in the Jendrassik-Grof method. This method is recommended because it is not falsely elevated by hemolysis and gives quantitative recovery of both conjugated and unconjugated bilirubin.

Which formula is most accurate in predicting plasma osmolality? A. Na + 2(Cl) + BUN + glucose B. 2(Na) + 2(Cl) + glucose + urea C. 2(Na) + (glucose ÷ 18) + (BUN ÷ 2.8) D. Na + Cl + K + HCO3

Correct Answer: C 70. C Calculated plasma osmolality is based upon measurement of sodium, glucose, and urea. Because sodium associates with a counter ion, two times the sodium estimates the millimoles per liter of electrolytes. Some laboratories multiply by 1.86 instead of 2 to correct for undissociated salts. Dividing glucose by 18 converts from milligrams per deciliter to millimoles per liter. Dividing blood urea nitrogen (BUN) by 2.8 converts from milligrams per deciliter BUN to millimoles per liter urea.

Quality control results for uric acid are as follows: Results should be reported from: Run1; Run2; Run3; Run4; Mean; s -QC1: 3.5; 3.8; 4.1; 4.2 mg/dL; 3.6 mg/dL; 0.40 QC2: 6.8; 7.2; 7.4; 7.5 mg/dL; 7.0 mg/dL; 0.25 A. Run 1 only B. Runs 1 and 2 C. Runs 1, 2, and 3 D. Runs 1, 2, 3, and 4

Correct Answer: C 9. C Although no single result exceeds the 2s limit, the 41s rule is broken on Run 4. This means that both QC1 and QC2 exceeded +1s on Run 3 and Run 4.

A drug that relaxes the smooth muscles of the bronchial passages is: A. acetaminophen B. lithium C. phenytoin D. theophylline

Correct Answer: D

Creatinine clearance is used to estimate the: A. tubular secretion of creatinine B. glomerular secretion of creatinine C. renal glomerular and tubular mass D. glomerular filtration rate

Correct Answer: D

The screen for adrenal cortical hyperfunction with the greatest and specificity is: A. 24-hour urine free cortisol B. plasma cortisol C. urinary 17-hydroxycorticosteroids D. plasma corticosterone

Correct Answer: D

The screening test for congenital hypothyroidism is based upon: A. TSH level in the newborn B. thyroid-binding globulin level in the newborn C. iodine level in the newborn D. total thyroxine (T4) level in the newborn

Correct Answer: D

What common substrate is used in the biosynthesis of adrenal steroids, including androgens and estrogens? A. cortisol B. catecholamines C. progesterone D. cholesterol

Correct Answer: D

Which of the following electrolytes is the chief plasma cation whose main function is maintaining osmotic pressure? A. chloride B. calcium C. potassium D. sodium

Correct Answer: D

Which enzyme is least useful in differentiating necrotic from obstructive jaundice? A. GGT B. ALT C. 5' Nucleotidase D. LD

Correct Answer: D 54. D GGT and 5 ́ nucleotidase are markedly elevated in both intra- and posthepatic obstruction. ALT is slightly elevated in obstructive jaundice but is markedly elevated in necrotic jaundice. Although LD is usually greater in necrotic jaundice than in obstructive jaundice, elevations in these conditions overlap frequently and result from many other causes.

Which photodetector is most sensitive to low levels of light? A. Barrier layer cell B. Photodiode C. Diode array D. Photomultiplier tube

Correct Answer: D 9. D The photomultiplier tube uses dynodes of increasing voltage to amplify the current produced by the photosensitive cathode. It is 10,000 times as sensitive as a barrier layer cell, which has no amplification. A photomultiplier tube requires a DC-regulated lamp because it responds to light fluctuations caused by the AC cycle.

If elevated, which laboratory test would support a diagnosis of congestive heart failure? A. Homocysteine B. Troponin C. Albumin cobalt binding D. B-type natriuretic peptide

D. Symptoms are sometimes nonspecific, making it difficult to diagnose congestive heart failure. B-type (brain) natriuretic peptide (BNP) is used to determine if physical symptoms are related to congestive heart failure. BNP is synthesized in and secreted by myocardial ventricles in response to ventricular volume expansion and pressure overload. An increase in BNP causes dilation of blood vessels and promotes sodium and water loss by the kidneys. This reduces fluid load on the heart in an attempt to improve cardiac function. Albumin cobalt binding is a test that measures ischemia-modified albumin, which is a marker for ischemic heart disease.

What is the active metabolite of the antiarrhythmic drug procainamide? A. Pronestyl B. Disopyramide C. PEMA D. NAPA

D. The major active metabolite of procainamide is TV-acetylprocainamide (NAPA). Procainamide is an antiarrhythmic drug that is used to treat such disorders as premature ventricular contractions, ventricular tachycardia, and atrial fibrillation. Because procainamide and its metabolite NAPA exhibit similar and cumulative effects, it is necessary that both be quantified to assess therapy. Methods for their analysis include GC, HPLC, FPIA, and EMIT.

Which of the following plasma proteins is not manufactured by the liver? A. Albumin B. Haptoglobin C. Fibrinogen D. IgG

D. The majority of the plasma proteins are manufactured by the liver. Albumin, fibrinogen, and most of the alpha- and beta-globulins are produced by the liver. The immunoglobulins, including IgG, IgA, IgM, IgD, and IgE, are produced by the lymphoid cells.

What compound may be detected by observing its orange-red fluorescence in acid solution? A. Porphobilinogen B. Uroporphyrinogen C. Aminolevulinic acid D. Coproporphyrin

D. The porphyrins that are of clinical significance include uroporphyrin, coproporphyrin, and protoporphyrin. These three porphyrin compounds may be detected in acid solution by irradiating the solution with long-wave ultraviolet light, which causes the porphyrins to fluoresce. The intense orange-red fluorescence of the porphyrins is due to the conjugated unsaturation of the tetrapyrrole ring structure.

prolonged diarrhea, renal tubular acidosis, adrenocortical hyperfunction

what are some diseases associated with Hyperchloremia?

diabetic acidosis, intravascular hemolysis, severe burns, renal failure, Addison's disease

what are some diseases associated with hyperkalemia

diabetic acidosis (excess acid production), chronic pyelonphritis, prolonged vomiting, aldosterone deficiency

what are some diseases associated with hypochloremia

insulin injections, alkalosis, GI loss, hyperaldosteronism, cushing's disease

what are some diseases associated with hypokalemia?

Cushing's syndrome, dehydration, hyperaldosteronism, insulin treatment( diabetes)

what are some diseases associated with increased levels of Na?

Na, K, Cl, Co2, Mg, Ca, phosphorous

what are the major electrolytes?

Which disorder is associated with thiamin deficiency? A. Beriberi B. Pellagra C. Rickets D. Dermatitis

A. A deficiency in thiamin (vitamin B]) is associated with beriberi and Wernicke-Korsakoff syndrome. In general, thiamin deficiency affects the nervous and cardiovascular systems. Thiamin deficiency is sometimes seen in chronic alcoholics and in the elderly.

A 46-year-old known alcoholic with liver damage is brought into the emergency department unconscious. In what way would you expect his plasma lipid values to be affected? A. Increased total cholesterol, triglyceride, LDL, and VLDL B. Increased total cholesterol and triglyceride, decreased LDL and VLDL C. Decreased total cholesterol, triglyceride, LDL, and VLDL D. Normal lipid metabolism, unaffected by the alcoholism

A. Hyperlipoproteinemia can be genetically inherited or secondary to certain diseases such as diabetes mellitus, hypothyroidism, or alcoholism. If the alcoholism has advanced to the state where there is liver damage, the liver can become inefficient in its metabolism of fats, leading to an increase of total cholesterol, triglyceride, LDL, and/or VLDL in the bloodstream. The elevation of these lipids along with the previous liver damage (e.g., cirrhosis) leads to a poor prognosis for the patient.

Cholesterol ester is formed through the esterification of the alcohol cholesterol with what substance? A. Protein B. Triglyceride C. Fatty acid D. Digitonin

C. Total cholesterol consists of two fractions, free cholesterol and cholesteryl ester. In the plasma, cholesterol exists mostly in the cholesteryl ester form. Approximately 70% of total plasma cholesterol is esterfied with fatty acids. The formation of cholesteryl esters is such that a transferase enzyme catalyzes the transfer of fatty acids from phosphatidylcholine to the carbon-3 alcohol function position of the free cholesterol molecule. Laboratories routinely measure total cholesterol by first using the reagent cholesterol esterase to break the ester bonds with the fatty acids.

Which of the following blood gas parameters are measured directly by the blood gas analyzer electrochemically as opposed to being calculated by the instrument? A. pH, HCC-3, total CO2 B. PCO2, HCOJ, PO2 C. pH, PCO2, K)2 D. PO2, HCOf, total CO2

C. pH, PCOl, and PO2 are measured directly from the specimen by utilizing electrodes. The pH and PCO2 electrodes are potentiometric where the voltage produced across a semipermeable membrane to hydrogen ions or CO2 gas is proportional to the "activity" of those ions in the patient's sample. Activity is measured in voltage whose value can be presented in terms of concentration. POi is measured similarly, but using an amperometric electrode. For PC>2 a small charge is put on a cathode, and electrons are drawn off the cathode in proportion to the oxygen present. The O2 becomes part of the circuit. The amount of electrons drawn is proportional to the amount of oxygen present. Bicarbonate and other parameters, such as base excess, are calculated by the instrument using pH and PCO2 values and the Henderson/Hasselbalch equation.

A 68-year-old man arrives in the emergency room with a glucose level of 722 mg/dL (39.7 mmol/L) and serum acetone of 4+ undiluted. An arterial blood gas form this patient is likely to be: A. low pH B. high pH C. low PO2 D. high PO2

Correct Answer: A

A low concentration of serum phosphorus is commonly found in: A. patients who are receiving carbohydrate hyperalimentation B. chronic renal disease C. hypoparathyroidism D. patients with pituitary tumors

Correct Answer: A

A patient has the following test results: -increased serum calcium levels -decreased serum phosphate levels -increased levels of parathyroid hormone This patient most likely has: A. hyperparathyroidism B. hypoparathyroidsim C. nephrosis D. steatorrhea

Correct Answer: A

Absorption of vitamin B12 requires the presence of: A. intrinsic factor B. gastrin C. secretin D. folic acid

Correct Answer: A

In familial hyperchlesterolemia, the hallmark finding is an elevation of: A. low-density lipoproteins B. chylomicrons C. high-density lipoproteins D. apolipoprotein A1

Correct Answer: A

In order to prepare 100 mL of 15 mg/dL BUN (5.35 mmol/L) working standard form a stock standard containing 500 mg/dL (178.5 mmol/L) of urea nitrogen, the number of mL of stock solution that should be used is: A. 3 mL B. 5 mL C. 33 mL D. 75 mL

Correct Answer: A

In the Jendrassik-Grof method for the determination of serum bilirubin concentration, quantitation is obtained by measuring the green color of: A. azobilirubin B. bilirubin glucuronide C. urobilin D. urobilinogen

Correct Answer: A

In the assay of lactate dehydrogenase (LD), the reaction is dependent upon which of the following coenzyme systems? A. NAD/NADH B. ATP/ADP C. Fe++/Fe+++ D. Cu/Cu++

Correct Answer: A

Plasma for cortisol determinations were collected at 7 AM, after waking the patient, and at 10 PM that evening. The cortisol level of the morning sample was higher than the evening sample. This is consistent with: A. a normal finding B. Cushing syndrome C. Addison disease D. hypopituitarism

Correct Answer: A

The different water content of erythrocytes and plasma makes true glucose concnetrations in whole blood a function of the: A. hematocrit B. leukocyte count C. erythrocyte count D. erythrocyte indices

Correct Answer: A

The greatest activities of serum AST and ALT are seen in which of the following? A. acute viral hepatitis B. primary biliary cirrhosis C. metastatic hepatic cirrhosis D. alcoholic cirrhosis

Correct Answer: A

The osmol gap is define as measure Osm/kg minus the calculate OSm/kg. Normally, the osmol gap is less than: A. 10 B. 20 C. 40 D. 60

Correct Answer: A

Which of the following methods is susceptible to the solvent displacing effect that results in falsely decreased electrolyte values? A. indirect ion-selective electrodes B. direct ion-selective electrodes C. spectrophotometric D. fluorescence

Correct Answer: A

Which analyte should be reported as a ratio using creatinine concentration as a reference? A. Urinary microalbumin B. Urinary estriol C. Urinary sodium D. Urinary urea

Correct Answer: A 14. A Measurement of urinary microalbumin concentration should be reported as a ratio of albumin to creatinine (e.g., mg albumin per g creatinine). This eliminates the need for 24-hour collection in order to avoid variation caused by differences in fluid intake. A dry reagent strip test for creatinine is available that measures the ability of a creatinine-copper complex to break down H2O2 , forming a colored complex. The strip uses buffered copper II sulfate, tetramethylbenzidine, and anhydrous peroxide. Binding of creatinine in urine to copper forms a peroxidase-like complex that results in oxidation of the benzidine compound. Also, 24-hour urinary metanephrines, vanillylmandelic acid, and homovanillic acid are reported per gram creatinine when measured in infants and children in order to compensate for differences in body size.

What is the recommended cutoff for the early detection of chronic kidney disease in diabetics using the test for microalbuminuria? A. >30 mg/g creatinine B. >80 mg/g creatinine C. >200 mg/g creatinine D. >80 mg/L

Correct Answer: A 18. A Microalbuminuria is the excretion of small quantities of albumin in the urine. In diabetics, excretion of albumin that is within allowable limits for healthy persons may signal the onset of chronic kidney disease. The term microalbuminuria is defined as albumin excretion ≥ 30 mg/g creatinine but ≤ 300 mg/g creatinine. The use of the albumin to creatinine ratio is preferred to measures of albumin excretory rate (μg/min) because the latter is subject to error associated with timed specimen collection. ADA recommends the test be done annually for all type 2 diabetics and type 1 diabetics who have had the disease for > 5 years.

Which of the following quality control (QC) rules would be broken 1 out of 20 times by chance alone? A. 12s B. 22s C. 13s D. 14s

Correct Answer: A 18. A The notation 12S means that one control is outside ±2 standard deviation units. QC results follow the bell- shaped curve called the Gaussian (normal) distribution. If a control is assayed 100 times, 68 out of 100 results would fall within +1 s and -1 s of the mean. Ninety- five (95.4) out of 100 results would fall within +2 s and -2 s. This leaves only 5 out of 100 results (1:20) that fall outside the ±2 s limit. Also, 99.7 out of 100 results fall within ±3 s of the mean.

Which is the most widely used screening test for Cushing's syndrome? A. Overnight low-dose dexamethasone suppression test B. Corticotropin-releasing hormone stimulation test C. Petrosal sinus sampling D. Metyrapone stimulation test

Correct Answer: A 28. A Dexamethasone is a synthetic corticosteroid that exhibits 30-fold greater negative feedback on the hypothalamus than cortisol. When an oral dose of 1 mg of the drug is given to a patient at 11 p.m., the 8 a.m. serum total cortisol level should be below 5.0 μg/dL. Patients with Cushing's syndrome almost always exceed this cutoff. Therefore, a normal response to dexamethasone excludes Cushing's syndrome with a sensitivity of about 98%. CRH stimulation and petrosal sinus sampling are confirmatory tests for Cushing's disease, and are used when the high-dose dexamethasone suppression test is inconclusive. The metyrapone stimulation test measures the patient's ACTH reserve. Metyrapone blocks cortisol formation by inhibiting 11-β hydroxylase. This causes an increase in ACTH output in normals. A subnormal ACTH response is seen in persons with Addison's disease caused by pituitary failure.

Which statement about glucose in cerebrospinal fluid (CSF) is correct? A. Levels below 40 mg/dL occur in septic meningitis, cancer, and multiple sclerosis B. CSF glucose is normally the same as the plasma glucose level C. Hyperglycorrhachia is caused by dehydration D. In some clinical conditions, the CSF glucose can be greater than the plasma glucose

Correct Answer: A 28. A High glucose in CSF is a reflection of hyperglycemia and not central nervous system disease. The CSF glucose is usually 50%-65% of the plasma glucose. Low levels are significant and are most often associated with bacterial or fungal meningitis, malignancy in the central nervous system, and some cases of subarachnoid hemorrhage, rheumatoid arthritis, and multiple sclerosis.

Which of the following enzymes is increased in persons with prostate and small-cell lung cancer? A. Creatine kinase-1 (CK-1) B. Gamma glutamyl transferase (GGT) C. Amylase D. Lactate dehydrogenase

Correct Answer: A 3. A CK-1 (CK-BB) is not normally found in plasma or serum except in neonates. It may be present in persons with central nervous system damage and some other disorders but its presence is often associated with various malignancies, especially prostate cancer and small-cell carcinoma of the lung. Several other commonly measured enzymes are elevated by malignancy. ALP and LD are associated with various tumors. GGT levels are very high in hepatoma, and amylase is elevated in pancreatic cancer.

The term reverse phase is used in HPLC to indicate that the mobile phase is: A. More polar than the stationary phase B. Liquid and the stationary phase is solid C. Organic and the stationary phase is aqueous D. A stronger solvent than the stationary phase

Correct Answer: A 56. A In reverse-phase HPLC, the separation takes place using a nonpolar sorbent (stationary phase) such as octadecylsilane (C18). Solutes that are nonpolar are retained longer than polar solutes. Most clinical separations of drugs, hormones, and metabolites use reverse phase because aqueous mobile phases are far less toxic and flammable.

Which statement regarding total and direct bilirubin levels is true? A. Total bilirubin level is a less sensitive and specific marker of liver disease than the direct level B. Direct bilirubin exceeds 3.5 mg/dL in most cases of hemolytic anemia C. Direct bilirubin is normal in cholestatic liver disease D. The ratio of direct to total bilirubin exceeds 0.40 in hemolytic anemia

Correct Answer: A 61. A Direct bilirubin measurement is a sensitive and specific marker for hepatic and posthepatic jaundice because it is not elevated by hemolytic anemia. In hemolytic anemia, the total bilirubin does not exceed 3.5 mg/dL, and the ratio of direct to total is less than 0.20. Unconjugated bilirubin is the major fraction in necrotic liver disease because microsomal enzymes are lost. Unconjugated bilirubin is elevated along with direct bilirubin in cholestasis because some necrosis takes place and some conjugated bilirubin is hydrolyzed back to unconjugated bilirubin.

In the enzymatic assay of bilirubin, how is measurement of both total and direct bilirubin accomplished? A. Using different pH for total and direct assays B. Using UDP glucuronyl transferase and bilirubin reductase C. Using different polarity modifiers D. Measuring the rate of absorbance decrease at different time intervals

Correct Answer: A 69. A Enzymatic methods use bilirubin oxidase to convert bilirubin back to biliverdin, and measure the decrease in absorbance that results. At pH 8, both conjugated, unconjugated, and delta bilirubin react with the enzyme, but at pH 4 only the conjugated form reacts.

Which of the following is the reference method for measuring serum glucose? A. Somogyi-Nelson B. Hexokinase C. Glucose oxidase D. Glucose dehydrogenase

Correct Answer: B 21. B The hexokinase method is considered more accurate than glucose oxidase methods because the coupling reaction using glucose-6-phosphate dehydrogenase (G-6-PD) is highly specific. The hexokinase method may be done on serum or plasma collected using heparin, EDTA, fluoride, oxalate, or citrate. The method can also be used for urine, cerebrospinal fluid, and serous fluids.

An HPLC assay for procainamide gives an internal standard peak that is 15% greater in area and height for sample 1 than sample 2. The technologist should suspect that: A. The column pressure increased while sample 2 was being analyzed B. Less recovery from sample 2 occurred in the extraction step C. The pH of the mobile phase increased during chromatography of sample 2 D. There was more procainamide in sample 1 than sample 2

Correct Answer: B 21. B The internal standard compensates for variation in extraction, evaporation, reconstitution, and injection volume. The same amount of internal standard is added to all samples and standards prior to assay. Increased column pH or pressure usually alters retention time, and may not affect peak quantitation.

When a therapeutic drug is suspected of causing toxicity, which specimen is the most appropriate for an initial investigation? A. Trough blood sample B. Peak blood sample C. Urine at the time of symptoms D. Gastric fluid at the time of symptoms

Correct Answer: B 21. B When a drug is suspected of toxicity, the peak blood sample (sample after absorption and distribution are complete) should be obtained because it is most likely to exceed the therapeutic limit. If the peak level is above the upper therapeutic limit, then toxicity is confirmed, and the drug dose is lowered. If the peak drug concentration is within the therapeutic range, toxicity is less likely, but cannot be ruled out. A high concentration of free drug, the presence of active metabolites, and abnormal response to the drug are causes of drug toxicity that may occur when the blood drug level is within the published therapeutic range.

Select the correct order of Hgb migration on citrate agar at pH 6.2. A. - F→S→C→A + B. - F→A→S→C + C. - A→S→F→C + D. - A→C→S→F +

Correct Answer: B 31. B In an acid buffer, the hemoglobins are expected to migrate to the cathode, with hemoglobin A being the slowest because it has the weakest net positive charge. However, Hgb C and Hgb S bind to sulfated pectins in the agar gel, forming a complex that is negatively charged causing them to migrate toward the anode. Hgb C migrates furthest toward the anode, followed by Hgb S. Hgb F migrates furthest toward the cathode. Hgbs A, A2, DPunjab, E, G, and HgbLepore migrate slightly toward the cathode.

Interference in atomic absorption spectrophotometry caused by differences in viscosity is called: A. Absorption interference B. Matrix effect C. Ionization interference D. Quenching

Correct Answer: B 31. B Significant differences in aspiration and atomization result when the matrix of sample and unknowns differ. Differences in viscosity and protein content are major causes of matrix error. Matrix effects can be reduced by using protein-based calibrators and diluting both standards and samples prior to assay.

Persistent noise from an ion-selective electrode is most often caused by: A. Contamination of sample B. Blocked junction at the salt bridge C. Overrange from high concentration D. Improper calibration

Correct Answer: B 43. B Electrode noise most often results from an unstable junction potential. Most reference electrodes contain a high concentration of KCl internal solution used to produce the reference potential. This forms a salt bridge with the measuring half cell by contacting sample, but is kept from equilibrating via a barrier called a junction. When this junction becomes blocked by salt crystals, the reference potential will be unstable, resulting in fluctuation in the analyzer readout.

The freezing point osmometer differs from the vapor pressure osmometer in that only the freezing point osmometer: A. Cools the sample B. Is sensitive to ethanol C. Requires a thermoelectric module D. Requires calibration with aqueous standards

Correct Answer: B 53. B Alcohol enters the vapor phase so rapidly that it evaporates before the dew point of the sample is reached. Therefore, ethanol does not contribute to osmolality as measured using the vapor pressure osmometer. Freezing-point osmometers measure alcohol and can be used in emergency department settings to estimate ethanol toxicity.

Which of the following total quality management tools can be used to calculate the analytical error rate for an analyte in the clinical laboratory? A. LEAN B. Six sigma C. ISO 9000 D. Laboratory information system

Correct Answer: B 53. B All four of the answer choices are total quality management (TQM) tools used in the clinical laboratory to improve performance. Six sigma is a measurement of the frequency of product defects. In clinical laboratories, it refers to the frequency of an erroneous result. At the six-sigma level, the analytical process has such small variance that an error of six times the standard deviation would still be within acceptable limits for total allowable error. For example, a six-sigma process for an analyte produces a significant error in test result only 3.4 out of 1 million times the test is performed. Conversely, a method performing at the three-sigma level would give 66,807 errors per million. The sigma of a method is calculated by subtracting its bias from the total method error and dividing by its standard deviation. It is the method's random error divided by its standard deviation.

What is the HDL cholesterol cutpoint recommend by NCEP? A. <30 mg/dL B. <40 mg/dL C. <30 mg/dL for males and < 40 mg/dL for females D. <45 mg/dL for males and < 50 mg/dL for females

Correct Answer: B 56. B The HDL cholesterol cutpoint recommended by NCEP is < 40 mg/dL regardless of sex. A result below 40 mg/dL counts as a risk factor for coronary artery disease. Conversely, if the HDL cholesterol is ≥ 60 mg/dL, then one risk factor is subtracted from the total number. The therapeutic goal for someone with low HDL cholesterol is still reduction of LDL cholesterol (if elevated), weight loss, and increased exercise.

Plate 6 shows an agarose gel on which immunofixation electrophoresis (IFE) was performed at pH 8.6. The gel contains the same serum sample as number 6 shown in Plate 2. What is the heavy and light chain type of the monoclonal protein present in this sample? A. IgA κ B. IgG κ C. IgG λ D. IgM λ

Correct Answer: B 6. B IFE is performed by placing the patient's sample in all six lanes and separating the proteins by electrophoresis. Following electrophoresis, the proteins in lane 1 are precipitated and fixed by overlaying sulfosalicylic acid onto the gel. Monospecific antiserum against each heavy or light chain is applied to the gel over the lanes as labeled and incubated to precipitate the immunoglobulins containing the corresponding chain. The gel is washed to remove unprecipitated proteins, then stained to visualize the precipitated bands. This IFE gel shows an insoluble immunoprecipitate restricted to a single band in lanes 2 and 5. The proteins in lane 2 reacted with anti-γ (anti-IgG), and the proteins in lane 5 reacted with anti-κ. Lane 5 also contains a faint restricted band anodal to the IgG band. This band is not present in lane 2 (does not contain γ chains) and represents free κ light chains.

The modification of diet in renal disease (MDRD) formula for calculating eGFR requires which four parameters? A. Urine creatinine, serum creatinine, height, weight B. Serum creatinine, age, gender, race C. Serum creatinine, height, weight, age D. Urine creatinine, gender, weight, age

Correct Answer: B 6. B The National Kidney Foundation recommends screening for chronic kidney disease using the estimated glomerular filtration rate (eGFR) because of the high frequency of sample collection errors associated with measuring creatinine clearance. The eGFR should be calculated according to the MDRD formula, and reported along with the serum or plasma creatinine. eGFR (mL/min/1.73m2) = 186 x Plasma Cr -1.154 x Age -0.203 x 0.742 (if female) x 1.21 (if Black)

Which reagent is used in thin-layer chromatography (TLC) to extract cocaine metabolites from urine? A. Acid and sodium chloride B. Alkali and organic solvent C. Chloroform and sodium acetate D. Neutral solution of ethyl acetate

Correct Answer: B 63. B Alkaline drugs such as cocaine, amphetamine, and morphine are extracted at alkaline pH. Ideally, the pH for extracting alkaline drugs into an organic solvent should be 2 pH units greater than the negative log of dissociation constant (pKa) of the drug. More than 90% of the drug will be nonionized and will extract in ethyl acetate or another organic solvent.

How soon following acute abdominal pain due to pancreatitis is the serum amylase expected to rise? A. 1-2 hours B. 2-12 hours C. 3-4 days D. 5-6 days

Correct Answer: B 69. B Serum amylase usually peaks 2-12 hours following acute abdominal pain resulting from pancreatitis. Levels reach 2-6 times the URL and return to normal within 3-4 days. Urinary amylase peaks concurrently with serum but rises higher and remains elevated for up to 1 week.

SITUATION: A GC-MS analysis using nitrogen as the carrier gas shows an extensively noisy baseline. A sample of the solvent used for the extraction procedure, ethyl acetate, was injected and showed the same noise. Results of an Autotune test showed the appearance of a base peak at 16 with two smaller peaks at 17 and 18. These results indicate: A. The solvent is contaminated B. The carrier gas is contaminated C. There is electrical noise in the detector D. The ion source is dirty

Correct Answer: B 72. B All of these situations are sources of baseline noise in GC-MS. However, the peak at 16 indicates the presence of oxygen in the carrier gas. Oxygen in the atmosphere also contains small quantities of two isotopes with molecular weights of 17 and 18 owing to one and two extra neutrons, respectively.

Which enzyme is measured in whole blood? A. Chymotrypsin B. Glucose-6-phosphate dehydrogenase C. Glycogen phosphorylase D. Lipase

Correct Answer: B 80. B Glucose-6-phosphate dehydrogenase deficiency is the most common inherited RBC enzyme deficiency and is X linked. The enzyme is measured on a whole- blood hemolysate using glucose-6-phosphate as the substrate, and forms 6-phosphogluconate as NADP+ is converted to NADPH. Persons with the deficiency are prone to a hemolytic episode upon exposure to certain oxidative drugs and fava beans and as a result of infections. Heinz bodies form in the RBCs, bite cells are seen in the circulation, and plasma haptoglobin is reduced in severe cases.

Which of the following dyes is the most specific for measurement of albumin? A. Bromcresol green (BCG) B. Bromcresol purple (BCP) C. Tetrabromosulfophthalein D. Tetrabromphenol blue

Correct Answer: B 9. B Tetrabromphenol blue and tetrabromosulfophthalein are dyes that change pKa in the presence of protein. Although they have greater affinity for albumin than globulins, they are not sufficiently specific to apply to measurement of serum albumin. BCG and BCP are anionic dyes that undergo a spectral shift when they bind albumin at acid pH. BCP is more specific for albumin than BCG. Reaction of both dyes with globulins requires a longer incubation time than with albumin, and reaction times are kept at 30 seconds or less to increase specificity. Both dyes are free of interference from bilirubin. However, BCG is the method used most often. One reason for this is that renal dialysis patients produce an organic acid that competes with BCP for the binding site on albumin, causing a falsely low result.

A 25-year-old man became nauseated and vomited 90 minutes after receiving a standard 75 g carbohydrate dose for an oral glucose tolerance test. The best course of action is to: A. give the patient a glass of orange juice and continue the test B. start the test over immediately with a 50g carbohydrate dose C. draw blood for glucose and discontinue test D. place the patient in a recumbent position, reassure him and continue the test

Correct Answer: C

Chylomicrons are present in which of the following dyslipidemias? A. familial hypercholestrolemia B. hypertirglyceridemia C. deficiency in lipoprotein lipase activity D. familial hypoalphalipoproteinemia

Correct Answer: C

To detect barbiturate abuse when analyzing urine specimens, immunoassay is the method of choice for screening. The method of choice for confirmation is: A. nephelometry B. thin-layer chromatography C. gas chromatography/ mass spectrometry D. ultraviolet absorption spectroscopy

Correct Answer: C

Which of the following procedures can be used to detect proportional error in a new method for glucose? A. Compare the standard deviation of 40 patient samples to the hexokinase method B. Measure a mixture made from equal parts of normal and high-QC sera C. Add 5.0 mg of glucose to 1.0 mL of a serum of known concentration and measure D. Compare the mean of 40 normal samples to the hexokinase method

Correct Answer: C 1. C Proportional error is percentage deviation from the expected result, and affects the slope of the calibration curve. It causes a greater absolute error (loss of accuracy) as concentration increases. It is measured by a recovery study in which a sample is spiked with known amounts of analyte. In the example, the concentration should increase by 500 mg/dL.

What is the PO2 of calibration gas containing 20.0% O2, when the barometric pressure is 30 in.? A. 60 mm Hg B. 86 mm Hg C. 143 mm Hg D. 152 mm Hg

Correct Answer: C 2. C Convert barometric pressure in inches to mm Hg by multiplying by 25.4 (mm/in.). Next, subtract the vapor pressure of H2O at 37°C, 47 mm Hg, to give dry gas pressure. Multiply dry gas pressure by the %O2: 25.4 mm/in. × 30 in. = 762 mm Hg 762 mm Hg - 47 mm Hg (vapor pressure) = 715 mm Hg (dry gas pressure) 0.20 × 715 mm Hg = 143 mm Hg PO2

Which substance is used in the CK assay to activate the enzyme? A. Flavin adenine dinucleotide (FAD) B. Imidazole C. N-acetylcysteine D. Pyridoxyl-5 ́-phosphate

Correct Answer: C 20. C In addition to Mg+2, CK requires a thiol compound to reduce interchain disulfide bridges and bind heavy metals that inactivate the enzyme. N-acetylcysteine is an activator of CK used for this purpose in the IFCC recommended method. Pyridoxyl-5 ́-phosphate is a prosthetic group of AST and ALT. FAD is a prosthetic group of glucose oxidase. Imidazole is used to buffer the CK reagent.

The EMIT for drugs of abuse uses an: A. Antibody conjugated to a drug B. Enzyme conjugated to an antibody C. Enzyme conjugated to a drug D. Antibody bound to a solid phase

Correct Answer: C 25. C In EMIT, enzyme-labeled drug competes with drug in the sample for a limited amount of reagent antibodies. When antibody binds to the enzyme-drug conjugate, it blocks the catalytic site of the enzyme. Enzyme activity is directly proportional to sample drug concentration because the quantity of unbound drug-enzyme conjugate will be highest when drug is present in the sample.

A gastric fluid from a patient suspected of having taken an overdose of amphetamine is sent to the laboratory for analysis. The technologist should: A. Perform an EMIT assay for amphetamine B. Refuse the sample and request serum or urine C. Dilute 1:10 with H2O and filter; perform TLC for amphetamines D. Titrate to pH 7.0, then follow procedure for measuring amphetamine in urine

Correct Answer: C 30. C The gastric sample can be measured by TLC, but such a sample should not be used in place of serum or urine without documentation of acceptability by the reagent manufacturer or laboratory. A positive amphetamine result by a screening test such as TLC or immunoassay may be caused by a related drug which interferes, and therefore, the result should be confirmed by GC-MS if there is a medicolegal implication.

SITUATION: A digoxin result from a stable patient with a normal electrocardiogram (EKG) is reported as 7.4 ng/mL (URL 2.6 ng/mL) using an immunofluorescent method. Renal function tests were normal and the patient was not taking any other medications. The assay was repeated and results were the same. The sample was frozen and sent to a reference laboratory for confirmation. The result was 1.6 ng/mL measured by a competitive chemiluminescent procedure. Which best explains the discrepancy in results? A. The fluorescent immunoassay was performed improperly B. Digoxin was lower by the chemiluminescent method because it is less sensitive C. An interfering substance was present that cross-reacted with the antibody in the fluorescent immunoassay D. Freezing the specimen caused lower results by converting the digoxin to an inactive metabolite

Correct Answer: C 34. C An error was suspected because there was a discrepancy between the test result and the patient's clinical status (i.e., signs of digoxin toxicity such as ventricular arrhythmia were not present.) Some substances called DLIFs (digoxin-like immunologic factors) can cross-react with antibodies used to measure digoxin. The extent of interference varies with the source of anti-digoxin used. In addition, falsely elevated digoxin results may result from accidental ingestion of plant poisons such as oleandrin and from administration of Digibind, a Fab fragment against digoxin that is used to reverse digoxin toxicity.

Select the equation describing the potential that develops at the surface of an ion-selective electrode. A. van Deemter equation B. van Slyke equation C. Nernst equation D. Henderson-Hasselbalch equation

Correct Answer: C 35. C The van Deemter equation describes the relation between the velocity of mobile phase to column efficiency in gas chromatography. The Henderson-Hasselbalch equation is used to determine the pH of a solution containing a weak acid and its salt. van Slyke developed an apparatus to measure CO2 and O2 content using a manometer.

In which condition would hypochloremia be expected? A. Respiratory alkalosis B. Metabolic acidosis C. Metabolic alkalosis D. All of these options

Correct Answer: C 36. C Chloride is the major extracellular anion and is retained or lost to preserve electroneutrality. Low chloride will occur in metabolic alkalosis because excess bicarbonate is present. Low chloride also will occur in partially compensated respiratory acidosis because the kidneys compensate by increased retention of bicarbonate.

Analysis of normal and abnormal QCs performed at the beginning of the evening shift revealed a 22s error across levels for triglyceride. Both controls were within the 3s limit. The controls were assayed again, and one control was within the acceptable range and the other was slightly above the 2s limit. No further action was taken and the patient results that were part of the run were reported. Which statement best describes this situation? A. Appropriate operating procedures were followed B. Remedial evaluation should have been taken, but otherwise, the actions were appropriate C. Corrective action should have been taken before the controls were repeated D. The controls should have been run twice before reporting results

Correct Answer: C 36. C Quality control limits are chosen to achieve a low probability of false rejection. For example, a 22s error occurs only once in 1,600 occurrences by chance. Therefore, such an error can be assumed to be significant. However, this does not mean the error will occur if the controls are repeated again. The error detection rate (power function) of the 22s rule is only about 30% for a single run. This means that there is a greater chance the repeated controls will be within range than outside acceptable limits. Therefore, controls should never be repeated until the test system is evaluated for potential sources of error. Calibration should have been performed prior to repeating the controls, and patient samples should have been evaluated to determine the magnitude of the error before reporting.

Which statement about sample collection for catecholamines and metabolites is true? A. Blood for catecholamines is collected in the usual manner following a 12-hour fast B. Twenty-four-hour urine for vanillylmandelic acid, catecholamines, or metanephrines is collected in 1 mL of boric acid C. Twenty-four-hour urine creatinine should be measured with vanillylmandelic acid, homovanillic acid, or metanephrines D. There is no need to discontinue medications if a 24-hour urine collection is used

Correct Answer: C 36. C Stress, exercise, and an upright position induce catecholamine elevation, and therefore, patients must be resting supine for at least 30 minutes prior to blood collection. The preferred method of collection is catheterization, so that the anxiety of venipuncture is not a factor. A 4-hour fast is also recommended. Many drugs contain epinephrine, which may falsely elevate catecholamine measurements. In addition, many drugs inhibit monoamine oxidase, which is needed to convert metanephrines to VMA. Therefore, medications should be removed prior to testing whenever possible. Twenty-four-hour urine samples for catecholamines are usually preserved with 10 mL of 6N HCl because some degradation occurs during storage when pH is greater than 3. Renal clearance affects excretion of catecholamine metabolites; it is preferable to report VMA, HVA, and metanephrines, in μg/mg creatinine. The urinary creatinine measurement should be at least 0.8 g/day, to validate the completeness of the 24-hour urine sample.

Which of the following is likely to occur first in iron deficiency anemia? A. Decreased serum iron B. Increased TIBC C. Decreased serum ferritin D. Increased transferrin

Correct Answer: C 42. C Body stores must be depleted of iron before serum iron falls. Thus, serum ferritin falls in the early stages of iron deficiency, making it a more sensitive test than serum iron in uncomplicated cases. Ferritin levels are low only in iron deficiency. However, concurrent illness such as malignancy, infection, and inflammation may promote ferritin release from the tissues, causing the serum ferritin to be normal in iron deficiency.

A new tumor marker for ovarian cancer is evaluated for sensitivity by testing serum samples from patients who have been diagnosed by staging biopsy as having malignant or benign lesions. The following results were obtained: Number of malignant patients who are positive for CA 125 = 21 out of 24 Number of benign patients who are negative for CA 125 = 61 out of 62 What is the sensitivity of the new CA 125 test? A. 98.4% B. 95.3% C. 87.5% D. 85.0%

Correct Answer: C 47. C Sensitivity is defined as the percentage of persons with the disease who have a positive test result. It is calculated as true positives (TP) divided by the sum of TP and false negatives (FN). % Sensitivity = TP × 100 / TP + FN Sensitivity = (21 × 100) ÷ (21 + 3) = 87.5%

The term δ-bilirubin refers to: A. Water-soluble bilirubin B. Free unconjugated bilirubin C. Bilirubin tightly bound to albumin D. Direct-reacting bilirubin

Correct Answer: C 50. B UDP-glucuronyl transferase esterifies glucuronic acid to unconjugated bilirubin, making it water soluble. Most conjugated bilirubin is diglucuronide; however, the liver makes a small amount of monoglucuronide and other glycosides. β-Glucuronidase hydrolyzes glucuronide from bilirubin, hormones, or drugs. It is used prior to organic extraction to deconjugate urinary metabolites (e.g., total cortisol). Biliverdin reductase forms bilirubin from biliverdin (and heme oxygenase forms biliverdin from heme). Bilirubin oxidase is used in an enzymatic bilirubin assay in which bilirubin is oxidized back to biliverdin and the rate of biliverdin formation is measured at 410 nm.

In which circumstances is a validation study (versus performing routine quality control) required? A. Instrument recalibration B. Source lamp or ion selective electrode change C. Change in reagent lot D. Change in calibrator lot

Correct Answer: C 54. C All of the listed conditions except a change in the reagent lot number can be validated by assaying two levels of control material following the procedure. A change in reagent lot number may alter the test system more dramatically, especially when the reagent was subjected to storage and shipping conditions that alter its performance. Therefore, both controls and patient samples should be analyzed and the results compared to the reagent in current use using criteria determined by the total allowable error for the analyte.

Which condition is caused by deficient secretion of bilirubin into the bile canaliculi? A. Gilbert's disease B. Neonatal hyperbilirubinemia C. Dubin-Johnson syndrome D. Crigler-Najjar syndrome

Correct Answer: C 56. C Dubin-Johnson syndrome is an autosomal recessive condition arising from mutation of an ABC transporter gene. It produces mild jaundice from accumulation of conjugated bilirubin that is not secreted into the bile canaliculi. Total and direct bilirubin are elevated, but other liver function is normal. Rotor syndrome is an autosomal recessive condition that also results in retention of conjugated bilirubin. The mechanism in Rotor syndrome is unknown, and like Dubin-Johnson syndrome it is commonly asymptomatic. It can be differentiated from Dubin-Johnson syndrome by the pattern of urinary coproporphyrin excretion and because it produces no black pigmentation in the liver.

A deficiency in which of the following leads to increased clotting time and may result in hemorrhagic disease in infancy? A. Riboflavin B. Pyridoxine C. Tocopherols D. Menaquinone

D. Adequate amounts of vitamin K are required for the synthesis of prothrombin by the liver. Because prothrombin is an essential component of the clotting system, a deficiency of vitamin K leads to a deficiency of prothrombin, which results in a delayed clot formation. Several closely related compounds having vitamin K properties include phylloquinones, which are synthesized in plants, and menaquinones, which are synthesized by bacteria. Because the intestinal flora may not be developed sufficiently in the newborn, vitamin K (menaquinone) deficiency can occur. This leads to increased clotting time, which may result in hemorrhagic disease in infancy.

Of the five immunoglobulin classes, IgG is the most structurally simple, consisting of how many light chains/heavy chains, respectively? A. 5/2 B. 1/1 C. 2/5 D. 2/2

D. All the immunoglobulins consist of heavyand light-chain polypeptides. The heavy chains are designated as gamma y, alpha a, mu (x, delta A, and epsilon e and are specific for the immunoglobulins IgG, IgA, IgM, IgD, and IgE, respectively. The light chains are designated as kappa K and lambda X, with both types being found in each of the immunoglobulin classes, although the two light chains attached to a particular set of heavy chains must be of the same type. Therefore, IgG consists of two heavy chains of the gamma type and two light chains of either the kappa or lambda type. The immunoglobulins IgA, IgD, and IgE have a structure similar to that of IgG in that they consist of two light chains and two heavy chains of the respective type. IgM is a macromolecule with a pentamer type of structure. IgM consists of five sets of two heavy-chain and two light-chain units, with the basic units being linked to each other by peptide fragments.

A patient is suspected of having Addison disease. His symptoms are weakness, fatigue, loss of weight, skin pigmentation, and hypoglycemia. His laboratory tests show low serum sodium and chloride, elevated serum potassium, and elevated urine sodium and chloride levels. The serum cortisol level is decreased and the plasma ACTH is increased. To make a definitive diagnosis, the physician orders an ACTH stimulation test, and serum cortisol levels are measured. If the patient has primary hypoadrenocortical function (Addison disease), what would be the expected level of serum cortisol following stimulation? If the patient has hypopituitarism and secondary hypoadrenocortical function, what would be the expected level of serum cortisol following stimulation? A. Increase from baseline; decrease from baseline B. Decrease from baseline; increase from baseline C. Slight increase from baseline; no change from baseline D. No change from baseline; slight increase from baseline

D. For differentiation of primary and secondary adrenal dysfunction, stimulation or suppression tests that depend on the feedback mechanism between cortisol and ACTH are performed. In the ACTH stimulation test, a patient with a low baseline serum cortisol level is given ACTH. The level of cortisol will increase slightly if the problem lies with the anterior pituitary gland, thus secondary adrenal insufficiency. This increase will be less than normal and may be somewhat delayed due to atrophy of the adrenal cortex as a result of the primary pituitary dysfunction. If the serum cortisol level does not change from baseline, the dysfunction is with the adrenal cortex, thus primary adrenal insufficiency.

Which of the following is not a component of the total anion content of serum? A. Acetoacetate B. Protein C. Lactate D. Iron

D. The largest fractions of the anion content of serum are normally provided by chloride and bicarbonate. The third largest anion fraction is contributed by the proteins that are negatively charged at physiological pH and that provide about 16 mmol ion charge per liter. Of the remaining organic anions, the largest contribution is generally from lactate, which ranges normally from 1 mmol/L up to 25 mmol/L in lactic acidosis. The ketone bodies, including acetoacetate, normally constitute only a small fraction of the total anions, but their total contribution may increase to 20 mmol/L in diabetic acidosis. Iron is present in the serum as a cation and does not contribute.

In a healthy individual, which protein fraction has the greatest concentration in serum? A. Alphai-globulin B. Beta-globulin C. Gamma-globulin D. Albumin

D. The serum proteins are divided into five principal fractions based on their electrophoretic mobilities. The five fractions are albumin, alpharglobulin, alpha2-globulin, beta-globulin, and gamma-globulin. Albumin constitutes the largest individual fraction of the serum proteins. The reference concentration of albumin in serum ranges between 3.5 and 5.0 g/dL, and the total globulin concentration is between 2.3 and 3.5 g/dL.

albumin, negative

______ has the smallest molecular weight, and therefore has the largest number of free ______ charges and will travel the fastest and furthest

low anion gap

a decrease in unmeasured anions: albumin loss; and increase in unmeasured cations: Mg, Ca, lithium therapy signifies a _____ anion gap

Alpha-fetoprotein (AFP)

a fetal protein that appears in the blood of adults with certain types of cancer

Alpha-1 Antitrypsin (AAT)

a serum protein produced by the liver normally found in the lungs that inhibits proteolytic enzymes of white cells from lysing lung tissue; genetic deficiency of this protein can cause emphysema.

acute phase and nephrotic syndrome

an increase in haptoglobin may indicate ___________ or __________

Posthepatic (obstructive) jaundice

bile flow is obstructed between liver and intestines; caused by strictures of bile duct, gallstones, and tumors in local area Conjugated bilirubin blocked from leaving liver Urine bilirubin = Positive + Urine urobilinogen = Decreased -

Haptoglobin

binds free hemoglobin.

concentration of reactants, pH, temperature, ionic strength, cofactors & coenzymes

enzyme reactions in lab measurements are affected by:

Hyperkalemia

high levels of potassium in the blood

Hypernatremia

high sodium level in the blood

pancreatitis

highest elevations of amylase seen in _________ and obstruction to pancreatic ducts

AST

highest value in hepatitis

ALT

highest value in hepatitis, live specific

LDL and HDL

how are cholesterols transported throughout the body?

glycolysis

if ______ occurs to prolonged testing the following can occur: - CO2 and pCO2--- increase - pH and pO2-- decrease

air exposure

if _________ occurs to blood sample the following can occur: -Co2 and pCO2 -- decrease -pH and pO2--- increase

cholersterol

important cellular physiology precursor to steroid hormones

higher

in PEP protein at pH ______ than its isoelectric point is negatively charged and migrates toward anode(positive charge)

Lipoproteins

in what form are human plasma lipids such as cholesterol transported through the body?

Urine urobilinogen

increase in bilirubin = an increase in ____________________

Albumin

largest plasma protein fraction (52-62%) function- regulates osmotic pressure

nephrotic syndrome

loss of large amounts of plasma protein, usually albumin, through urine due to an increased permeability of the glomerular membrane

Hypochloremia

low levels of chloride in the blood

Hypokalemia

low levels of potassium in the blood

salivary glands

lower elevations of amylase seen in obstruction of the _______ _______.

Potassium (K)

major cation for intracellular fluid

Sodium (Na)

major electrolyte found in extracellular fluid

CK-MB (CK2)

mass _____ measures concentration rather than activity

osmolality

measure of total concentration (number) of dissolved particles in a solution.

chloride shift

movement of chloride ions from the blood plasma into red blood cells as bicarbonate ions diffuse out of the red blood cells into the plasma

anaerobic

must use ________ collection for pH and blood gas studies?

30, 4-10

myoglobin rises within ______ mins of an Acute myocardial infraction (AMI). peaks within _____ hours and returns to normal within 24hrs

hemolyic anemia

pre-hepatic jaundice; plasma/serum--> unconjugated= increased conjugated= N Urine--> bili- 0 urobili- increased

Hypoparathyroidism, decrease serum albumin and vitamin D

what are some disease states associated with hypocalcemia?

Volumetric, manometric, colorimetric, pCO2 electrode measurements

what are some of the ways to measure total CO2

iron deficiency anemia

what disease state(s) would yield these results? serum iron: decreased transferrin saturation: decreased TIBC: increased serum ferritin: decreased

Sideroblastic anemia

what disease state(s) would yield these results? serum iron: increased transferrin saturation: increased TIBC: N serum ferritin: increased

Thalassemia & hemochromatosis

what disease state(s) would yield these results? serum iron: increased transferrin saturation: increased TIBC: decreased serum ferritin: increased

Ion Selective Electrodes (ISE)

what is the method of testing Na?

<100

what is the normal fasting plasma glucose?

10-120µg/dL

what is the normal range for serum ferritin in females?

≥126

what is the range for a patient to be diagnosed with diabetes mellitus

≥100 <126

what is the range for patients to be diagnosed as pre-diabetic?

fructosamine test

A blood test that measures average glucose levels over the past 3 weeks.

Procalcitonin (PCT)

- Normally made in thyroid and converted to calcitonin which causes ↓ blood Ca++ - In bacterial infection elevated levels (greater than 2.0 ng/mL) marker of high risk of sepsis

intraoperative PTH monitoring

- Rapid (POCT) assaying of PTH during surgery - use to determine if abnormal tissue removed - Baseline plasma PTH and then at five and ten minute intervals after removal of the parathyroid tissue - Look for >50% decline in PTH from 0-5min postexcision

LD isoenzymes

-2 chains (M and H) - 4 subunits - 5 forms (tissue-specific)

metabolic alkalosis compensation

-compensatory hypoventilation to retain CO2 and decrease pH -may cause hypoxemia

fetal newborn screening

-dried blood spot spicmen (Ex: PKU) used to screen for >25 genetic diseases. (ex: sickle cell, cystic fibrosis)

metabolic acidosis compensation

-lungs hyperventilate to eliminate CO2 and increase pH -effective in acute situations and some chronic situations

Iron (Fe)

-over 65% of total body _____ is in hemoglobin - O2 transport - transported by transferrin, haptoglobin, & hemopexin - storage and ferritin & hemosiderin

High-density lipoprotein (HDL)

20-35%(good cholesterol) blood fat that helps transport cholesterol out of the arteries, thereby protecting against heart disease

very low density lipoproteins (VLDL)

5-12% lipoproteins that transport triglycerides and other lipids from the liver to various tissues in the body

low-density lipoprotein (LDL)

60-70%(bad cholesterol) blood fat that transports cholesterol to organs and tissues; excess amounts result in the accumulation of fatty deposits on artery walls

Duchenne muscular dystrophy

A human genetic disease caused by a sex-linked recessive allele; characterized by progressive weakening and a loss of muscle tissue.

Phosphorus (PO4)

A major mineral that is essential to cell membranes, bone and tooth structure, DNA, RNA, ATP, lipid transport, and a variety of processes in the body

An active metabolite of amitriptyline is: A. nortriptyline B. protriptyline C. butrptyline D. norbutriptyline

Correct Answer: A

Active cirrhosis

A serum protein electrophoretic pattern displaying a beta gamma bridge or broad gamma is seen in which disease state?

Which of the following lamps provides a continuous spectrum of radiant energy in the visible, near IR, and near UV regions of the spectrum? A. Tungsten-filament B. Hydrogen C. Deuterium D. Mercury vapor

A. A tungsten-filament lamp is the most common light source for photometry in the visible region. It provides a continuous spectrum (360-800 nm) from the near infrared (IR) through the visible to the near ultraviolet (UV) region. Most of the radiant energy is in the near IR. Only about 15% is in the visible region—the region usually used. Because of the large emission in the near IR, tungsten lamps generate a significant amount of heat. Hydrogen and deuterium lamps are used for work in the 200-375 nm range. The mercury vapor lamp does not provide a continuous spectrum, emitting radiation at specific wavelengths.

What endogenous substance may cause a positive interference in the urease/glutamate dehydrogenase assay? A. Ammonia B. Creatinine C. Glucose D. Cholesterol

A. Adequate specificity is generally obtained when using the urease/glutamate dehydrogenase method. Because urease hydrolyzes urea to ammonia and water, a positive interference from endogenous ammonia will occur with elevated blood levels of ammonia. Such interference may occur from use of aged blood specimens and in certain metabolic diseases.

In the condition kernicterus, the abnormal accumulation of bilirubin occurs in what tissue? A. Brain B. Liver C. Kidney D. Blood

A. An abnormal accumulation of bilirubin in the body may be due to increased production or decreased excretion of bilirubin. Terms frequently associated with a buildup of bilirubin include "jaundice," "kernicterus," and "icterus." Both jaundice and icterus are characterized by the yellow coloration of the skin, sclera, and mucous membranes that results from increased plasma concentrations of either conjugated or unconjugated bilirubin or both. This yellow coloration is also visible in serum and plasma specimens in vitro. Kernicterus refers to the accumulation of bilirubin in brain tissue that occurs with elevated levels of unconjugated bilirubin. This condition is most commonly seen in newborns with hemolytic disease resulting from maternal-fetal Rh incompatibility. Newborns afflicted with kernicterus will exhibit severe neural symptoms.

During chemotherapy for leukemia, which of the following analytes would most likely be elevated in the blood? A. Uric acid B. Urea C. Creatinine D. Ammonia

A. An increase in serum uric acid levels may be seen during chemotherapy for leukemia. The cause of this is the accelerated breakdown of cell nuclei in response to the chemotherapy. Other proliferative disorders that may respond similarly are lymphoma, multiple myeloma, and polycythemia. It is important that serum uric acid be monitored during chemotherapy to avoid nephrotoxicity.

When the Jaffe reaction is employed as a kinetic assay to quantify serum creatinine, which of the following is used in the analysis? A. Serum sample used directly B. Folin-Wu filtrate C. Somogyi-Nelson filtrate D. Trichloroacetic acid filtrate

A. Because protein will interfere with the Jaffe reaction, serum for a manual creatinine analysis is treated with sodium tungstate and sulfuric acid to precipitate the proteins. The use of tungstic acid to make a protein-free filtrate is known as the Folin- Wu method. The protein-free filtrate, which still contains creatinine and other reducing substances, is then mixed with alkaline picrate reagent to yield the characteristic Jaffe reaction. Automated methods have replaced manual methods. These kinetic methods using the alkaline picrate reagent system have been adapted to use small volumes of serum and have readings taken within a short interval of 25-60 sec following initiation of the reaction. Because of the speed at which the analysis is performed and the small serum sample requirement, serum may be used directly, alleviating the need for a protein-free filtrate.

Which of the following is a copper transport protein that migrates as an alpha2-globulin? A. Ceruloplasmin B. Haptoglobin C. Transferrin D. Fibrinogen

A. Ceailoplasmin, a metalloprotein, is the principal transport protein of copper in the plasma. In the plasma, copper is primarily bound to ceruloplasmin, with only very small amounts of copper bound to albumin or in a dialyzable free state. When subjected to an electric field, ceruloplasmin migrates as an alpha2-globulin.

In what form must a drug be in order to elicit a pharmacologic response? A. Free B. Bound to albumin C. Bound to globulins D. Bound to fatty acids

A. Drugs in the free state are able to elicit a pharmacologic response. It is the free drug that is able to cross cell membranes and to bind at receptor sites. In the protein-bound state, drugs are unable to enter tissues and interact at receptor sites.

Which of the following represents a primary advantage of performing fluorometric over absorption spectroscopic methods of analysis? A. Increased specificity and increased sensitivity B. Increased specificity and decreased sensitivity C. Purity of reagents used not as critical D. Ease of performing assays

A. Fluorescence occurs when a molecule absorbs light of a particular wavelength and is thereby stimulated to emit light of a longer wavelength. The emitted light has a characteristic spectrum, the emission spectrum, that is unique for each fluorescing molecule. Hence, fluorometric methods are extremely sensitive and highly specific. Because of this extreme sensitivity, reagents used must be of a higher degree of purity than is required for spectroscopy, because even slight traces of impurities may fluoresce.

What would an individual with Gushing syndrome tend to exhibit? A. Hyperglycemia B. Hypoglycemia C. Normal blood glucose level D. Decreased 2-hour postprandial glucose

A. In dishing syndrome the adrenal cortex secretes an excessive amount of the hormone cortisol. Because cortisol has a stimulatory effect on gluconeogenesis, hyperglycemia commonly occurs as a secondary disorder. Hypoglycemia frequently characterizes Addison disease in which there is decreased production of cortisol.

A 75-year-old woman comes to her physician complaining of abdominal pain. She says she has had a sore stomach for the last 3 weeks and has been taking increasing doses of antacid pills to control it. Now she is taking a box of pills a day. Blood gases are drawn with the following results: pH = 7.49, PCO2 = 59 mm Hg, HCOJ = 25 mmol/L. What do these data indicate? A. Metabolic alkalosis, partially compensated B. Respiratory acidosis, uncompensated C. A dual problem of acidosis D. An error in one of the blood gas measurements

A. In this case the pH is increased indicating alkalosis. HCO^ is increased, which means it is a metabolic problem. The PCO2 is also increased, which indicates that the lungs are trying to compensate by retaining PCO2 thus bringing the pH closer to 7.4.

Which of the following may be described as a variant form of LDL, associated with increased risk of atherosclerotic cardiovascular disease? A. Lp(a) B. HDL C. ApoA-I D. ApoA-II

A. Lipoprotein (a) is an apolipoprotein that is more commonly referred to as Lp(a). Although it is related structurally to LDL, Lp(a) is considered to be a distinct lipoprotein class with an electrophoretic mobility in the prebeta region. Lp(a) is believed to interfere with the lysis of clots by competing with plasminogen in the coagulation cascade, thus increasing the likelihood of atherosclerotic cardiovascular disease.

Which of the following constituents normally present in serum must be chemically eliminated so that it will not interfere with the measurement of serum magnesium? A. Calcium B. Chloride C. Iron D. Potassium

A. Magnesium measurements are commonly done spectrophotometrically using reagent systems such as calmagite, methylthymol blue, and chlorophosphonazo III. Calcium will interfere and is eliminated by complexing with a chelator that binds calcium and not magnesium. Atomic absorption is a specific and sensitive method for analysis of magnesium, with the only significant interference being phosphate ions, which are removed by complexing with a lanthanum salt.

To aid in the diagnosis of skeletal muscle disease, which of the following serum enzyme measurements would be of most use? A. Creatine kinase B. Alkaline phosphatase C. Aspartate aminotransferase D. Alanine aminotransferase

A. To aid in the diagnosis of skeletal muscle disease, measurement of creatine kinase would be most useful. CK yields the most reliable information when skeletal muscle disease is suspected. Other enzymes that are also useful to measure are aspartate aminotransferase and lactate dehydrogenase. Both of these enzymes will be moderately elevated, whereas CK is significantly increased.

Which of the following is an example of a long-acting barbiturate? A. Phenobarbital B. Amobarbital C. Secobarbital D. Pentobarbital

A. The barbiturates are classified pharmacologically according to their duration of action. Phenobarbital is long acting, amobarbital and butabarbital are intermediate acting, and pentobarbital and secobarbital are short acting. In general, the long-acting barbiturates have higher therapeutic and toxic levels than the shorter-acting barbiturates. In cases of overdose, it is important to be able to identify the type of barbiturate in the blood for correct therapy. Measurement of specific barbiturates usually requires chromatography or immunoassay.

Which of the following functions as a transport protein for bilirubin in the blood? A. Albumin B. Alpha]-globulin C. Beta-globulin D. Gamma-globulin

A. The cells of the reticuloendothelial system are able to phagocytize aged red blood cells and convert the hemoglobin to the excretory product bilirubin. It is then necessary for the bilirubin to be transported to the liver, where it is conjugated for excretion in the bile. Albumin acts as the transport vehicle for unconjugated bilirubin in the blood, with each mole of albumin capable of binding two moles of bilirubin.

In a patient who is suspected of having pheochromocytoma, measurement of which of the following would be most useful? A. Metanephrine B. Homovanillic acid C. 5-Hydroxyindoleacetic acid D. Homogentisic acid

A. The majority of pheochromocytomas (rare tumors) occur in the adrenal medulla, causing increased secretion of the catecholamines. As a screening test for this disorder, quantification of urinary metanephrine, the methylated product of epinephrine, is suggested because false negatives seldom occur. Follow-up testing should include measurement of urinary vanillylmandelic acid (VMA), because VMA is the primary metabolite of epinephrine and norepinephrine.

Which of the following lipid tests is least affected by the fasting status of the patient? A. Cholesterol B. Triglyceride C. Fatty acid D. Lipoprotein

A. Total cholesterol screenings are commonly performed on nonfasting individuals. Total cholesterol is only slightly affected by the fasting status of the individual, whereas triglycerides, fatty acids, and lipoproteins are greatly affected. Following a meal, chylomicrons would be present, which are rich in triglycerides and fatty acids and contain very little cholesterol. The majority of cholesterol is produced by the liver and other tissues. High levels of exogenous triglycerides and/or fatty acids will interfere with the measurement of lipoproteins. Chylomicrons are normally cleared from the body 6 hours after eating.

If a blood gas specimen is left exposed to air, which of the following changes will occur? A. PO2 and pH increase; PCO2 decreases B. PO2 and pH decrease; PCO2 increases C. PO2 increases; pH and PCO2 decrease D. PO2 decreases; pH and PCO2 increase

A. When a blood specimen is drawn for gas analysis, it is important to avoid exposure of the specimen to air because of the differences in the partial pressures of carbon dioxide and oxygen in air and in blood. The PCO2 m blood is much greater than the PCO2 in air. Hence on exposure of blood to air, the total CO2 and the PCO2 both decrease, causing an increase in pH. Similarly, the PO2 of air is much greater than that of blood, thus, the blood PO2 increases on exposure to air.

What is the glucose concentration in fasting whole blood? A. Less than the concentration in plasma or serum B. Greater than the concentration in plasma or serum C. Equal to the concentration in plasma or serum D. Meaningless because it is not stable

A. When highly specific analytical methods are used, the glucose concentration in fasting whole blood is approximately 12-15% lower than in plasma or serum. Although glucose diffuses freely between the water phase of plasma and red blood cells, there is a higher concentration of water in plasma (approximately 12%) than in whole blood, accounting for the increased glucose concentration in plasma. The water content of whole blood depends on the hematocrit.

What term is used to describe the accumulation of bilirubin in the skin? A. Jaundice B. Hemolysis C. Cholestasis D. Kernicterus

A. When total bilirubin levels exceed 2.5 mg/dL, the clinical manifestation of jaundice develops. Characteristically, such body areas as the skin and sclera develop a yellow-pigmented appearance. Jaundice may be caused by an increase in either the unconjugated or conjugated form of bilirubin. Such increases in bilirubin levels may be caused by prehepatic, hepatic, or posthepatic disorders.

What substance is the precursor to all steroid hormones? A. Fatty acid B. Cholesterol C. Triglyceride D. Phospholipid

B. The 27-carbon, ringed structure of cholesterol is the backbone of steroid hormones. The nucleus is called the cyclopentanoperhydrophenanthrene ring. The steroid hormones having this ring include estrogens (18 carbons), androgens (19 carbons), glucocorticoids (21 carbons), and mineralocorticoids (21 carbons).

alpha 1 globulins

Alpha-1 Antitrypsin (AAT) Alpha-1 Fetoprotein (Alpha Fetoprotein, AFP)

myocardia infraction, liver disease, muscle trauma, renal infarct, hemolytic diseases, pernicious anemia

An increase in lactase dehydrogenase case result in the following:

Increase concentration of alpha-fetoprotein (AFP) in adults are most characteristically associated with: A. hepatocellular carcinoma B. alcoholic cirrhosis C. chronic active hepatitis D. multiple myeloma

Correct Answer: A

Night blindness is associated with deficiency of which of the following vitamins? A. A B. C C. niacin D. thiamine

Correct Answer: A

Of the following disease, the one most often associate with elevations of lactate dehydrogenase isoenzymes 4 and 5 on electrophoresis is: A. liver disease B. hemolytic anemia C. myocardial infarction D. pulmonary edema

Correct Answer: A

Phenobarbital is a metabolite of: A. primidone B. phenytoin C. amobarbital D. secobarital

Correct Answer: A

Which of the following determinations is useful in prenatal diagnosis of open neural tube defects? A. amniotic fluid alpha-fetaprotein B. amniotic fluid estriol C. maternal serum estradio D. maternal serum estrone

Correct Answer: A

Which of the following is a low-weight protein that is found on the cell surfaces of nucleated cells? A. C-reactive protein B. p2-Microglobulin C. Ceruloplasmin D. G^-Macroglobulin

B. (32-Microglobulin is a single polypeptide chain that is the light chain component of human leukocyte antigens (HLAs). It is found on the surface of nucleated cells and is notably present on lymphocytes. Increased plasma levels of fi2~ microglobulin are associated with renal failure, lymphocytosis, rheumatoid arthritis, and systemic lupus erythematosus.

In cases of hepatoma, which protein not normally found in adult serum is synthesized by liver cells? A. cq-Acid glycoprotein B. aj-Fetoprotein C. a2-Macroglobulin D. Carcinoembryonic antigen

B. AFP is normally produced only by the fetus, with blood levels disappearing shortly after birth. However, in the adult, such conditions as hepatoma or teratoma stimulate the production of this primitive protein by the tumor cells. The quantification of AFP may be used both diagnostically and as a monitor of chemotherapy.

In the United States, most cases of scurvy occur in children between the ages of 7 months to 2 years. Scurvy is a disease caused by a deficiency in which of the following? A. Vitamin A B. Vitamin C C. Vitamin D D. Vitamin K

B. Ascorbic acid is commonly known as vitamin C. Because humans are unable to synthesize ascorbic acid, it is necessary that it be taken in through the diet. If ascorbic acid is not ingested in a sufficient amount, a deficiency develops that leads to the disease known as scurvy. Scurvy is characterized by bleeding gums, loose teeth, and poor wound healing.

A cerebrospinal fluid specimen is sent to the lab at 9:00 P.M. for glucose analysis. The specimen is cloudy and appears to contain red blood cells. Which of the following statements is true? A. Glucose testing cannot be performed on the specimen. B. Specimen should be centrifuged and glucose assayed immediately. C. Specimen can be refrigerated as received and glucose assayed the next day. D. Specimen can be frozen as received and glucose assayed the next day.

B. Because of the critical reasons for aspirating a CSF specimen, the testing is performed as soon as possible upon receipt of the specimen in the laboratory. In this case, the cloudy appearance would be most likely due to the presence of bacteria. Both bacteria and red blood cells can use glucose in vitro. Thus any delay in glucose testing could result in a falsely low result. The CSF specimen should be centrifuged to remove cellular material and assayed immediately.

To what metal does ceruloplasmin firmly bind? A. Chromium B. Copper C. Zinc D. Iron

B. Copper is found in the plasma mainly in two forms: a minor fraction loosely bound to albumin and the majority, representing about 80-95%, firmly bound to the enzyme ceruloplasmin, an a2-globulin, which is important in the oxidation of iron from the ferrous to the ferric state. Copper is also an essential constituent of a variety of other enzymes found in erythrocytes and in other sites throughout the body. The major clinical usefulness of determining serum copper or ceruloplasmin levels is that the decreased level of both is associated with Wilson disease. Decreased levels of copper are also found in protein malnutrition and malabsorption and in nephrosis.

Which of the following methods allows for the separation of charged particles based on their rates of migration in an electric field? A. Rheophoresis B. Electrophoresis C. Electroendosmosis D. Ion exchange

B. Electrophoresis is a method of separating charged particles by their rates of migration in an electric field. An electrophoretic chamber consists of two electrodes, two reservoirs to hold buffer, a means of supporting a strip in the chamber so that the ends are dipping into the reservoirs, and a means of applying an electric current to the strip. The whole chamber is sealed to make it vaporproof.

Which of the following enzymes does not belong to the class of enzymes known as the hydrolases? A. Alkaline phosphatase B. Aldolase C. Amylase D. Lipase

B. Hydrolases are enzymes that split molecules with the addition of water—for example, amylase, lipase, alkaline phosphatase, acid phosphatase, 5'-nucleotidase, and trypsin. They do not usually require coenzymes but often need activators. Aldolase and carbonic anhydrase are examples of the class of enzymes known as the lyases. Lyases are enzymes that split molecules between carbon-to-carbon bonds without the addition of water. The resulting products usually contain carbon double bonds.

Given the following information on a particular compound that has been visualized by means of thin-layer chromatography, calculate the /fyof the compound. Distance from origin to spot center = 48 mm Distance from spot center to solvent front = 93 mm Distance from origin to solvent front = 141 mm A. 0.29 B. 0.34 C. 0.52 D. 0.66

B. In thin-layer chromatography (TLC), the Rf (retention factor) describes the distance traveled by the solute (compound of interest) in relation to the distance traveled by the solvent (mobile phase). Measurements of the TLC plate are made from the origin or point of sample application to the center of the developed spot and from the origin to the solvent front. An Rf may be calculated by means of the following formula: Rf = Distance from origin to spot center/ Distance from origin to solvent front Rf = 48mm/ 141 mm = 0.34 The Rf of the compound of interest, along with chromogenic spray characteristics, may then be compared with standards for identification of the unknown compound.

Which of the following is used in the treatment of manic depression? A. Potassium B. Lithium C. Calcium D. Chloride

B. Lithium is used in the treatment of manic depression. Because of the small difference between therapeutic and toxic levels in the serum, accurate measurements of lithium concentrations are essential. It is also important to standardize the sample drawing time in relation to the previous dose. Measurement is made by ion-selective electrode electrochemical analysis or by atomic absoiption spectrophotometric analysis.

Analysis of CSF for oligoclonal bands is used to screen for which of the following disease states? A. multiple myeloma B. multiple sclerosis C. myasthenia gravis D. von Willebrand disease

Correct Answer: B

After absorption, codeine is rapidly metabolized to what compound? A. Phencyclidine B. Morphine C. Methadone D. Propoxyphene

B. Morphine, codeine, and heroin are collectively referred to as opiates. Codeine is found in many prescription medicines and is rapidly metabolized after absorption into morphine and norcodeine. Because blood concentrations of most opiates are low even in overdose, screening is usually done on the urine. Immunoassay or colorimetric methods can be used for screening purposes, but chromatography is generally required for quantification of specific compounds. Gas chromatography/mass spectroscopy (GC/MS) is useful for the quantification of morphine and codeine.

In which of the following conditions would a normal level of creatine kinase be found? A. acute myocardial infarct B. hepatitis C. progressive muscular dystrophy D. intramuscular injection

Correct Answer: B

Which of the following is not descriptive of a photomultiplier tube? A. Emits electrons proportionally to initial light absorbed B. Must be shielded from stray light C. Cannot be used with a chopper D. Amplifies the initial signal received

B. Photometric methods are based on the use of Beer's law, which is applicable only for monochromatic light. A monochromator is a device for selecting a narrow band of wavelengths from a continuous spectrum. The three kinds of monochromators are filters, prisms, and diffraction gratings.

Which vitamin is a constituent of two redox coenzymes? A. Vitamin A B. Vitamin B2 C. Vitamin B6 D. Vitamin C

B. Riboflavin (vitamin 62) is a constituent of two redox coenzymes, flavin mononucleotide (FMN) and flavin-adenine dinucleotide (FAD). These coenzymes, in combination with appropriate proteins, form the flavoprotein enzymes, which participate in tissue respiration as components of the electron-transport system. The property that enables them to participate in electron-transport is their ability to exist in the half-reduced form (FADH) and in the fully reduced form (FADH2).

Which of the following chromogens will not produce a colored complex with iron that can be measured spectrophotometrically? A. Bathophenanthroline B. 8-Hydroxyquinoline C. Tripyridyltriazine D. Ferrozine

B. Serum iron concentrations are most often determined by the colorimetric reaction with ferrozine, bathophenanthroline, or tripyridyltriazine. The same reagent is usually used in the determination of serum TIBC by saturating the transferrin in the serum with an excess of iron, removing any unbound iron, and measuring the iron bound to transferrin. This measurement of TIBC provides a measure of transferrin concentration. Several magnesium methods require the precipitation of magnesium as part of the analysis, and 8-hydroxyquinoline effectively precipitates magnesium.

Which enzyme is measured by methodologies that use small oligosaccharides and 4-nitrophenyl-glycoside for substrates? A. Lipase B. Amylase C. Creatine kinase D. Cholinesterase

B. The function of amylase to catalyze the hydrolysis of starch to dextrins, maltose, and glucose has been used as the basis for several methods over the years. The more commonly used methods today employ small oligosaccharides and 4-nitrophenyl-glycoside as substrates. In general, these methods can be automated, using an oxygen electrode system and UV or visible wavelength spectrophotometry to determine amylase activity.

What compound is a crucial intermediary in the metabolism of triglyceride to form energy? A. Bile B. Acetyl-coenzyme A C. Acetoacetate D. Pyruvate

B. The long-chain fatty acids of triglycerides can be broken down to form energy through the process of beta/oxidation, also known as the fatty acid cycle. In this process, two carbons at a time are cleaved from long-chain fatty acids to form acetyl-coenzyme A. Acetyl-coenzyme A, in turn, can enter the Krebs cycle to be converted to energy or be converted to acetoacetyl-Co-A and converted to energy by an alternate pathway, leaving behind the acidic by-product ketones composed of beta-hydroxybutyrate, acetoacetate, and acetone. Under proper conditions, pyruvate can be converted to acetyl-coenzyme A at the end of glycolysis of glucose. Bile is a breakdown product of cholesterol used in the digestion of dietary cholesterol.

Thyroid-releasing hormone (TRH) is given to a patient. Serum thyroid-stimulating hormone (TSH) levels are taken before and after the injection, and the values are the same—low. This patient probably has which of the following disorders? A. Primary hypothyroidism B. Secondary hypothyroidism C. Tertiary hypothyroidism D. Iodine deficiency

B. To distinguish between a hypothalamic disorder and a disorder of the pituitary gland, thyroidreleasing hormone (TRH) is administered. In the case of a hypothalamic disorder (tertiary hypothyroidism), the TRH administered will cause an increased excretion of pituitary hormone, TSH. However, if the disorder originates in the pituitary gland (secondary hypothyroidism), the administration of TRH will have no effect on the pituitary gland and thus no increased excretion of TSH. Because the values of TSH were low before and remained low after administration of TRH, the disorder is secondary hypothyroidism. Primary hypothyroidism is caused by failure of the thyroid gland itself and is not evaluated by use of the TRH stimulation test. Iodine deficiency would cause high levels of TSH, and administration of TRH is not used to evaluate this disorder.

As part of a routine physical, a fasting plasma glucose is performed on a 45-yearold male and the test result is 105 mg/dL. How should this individual be classified? A. Normal for his age B. Impaired fasting glucose C. Type 1 diabetes mellitus D. Type 2 diabetes mellitus

B. When a fasting plasma glucose test is performed and the glucose value is between 100-125 mg/dL, the individual is considered to have impaired fasting glucose (IFG). This is less than the value associated with diagnosis of diabetes mellitus, which is a fasting plasma glucose > 126 mg/dL. IFG is considered a risk factor and a stage between normal glucose metabolism and development of diabetes mellitus.

An electrophoretic separation of lactate dehydrogenase isoenzymes that demonstrates and elevation in LD-1 and LD-2 in a "flipped pattern is consistent with: A. myocardial infartion B. viral hepatitis C. pancreatis D. renal failure

Correct Answer: A

ALP

Biliary obstruction; may be slightly elevated in hepatitis

osteoblastic tumors

Bone forming tumor (osteochondroma)

Which of the following is the most potent estrogen and is considered to be the true ovarian hormone? A. Estriol (E3) B. Estrone (Ej) C. 17p-Estradiol (E2) D. 16a-Hydroxyestrone

C. 17(3-Estradiol (E2) is the most potent estrogen. 17(3-Estradiol is considered to be the true ovarian hormone because it is secreted almost entirely by the ovaries. In contrast, estrone (Ej) is produced from circulating €19 neutral steroids (e.g., androstenedione) and is also synthesized from 17(3-estradiol. Estriol (£3) is derived almost exclusively from 17(3-estradiol and has little clinical significance except in pregnancy. The measurement of 17(3-estradiol is used to evaluate ovarian function.

Which of the following is an example of an antiarrhythmic drug that has a metabolite with the same action? A. Quinidine B. Digoxin C. Procainamide D. Nortriptyline

C. Although digoxin, nortriptyline, and quinidine have various effects on cardiac arrhythmias, they do not have metabolites with similar activity. Procainamide is an antiarrhythmic drug and has at least one metabolite with the same activity, namely, ,/V-acetylprocainamide (NAPA). Because of differences in half-life, NAPA may accumulate in the blood and produce toxic effects even with therapeutic levels of procainamide. Therefore, both procainamide and NAPA need to be quantified for therapeutic drug monitoring.

Bile acids that are synthesized in the liver are derived from what substance? A. Bilirubin B. Fatty acid C. Cholesterol D. Triglyceride

C. Bile acids are synthesized in the hepatocytes of the liver. They are C24 steroids that are derived from cholesterol. With fat ingestion, the bile salts are released into the intestines, where they aid in the emulsification of dietary fats. Thus bile acids also serve as a vehicle for cholesterol excretion. A majority of the bile acids, however, are reabsorbed from the intestines into the enterohepatic circulation for reexcretion into the bile. The two principal bile acids are cholic acid and chenodeoxycholic acid. These acids are conjugated with one of two amino acids, glycine or taurine. Measurement of bile acids is possible via immunotechniques and may aid in the diagnosis of some liver disorders such as obstructive jaundice, primary biliary cirrhosis, and viral hepatitis.

An increase in serum acetone is indicative of defect in the metabolism of: A. carbohdrates B. fat C. urea nitrogen D. uric acid

Correct Answer: A

Which of the following statements is false about the electrolyte chloride? A. Main anion of the extracellular fluid B. Can shift from the extracellular plasma to the intracellular fluid of red blood cells C. Unable to be reabsorbed by active transport D. Measured in serum, urine, and sweat

C. Chloride is the principal plasma anion. The average concentration of chloride in plasma is 103 mmol/L. In the kidneys, chloride ions are removed from the blood through the glomerulus and then passively reabsorbed by the proximal tubules. The chloride pump actively reabsorbs chloride in the thick ascending limb of the loop of Henle. In the lungs, chloride ions participate in buffering the blood by shifting from the plasma to the red blood cells to compensate for ionic changes that occur in the alveoli when the HCO^ from the red blood cells enters the plasma. This is termed the chloride shift. Chloride can be measured in a variety of body fluids, including serum, plasma, urine, and sweat.

An endogenous substance assayed to assess the glomerular filtration rate may be described as being filtered by the glomeruli, not reabsorbed by the tubules, and only secreted by the tubules when plasma levels become elevated. What is this frequently assayed substance? A. Inulin B. Uric acid C. Creatinine D. Urea

C. Creatinine is an endogenous substance that is filtered by the glomeruli and normally is neither reabsorbed nor secreted by the tubules. When plasma levels of creatinine rise, some secretion of creatinine by the tubules will occur. The filtration properties of creatinine and the fact that it is a substance normally present in blood make the creatinine clearance test the method of choice for assessing the glomerular nitration rate.

What would be an example of ectopic hormone production? A. Prolactin production by pituitary tumors B. Calcitonin production by thyroid tumors C. Growth hormone production by lung tumors D. Cortisol production by adrenal tumors

C. Ectopic hormones are hormonal substances produced by benign and malignant tumors derived from tissues that do not normally secrete those hormones. Examples of ectopic hormone production would be ACTH production by oat cell carcinoma of the lung and growth hormone production by bronchogenic carcinomas of the lung. Cortisol and growth hormone are normally secreted by the adrenal gland and anterior pituitary gland, respectively. Ectopic hormones are not in all cases chemically identical to the native hormone but may be similar enough to crossreact in immunoassay methods for the native hormone.

Which bilirubin fraction is unconjugated and covalently bound to albumin? A. Alpha B. Beta C. Delta D. Gamma

C. Four bilirubin fractions represented by Greek letters have been identified: unconjugated (alpha), monoconjugated (beta), diconjugated (gamma), and unconjugated bilirubin covalently bound to albumin (delta). Delta-bilirubin is normally present in low concentration in the blood, and it is known to react directly with diazotized sulfanilic acid. Increased serum levels of delta-bilirubin are associated with liver-biliary disease.

Zollinger-Ellison syndrome is characterized by an elevated blood level of which of the following? A. Trypsin B. Pepsin C. Gastrin D. Cholecystokinin-pancreozymin

C. Gastrin is the designation given to a family of protein hormones produced by the mucosal cells of the gastric antrum. Once secreted, gastrin is earned in the blood to the fundic cells, causing release of hydrochloric acid. Serum gastrin levels are markedly elevated in the Zollinger-Ellison syndrome, a neoplastic proliferation of the nonbeta cells of the pancreatic islets. Gastrin levels may also be elevated in pernicious anemia, duodenal ulcer disease, and gastric ulcer disease.

How would blood gas parameters change if a sealed specimen is left at room temperature for 2 or more hours? A. PO2 increases, PCO2 increases, pH increases B. PO2 decreases, PCO2 decreases, pH decreases C. PO2 decreases, PCO2 increases, pH decreases D. PO2 increases, PCO2 increases, pH decreases

C. Glycolysis and other oxidative metabolic processes will continue in vitro by red blood cells when a whole blood specimen is left standing at room temperature. Oxygen is consumed during these processes, resulting in a decrease in PO2 levels. A decrease of 3-12 mm Hg/hr at 37°C has been observed for blood specimens exhibiting normal /3O2 ranges. This rate of decrease is accelerated with elevated PO2 levels. Additionally, carbon dioxide is produced as a result of continued metabolism. An increase in /3CO2 levels of approximately 5 mm Hg/hr at 37°C has been demonstrated. The increased production of carbonic acid and lactic acid during glycolysis contributes to the decrease in blood pH.

In a patient with suspected primary hyperthyroidism associated with Graves disease, one would expect the following laboratory serum results: free thyroxine (FT4) _______ , thyroid hormone binding ratio (THBR) ____________, and thyroid-stimulating hormone (TSH) A. Increased, decreased, increased B. Increased, decreased, decreased C. Increased, increased, decreased D. Decreased, decreased, increased

C. Graves disease is a name given to a diffusely hyperactive thyroid that produces thyrotoxicosis. Thyrotoxicosis results from elevated levels of thyroid hormone; therefore, laboratory results for free thyroxine (FT4) and free triiodothyronine (FT3) would be increased, thyroid hormone binding ratio (THBR) increased, and thyroidstimulating hormone (TSH) decreased. In hyperthyroidism, the THBR is increased because thyroxine-binding globulin (TBG) is saturated with endogenous T4. TSH levels are decreased because of the negative-feedback control of the thyroid hormones on the anterior pituitary.

Which of the following instruments has a sample-introduction system, solventdelivery system, column, and detector as components? A. Atomic absorption spectrometer B. Mass spectrometer C. High-performance liquid chromatograph D. Nephelometer

C. High-performance liquid chromatography (HPLC) systems are composed of four basic units: sample-introduction system, solventdelivery system, column, and detector. The sample-introduction system is generally a fixedloop injection valve, which allows the sample to be injected into a stainless steel external loop for flushing onto the column by the solvent. The solvent-delivery system may be composed of one or two pumps for the purpose of forcing the mobile phase and sample through the column. Photometric, fluorometric, and electrochemical detectors are available for monitoring the eluate as it emerges from the column.

A mother brings her obese, 4-year-old child who is a known type 1 diabetic to the laboratory for a blood workup. She states that the boy has been fasting for the past 12 hours. After centrifugation the tech notes that the serum looks turbid. The specimen had the following results: blood glucose = 150 mg/dL, total cholesterol = 250 mg/dL, HDL cholesterol = 32 mg/dL, triglyceride = 395 mg/dL. What best explains these findings? The boy A. Is a low risk for coronary artery disease B. Is a good candidate for a 3-hour oral glucose tolerance test C. Has secondary hyperlipidemia due to the diabetes D. Was not fasting when the specimen was drawn

C. In this case, the child fits the description of a suspected hyperlipemic patient. He is known to have diabetes mellitus, and the mother has assured the laboratory that the boy has followed the proper fasting protocol before the test. Hyperlipoproteinemia can be secondary to diabetes mellitus. The boy has a relatively high risk to develop CAD, and, as a known diabetic, should never undergo an oral 3-hour glucose tolerance test.

A 30-year-old pregnant woman has a gestational diabetes mellitus screening test performed at 26 weeks of gestation. Her physician chooses to order a 50-g oral glucose load. Her serum glucose level is 150 mg/dL at 1 hour. What should occur next? A. This confirms diabetes mellitus; give insulin. B. This confirms diabetes mellitus; dietary intake of carbohydrates should be lessened. C. This is suspicious of diabetes mellitus; an oral glucose tolerance test should be performed. D. This is an expected glucose level in a pregnant woman.

C. Increased insulin resistance is commonly seen in the late second and third trimesters of pregnancy. Most women are able to compensate by secreting additional insulin and, thus, are able to maintain normal blood glucose levels. In cases of gestational diabetes mellitus, women are unable to make sufficient insulin to meet their needs. In the screening test, serum glucose is assessed at 1 hour following the ingestion of a 50-gram glucose load (glucose challenge test). If the serum glucose is >140 mg/dL, the next step is to perform an oral glucose tolerance test.

Which of the following is false about ion-selective electrode analysis of sodium? A. Uses a glass membrane B. Errors occur from protein buildup on the membrane. C. Membrane coated with valinomycin D. Principle based on potentiometry

C. Ion-selective electrodes for the measurement of sodium are glass membrane electrodes with selective capability. They are constructed from glass that consists of silicon dioxide, sodium oxide, and aluminum oxide. This type of electrode is based on the principle of potentiometry. Measurement errors may occur from protein buildup on the membrane surface. Potassium is measured using an ion-exchange electrode where the liquid ion-exchange membrane consists of valinomycin as the ionselective carrier.

Which of the following disorders is characterized by increased production of chloride in sweat? A. Multiple myeloma B. Hypoparathyroidism C. Cystic fibrosis D. Wilson disease

C. Measuring the concentration of chloride in sweat is a commonly used diagnostic procedure for determining the disorder of cystic fibrosis (CF). The majority of patients with CF will present with increased concentrations of sodium and chloride in their sweat. Generally, children with CF will manifest sweat chloride levels that are two to five times the reference interval. In sweat testing, sweat production is stimulated by iontophoresis with pilocarpine. Then the sweat is either collected and analyzed for chloride or an ion-selective electrode is applied to the skin surface to quantify chloride. It has been established that the gene abnormality causing CF is located on chromosome 7.

Which statement best describes the clinical use of measuring NT-proBNP? A. Used to assess risk of coronary heart disease B. Used to assess risk of angina C. Used to assess individuals treated with nesiritide D. Used to assess individuals treated with vitamin B

C. N-terminal proBNP is released when BNP (Btype or brain natriuretic peptide) is cleaved from precursor proBNP. NT-proBNP is released in a 1:1 ratio to BNP; however, NT-proBNP has a half-life of hours as compared to BNP's short half-life of approximately 22 minutes. The longer half-life of NT-proBNP contributes to its clinical utility. In addition, when measuring NT-proBNP, there is no interference when an individual is being treated with nesiritide, which is human recombinant BNP used in treating congestive heart failure.

A 53-year-old female presents with fatigue, pruritus, and an enlarged, nontender liver. The physician orders a series of blood tests. Based on the following serum test results, what is the most likely diagnosis? Alkaline phosphatase—markedly elevated Alanine aminotransferase—slightly elevated Lactate dehydrogenase—slightly elevated Gamma-glutamyltransferase— markedly elevated Total bilirubin—slightly elevated A. Alcoholic cirrhosis B. Infectious mononucleosis C. Intrahepatic cholestasis D. Viral hepatitis

C. Obstruction of the biliary tree is also referred to as intrahepatic cholestasis. This disorder is characterized by significant elevations in the serum levels of alkaline phosphatase and gamma-glutamyltransferase. The serum levels of alanine and aspartate aminotransferases and lactate dehydrogenase are only slightly elevated. Early in the disease, the serum bilirubin level may be normal or only slightly elevated. In alcoholic cirrhosis, viral hepatitis, and infectious mononucleosis, only a slight to moderate elevation of alkaline phosphatase would be seen.

What does the concentration of urinary free cortisol mainly reflect? A. Total serum cortisol B. Conjugated cortisol C. Unbound serum cortisol D. Protein-bound serum cortisol

C. Only very small quantities, normally less than 2%, of the total adrenal secretion of cortisol appear in the urine as free cortisol. The majority of cortisol is either metabolized in various tissues or conjugated in the liver and excreted. It is only the serum unconjugated cortisol not bound to corticotropin binding globulin (CBG) or the conjugated cortisol that can be cleared by glomerular filtration in the kidney. Therefore, the measurement of free cortisol in the urine is a sensitive reflection of the amount of unbound cortisol in the serum. It is not a reflection of the amount of conjugated cortisol or the serum total cortisol but, rather, only the increased cortisol production that is not accompanied by an increase in serum levels of CBG.

Which is a compensatory mechanism in respiratory acidosis? A. Hypoventilation B. Decreased reabsorption of bicarbonate by the kidneys C. Increased Na+/H+ exchange by the kidneys D. Decreased ammonia formation by the kidneys

C. Respiratory acidosis is a disturbance in acidbase balance that is caused by the retention of CO2 by the lungs. This imbalance is associated with such conditions as bronchopneumonia, pulmonary emphysema, pulmonary fibrosis, and cardiac insufficiency. Respiratory acidosis is characterized by a primary excess in physically dissolved CO2, which is quantified by measuring the blood PCO2 level. The primary problem leading to an increase in the PCO2 level is hypoventilation. This retention of CO2 alters the normal 20:1 ratio of cHCOJ/PCO2, causing a decrease in blood pH level. In respiratory acidosis, because the initial defect is associated with the lungs, the kidneys respond as the major compensatory system. The production of ammonia, the exchange of Na+ for H+ with the excretion of H+, and the reabsorption of bicarbonate are all increased in the kidneys to compensate for the malfunction of the lungs. In cases where the defect is not within the respiratory center, the excess of /3CO2 in the blood can actually have a stimulatory effect on the center, causing an increase in the respiration rate. Thus compensation can also occur through CO2 elimination by the lungs.

Which of the following is not associated with silver stains? A. Reactive to nanogram concentrations of proteins B. Polypeptides stain a variety of colors C. Not as sensitive as Coomassie brilliant blue D. Preconcentration of CSF not necessary

C. Silver stains react with nanogram concentrations of proteins and nucleic acids, staining them shades of green, yellow, blue, and red. Silver stains are approximately 30 times more sensitive than Coomassie blue stains. Because of their sensitivity, silver stains are being used in electrophoretic methods to identify cerebrospinal fluid and urine proteins without preconcentration of the specimens.

In the Berthelot reaction, what contaminant will cause the urea level to be falsely elevated? A. Sodium fluoride B. Protein C. Ammonia D. Bacteria

C. The Berthelot reaction is based on the production of a blue-indophenol compound when ammonia reacts in an alkaline medium with phenol and sodium hypochlorite. This basic colorimetric reaction can be used to quantify both urea and blood ammonia levels. Therefore, any ammonia contamination (i.e., in the distilled water used to make reagents for the urea procedure and on glassware) must be avoided so that falsely elevated urea values will not be obtained.

Indirect-reacting bilirubin may be quantified by reacting it initially in which reagent? A. Dilute hydrochloric acid B. Dilute sulfuric acid C. Caffeine-sodium benzoate D. Sodium hydroxide

C. Unlike direct bilirubin, indirect-reacting bilirubin is insoluble in deionized water and dilute hydrochloric acid. Indirect-reacting bilirubin must first be mixed with methanol or caffeine-sodium benzoate to solubilize it before proceeding with the diazo reaction. Because of these properties, total bilirubin and direct bilirubin are usually chemically analyzed, and the indirect, or unconjugated, fraction is calculated from the difference between the total and direct values. The total value represents the reaction of both conjugated and unconjugated bilirubin, whereas the direct value represents only the reaction of conjugated bilirubin.

Which of the following is another name for vitamin Bj2? A. Retinol B. Pyridoxine C. Cyanocobalamin D. Riboflavin

C. Vitamin B12 (cyanocobalamin) is a cobaltcontaining vitamin that is necessary for normal erythropoiesis. Intrinsic factor is a gastric protein that specifically binds vitamin B12 and carries it to the ileurn for absorption. The transcobalamins are a group of plasma proteins, some of which bind vitamin B]2 and some of which bind both vitamin B12 and cobalamin analogs. The cobalophilins (R proteins) are those transcobalamins that can also bind the cobalamin analogs.

Which of the following serum components is able to alter the free drug level in plasma? A. Creatinine B. Urea C. Albumin D. Calcium

C. Within the systemic circulation a drug will either remain free or will bind to protein. Generally, acidic drugs bind to albumin, and basic drugs bind to such globulins as alpha j -acid glycoprotein (AAG). Occasionally, a particular drug may bind to both types of protein.

6-10

CK-MB rises within _____ hrs of AMI; peaks within 24hrs; returns to normal within 2-3 days

skeletal muscles

CK-MM

cardiac muscles

CK-MM and Ck-MB

Total carbon dioxide (CO2)

CO2+ HCO3- +H2CO3= ________ reflects bicarbonate (HCO3-) concentration.

Magnesium (Mg)

Ca channel blocking agent increases in renal failure decreased in cardiac disorders, D. mellitus, and diuretics

A potassium level of 6.8 mEq/L (6.8 mmol/L) is obtained. Before reporting the results, the first step the technologist should take is to: A. check the serum for hemolysis B. rerun the test C. check the age of the patient D. do nothing, simply report out the result

Correct Answer: A

Fetal fibronectin (fFN)

Cervical swab for a protein that acts as a "glue" during pregnancy attaches the amniotic sac to the lining of the uterus. After the 35th week of pregnancy, it begins to break down naturally, and is detectable. If premature birth is imminent, fetal fibronectin may be detected before week 35.

proteins

Chains of amino acids linked together by amide bonds

An infant with diarrhea is being evaluated for a carbohydrate intolerance. His tool yields a positive copper reduction test and a pH of 5.0. It should be concluded that: A. further test are indicated B. results are inconsistent- repeat both tests C. the diarrhea is not due to carbohydrate intolerance D. the test provided no useful information

Correct Answer: A

A serum sample drawn in the emergency room form a 42-year-old man yielded the following: Patient; Reference range -CK: 185 U/L; 15-60 U/L -AST: 123 U/L; 0-48 U/L -CKMB: 6 U/L; 2-12 U/L Which of the following conditions might account for these values? A. crush injury to the thigh B. cerebrovascular accident C. pulmonary infarction D. early acute hepatitis

Correct Answer: A

respiratory acidosis compensation

Compensation: Metabolic kidneys conserve bicarb and excrete H Deep and Slow Resps

respiratory alkalosis compensation

Compensation: Metabolic kidneys excrete bicarb and conserve H; cellular buffering Shallow RapidBreaths

"Laboratory A" measures maternal serum alpha-fetoprotein (MSAFP) at 16-18 weeks gestation as a screen for fetal disorders. The 16-weeks MSAFP medium for Lab A is 32 ug/L. A 37-year-old woman has an MSAFP level of 34 ug/L at her 16th week. This reuslt is consistent with: A. a normal MSAFP level for 16-week gestation B. possible neural tube defect, including spina bifida C. possible multiple birth (ie, twins) D. possible trisomy disorder, including Down syndrome

Correct Answer: A

A 24-hour urine specimen (total volume =1,136 mL) is submitted to the laboratory for quantitative urine protein. Calculate the amount of protein excreted per day if the total protein is 52 mg/dL. A. 591 mg B. 487 mg C. 220 mg D. 282 mg

Correct Answer: A

A 45-year-old male of average heigh and weight was admitted to the hospital for renal function studies. He had the following lab results: -urine creatinine: 120 mg/dL (10680 umol/L) -serum creatinine: 1.5 mg/dL (132.6 umol/L) -total urine volume in 24 hours: 1800/mL Calculate the creatinine clearance for this patient in mL/min. A. 100 B. 144 C. 156 D. 225

Correct Answer: A

Which of the following biochemical processes is promoted by insulin? A. Glycogenolysis B. Gluconeogenesis C. Lipolysis D. Uptake of glucose by cells

Correct Answer: D 1. D Insulin reduces blood glucose levels by increasing glucose uptake by cells. It promotes lipid and glycogen production, induces synthesis of glycolytic enzymes, and inhibits formation of glucose from pyruvate and Krebs cycle intermediates.

In the liver, bilirubin is conjugated by addition of: A. Vinyl groups B. Methyl groups C. Hydroxyl groups D. Glucuronyl groups

Correct Answer: D 49. D The esterification of glucuronic acid to the propionyl side chains of the inner pyrrole rings (I and II) makes bilirubin water soluble. Conjugation is required before bilirubin can be excreted via the bile.

What is the primary clinical utility of measuring CEA? A. Diagnosis of liver cancer B. Diagnosis of colorectal cancer C. Screening for cancers of endodermal origin D. Monitoring for recurrence of cancer

Correct Answer: D 9. D CEA is a glycoprotein that is secreted into plasma by various cancers of endodermal origin, including breast, lung, colorectal, and stomach cancer. However, it is present in only 40%-60% of such cancers, is present at low levels (<3.0 ng/mL) in normal adults, and is increased by causes other than cancer (e.g., smoking). Its clinical use is to detect recurrence and the need for second-look surgery in persons who have been treated and to evaluate the response to treatment.

The preparation of a patient for standard glucose tolerance testing should include: A. a high carbohydrate diet for 3 days B. a low carbohydrate diet for 3 days C. fasting for 48 hours prior to testing D. bed rest for 3 days

Correct answer: A

What abbreviation has been used in the past to designate alanine aminotransf erase? A. AST B. A AT C. GOT D. GPT

D. Alanine aminotransferase (ALT), formerly known as glutamate pyruvate transaminase (GPT), and aspartate aminotransferase (AST), formerly known as glutamate oxaloacetate transaminase (GOT), are categorized as transferase enzymes. These older designations are still seen in conjunction with the current terminology on reagent packaging, on physician test request forms, and on laboratory test result forms. Through the transfer of amino groups, ALT and AST catalyze the interconversion of amino acids and keto acids. ALT catalyzes the interconversion of alanine and oxoglutarate to pyruvate and glutamate. The reaction is reversible. In viral hepatitis, both ALT and AST are elevated. In acute myocardial infarction, AST is elevated and ALT is normal or slightly increased. L-Alanine + a-oxoglutarate ALT P-5'-P pyruvate + L-glutamate

What is the basis for the Kjeldahl technique for the determination of serum total protein? A. Quantification of peptide bonds B. Determination of the refractive index of proteins C. Ultraviolet light absorption by aromatic rings at 280 nm D. Quantification of the nitrogen content of protein

D. Although the Kjeldahl technique for the determination of protein nitrogen is too cumbersome for use in routine testing, it is considered to be the reference method of choice to validate materials used with the biuret method. The Kjeldahl technique is based on the quantification of the nitrogen content of protein. It is estimated that the average nitrogen content of protein is 16% of the total weight. In the Kjeldahl technique, protein undergoes a digestion process with sulfuric acid through which the nitrogen content of the protein is converted to ammonium ion. The ammonium ion in turn may be reacted with Nessler's reagent, forming a colored product that is read spectrophotometrically, or the ammonium ion may undergo distillation, liberating ammonia that is titrated.

What is the purpose of using ampholytes in isoelectric focusing? A. Maintain the polyacrylamide gel in a solid state B. Maintain the protein sample in a charged state C. Maintain the pH of the buffer solution D. Establish a pH gradient in the gel

D. Ampholytes are mixtures of polyanions and polycations used to establish a pH gradient within the gel media in isoelectric focusing. When an electrical field is applied to the gel, ampholytes seek their own isoelectric point where they become stationary, establishing a pH gradient. Similarly, proteins will migrate within the gel-gradient until they reach the pH of their isoelectric point, thus becoming stationary or focused. This system is most useful in separating proteins that have close isoelectric points.

Which of the following does not properly describe type 1 diabetes mellitus? A. Insulin deficiency B. Associated with autoimmune destruction of pancreatic (3-cells C. Ketoacidosis prone D. Occurs more frequently in adults

D. Based on the biochemistry of the disease, diabetes mellitus has been classified as type 1 and type 2. Type 1 occurs more commonly in individuals under 20 years of age. Studies suggest that type 1 is associated with autoimmune destruction of (3-cells, and it is characterized by insulin deficiency and thus a dependency on injection of insulin. Unlike people afflicted with type 2, type 1 individuals are prone to ketoacidosis and to such complications as angiopathy, cataracts, nephropathy, and neuropathy.

What is the compound that comprises the majority of the nonprotein-nitrogen fractions in serum? A. Uric acid B. Creatinine C. Ammonia D. Urea

D. Constituents in the plasma that contain the element nitrogen are categorized as being protein- or nonprotein-nitrogen compounds. The principal substances included among the nonprotein- nitrogen compounds are urea, amino acids, uric acid, creatinine, creatine, and ammonia. Of these compounds, urea is present in the plasma in the greatest concentration, comprising approximately 45% of the nonprotein-nitrogen fraction.

Which of the following classes of compounds has a sedative effect and as such is used to treat anxiety? A. Amphetamines B. Opiates C. Cannabinoids D. Benzodiazepines

D. Diazepam (Valium ) is an example of a benzodiazepine. This group of drugs is used for the treatment of anxiety. Oxazepam is an active metabolite of diazepam and is also available as a prescribed drug (Serax®). Detection of oxazepam glucuronide in the urine is used as a screening method for diazepam. Quantification of the benzodiazepines may be achieved using HPLC.

Which of the following is not associated with the enzyme-multiplied immunoassay technique (EMIT)? A. Is a homogeneous enzyme immunoassay B. Determines antigen concentration C. Employs a labeled reactant D. Enzyme reacts with drug in serum sample

D. EMIT employs a homogeneous enzyme immunoassay method. This means that physical separation of the free labeled antigen from the antibody-bound-labeled antigen is not necessary for measurement. This is possible because only the free labeled antigen remains active. In the EMIT system the antigen is labeled with an enzyme (e.g., glucose-6-phosphate dehydrogenase). Determination of the drug concentration in the serum sample is made when the free enzymelabeled drug reacts with substrate and coenzyme, resulting in an absorbance change that is measured spectrophotometrically. The drug in the serum sample is the unlabeled antigen in the assay, and it competes with the labeled drug for the binding sites on the antibody.

Which of the following statements applies to the preferred use of plasma or serum, rather than whole blood, for glucose determination? A. Glucose is more stable in separated plasma or serum. B. Specificity for glucose is higher with most methods when plasma or serum is used. C. It is convenient to use serum or plasma with automated instruments because whole blood requires mixing immediately before sampling. D. All the above.

D. Glucose determinations are generally performed on serum or plasma rather than whole blood. Serum or plasma is more convenient to use than whole blood in most automated systems because serum does not require mixing before sampling. Glucose stability is greater in separated plasma than in whole blood because glycolysis is minimized. Specificity for glucose is higher when plasma or serum is used because variations attributable to interfering substances in the red cells are avoided.

Which of the following is not characteristic of severe hyperglycemia? A. Polyuria B. Ketonuria C. Glycosuria D. Hypoglucagonemia

D. In uncontrolled diabetes mellitus, the blood glucose level exceeds the renal threshold of approximately 180 mg/dL for glucose, leading to glycosuria and polyuria. The excess secretion of glucagon stimulates lipolysis, with increased formation of acetoacetic acid. In the blood, the ketoacids dissociate, with the hydrogen ions being buffered by bicarbonate. This causes the bicarbonate to become depleted and leads to metabolic acidosis.

When a blood ammonia determination is performed, the blood specimen must be treated in a manner that will ensure that A. The deamination process continues in vitro B. Glutamine formation in vitro is avoided C. The transamination process continues in vitro D. Ammonia formation in vitro is avoided

D. It is necessary that certain precautions in specimen handling be exercised because the enzymatic process of deamination of amides continues at room temperature after a blood sample is drawn. When blood is drawn for ammonia analysis, it is critical that any in vitro ammonia formation be prevented. It is recommended that the tube containing the blood specimen be placed in an ice bath immediately after the blood is drawn, because the cold environment will help retard metabolic processes. It is also important that the chemical analysis of the specimen be started within 20 minutes of drawing the specimen.

Singlet oxygen reacting with a precursor chemiluminescent compound to form a decay product whose light energizes a fluorophore best describes A. Fluorescent polarization immunoassay B. Enzyme-multiplied immunoassay technique C. Electrochemiluminescence immunoassay D. Luminescent oxygen channeling immunoassay

D. Luminescent oxygen channeling immunoassay (LOCI™) is a homogeneous technique that is an adaptation of the chemiluminescent immunoassay. Singlet oxygen reacts with the precursor chemiluminescent compound to form a chemiluminescent product that decays and emits light. This light energy is accepted by a fluorophore, which results in light emission of a longer wavelength. In this assay, the chemiluminescent signal is enhanced by the resulting fluorescent signal which is proportional to the concentration of analyte in the serum sample.

The placenta secretes numerous hormones both protein and steroid. Which of the following hormones is not secreted by the placenta? A. Human chorionic gonadotropin (hCG) B. Estrogen C. Human placental lactogen (HPL) D. Luteinizing hormone (LH)

D. Luteinizing hormone (LH) is secreted only by the anterior pituitary. A protein hormone, human chorionic gonadotropin (hCG), appears soon after conception and is thus used for early detection of pregnancy. Human placental lactogen (HPL), also a protein hormone, is produced only by the placenta and is measurable between the seventh and ninth weeks. HPL steadily increases throughout pregnancy and peaks near term. Analysis of HPL for placental dysfunction has been successful; however, it is not widely used for this purpose. During pregnancy the placenta is the main source of estrogen and progesterone. Both hormones are needed for the maintenance of pregnancy.

Which of the following is a cause of metabolic alkalosis? A. Late stage of salicylate poisoning B. Uncontrolled diabetes mellitus C. Renal failure D. Excessive vomiting

D. One of the primary reasons for metabolic alkalosis, especially in infants, is vomiting. Hydrogen ions are lost in the vomit, and the body reacts to replace them in the stomach. Consequently, hydrogen is lost from the plasma. This loss of hydrogen is due to a metabolic as opposed to a respiratory reason. Salicylate poisoning, uncontrolled diabetes mellitus, and renal failure all lead to metabolic acidosis either through an overproduction of ketone bodies, such as acetoacetic acid and beta-hydroxybutyric acid, or because of a reduced excretion of acid by the kidneys.

Of the total serum osmolality, sodium, chloride, and bicarbonate ions normally contribute approximately what percent? A. 8 B. 45 C. 75 D. 92

D. Osmolality is a measure of the total number of solute particles per unit weight of solution and is expressed as milliosmoles per kilogram of water. The normal osmolality of serum is in the range of 275-295 mOsm/kg water. For monovalent cations or anions the contribution to osmolality is approximately 92%. Other serum electrolytes, serum proteins, glucose, and urea contribute to the remaining 8%.

Which of the following is an effect of increased parathyroid hormone secretion? A. Decreased blood calcium levels B. Increased renal reabsorption of phosphate C. Decreased bone resorption D. Increased intestinal absorption of calcium

D. PTH has physiological actions on bone, kidney, and intestine. Its overall effect is to raise serum ionized calcium levels and lower serum phosphorus levels. Its actions on various organs are the result of a combination of both direct and indirect effects. In bones, PTH directly acts to increase bone resorption, thereby increasing both calcium and phosphorus in the blood. In the kidneys, PTH directly acts on the renal tubules to decrease phosphate reabsorption. In combination with the effect on bone, the overall result is a decrease in blood phosphorus levels. In the intestines, PTH acts to increase absoiption of calcium by its action in increasing 1,25-dihydroxyvitamin D3 synthesis in the kidneys, which in turn stimulates intestinal absorption of calcium.

Primary aldosteronism results from a tumor of the adrenal cortex. How would the extracellular fluid be affected? A. Normal sodium, decreased potassium levels B. Decreased sodium, decreased potassium levels C. Decreased sodium, increased potassium levels D. Increased sodium, decreased potassium levels

D. Primary aldosteronism is characterized by the hypersecretion of aldosterone, a mineralocorticoid, by the zona glomerulosa cells of the adrenal cortex. Excessive secretion of aldosterone will increase renal tubular reabsorption of sodium, resulting in a decrease in the loss of sodium in the urine. The net result of this mechanism is increased sodium in the extracellular fluid. Additionally, there will be increased renal excretion of potassium, causing a decrease of potassium in the extracellular fluid.

Which of the following functions as an inhibiting factor for somatotropin release? A. Gonadotropin-releasing hormone B. Growth hormone-releasing hormone C. Somatomedin D. Somatostatin

D. Somatostatin is also known as growth hormone- inhibiting hormone (GHIH). Somatostatin is a 14-amino-acid peptide that is secreted by the hypothalamus and is an inhibitor of growth hormone (somatotropin) secretion by the pituitary. It is also secreted by a variety of other organs and is a powerful inhibitor of insulin and glucagon secretion by the pancreas. Somatostatin can be measured by immunoassay methods, but its concentration in the peripheral circulation is extremely low, making it likely that its action is mostly at or near the site of secretion.

Which of the following disorders is not associated with an elevated blood ammonia level? A. Reye syndrome B. Renal failure C. Chronic liver failure D. Diabetes mellitus

D. The gastrointestinal tract is the primary source of blood ammonia. With normal liver function, ammonia is metabolized to urea for urinary excretion. When blood ammonia levels become elevated, toxicity of the central nervous system occurs. Diseases associated with elevated blood ammonia levels include Reye syndrome, renal failure, chronic liver failure, cirrhosis, and hepatic encephalopathy.

Levels of 8-9% carboxyhemoglobin saturation of whole blood are commonly found in which of the following situations? A. Fatal carbon monoxide poisoning B. Acute carbon monoxide poisoning C. Nonsmoking residents of rural areas D. Cigarette smokers

D. The term "carboxyhemoglobin saturation" refers to the fraction of circulating hemoglobin combined with carbon monoxide. Nonsmokers generally have carboxyhemoglobin saturations ranging from 0.5 to 1.5%. Fatal carbon monoxide poisoning is usually associated with carboxyhemoglobin saturations of more than 60%, and acute symptoms begin to appear at saturations of 20%. Cigarette smokers exhibit levels of 8-9% carboxyhemoglobin, but occasionally saturations of greater than 16% have been reported in heavy smokers.

Which of the following is not a problem inherent in turbidimetry? A. Variation in particle size of samples B. Variation in particle size of standards C. Rate of aggregation or settling of particles D. Need to maintain a constant and specific temperature

D. Turbidimetry is the measurement of the amount of light blocked by particulate matter in passing through a turbid solution. The amount of light blocked depends on the number and the size of the particles. Hence the particle size in samples and standards must be comparable. Consistent timing of sample preparation and assay helps to avoid errors resulting from aggregation or settling of particles. The procedure is usually carried out at room temperature. Slight variations in temperature are not critical.

Bromcresol green (BCG) & HABA

Dye-Binding methods for albumin

Amylase

Enzyme in saliva that breaks the chemical bonds in starches

aspartate transaminase (AST)

Found in cardiac muscle, liver, RBC, and other tissues.

hs-CRP (high sensitivity C-reactive protein)

Glycoprotein made by liver, when acute inflammation. Link in development atherosclerosis- stable plaque weakening and rupturing. > 3.0 mg/dL greatest risk 1.0-3.0 mg/dL moderate risk < 1.0 mg/dL low risk

alpha 2 globulins

Haptoglobin and Ceruloplasmin

respiratory alkalosis causes

Hyperventilation (causes hypocapnia); Contributing Causes: Overventilation on a ventilator, Response to acidosis, Bacteremia, Thyrotoxicosis, Fever, Hepatic failure, Response to hypoxia, Hysteria

respiratory acidosis causes

Hypoventilation (causes hypercapnia); Contributing Causes: COPD, Pulmonary dz, Drugs, Obesity, Mechanical asphyxia, Sleep Apnea

Lactic acid test

Indicator of oxygen deprivation. End product of glucose metabolism when oxygen deprivation occurs. Build up results in lactic acidosis. Sample collection and handling critical.

hepatic jaundice

Jaundice resulting from conditions that affect the liver cells directly, such as viral or toxic hepatitis. liver function= decreased unconjugated/urobili= increased conjugated= increased in urine= yes

23

K is _____ times higher in cells than in plasma

spectrophotometric method

LD converts pyruvate to lactate while oxidizing NADH to NAD. rate of decrease in absorbance of NADH at 340nm is proportional to LD activity.

Zero order kinetics

Large excess of substrate so that the amount of enzyme activity is only rate-limiting factor

GGT

Liver specific, highest from biliary obstruction or after alcohol ingestion.

Monoclonal gammopathy

M component (spike); sign of plasma cell disorder

Ceruloplasmin

Major copper carrying protein in the blood; plays a role in iron metabolism.

alanine transaminase (ALT)

More liver specific than AST increase in liver disease

Acid Phosphatase (ACP)

Old marker for prostatic carcinoma and treatment monitoring

inverse

PO4 and Ca have a ______ relationship

bone resorption, renal tubular reabsorption Ca

PTH causes increase blood Calcium by ______ & ______

alcoholism, biliary obstruction, cirrohosis

REMEMBER: elevated liver enzymes are as easy as ABC

Total Iron Binding Capacity (TIBC)

Represents the available iron binding sites on transferrin

biuret reaction

Serum total protein. used most frequently. Depends on presence of >2 peptide bonds which bonds to form a purple complex with copper salts in alkaline solution.

parathyroid hormone (PTH)

Synthesized and released by the parathyroid glands stimulated by low Ca. suppressed by high Ca concentrations

anion gap

The difference between unmeasured plasma cations and anions that is used to distinguish different causes of metabolic acidosis.

C-peptide test

This peptide is created in a 1:1 ratio with insulin and is released into the bloodstream with insulin, therefor it is a proxy for insulin secretion levels.

4-8

Troptonin level rise withing _____ hours of AMI; peak at 12-14 hours; remain elevated for 10 days

Amnisure test

Vaginal secretions obtained using swab, swab paced in vial with solvent, solvent extracts sample after one minute, swab discarded and test strip placed in solvent fo 5 to 10 minutes, one line on strip indicates no rupture, and 2 lines on control strip indicates ROM.

Xylose absorption test

Xylose absorbed and ecreted into urine without need of pancreatic enzymes. Helps distinguish intestinal malabsorption and pancreatic-based malabsorption.

85

_____ % of Na is absorbed by the kidney tubules

opiates

_____ such as morphine can cause elevations in amylase.

Hypogammaglobulinemia

an abnormally low concentration of gamma globulin in the blood and increased risk of infection

Homocysteine

an amino acid normally present in the blood that, when found at high levels, may be related to higher risk of cardiovascular disease associated with vitamin b6, b12, and folic acid

high anion gap

an increase concentration in unmeasured anions: ethanol, ketones, lactic acid; and decreased level in unmeasured cations: Mg, Ca signifies a _______ anion gap

LDL and Iron deficiency

an increase in beta globulin results in and elevation in _____ and ________

acute phase, pregnancy

an increase in ceruloplasmin may be an indication of ______ and/or ______

troponin

best test for diagnosing AMI. -- 3 muscle fiber proteins

CK-BB

brain, GI, colon, prostate, uterus =

beta globulins

carrier protein for iron (transferrin) and lipids (lipoproteins)

Hypercalcemia

causes muscle weakness, disorientation.

human plasma lipids

cholesterol, triglycerides, phospholipids, and nonesterified fatty acids are ___________________

down's syndrome

decreased levels of AFP test may indicate this syndrome

Wilson's disease

decreased serum ceruloplasmin is an indication of what disease?

Urinary albumin

detects above normal albumin in urine; may signify renal damage due to HTN or diabetes

conjugated bilirubin

direct bilirubin; water soluble; never normal in urine - reacts rapidly with Diazo reagent

protein electrophoresis (PEP)

direction of migration of proteins in an electrical field determined by surface charge of proteins.

Lactose Tolerance Test

evaluates defiency of lactase in small bowel. --Ingest lactose and measure blood glucose.

hyperparathyroidism

excessive production of parathormone

70-99

fasting reference range for serum or plasma glucose is ________ mg/dL

Rickets disease

faulty mineral deposit in bones of *children due to a vitamin D deficiency*. This results in soft, deformed bones, bowed legs.

A1C test

for diabetes by checking sugar on red blood cells to get an average glucose level over several months

Alkaline Phosphatase (ALP)

found in bone, intestinal mucosa, renal tubule cells, biliary tree (liver), leukocytes, placenta, some tumors -optimum pH= 10, Mg++ activation

LD

found in many tissues other than liver (ex: heart, skeletal muscle)

10

glucose levels decrease about _____ mg/dL per hour in whole blood

22-26 mM/L

normal range for bicarbonate (HCO3)

35-45 mmHg

normal range for partial CO2 (pCO2)?

85-105 mmHg

normal range for partial oxygen (pO2)?

enzyme

organic catalysts responsible for most reactions in the body

Henderson-Hasselbalch equation

pH = pKa + log ([HCO3-]/[H2CO3]

metabolic acidosis lab findings

pH- decreased (<7.35) HCO3- decreased (<22 mM/L)

Respiratory acidosis lab findings

pH- decreased (<7.35) pCO2- increased (>45mmHg)

metabolic alkalosis lab findings

pH- increased (>7.43) HCO3- increased (<26mM/L)

respiratory alkalosis lab findings

pH- increased (>7.45) pCO2- decreased (<35 mmHg)

lipase

pancreatic enzyme necessary to digest fats. more specific test for acute pancreatitis

competitive, noncompetitive, uncompetitive inhibitors

substances that can reduce the rate of the enzyme reaction.

sweat chloride

test of sweat production stimulation by electrical means then measuring chloride content to diagnose cystic fibrosis (>60 mM/L)

20-55

that is the normal range for the percentage of transferrin saturation?

Glucose

the form of sugar that circulates in the blood and provides the major source of energy for body tissues. When its level is low, we feel hunger.

Apolipoproteins

the protein components of lipoproteins

Ergocalciferol

vitamin D2, a form of vitamin D found exclusively in plant foods

Cholecalciferol

vitamin D3, a form of vitamin D found in animal foods and the form we synthesize from the sun

PTH v. Calcitonin, Vitamin D

what 3 hormones control Ca?

sodium heparinate

what anticoagulant is used in collection blood for acid base testing.

sodium fluoride

what anticoagulant slows down glycolysis?

hyperparathyroidism, cancer with bone metastasis, multiple myeloma, renal failure

what are disease states associated with Hypercalcemia

diabetic acidosis(metabolic), diarrhea, addison's disease, renal tubular disease

what are diseases associated with low Na?

hyperparathyroidism, imparied renal absorption

what are some disease states associated with decreased levels of PO4?

A 45-year-old woman has a fasting serum glucose concentration of 95 mg/dL (5.2 mmol/L) and a 2-hour postprandial glucose concentration of 105 mg/dL 95.8 mmol/L). The statement which best describes this patient's fasting serum glucose concentration is: A. normal; reflecting glycogen breakdown by the liver B. normal; reflecting glycogen breakdown by skeletal muscle C. abnormal; indicating diabetes mellitus D. abnormal; indicating hypoglycemia

Correct Answer: A

A patient is admitted to the emergency room in a stage of metabolic alkalosis. Which of the following would be consistent with this diagnosis? A. high TCO2, increased HCO3 B. low TCO2, increased HCO3 C. high TCO2, decreased H2CO3 D. low TCO2, decreased H2CO3

Correct Answer: A

A patient with hemolytic anemia will A. show a decrease in glycated Hgb value B. show an increase in glycated Hgb value C. show little or no change in glycated Hgb value D. demonstrate an elevated Hgb A1

Correct Answer: A

An emphysema patient suffering form fluid accumulation in the alveolar spaces is likely to be in what metabolic state? A. respiratory acidosis B. respiratory alkalosis C. metabolic acidosis D. metabolic alkalosis

Correct Answer: A

Refer to the following pattern: See BOC pg 89 Pic 1 Patient values; Reference Values -Total Protein: 6.1 g/dL (61 g/L); 6.0-8.0 g/dL (60-80 g/L) -Albumin: 2.3 g/dL (23 g/L); 3.6-5.2 g/dL (36-52 g/L) -Alpha-1: 0.2 g/dL (2 g/L); 0.1-0.4 g/dL (1-4 g/L) -Alpha-2: 0.5 g/dL (5 g/L); 0.4-1.0 g/dL (4-10 g/L) -Beta: 1.2 g/dL (12 g/L); 0.5-1.2 g/dL (5-12 g/L) -Gamma: 1.9 g/dL (19 g/L); 0.6-1.6 g/dL (6-16 g/L) This pattern is consistent with: A. cirrhosis B. acute inflammation C. polyclonal gammopathy (eg, chronic inflammation) D. alpha-1 antitrypsin deficiency; severe emphysema

Correct Answer: A

The cellulose acetate electrophoresisi at pH 8.6 of serum proteins will show an order of migration beginning with the fastest migration as follows: A. albumin, alpha-1 globulin, alpha -2 globulin, beta globulin, gamma globulin B. alpha-1 globulin, alpha-2 globulin, beta globulin, gamma globulin, albumin C. albumin, alpha-2 globulin, alpha -1 globulin, beta globulin, gamma globulin D. gamma globulin, beta globulin, alpha -2 globulin, alpha -1 globulin, albumin

Correct Answer: A

The expected blood gas results for a patient in chronic renal failure would match the patter of: A. metabolic acidosis B. respiratory acidosis C. metabolic alkalosis D. respiratory alkalosis

Correct Answer: A

The following data was obtained from a cellulose acetate protein electrophoresis scan: -albumin are: 75 units -gamma globulin are: 30 units -total area: 180 units -total protein: 6.5 g/dL (65 g/L) The gamma globulin content in g/dL is: A. 1.1 g/dL (11 g/L) B. 2.7 g/dL (27 g/L) C. 3.8 g/dL (38 g/L) D. 4.9 g/dL (49 g/L)

Correct Answer: A

The primary function of serum albumin in the peripheral blood is to: A. maintain colloidal osmotic pressure B. increase antibody production C. increase fibrinogen formation D. maintain blood viscosity

Correct Answer: A

The protein that has the highest dye-binding capacity is: A. albumin B. alpha globulin C. beta globulin D. gamma globulin

Correct Answer: A

Total glycosylated hemoglobin levels in a hemolysate reflect the: A. average blood glucose levels of the past 2-3 months B. average blood glucose levels for the past week C. blood glucose level at the time the sample is drawn D. hemoglobin A1C level at the time the sample is drawn

Correct Answer: A

Which of the following amino acids is associated with sulfhydryl group? A. cysteine B. glycine C. serine D. tyrosine

Correct Answer: A

Which of the following is true about direct ion selective electrodes for electrolytes? A. whole blood specimens are acceptable B. elevated lipids cause falsely decreased results C. elevated proteins cause falsely decreased results D. elevated platelets cause falsely increased results

Correct Answer: A

A healthy person with a blood glucose of 80 mg/dL (4.4 mmol/L) would have a simultaneously determined cerebrospinal fluid glucose value of: A. 25 mg/dL ( 1.4 mmol/L) B. 50 mg/dL (2.3 mmol/L) C. 100 mg/dL (5.5 mmol/L) D. 150 mg/dL (8.3 mmol/L)

Correct Answer: B

In electrophoresis of proteins, when the sample is placed in an electric field connected to a buffer of pH 8.6, all of the proteins: A. have a positive charge B. have a negative charge C. are electrically neutral D. migrate toward the cathode

Correct Answer: B

Osmolal gap is the difference between: A. the ideal and real osmolality values B. calculated and measured osmolality values C. plasma and water osmolality values D. molality and molarity at 4C

Correct Answer: B

Refer to the following illustration: See BOC pg 90 Pic 1 Patient values; Reference Values -Total Protein: 7.8 g/dL (78 g/L); 6.0-8.0 g/dL (60-80 g/L) -Albumin: 3.0 g/dL (30 g/L); 3.6-5.2 g/dL (36-52 g/L) -Alpha-1: 0.4 g/dL (4 g/L); 0.1-0.4 g/dL (1-4 g/L) -Alpha-2: 1.8 g/dL (18 g/L); 0.4-1.0 g/dL (4-10 g/L) -Beta: 0.5 g/dL (5 g/L); 0.5-1.2 g/dL (5-12 g/L) -Gamma: 1.1 g/dL (11 g/L); 0.6-1.6 g/dL (6-16 g/L) The serum protein electrophoresis pattern is consistent with: A. cirrhosis B. acute inflammation C. polyclonal gammopathy (eg, chronic inflammation) D. alpha-1-antitrypsin deficiency; severe emphysema

Correct Answer: B

The direction in which albumin migrates (ie, toward anode or cathode) during electrophoretic separation of serum proteins, at pH 8.6, is determined by: A. the ionization of the amine groups, yielding a net positive charge B. the ionization of the carboxyl groups, yielding a net negative charge C. albumin acting as a switterion D. the density of the gel layer

Correct Answer: B

The following laboratory results were obtained: Serum electrolytes -sodium: 136 mEq/L (136 mmol/L) -potassium: 4.4 mEq/L (4.4 mmol/L) -chloride: 92 mEq/L (92 mmol/L) -bicarbonate: 40 mEq/L (40 mmol/L) Arterial blood -pH: 7.32 -PCO2: 79 mm Hg These results are most compatible with: A. respiratory alkalosis B. respiratory acidosis C. metabolic alkalosis D. metabolic acidosis

Correct Answer: B

A patient with Type-I, insulin-dependent diabetes mellitus has the following results: Test; Patient; Reference Range -fasting blood glucose: 150 mg/dL (8.3 mmol/L); 70-110 (3.9-6.1 mmol/L) -hemoglobin A1C: 8.5%; 4.0%-6.0% -fructosamine: 2.5 mmol/L; 2.0-2.9 mmol/L After reviewing these test results, the technologist concluded that the patient is in a: A. "steady state" of metabolic control B. state of flux, progressively worsening metabolic control C. improving stat of metabolic control as indicated by fructosamine D. state of flux as indicted by the fasting glucose level

Correct Answer: C

A physician requested that electrolytes on a multiple myeloma patient specimen be run by direct ISE and not indirect ISE because: A. excess protein binds Na in indirect ISE B. Na is falsely increased by indirect ISE C. Na is falsely decreased by indirect ISE D. excess protein reacts with diluent in indirect ISE

Correct Answer: C

At a pH of 8.6 the gamma globulins move toward the cathode, despite the fact that they are negatively charged. What is the phenomenon called? A. reverse migration B. molecular sieve C. endosmosis D. migratory inhibition factor

Correct Answer: C

Refer to the following illustration: See BOC pg 88 Pic 2 Patient values; Reference Values -Total Protein: 8.9 g/dL (89 g/L); 6.0-8.0 g/dL (60-80 g/L) -Albumin: 4.8 g/dL (48 g/L); 3.6-5.2 g/dL (36-52 g/L) -Alpha-1: 0.3 g/dL (3 g/L); 0.1-0.4 g/dL (1-4 g/L) -Alpha-2: 0.7 g/dL (7 g/L); 0.4-1.0 g/dL (4-10 g/L) -Beta: 0.8 g/dL (8 g/L); 0.5-1.2 g/dL (5-12 g/L) -Gamma: 2.3 g/dL (23 g/L); 0.6-1.6 g/dL (6-16 g/L) The serum protein electrophoresis pattern is consistent with: A. cirrhosis B. acute inflammation C. monoclonal gammopathy D. polyclonal gammopathy (eg, chronic inflammation)

Correct Answer: C

Refer to the following illustration: See BOC pg 90 Pic 2 Patient values; Reference Values -Total Protein: 8.5 g/dL (85 g/L); 6.0-8.0 g/dL (60-80 g/L) -Albumin: 4.3 g/dL (43 g/L); 3.6-5.2 g/dL (36-52 g/L) -Alpha-1: 0.3 g/dL (3 g/L); 0.1-0.4 g/dL (1-4 g/L) -Alpha-2: 0.7 g/dL (7 g/L); 0.4-1.0 g/dL (4-10 g/L) -Beta: 0.9 g/dL (9 g/L); 0.5-1.2 g/dL (5-12 g/L) -Gamma: 2.3 g/dL (2.3 g/L); 0.6-1.6 g/dL (6-16 g/L) The above serum protein electrophoresis pattern is consistent with: A. cirrhosis B. monoclonal gammopathy C. polyclonal gammopathy (eg, chronic inflammation) D. alpha-1-antitrypsin deficiency; severe emphysema

Correct Answer: C

The reference ranges for the pH of arterial blood measure at 37C is: A. 7.28-7.34 B. 7.33-7.37 C. 7.35-7.45 D. 7.45-7.50

Correct Answer: C

The relative migration rate of proteins on cellulose acetate is based on: A. molecular weight B. concentration C. ionic charge D. particle size

Correct Answer: C

A blood gas sample was sent to the lab on ice, and a bubble was present in the syringe. The blood had been exposed to room air for at least 30 minutes. The following change in blood gases will occur: A. CO2 content increased/PCO2 decreased B. CO2 content and PO2 increased/pH increased C. CO2 content and PCO2 decreased/pH decreased D. PO2 increased/HCO3 decreased

Correct Answer: D

A characteristic of Bence Jones protein that is used to distinguid it form other urinary proteins is its solutiblity: A. in ammonium sulfate B. in sulfuric acid C. at 40 - 60 C D. at 100 C

Correct Answer: D

A sweat chloride result of 55 mEq/L (55 mmol/L) and a sweat sodium of 52 mEq/L (52 mmol/L) were obtained on a patient who has a history of respiratory problems. The best interpretation of these result is: A. normal B. normal sodium and an abnormal chloride test should be repeated C. abnormal results D. borderline results, the test should be repeated

Correct Answer: D

Blood samples were collected at the beginning of an exercise class and after thirty minutes of aerobic activity. Which of the following would be most consistent with the post-exercise sample? A. normal lactic acid, low pyruvate B. low lactic acid, elevated pyruvate C. elevated lactic acid, low pyruvate D. elevated lactic acid, elevated pyruvate

Correct Answer: D

Following overnight fasting, hypoglycemia in adults is defined as a glucose of: A. </= 70 mg/dL (</= 3.9 mmol/L) B. </= 60 mg/dL (</= 3.3 mmol/L) C.</= 55 mg/dL (</= 3.0 mmol/L) D.</= 45 mg/dL (</= 2.5 mmol/L)

Correct Answer: D

Metabolic acidosis is described as a(n): A. increase in CO2 content and PCO2 with a decreased pH B. decrease in CO2 content with an increased pH C. increase in CO2 with an increased pH D. decrease in CO2 content and PCO2 with a decreased pH

Correct Answer: D

Monitoring long-term glucose control in patients with adult onset diabetes mellitus can best be accomplished by measuring: A. weekly fasting 7 AM serum glucose B. glucose tolerance testing C. 2-hour postprandial serum glucose D. hemoglobin A 1C

Correct Answer: D

Pregnant women with symptoms of thirst, frequent urination or unexplained weight loss should have which of the following tests performed? A. tolbutamide test B. lactose tolerance test C. epinephrine tolerance test D. glucose tolerance test

Correct Answer: D

Refer to the following illustration: See BOC pg 88 Pic 1 Patient values; Reference Values -Total Protein: 7.3 g/dL (73 g/L); 6.0-8.0 g/dL (60-80 g/L) -Albumin: 4.2 g/dL (42 g/L); 3.6-5.2 g/dL (36-52 g/L) -Alpha-1: 0.0 g/dL (0 g/L); 0.1-0.4 g/dL (1-4 g/L) -Alpha-2: 0.9 g/dL (9 g/L); 0.4-1.0 g/dL (4-10 g/L) -Beta: 0.8 g/dL (8 g/L); 0.5-1.2 g/dL (5-12 g/L) -Gamma: 1.4 g/dL (14 g/L); 0.6-1.6 g/dL (6-16 g/L) This electrophoresis pattern is consistent with: A. cirrhosis B. monoclonal gammopathy C. polyclonal gammopathy (eg, chronic inflammation D. alpha-1 antitrypsin deficiency; severe emphysema

Correct Answer: D

Serum "anion gap" is increased in patients with: A. renal tubular acidosis B. diabetic alkalosis C. metabolic acidosis due to diarrhea D. lactic acidosis

Correct Answer: D

The buffering capacity of blood is maintained by a reversible exchange process between bicarbonate and; A. sodium B. potassium C. calcium D. chloride

Correct Answer: D

The conversion of glucose or other hexoses into lactate or pyruvate is called: A. glycogenesis B. glycogenolysis C. gluconeogenesis D. glycolysis

Correct Answer: D

The degree to which the kidney concentrates the glomerular filtrate can be determined by: A. urine creatine B. serum creatinine C. creatinine clearance D. urine to serum osmolality ration

Correct Answer: D

The following laboratory resutls were obtained: BF type; Calcium; Alkaline Phosphate; Alkaline phosphatase -serum: increased: decrease; normal or increased -urine: increased; increased These result are most compatible with: A. multiple myeloma B. milk-alkali syndrome C. sarcoidosis D. primary hyperparathyroidism

Correct Answer: D

The most specific enzyme test of acute pancreatitis is: A. acid phosphates B. trypsin C. amylase D. lipase

Correct Answer: D

Which of the following hemoglobins has glucose-6-phosphate on the amino-terminal valine of the beta chain? A. S B. C C. A2 D. A1c

Correct Answer: D


Kaugnay na mga set ng pag-aaral

ASP Associate Safety Professional Exam

View Set

Insurance IL statutes and regukations for all lines

View Set

Finance 2000 chapter 1 questions

View Set